Sunteți pe pagina 1din 190

SHIPSIDE INCORPORATED same being not a mandatory and jurisdictional

vs. requirement, and as to the non-forum shopping the


THE HON. COURT OF APPEALS, HON. same was considered to be valid because the case
REGIONAL TRIAL COURT, BRANCH 26 & the of the petitioner must be litigated based on its merit
REPUBLIC OF THE PHILIPPINES and must not be dismissed based on technical and
G.R. No. 143377, February 20, 2001 procedural infirmities, which were actually cured.

FACTS:
Pilipinas Loan Company vs SEC
Originally four lots were owned Rafael G.R. No. 104720
Galvez. He subsequently sold lot 1 and 4 in favor of April 4, 2001
Filipina Mamaril, Cleopatra Llana, Regina Bustos,
and Erlinda Balatbat in a deed of sale. Mamaril later
sold lot 1 to Lepanto Consolidated Mining FACTS:
Company. Later, unknown to Lepanto, the RTC
declared the OCT registered in the name of Galvez Private respondent Filipinas Pawnshop, Inc. is a
as null and void and ordered the cancellation duly organized corporation registered with the
thereof. On October 28, 1963, Lepanto Securities and Exchange Commission on February
Consolidated Mining Company sold to herein 9, 1959. The articles of incorporation of private
petitioner Lots No. 1 and 4. Meanwhile the decision respondent states that its primary purpose is to
of the CA became final and executory and a writ extend loans at legal interest on the security of
was issued, however said writ remained unsatisfied either personal properties or on the security of real
for 24 years. properties, and to finance installment sales of motor
Office of the Solicitor General filed a vehicles, home appliances and other chattels.
complaint for revival of judgment and cancellation of
titles before the Regional Trial Court of the First Petitioner is a lending corporation duly registered
Judicial Region against the successors of Galvez with the SEC on July 27, 1989. Based on its articles
and herein petitioner and its motion for of incorporation, the primary purpose of petitioner is
reconsideration was likewise turned down. The CA “to act as a lending investor or, otherwise, to
affirmed the same, hence this petition. engage in the practice of lending money or
extending loans on the security of real or personal,
ISSUE: tangible or intangible properties whether as pledge,
real or chattel mortgage or otherwise, xxx without
Whether or not the filing of the petition was however, engaging in pawnbroking as defined
authorized by the BOD of petitioner. under PD 114."

RULING: Private respondent filed a complaint with the


Prosecution and Enforcement Department (PED) of
YES. the SEC and alleged that: (1) petitioner, contrary to
the restriction set by the Commission, has been
The Court of Appeals dismissed the petition operating and doing business as a pawnbroker,
for certiorari on the ground that Lorenzo Balbin, the pawnshop or "sanglaan" in the same neighborhood
resident manager for petitioner, who was the where private respondent has had its own
signatory in the verification and certification on non- pawnshop for 30 years in violation of its primary
forum shopping, failed to show proof that he was purpose and without the imprimatur of the Central
authorized by petitioner's board of directors to file Bank to engage in the pawnshop business thereby
such a petition. causing unjust and unfair competition with private
It was clear from the record that when the respondent. Petitioner denied that it is engaged in
general manager filed the petition, there was no the pawnshop business, alleging that it is a lending
proof attached as to the authorization by the Board. investor duly registered with the Central Bank.
However, when the petitioner filed its motion for
reconsideration a resolution or secretary’s ISSUE:
certification stating that that on October 11, 1999, or
ten days prior to the filing of the petition, Balbin had Whether or not petitioner violated its primary
been authorized by petitioner's board of directors to franchise.
file said petition. The Court accepted this
certification, although belatedly presented, as a RULING:
valid authorization. The Court was reiterated that
belated submission of a verification is allowed the
A corporation, under the Corporation Code, has ruled that: (1) Lourdes M. de Leon was not
only such powers as are expressly granted to it by authorized to issue the subject checks in favor of
law and by its articles of incorporation, those which E.T. Henry, Inc.; (2) The issuance of the subject
may be incidental to such conferred powers, those checks by Lourdes M. de Leon and the late Antonio
reasonably necessary to accomplish its purposes de las Alas constituted ultra vires acts; and (3) The
and those which may be incident to its existence. subject checks were not issued for valuable
consideration. Hence, Atrium filed the petition.
In the case at bar, the limit of the powers of
petitioner as a corporation is very clear, it is Issue [1]: Whether the issuance of the checks was
categorically prohibited from "engaging in an ultra vires act.
pawnbroking as defined under PD 114". Hence, in
determining what constitutes pawnbrokerage, the Held [1]: The record reveals that Hi-Cement
relevant law to consider is PD 114.
Corporation issued the four (4) checks to extend
financial assistance to E.T. Henry, not as payment
Indispensable therefore to the determination of
of the balance of the P30 million pesos cost of
whether or not petitioner had violated its articles of
hydro oil delivered by E.T. Henry to Hi-Cement.
incorporation, was an inquiry by the SEC if Why else would petitioner de Leon ask for
petitioner was holding out itself to the public as a counterpart checks from E.T. Henry if the checks
pawnshop. It must be stressed that the
were in payment for hydro oil delivered by E.T.
determination of whether petitioner violated PD 114
Henry to Hi-Cement? Hi-Cement, however,
was merely incidental to the regulatory powers of
maintains that the checks were not issued for
the SEC, to see to it that a corporation does not go
consideration and that Lourdes and E.T. Henry
beyond the powers granted to it by its articles of engaged in a "kiting operation" to raise funds for
incorporation.
E.T. Henry, who admittedly was in need of financial
assistance. There was no sufficient evidence to
show that such is the case. Lourdes M. de Leon is
Atrium Management Corporation vs. Court of the treasurer of the corporation and is authorized to
Appeals sign checks for the corporation. At the time of the
[GR 109491, 28 February 2001], also De Leon vs. issuance of the checks, there were sufficient funds
Court of Appeals [GR 121894] in the bank to cover payment of the amount of P2
First Division, Pardo (J): 4 concur million pesos. Thus, the act of issuing the checks
was well within the ambit of a valid corporate act, for
Facts: Hi-Cement Corporation through its corporate it was for securing a loan to finance the activities of
signatories, Lourdes M. de Leon, treasurer, and the the corporation, hence, not an ultra vires act. An
late Antonio de las Alas, Chairman, issued checks ultra vires act is one committed outside the object
in favor of E.T. Henry and Co. Inc., as payee. E.T. for which a corporation is created as defined by the
Henry and Co., Inc., in turn, endorsed the four law of its organization and therefore beyond the
checks to Atrium Management Corporation for power conferred upon it by law" The term "ultra
valuable consideration. Upon presentment for vires" is "distinguished from an illegal act for the
payment, the drawee bank dishonored all four former is merely voidable which may be enforced by
checks for the common reason "payment stopped". performance, ratification, or estoppel, while the
On 3 January 1983, Atrium Management latter is void and cannot be validated.
Corporation filed with the Regional Trial Court,
Manila an action for collection of the proceeds of Issue [2]: Whether Lourdes M. de Leon and
four postdated checks in the total amount of P2 Antonio de las Alas were personally liable for the
million, after its demand for payment of the value of checks issued as corporate officers and authorized
the checks was denied. After due proceedings, on signatories of the check.
20 July 1989, the trial court rendered a decision
ordering Lourdes M. de Leon, her husband Rafael Held [2]: Personal liability of a corporate director,
de Leon, E.T. Henry and Co., Inc. and Hi-Cement trustee or officer along (although not necessarily)
Corporation to pay Atrium jointly and severally, the
with the corporation may so validly attach, as a rule,
amount of P2 million corresponding to the value of
only when: (1) He assents (a) to a patently unlawful
the four checks, plus interest and attorney's fees.
act of the corporation, or (b) for bad faith or gross
On appeal to the Court of Appeals, on 17 March
negligence in directing its affairs, or (c) for conflict of
1993, the Court of Appeals promulgated its decision interest, resulting in damages to the corporation, its
modifying the decision of the trial court, absolving stockholders or other persons; (2) He consents to
Hi-Cement Corporation from liability and dismissing
the issuance of watered down stocks or who, having
the complaint as against it. The appellate court
knowledge thereof, does not forthwith file with the
corporate secretary his written objection thereto; (3) 3. ID.; ID.; DONATION DISTINGUISHED FROM
He agrees to hold himself personally and solidarily GRATUITY. — While a donation may technically be
liable with the corporation; or (4) He is made, by a different from a gratuity, in substance they are the
specific provision of law, to personally answer for same. They are even similar to a pension. Thus, it
his corporate action." Herein, Lourdes M. de Leon was said that "A pension is a gratuity only when it is
and Antonio de las Alas as treasurer and Chairman granted for services previously rendered, and which
of Hi-Cement were authorized to issue the checks. at the time they were rendered gave rise to no legal
However, Ms. de Leon was negligent when she obligation." (Words and Phrases, Permanent
signed the confirmation letter requested by Mr. Yap Edition, p. 675; O’Dea v. Cook, 169 Pac., 306, 176
of Atrium and Mr. Henry of E.T. Henry for the Cal., 659.)
rediscounting of the crossed checks issued in favor
of E.T. Henry. She was aware that the checks were 4. ID.; POWERS OF A CORPORATION; ACTS
strictly endorsed for deposit only to the payee's PERFORMED WITHIN THE POWERS GRANTED
account and not to be further negotiated. What is ARE NOT "ULTRA VIRES. — Where the
more, the confirmation letter contained a clause that corporation was given broad and almost unlimited
was not true, that is, "that the checks issued to E.T. powers to carry out the purposes for which it was
Henry were in payment of Hydro oil bought by Hi- organized among them, to aid in any other manner
Cement from E.T. Henry". Her negligence resulted any person in the affairs and prosperity of whom it
in damage to the corporation. Hence, Ms. de Leon has a lawful interest, a donation made to the heirs
may be held personally liable therefor. of its late president in recognition of the valuable
services rendered by the latter which had
immensely contributed to its growth, comes within
this broad grant of power and can not be considered
an ultra vires act.
G.R. No. L-5377 December 29, 1954
5. ID.; ID.; "ULTRA VIRES" ILLEGAL ACTS
MARIA CLARA PIROVANA ET AL., plaintiffs- DISTINGUISHED; EFFECT OF RATIFICATION BY
appellees, STOCKHOLDERS. — Illegal acts of a corporation
vs. contemplate the doing of an act which is contrary to
THE DE LA RAMA STEAMSHIP CO., defendant- law, morals, or public order, or contravene some
appellant. rules of public policy or public duty, and are, like
similar transactions between individuals, void. They
SYLLABUS can not serve as basis of a court action, nor acquire
validity by performance, ratification, or estoppel. On
the other hand, ultra vires acts or those which are
1. CORPORATIONS; DONATIONS; DONATION not illegal and void ab initio but are merely within
GIVEN "OUT OF GRATITUDE FOR SERVICES the scope of the article of incorporation, are merely
RENDERED" IS REMUNERATIVE. — A donation voidable and may become binding and enforceable
given by the corporation to the minor children of its when ratified by the stockholders.
late president because he "was to a large extent
responsible for the rapid and very successful 6. ID.; ID.; "ULTRA VIRES" ACTS; RATIFICATION
development and expansion of the activities of this BY STOCKHOLDERS OF "ULTRA VIRES" ACTS
company" is remunerative in nature in CURES INFIRMITY. — The ratification by the
contemplation of law. stockholders of an ultra vires act which is not illegal
cures the infirmity of the corporate act and makes it
2. ID.; ID.; PERFECTED DONATION CAN ONLY perfectly valid and enforceable, specially so if it is
BE RESCINDED ON LEGAL GROUNDS. — Where not merely executory but executed and
the donation made by the corporation has not only consummated and no creditors are prejudiced
been granted in several resolutions duly adopted by thereby.
its board of directors but also it has been formally
ratified by its stockholders, with the concurrence of 7. ATTORNEY’S FEES, WHEN MAY BE
its only creditor, and accepted by the donee, the AWARDED AS DAMAGES. — When the
donation has reached the stage of perfection which defendant’s act or omission has compelled the
is valid and binding upon the corporation and as plaintiff to litigate with third persons or to incur
such cannot be rescinded unless there exist legal expenses to protect his interest, attorney’s fees may
grounds for doing so. be awarded as damages (Article 2208, paragraph 2,
of the new Civil Code).
Facts: demanded the payment of the credit due them,
amounting to P564,980.89, but the company
Enrico Pirovano was the President and General refused to pay. Thus, they instituted an action in the
Manager of the De la Rama Steamship Company. Court of First Instance of Rizal.
Early in 1941 the company insured the life of said
Enrico Pirovano in various Philippine and American Issue: Can defendant corporation give by way of
Life Insurance companies. Enrico Pirovano was donation the proceeds of said insurance policies to
largely responsible for the rapid and very successful the minor children of the late Enrico Pirovano under
development of the activities of the company. He the law or its articles of corporation, or is that
was killed by the Japanese in Manila sometime in donation an ultra vires act?
1944 leaving as his only heirs four minor children.

In view of the fact that Enrico Pirovano left


practically nothing to his heirs, the current President Held: After a careful perusal of the provisions of the
of De la Rama Steamship proposed that it is but fit articles of incorporation of the De la Rama
and proper that the company which owes so much company, we find that the corporation was given
to the deceased should make some provision for his broad and almost unlimited powers to carry out the
children. He proposed that out of the proceeds of purposes for which it was organized among them,
the insurance policies the sum of P400,000 be set (1) "To invest and deal with the moneys of the
aside for Pirovano’s minor children, said sum of company not immediately required, in such manner
money to be convertible into 4,000 shares of the as from time to time may be determined" and, (2)
stock of the Company, at par, or 1,000 shares for "to aid in any other manner any person, association,
each child. or corporation of which any obligation or in which
any interest is held by this corporation or in the
A resolution was adopted to carry out the proposal affairs or prosperity of which this corporation has a
and submitted to the stockholders of the De la lawful interest."
Rama company at a meeting properly convened,
and on that same date the same was duly The world deal is broad enough to include any
approved. manner of disposition, and refers to moneys not
immediately required by the corporation, and such
disposition may be made in such manner as from
time to time may be determined by the corporations.
Sometime in March 1950, the President of the
corporation, Sergio Osmeña, Jr., inquired to the The donation in question undoubtedly comes within
Securities and Exchange Commission asking for the scope of this broad power for it is a fact
opinion regarding the validity of the donation of the appearing in the evidence that the insurance
proceeds of the insurance policies to the Pirovano proceeds were not immediately required when they
children. were given away.

SEC rendered its opinion that the donation was void Granting arguendo that the donation given by
because the corporation could not dispose of its Pirovano children is outside the scope of the powers
assets by gift and therefore the corporation acted of the defendant corporation, or the scope of the
beyond the scope of its corporate powers. powers that it may exercise under the law, or it is an
ultra vires act, still it may said that the same can not
In 1951, in view of the failure of compliance with the be invalidated, or declared legally ineffective for the
conditions to which the above donation was made reason alone, it appearing that the donation
subject, and in view of the opinion of the SEC represents not only the act of the Board of Directors
Commissioner, the majority of the stockholders' but of the stockholders themselves as shown by the
voted to revoke the resolution approving the fact that the same has been expressly ratified in a
donation to the Pirovano children. resolution duly approved by the latter. By this
ratification, the infirmity of the corporate act, it may
The minor children of the late Enrico Pirovano,
has been obliterated thereby making the act
represented by their mother and guardian, Estefania
perfectly valid and enforceable. This is specially so adopted a resolution authorizing its president,
if the donation is not merely executory but executed among other things, to purchase the bonds in the
and consummated and no creditors are prejudice, Mindoro Sugar Company that was about to issue,
or if there are creditors affected, the latter has and to resell them, with or without the guarantee of
expressly given their confirmity. said trust corporation, at a price not less than par,
and to guarantee to the Philippine National Bank the
A distinction should be made between corporate payment of the indebtedness to said bank by the
acts or contracts which are illegal and those which Mindoro Sugar Company. Pursuance of this
are merely ultra vires. resolution, the Mindoro Sugar Company executed in
favor of the Philippine Trust Company the deed of
The former contemplates the doing of an act which
trust transferring all of its property to it in
is contrary to law, morals, or public policy or public
consideration of the bonds it had issued. Philippine
duty, and are, like similar transactions between the
Trust Company sold thirteen bonds, to Ramon Diaz.
individuals void. They cannot serve as basis of a
The Philippine Trust Company paid the appellant,
court action, nor require validity.
upon presentation of the coupons, the stipulated
Ultra vires acts on the other hand, or those which interest from the date of their maturity then it
are not illegal and void ab initio, but are not merely stopped payments; and thenceforth it alleged that it
within the scope of the articles of incorporation, are did not deem itself bound to pay such interest or to
merely voidable and may become binding and redeem the obligation because the guarantee given
enforceable when ratified by the stockholders. Said for the bonds was illegal and void.
donation, even if ultra vires in the supposition we
Issue: Whether or not PTC has the power to
have adverted to, is not void, and if voidable its
guarantee and does this act constitute an ultra vires
infirmity has been cured by ratification and
act?
subsequent acts of the defendant corporation. The
defendant corporation, therefore, is now prevented Held: No. It is not ultra vires for a corporation to
or estopped from contesting the validity of the enter into contracts of guaranty or suretyship where
donation. it does so in the legitimate furtherance of its
purposes and business. And it is well settled that
where a corporation acquires commercial paper or
IRINEO G. CARLOS, plaintiff-appellant, vs. bonds in the legitimate transaction of its business it
may sell them, and in furtherance of such a sale it
MINDORO SUGAR CO., ET AL., defendants- may, in order to make them the more readily
appellees. marketable, indorse or guarantee their payment.

Facts: The plaintiff brought this action to recover Whenever a corporation has the power to take and
from the defendants the value of four bonds, with dispose of the securities of another corporation, of
due and unpaid interest thereon, issued by the whatsoever kind, it may, for the purpose of giving
Mindoro Sugar Company and placed in trust with them a marketable quality, guarantee their payment,
the Philippine Trust Company which, in turn, even though the amount involved in the guaranty
guaranteed them for value received. Mindoro Sugar may subject the corporation to liabilities in excess of
Company is a corporation constituted in accordance the limit of indebtedness which it is authorized to
with the laws of the country. According to its articles incur. A corporation which has power by its charter
of incorporation one of its principal purposes was to to issue its own bonds has power to guarantee the
acquire and exercise the franchise granted by Act bonds of another corporation, which has been taken
No. 2720 to George H. Fairchild, to substitute the in payment of a debt due to it, and which it sells or
organized corporation. transfers in payment of its own debt, the guaranty
being given to enable it to dispose of the bond to
Philippine Trust Company is another domestic better advantage. And so guaranties of payment of
corporation its principal purpose, then, as its name bonds taken by a loan and trust company in the
indicates, is to engage in the trust business. The ordinary course of its business, made in connection
board of directors of the Philippine Trust Company, with their sale, are not ultra vires, and are binding.
When a contract is not on its face necessarily board of directors of the company expressing
beyond the scope of the power of the corporation by conformity to the above condition relative to the
which it was made, it will, in the absence of proof to responsibility to be assumed buy it in the event a
the contrary, be presumed to be valid. post office branch is opened as requested. To which
the company agreed and adopted a board
resolution which provides among others “That the
requirement of the Bureau of Posts that the
Republic v. Acoje Mining Co.,
3 SCRA 361 (1963) Company should accept full responsibility for all
cash received by the Postmaster be complied with,
and that a copy of this resolution be forwarded to
the Bureau of Posts.”
IX. CORPORATE POWERS, AUTHORITY AND ACTIVITIES
Ultra Vires Acts

(b) Ratification of Ultra Vires Acts: The post office branch was opened at the camp on
October 13, 1949 with one Hilario M. Sanchez as
postmaster. He is an employee of the company,
who went on a three-day leave but never returned.
The company immediately informed the officials of
Facts; the Manila Post Office and the provincial auditor of
Zambales of Sanchez' disappearance with the
result that the accounts of the postmaster were
checked and a shortage was found in the amount of
Acoje Mining Company, Inc. wrote the Director of P13,867.24. Several demands made upon the
Posts requesting the opening of a post, telegraph company for the payment of the shortage in line
and money order offices at its mining camp at Sta. with the liability it has assumed having failed, the
Cruz, Zambales, to service its employees and their government commenced the present action seeking
families that were living in said camp. Acting on the to recover the amount.
request, the Director of Posts wrote in reply stating
that if aside from free quarters the company would
provide for all essential equipment and assign a
responsible employee to perform the duties of a The company in its answer denied liability for said
postmaster without compensation from his office amount contending that the resolution of the board
until such time as funds therefor may be available of directors wherein it assumed responsibility for the
he would agree to put up the offices requested. The act of the postmaster is ultra vires, and in any event
company in turn replied signifying its willingness to its liability under said resolution is only that of a
comply with all the requirements outlined in the guarantor who answers only after the exhaustion of
letter of the Director of Posts. the properties of the principal, aside from the fact
that the loss claimed by the plaintiff is not supported
by the office record.

The Director of Posts again wrote a letter to the


company stating among other things that "In cases
where a post office will be opened under Issue;
circumstances similar to the present, it is the policy
of this office to have the company assume direct
responsibility for whatever pecuniary loss may be Whether or not the ultra vires act of the company
suffered by the Bureau of Posts by reason of any which in the sense that it has no authority to act on
act of dishonesty, carelessness or negligence on a matter which may render the company liable as a
the part of the employee of the company who is guarantor is valid.
assigned to take charge of the post office," thereby
suggesting that a resolution be adopted by the
Held; responsibility for all cash received by the
Postmaster, be complied with, and that a copy of
this resolution be forwarded to the Bureau of Posts."
On the basis of the foregoing facts, it is evident that
The claim that the resolution adopted by the board
the company cannot now be heard to complain that
of directors of appellant company is an ultra
it is not liable for the irregularity committed by its
vires act cannot be entertained it appearing that the
employee upon the technical plea that the resolution
same covers a subject which concerns the benefit,
approved by its board of directors is ultra vires. The
convenience and welfare of its employees and their
least that can be said is that it cannot now go back
families. While as a rule an ultra vires act is one
on its plighted word on the ground of estoppel.
committed outside the object for which a corporation
is created as defined by the law of its organization The Court agreed with the following findings of the a
and therefore beyond the powers conferred upon it quo the: "A mere reading of the resolution of the
by law (19 C.J.S., Section 965, p. 419), there are Board of Directors dated August 31, 1949, upon
however certain corporate acts that may be which the plaintiff based its claim would show that
performed outside of the scope of the powers the responsibility of the defendant company is not
expressly conferred if they are necessary to just that of a guarantor. Notice that the phraseology
promote the interest or welfare of the corporation. and the terms employed are so clear and sweeping
Thus, it has been held that "although not expressly and that the defendant assumed 'full responsibility
authorized to do so a corporation may become a for all cash received by the Postmaster.' Here the
surety where the particular transaction is reasonably responsibility of the defendant is not just that of a
necessary or proper to the conduct of its guarantor. It is clearly that of a principal."
business,"1 and here it is undisputed that the
establishment of the local post office is a
reasonable and proper adjunct to the conduct of the
business of appellant company. Indeed, such post Crisologo-Jose VS. CA (1989)
office is a vital improvement in the living condition of
Facts:
its employees and laborers who came to settle in its Ricardo S. Santos Jr. was the vice-
mining camp which is far removed from the postal president of Mover Enterprises, Inc., in charge of
facilities or means of communication accorded to marketing sales; the president was Atty. Oscar Z.
people living in a city or municipality.. Benares. Atty. Benares, in accommodation of his
clients, spouses Ong, issued check against Traders
After the company had signified its willingness to Royal Bank in the amount of Php 45,000 payable to
comply with the requirement of the government that petitioner Ernestina Crisologo-Jose. Since the
it furnish free quarters and all the essential check was under the account of their corporation,
Atty. Benares made Santos to sign the same as an
equipment that may be necessary for the operation
alternative to the treasurer who is unavailable. This
of the office including the assignment of an is in consideration of the waiver or quitclaim by
employee who will perform the duties of a petitioner over a certain parcel of property which the
postmaster, the Director of Posts agreed to the GSIS agreed to sell to the clients of Atty. Benares,
opening of the post office stating that "In cases Spouses Ong, with the understanding that upon
where a post office will be opened under approval by the GSIS of the compromise agreement
circumstances similar to the present, it is the policy with that spouses, the check will be encashed
accordingly. However, since the compromise
of this office to have the company assume direct
agreement was not approved within the expected
responsibility for whatever pecuniary loss may be period of time, the said check was replaced by
suffered by the Bureau of Posts by reason of any another check with the same amount and the same
act of dishonesty, carelessness or negligence on was signed again by both Santos and Atty.
the part of the employee of the company who is Benares. When petitioner deposited the
assigned to take charge of the post office," and replacement check, it was dishonored for
insufficiency of funds. Petitioner then constrained to
accepting this condition, the company, thru its board
file a criminal complaint for violation of B.P. 22
of directors, adopted forthwith a resolution of the against Santos and Benares. During the preliminary
following tenor: "That the requirement of the Bureau investigation, Santos tendered a check to Crisologo-
of Posts that the company should accept full Jose in which the latter refused the same. Hence,
Santos deposited it with the Clerk of Court. The trial The instant case falls squarely within the
court a quo dismissed the complaint filed by Santos purview of the aforesaid decisional rules. If we
and not persuaded to believe that consignation indulge petitioner in her aforesaid postulation, then
referred under Article 1256 is applicable to this she is effectively barred from recovering from Mover
case. CA reversed and set aside judgment of Enterprises, Inc. the value of the check. Be that as it
dismissal and revived Santos’s complaint for may, petitioner is not without recourse.
consignation, directing the court a quo to give due
course. Hence, this instant petition by Crisologo-
Jose who avers that the accommodation party in Harden vs benguet Consolidated Mining Co.
this case is Mover Enterprises, Inc. and not private
respondent who merely signed the check in Facts:
question in a representative capacity, that is, as
vice-president of said corporation, hence, he is not A petition was filed by F. M. Harden here in
liable. known as the petitioner, acting in his own behalf
and that of all other stockholders of the Balatoc
Issue:
Mining Co. who might join in the action and
W/N the corporation may be held liable on
the accommodation instrument? contribute to the expense of the suit against , The
defendants are the Benguet Consolidated Mining
Ruling: Co., the Balatoc Mining Co., H. E. Renz, John W.
No. Haussermann, and A. W. Beam. The connection of
The provision of the Negotiable Instruments Renz to this case, was that he was made as trustee
Law which holds an accommodation party liable on to the certificate of shares which was the subject of
the instrument to a holder for value, although such
holder at the time of taking the instrument knew him this case.
to be only the accommodation party, does not
include nor apply to corporations which are The principal purpose the original action
accommodation parties. This is because the issue was to annul a certificate covering 600,000 shares
or indorsement of negotiable paper by a corporation of the stock of the Balatoc Mining Co., which have
without consideration and for the accommodation of been issued to the Benguet Consolidated Mining
another is ultra vires. Hence, one who has taken the Co., and to secure to the Balatoc Mining Co.
instrument with knowledge of the accommodation
nature thereof cannot recover against a corporation The Benguet Consolidated Mining Co. was
where it is only an accommodation party. If the form organized in June, 1903, as a sociedad anonima in
of the instrument, or the nature of the transaction, is
conformity with the provisions of Spanish law; while
such as to charge the indorsee with knowledge that
the issue or indorsement of the instrument by the the Balatoc Mining Co. was organized in December
corporation is for the accommodation of another, he 1925, as a corporation, in conformity with the
cannot recover against the corporation thereon. provisions of the Corporation Law (Act No. 1459).
By way of exception, an officer or agent of a Both entities were organized for the purpose of
corporation shall have the power to execute or engaging in the mining of gold in the Philippine
indorse a negotiable paper in the name of the Islands, and their respective properties are located
corporation for the accommodation of a third person
only a few miles apart in the sub province of
only if specifically authorized to do so. Corollarily,
corporate officers, such as the president and vice- Benguet.
president, have no power to execute for mere
accommodation a negotiable instrument of the The Balatoc Corporation on its first years of
corporation for their individual debts or transactions operations was not so successful that led to the
arising from or in relation to matters in which the order of suspension made by the board of directors.
corporation has no legitimate concern. Since such After the said suspension order, a meeting was held
accommodation paper cannot thus be enforced by the board of directors in order to get capital.
against the corporation, especially since it is not
They then approach A.W. Beam president of the
involved in any aspect if the corporate business or
operations, the inescapable conclusion in law and in Benget Company to help in developing the
logic is that the signatories thereof shall be properties of Balatoc Corporation.
personally liable therefor, as well as the
consequences arising from their acts in connection Negations was made between them and
therewith. come up with the following contract wherein the
Benguet Company was to proceed with the secure in the possession of that improvement, and
development and construct a milling plant for the talk about putting the parties in status quo ante by
Balatoc mine, and to erect an appropriate power restoring the consideration with interest, while the
plant. In return Benguet Company should receive Balatoc Company remains in possession of what it
from the treasurer of the Balatoc Company shares obtained by the use of that money, does not quite
of a par value of P600,000, in payment for the first meet the case. Also, to mulct the Benguet Company
P600,000 be thus advanced to it by the Benguet in many millions of dollars in favor of individuals
Company. who have not the slightest equitable right to that
money in a proposition to which no court can give a
The petitioner in this case as one of the ready assent.
major stockholder of the Balatoc company together
with other stock holders to annul the issuance of the The penalties imposed on what is now Sec. 190 (A)
shares of stock for it was unlawful for Benguet of the Corporation Law for the violation of the
Company to possess shares of Balatoc company by prohibition in question are of such nature that they
can be enforced only by a criminal prosecution or by
the provision under a certain Philippine bill act
an action of Quo warranto. However these
specifically section 75 of the said act which states “it proceedings can be maintained only by the Attorney
shall be unlawful for any member of a corporation General in representation of the government.
engaged in agriculture or mining and for any
corporation organized for any purpose except
irrigation to be in any wise interested in any other HARDEN v BENGUET CONSOLIDATED MINING
corporation engaged in agriculture or in mining." COMPANY
G.R. No. L-37331, March 18, 1933
Issue:
FACTS: Benguet Consolidated Mining Co. was
Whether or not the issuance of the shares of stock organized in June, 1903, as a sociedad anonima in
conformity with the provisions of Spanish law.
should be annulled
Balatoc Mining Co. was organized in December
1925, as a corporation, in conformity with the
Held:
provisions of the Corporation Law (Act No. 1459).
Both were organized for mining of gold and their
Although arrangements between the two respective properties are located only a few miles
mining companies was prohibited under the terms apart in Benguet. Balatoc capital stock consists of
of the Corporation law, the supreme court will not one million shares of the par value of one peso (P1)
declare the nullity of the arrangements on the each.
ground that only private rights and interests, not When the Balatoc was first organized, its properties
public interests are involved in the case. The were largely undeveloped. To improve its
operations, the company’s committee approached
provision referred to was adopted by the lawmakers
A. W. Beam, then president and general manager of
with a sole view to the public policy that should the Benguet Company, to secure the capital
control in the granting of mining rights. necessary to the development of the Balatoc
property. A contract was entered into wherein
The defendant Benguet Company has Benguet will (1) construct a milling plant for the
committed no civil wrong against the plaintiffs, and if Balatoc mine, of a capacity of 100 tons of ore per
a public wrong has been committed, the directors of day, and with an extraction of at least 85 per cent of
the Balatoc Company, and the plaintiff Harden the gold content; (2) erect an appropriate power
himself, were the active inducers of the commission plant. In return, Benguet will receive from Balatoc
shares of a par value of P600,000.
of that wrong. The contract, supposing it to have
The total cost incurred by Benguet in developing
been unlawful in fact, has been performed on both Balatoc was P1,417,952.15. A certificate for
sides, by the building of the Balatoc plant by the 600,000 shares of the stock of the Balatoc
Benguet Company and the delivery to the latter of Company was given to Benguet and the excess
the certificate of 600,000 shares of the Balatoc value was paid to Benguet by Balatoc in cash. Due
Company. There is no possibility of really undoing to the improvements made by Benguet, the value of
shares of Balatoc increased in the market (from P1
what has been done. Nobody would suggest the
to more than P11) and dividends enriched its
demolition of the mill. The Balatoc Company is stockholders. Harden, the owner of thousands of
shares of Balatoc, questioned the transfer of liberalization. Thus, the inhibition contained in the
600,000 shares to Benguet with the success of the original provision against members of a corporation
development. engaged in agriculture or mining from being
interested in other corporations engaged in
ISSUE: W/N it is unlawful for Benguet Company to agriculture or in mining was so modified as merely
hold any interest in a mining corporation. to prohibit any such member from holding more
W/N, assuming the first question to be than fifteen per centum of the outstanding capital
answered in the affirmative, the Benguet Company, stock of another such corporation. Moreover, the
which was organized as a sociedad anonima, is a explicit prohibition against the holding by any
corporation within the meaning of the corporation (except for irrigation) of an interest in
language used by the Congress of the United any other corporation engaged in agriculture or in
States, and later by the Philippine Legislature, mining was so modified as to limit the restriction to
prohibiting a corporations organized for the purpose of engaging
mining corporation from becoming in agriculture or in mining.
interested in another mining corporation.

RULING:
1st Issue: The defendant Benguet Company has
committed no civil wrong against the plaintiffs, and if Safic Alcan & Cie vs Imperial Vegetable Oil Co.
a public wrong has been committed, the directors of
the Balatoc Company, and the plaintiff Harden
himself, were the active inducers of the commission
of that wrong. The contract, supposing it to have Facts:
been unlawful in fact, has been performed on both
sides.
2nd Issue: Having shown that the plaintiffs in this
case have no right of action against the Benguet In 1985, Safic Alcan & Cie (SAC), a corporation,
Company for the infraction of law supposed to have entered into an agreement with Imperial Vegetable
been committed, we forego any discussion of the Oil Co., Inc. (IVO) whereby the latter shall deliver
further question whether a sociedad anonima
created under Spanish law, such as the Benguet tones of coconut oil to SAC. Both parties complied.
Company, is a corporation within the meaning of the IVO was represented by its president, Dominador
prohibitory provision already so many times Monteverde. In 1986, SAC again entered into an
mentioned. several agreements with IVO but this time it was
A sociedad anonima is something very much like agreed that IVO shall deliver the coconut oil 8
the English joint stock company, with features months from the agreement or sometime in 1987.
resembling those of both the partnership is shown in
This time, IVO failed to deliver and SAC sued IVO.
the fact that sociedad, the generic component of its
name in Spanish, is the same word that is used in
that language to designate other forms of
partnership, and in its organization it is constructed
IVO in its defense aver that Monteverde was acting
along the same general lines as the ordinary
partnership. beyond his power as president when he made the
In section 75 of the Corporation Law, a provision is 1986 agreement with SAC; that Monteverde is
found making the sociedad anonima subject to the acting beyond his power because the 1986
provisions of the Corporation Law "so far as such contracts were speculative in nature and
provisions may be applicable", and giving to the speculative contracts are prohibited by the by-laws
sociedades anonimas previously created in the of IVO.
Islands the option to continue business as such or
to reform and organize under the provisions of the
Corporation Law.
The provision in Section 75 of the Act Congress of
SAC insists that there is an implied agency between
July 1, 1902 (Philippine Bill), generally prohibiting
corporations engaged in mining and members of IVO and Monteverde because SAC and Monteverde
such from being interested in any other corporation has been transacting since 1985 and that IVO
engaged in mining, was amended by section 7 of benefited from said transactions.
Act No. 3518 of the Philippine Legislature, approved
by Congress March 1, 1929. The change in the law
effected by this amendment was in the direction of
the manufacture of garments for domestic and
foreign consumption. The spouses by virtue of an
ISSUE: Whether or not Monteverde’s act in entering SPA appointed and authorized their daughter to
into the 1986 contracts is ultra vires. obtain loan from respondent Pacific Bank. A loan
was secured and as security therefore a REM was
executed over the property of the spouses.
Sometime after, BET was incorporated into a family
corporation named Bela’s Export Corporation (BEC)
and the loan was restructured in its name.
HELD: Yes. It was proven by IVO, when they Subsequent loans were obtained in behalf of BEC
presented a copy of their by-laws, that Monteverde all secured by the previous REM. BEC defaulted in
its payments which led to the foreclosure and sale
acted beyond his authority when he entered into
of the mortgaged property. The spouses moved to
speculative contracts with SAC in 1986. The 1986 annul the sale alleging that BEC is a distinct and
contracts are speculative because at the time of the separate personality from them and that the REM
contracts, the coconuts are not even growing at that was executed only to secure BET’s loan. Both trial
time and are yet to be harvested. Hence, the 1986 court and CA ruled to pierce the corporate veil to
contracts are sales of mere expectations – and this hold petitioner spouses liable for BEC’s obligations.
is something prohibited by the by-laws and the
ISSUE:
Board of Directors of IVO.
Whether or not the doctrine of piercing the veil of
corporate fiction is applicable in this case.
There can be no implied agency too simply because
RULING:
there has been a previous transaction between SAC
and IVO where IVO was represented by YES.
Monteverde. This is because the 1985 contract and
the 1986 contracts are very different. The 1985 We find that the evidence on record demolishes,
contract is not speculative while the 1986 contracts rather than buttresses, petitioners’ contention that
are speculative hence, SAC should have secured BET and BEC are separate business entities. Note
that Estelita Lipat admitted that she and her
the confirmation by IVO’s Board that Monteverde is
husband, Alfredo, were the owners of BET and
indeed authorized to enter into such agreements. were two of the incorporators and majority
Further, Monteverde did not even present the said stockholders of BEC. It is also undisputed that
1986 agreements before the Board of Directors so Estelita Lipat executed a special power of attorney
there was, in fact, no occasion at all for ratification. in favor of her daughter, Teresita, to obtain loans
The contracts were not even reported in IVO’s and credit lines from Pacific Bank on her behalf.
export sales book and turn-out book. Neither were Incidentally, Teresita was designated as executive-
vice president and general manager of both BET
they reflected in other books and records of the
and BEC, respectively.
corporation. It must be pointed out that the Board of
Directors, not Monteverde, exercises corporate It could not have been coincidental that BET and
power. Clearly, Monteverde’s speculative contracts BEC are so intertwined with each other in terms of
with Safic never bound IVO and Safic cannot ownership, business purpose, and management.
therefore enforce those contracts against IVO. Apparently, BET and BEC are one and the same
and the latter is a conduit of and merely succeeded
the former. Petitioners’ attempt to isolate
themselves from and hide behind the corporate
Lipat vs Pacific Bank Corporation personality of BEC so as to evade their liabilities to
G.R. No. 104720 Pacific Bank is precisely what the classical doctrine
April 4, 2001 of piercing the veil of corporate entity seeks to
prevent and remedy.

FACTS: In our view, BEC is a mere continuation and


successor of BET and petitioners cannot evade
Petitioner spouses Lipat owned Bela’s Export their obligations in the mortgage contract secured
Trading (BET) a single proprietorship engaged in under the name of BEC on the pretext that it was
signed for the benefit and under the name of BET. lawyers of HI Cement declared in open court that
We are thus constrained to rule that the Court of they are authorized to enter into a compromise
Appeals did not err when it applied the agreement for HI Cement; that one of the lawyers of
instrumentality doctrine in piercing the corporate veil
HI Cement, Atty. Florentino Cardenas, is an
of BEC.
executive official of HI Cement; that Cardenas even
nominated one of the commissioners; that such act
ratified the compromise agreement even if it was
VICENTE VS GERALDEZ not approved by the Board. HI Cement, in its
defense, averred that the lawyers were not
authorized and that in fact there was no special
power of attorney executed in their favor for the
Facts:
purpose of entering into a compromise agreement.
Judge Ambrosio Geraldez ruled in favor of HI
Cement.
In 1967, HI Cement Corporation was
granted authority to operate mining facilities in
Bulacan. However, the areas allowed for it to
ISSUE: Whether or not a compromise agreement
explore cover areas which were also being explored
entered into by a lawyer purportedly in behalf of the
by Ignacio Vicente, Juan Bernabe, and Moises
corporation is valid without a written authority.
Angeles. And so a dispute arose between the three
and HI Cement as neither side wanted to give up
their mining claims over the disputed areas.
Eventually, HI Cement filed a civil case against the HELD: No. Corporations may compromise only in
three. During pre-trial, the possibility of an amicable the form and with the requisites which may be
settlement was explored where HI Cement offered necessary to alienate their property. Under the
to purchase the areas of claims of Vicente et al at corporation law the power to compromise or settle
the rate of P0.90 per square meter. Vicente et al claims in favor of or against the corporation is
however wanted P10.00 per square meter. ordinarily and primarily committed to the Board of
Directors but such power may be delegated. The
delegation must be clearly shown for as a general
rule an officer or agent of the corporation has no
In 1969, the lawyers of HI Cement agreed
power to compromise or settle a claim by or against
to enter into a compromise agreement with the
the corporation, except to the extent that such
three whereby commissioners shall be assigned by
power is given to him either expressly or by
the court for the purpose of assessing the value of
reasonable implication from the circumstances. In
the disputed areas of claim. An assessment was
the case at bar, there was no special power of
subsequently made pursuant to the compromise
attorney authorizing the three lawyers to enter into a
agreement and the commissioners recommended a
compromise agreement. This is even if the lawyers
price rate of P15.00 per square meter.
declared in open court that they are authorized to
do so by the corporation (in this case, the transcript
of stenographic notes does not show that the
One of the lawyers of HI Cement, Atty. lawyers indeed declare such in open court).
Francisco Ventura, then notified the Board of
Directors of HI Cement for the approval of the The fact that Cardenas, an officer of HI Cement,
compromise agreement. But the Board disapproved acted in effecting the compromise agreement, i.e.
the compromise agreement hence Atty. Ventura nominating a commissioner, does not ratify the
filed a motion with the court to disregard the compromise agreement. There is no showing that
compromise agreement. Vicente et al naturally Cardenas’ act binds HI Cement; no proof that he is
assailed the motion. Vicente et al insisted that the authorized by the Board; no proof that there is a
compromise agreement is binding because prior to provision in the articles of incorporation of HI
entering into the compromise agreement, the three Cement that he can bind the corporation.
ISSUE : Was the promissory note dated 21 March
1978 secured and signed by Arrieta and Lilia Perez
Aguenza vs. Metropolitan Bank and Trust Co. et al. a corporate liability of Intertrade and Aguenza?
G.R. No. 74336, 4/7/1997
RULING:
FACTS: On March 21, 1978, private respondents NO. Arrieta and Perez were never authorized by
Vitaliado Arrieta, VP of Intertrade and Lilia P. Perez, Intertrade through a board resolution of the Board of
a bookkeeper in the employ of Intertrade, obtained Directors of Intertrade authorizing the former to
a P500,000.00 loan from private respondent transact said loan for and in behalf of the
Metrobank. Both executed a Promissory Note in corporation. It is a well-settled rule that a
favor of said bank in the amount of P500,000.00. corporation transacts its business only through its
Under said note, private respondents Arrieta and officers or agents. And the authority of such
Perez promised to pay said amount, jointly and officers or agents is derived from the BOD or other
severally, in twenty five (25) equal installments governing body unless conferred by the charter of
of P20,000.00 each starting on April 20, 1979 with the corporation. It is to be noted that the promissory
interest of 18.704% per annum, and in case of note dated 21 March 1977 was signed by Arrieta
default, a further 8% per annum. and Lilia Perez only with no indication as to what
Private respondents Arrieta and Perez capacity the two signatories had in affixing their
defaulted in the payment of several installments, signatures thereon. There is no record that
thus resulting in the entire obligation becoming due Intertrade through its BOD, conferred upon Arrieta
and demandable. In 1979, private respondent and Lilia Perez the authority to contract a loan with
Metrobank instituted suit against Intertrade, Metrobank and execute the promissory note as a
Vitaliado Arrieta, Lilia Perez and her husband, security therefor.
Patricio Perez, to collect not only the unpaid
principal obligation, but also interests, fees and Metrobank in turn never presented a board
penalties, exemplary damages, as well as attorney's resolution nor a stockholder's resolution showing
fees and costs of suit. that Arrieta and Lilia Perez were empowered by
Intertrade to execute the promissory note. Being
More than a year after private respondent that the promissory note was not the responsibility
Metrobank filed its original complaint, it filed an of Intertrade, it follows that the same was not
Amended Complaint dated August 30, 1980 for the covered by the Continuing Suretyship Agreement.
sole purpose of impleading petitioner as liable for
the loan made by private respondents Arrieta and CA decision is reversed and trial court decision is
Perez on March 21, 1978, notwithstanding the fact REINSTATED.
that such liability is being claimed on account of a
Continuing Suretyship Agreement dated March 14,
1977 executed by petitioner and private respondent GR 68555, 19 March 1993
Arrieta specifically to guarantee the credit line
applied for by and granted to, Intertrade, through
petitioner and private respondent Arrieta who were
specially given authority by Intertrade on February Prime White Cement Corporation
28, 1977 to open credit lines with private
respondent Metrobank. The obligations incurred by vs. Intermediate Appellate Court
Intertrade under such credit lines were completely
paid as evidenced by private respondent
Metrobank's debit memo in the full amount
of P562,443.46. FACTS:
RTC ruled that petitioner and Intertrade are not
liable for the promissory note executed by Arrieta
and Lilia Perez in the amount of ₱ 500,000 as the
same was the personal liability of the latter. On or about 16 July 1969, Alejandro Te and
Prime White Cement Corporation (PWCC) thru its
CA reversed the trial court and ordered Intertrade President, Mr. Zosimo Falcon and Justo C. Trazo,
and Marketing Co., Inc. and J. Antonio Aguenza to
as Chairman of the Board, entered into a dealership
pay, jointly and severally, the promissory note
contracted by Arrieta and Lilia Perez. agreement whereby Te was obligated to act as the
exclusive dealer and/or distributor of PWCC of its
cement products in the entire Mindanao area for a by ratification, the President as such may, as a
term of 5 years. general rule, bind the corporation by a contract in
the ordinary course of business, provided the same
Right after Te entered into the dealership is reasonable under the circumstances. These rules
agreement, he placed an advertisement in a are basic, but are all general and thus quite flexible.
national, circulating newspaper the fact of his being They apply where the President or other officer,
the exclusive dealer of PWWC's white cement purportedly acting for the corporations, is dealing
products in Mindanao area, more particularly, in the with a third person, i.e., a person outside the
Manila Chronicle dated 16 August 1969 and was corporation. The situation is quite different where a
even congratulated by his business associates, so director or officer is dealing with his own
much so, he was asked by some of his corporation. Herein, Te was not an ordinary
businessmen friends and close associates if they stockholder; he was a member of the Board of
can be his sub-dealer in the Mindanao area. Directors and Auditor of the corporation as well. He
was what is often referred to as a "self-dealing"
director.
ISSUE:

Whether the "dealership agreement" G.R. No. L-18287 March 30, 1963
referred by the President and Chairman of the
Board of PWCC is a valid and enforceable contract. TRINIDAD J. FRANCISCO, plaintiff-appellee,
vs.
GOVERNMENT SERVICE INSURANCE SYSTEM,
defendant-appellant.
RULING:
1. CORPORATIONS; BINDING EFFECT OF ACTS
OF CORPORATE OFFICERS. — A corporation
cannot evade the binding effect produced by a
NO. telegram sent by its board secretary, and the
addressee of such telegram cannot be blamed for
relying upon it, because if every person dealing with
a corporation were held duty-bound to disbelieve
The “dealership agreement” is not valid and every act of its responsible officers no matter how
unenforceable. Under the Corporation Law, which regular it should appear on its face, corporate
was then in force at the time the case arose, as well transactions would speedily come to a standstill.
as under the present Corporation Code, all
2. ID.; ID.; WHEN CORPORATION ESTOPPED TO
corporate powers shall be exercised by the Board of
DENY APPARENT AUTHORITY OF ITS
Directors, except as otherwise provided by law. OFFICERS. — If a private corporation intentionally
Although it cannot completely abdicate its power or negligently clothes its officers or agents with
and responsibility to act for the juridical entity, the apparent power to perform acts for it, the
Board may expressly delegate specific powers to its corporation will be estopped to deny that such
President or any of its officers. apparent authority is real, as to innocent third
persons dealing in good faith with such officers or
In the absence of such express delegation, agents. (2 Fletcher’s Cyclopedia, Priv. Corp, 255,
a contract entered into by its President, on behalf of perm. Ed.)
the corporation, may still bind the corporation if the
3. ID.; ID.; ID.; WHEN NOTICE OF FACTS BY A
board should ratify the same expressly or impliedly.
CORPORATE OFFICER IS NOTICE TO
Implied ratification may take various forms — like CORPORATION. — Knowledge of facts acquired or
silence or acquiescence; by acts showing approval possessed by an officer or agent of a corporation in
or adoption of the contract; or by acceptance and the course of his employment, and in relation to
retention of benefits flowing therefrom. Furthermore, matters within he communicates such knowledge or
even in the absence of express or implied authority not. (Ballentine, Law on Corporations, section 112).
installments of P3,902.41, and with interest of 7%
4. ID.; ID.; ID.; SILENCE OF CORPORATION AS per annum compounded monthly.
RATIFICATION OF AGREEMENT. — The silence
of the corporation, taken together with the On 6 January 1959, the GSIS extrajudicially
unconditional acceptance of three subsequent foreclosed the mortgage on the ground that up to
remittances from plaintiff, constitutes a binding that date the plaintiff-mortgagor was in arrears on
ratification of the original agreement between them her monthly installments in the amount of
(Civil Code, Article 1393). P52,000.00. Payments made by the plaintiff at the
time of foreclosure amounted to P130,000.00. GSIS
5. ID.; ID.; ID.; MAXIM THAT THE ONE WHO itself was the buyer of the property in the
MADE IT POSSIBLE FOR A WRONG TO BE foreclosure sale.
DONE SHOULD SUFFER. — The equitable maxim
that between two innocent parties the one who But then, Trinidad’s father, Atty. Vicente Francisco,
made it possible for the wrong to be done should be wrote a letter to general manager of the defendant
the one to bear the resulting loss, applies when—as corporation, Mr. Rodolfo P. Andal, offering that he
in the instant case—a corporation allows one of its pay P30k off the loan and then allow GSIS to
officers, now alleged to be without the proper administer the mortgaged property instead of
authority, to send a telegram binding the foreclosing it; that thereafter, GSIS shall receive
corporation. rents from the tenants of the land until the arrears
are paid and the account is made current or up to
6. DAMAGES; BREACH OF CONTRACT; MORAL date (because the total of the monthly rents is
DAMAGES NOT WARRANTED IF BREACH IS bigger than the monthly loan payments supposed to
NOT MALICIOUS OR FRAUDULENT. — Award of be paid by Trinidad to GSIS).
moral damages under Article 2220 of the Civil Code
is not warranted if the breach of contract is not GSIS, through its general manager Rodolfo Andal,
malicious or fraudulent (Ventanilla v. Centeno, 110 responded with a letter which states that the GSIS
Phil., 811; Flores v. Miranda, 105 Phil., 266). Board had accepted Vicente’s offer. But GSIS for
some reason did not take over the property.
7. ID.; ID.; WHEN EXEMPLARY DAMAGES Nevertheless, the Franciscos collected rents and
ALLOWED. — Exemplary damages are only turned them over to GSIS.
allowed in addition to moral, temperate liquidated,
or compensatory damages (Art. 2234, Civil Code; Then in 1960, GSIS demanded Trinidad Francisco
Velayo v. Shell Co. of P.I. 100 Phil., 186; Singson, to pay off the loan. Vicente then reminded GSIS of
Et. Al. v. Aragon and Lorza, 92 Phil. 514; 49 Off. the concluded contract generated by his offer and
Gaz. No. 2, 515). its acceptance by telegram of the same date, the
compliance of the terms of the offer already
8. ATTORNEY’S FEES; AWARD ESSENTIALLY commenced by the plaintiff.
DISCRETIONARY ALLOWED. — The award of
attorney’s fees is essentially discretionary with the Defendant countered that the telegram should be
trial court, and no abuse of discretion is committed disregarded in view of its failure to express the
when the court refuses to make an award because contents of the board resolution due to the error of
of the absence of gross and evident bad faith in its minor employees in couching the correct wording
defendant’s refusal to satisfy plaintiff’s claim, or of of the telegram.
any of the other grounds enumerated in Article 2208
of the Civil Code.
Hence, the plaintiff instituted the present suit, for
specific performance and damages. The defendant
FACTS: answered, pleading that the binding acceptance of
Francisco's offer was the resolution of the Board,
On 10 October 1956, the plaintiff, Trinidad J. and that Andal's telegram, being erroneous, should
Francisco, in consideration of a loan in the amount be disregarded.
of P400,000.00, mortgaged in favor of the
defendant, Government Service Insurance System, After trial, the court below found that the offer of
a parcel of land containing an area of 18,232 square Atty. Francisco, dated 20 February 1959, made on
meters, with twenty-one (21) bungalows, known as behalf of his daughter, had been unqualifiedly
Vic-Mari Compound, located at Baesa, Quezon accepted, and was binding, and rendered judgment
City, payable within ten (10) years in monthly as noted at the start of this opinion.
ISSUE: Whether or not the telegram generated a public as possessing power to do those acts, the
contract that is valid and binding upon the parties. corporation will, as against any one who has in
good faith dealt with the corporation through such
RULING: agent, be estopped from denying his authority; and
where it is said "if the corporation permits" this
In the case at bar, the response by GSIS to Vicente means the same as "if the thing is permitted by the
directing power of the corporation."
by way of a telegram, is within the apparent
authority of Andal. If there are any irregularities in
the telegraph i.e., the sending of the secretary A person who knows that the officer or agent of the
without the authority of Andal, Vicente is not corporation habitually transacts certain kinds of
expected to know it because the telegram on its business for such corporation under circumstances
face is clear as to the acceptance. Vicente cannot which necessarily show knowledge on the part of
therefore be faulted for relying on the telegram; that those charged with the conduct of the corporate
GSIS accepted his offer. Hence, GSIS cannot now
businesS.
ask Francisco to suddenly pay off the debt.

GSIS cannot also deny that it has knowledge of the


acceptance. A corporation cannot see, or know, PRIME WHITE VS IAC
anything except through its officers. Knowledge of
facts acquired or possessed by an officer or agent
of a corporation in the course of his employment, Facts:
and in relation to matters within the scope of his A director (Te) entered into an agreement
authority, is notice to the corporation, whether he of Dealership agreement with PWCC, signed by
communicates such knowledge or not. Andal is its chairman and president of the corporation to
presumed to have knowledge of the acceptance supply 20,000 bags of white cement per month for
because it was his office which sent it to Vicente. five years at a fixed price of P9.70 per bag.
Knowledge of Andal, an officer of GSIS, is deemed Subsequently, the Board refused to abide by the
knowledge of GSIS. contract unless new conditions are accepted
providing for a new price formula. The dealing
At any rate, even if the compromise agreement is director sued for specific performance on the
void because of the “unauthorized” telegram, contract.
GSIS’s silence and acceptance of the subsequent
remittances of the Franciscos ratified the Issue:
compromise agreement. W/N the terms of said dealership
agreement binds the corporation.
The offer of compromise made by plaintiff in the Ruling:
letter had been validly accepted, and was binding No, since the terms were unreasonable for
on the defendant. There was nothing in the the corporation and that the unfairness in the
telegram that hinted at any anomaly, or gave contract was a basis which renders a contract
ground to suspect its veracity, and the plaintiff, entered into the President without authority from
therefore, cannot be blamed for relying upon it. the Board, void or voidable, although it may have
been in the ordinary course of business.
There is no denying that the telegram was within The Court held that under both the
Andal's apparent authority, but the defense is that Corporation Law then and the present
he did not sign it, but that it was sent by the Board Corporation Code, the doctrine is that all corporate
Secretary in his name and without his knowledge. powers shall be exercised by the Board of Directors,
Assuming this to be true, how was appellee to know except as those provided by law. Although it
it? Corporate transactions would speedily come to a cannot completely abdicate its powers and
standstill were every person dealing with a responsibility to act for the juridical entity, the
corporation held duty-bound to disbelieve every act Board may expressly delegate specific powers to
of its responsible officers, no matter how regular its president or any of its officers. In the absence
they should appear on their face. of such express delegation, a contract entered into
by its President on behalf of the corporation may
still bind the corporation if the Board should ratify
As already observed, it is familiar doctrine that if a
the same expressly or impliedly.
corporation knowingly permits one of its officers, or
On the other hand, a director's contract with
any other agent, to do acts within the scope of an
his corporation is not in all instances void or
apparent authority, and thus holds him out to the
voidable. If the contract is fair and reasonable under embodies apparent authority. On the other hand,
the circumstances, it may be ratified by the the general manager on its own may or may not
stockholders provided a full disclosure of his embody such authority depending on the
adverse interest is made. Section 32 of the circumstances that go with it. The corporate
Corporation Code provides, thus: secretary and lawyer enjoy no such presumption
Sec. 32. Dealings of directors, trustees or because their positions do entail much commercial
officers with the corporation. — A contract of the significance.
corporation with one or more of its directors or Prime white vs iac
trustees or officers is voidable, at the option of such
corporation, unless all the following conditions are 220 SCRA 103 – Commercial Law – Corporation
present: Code – Award of Moral Damages to Corporations –
1. That the presence of such director or Self-Dealing Director
trustee in the board meeting in which the contract In July 1969, Zosimo Falcon and Justo Trazo
was approved was not necessary to constitute a entered into an agreement with Alejandro Te
quorum for such meeting; whereby it was agreed that from 1970 to 1976, Te
2. That the vote of such director or trustee shall be the sole dealer of 20,000 bags Prime White
was not necessary for the approval of the contract; cement in Mindanao. Falcon was the president of
3. That the contract is fair and reasonable Prime White Cement Corporation (PWCC) and
under the circumstances; and Trazo was a board member thereof. Te was
4. That in the case of an officer, the contract likewise a board member of PWCC. It was agreed
with the officer has been previously authorized by that the selling price for a bag of cement shall be
the Board of Directors. P9.70.
Where any of the first two conditions set Before the bags of cement can be delivered, Te
forth in the preceding paragraph is absent, in the already made known to the public that he is the sole
case of a contract with a director or trustee, such dealer of cements in Mindanao. Various hardwares
contract may be ratified by the vote of the then approached him to be his sub-dealers, hence,
stockholders representing at least two-thirds (2/3) of Te entered into various contracts with them.
the outstanding capital stock or of two-thirds (2/3) of But then apparently, Falcon and Trazo were not
the members in a meeting called for the purpose: authorized by the Board of PWCC to enter into such
Provided, That full disclosure of the adverse interest contract. Nevertheless, the Board wished to retain
of the directors or trustees involved is made at such the contract but they wanted some amendment
meeting: Provided, however, That the contract is fair which includes the increase of the selling price per
and reasonable under the circumstances. bag to P13.30 and the decrease of the total amount
Although the old Corporation Law which of cement bags from 20k to 8k only plus the
governs the instant case did not contain a similar contract shall only be effective for a period of three
provision, yet the cited provision substantially months and not 6 years.
incorporates well-settled principles in corporate law. Te refused the counter-offer. PWCC then awarded
Implied ratification takes various forms (1) the contract to someone else.
silence or acquiescence (2) by acts showing Te then sued PWCC for damages. PWCC filed a
approval or adoption of the contract or (3) by counterclaim and in said counterclaim, it is claiming
acceptance and retention of the benefits flowing for moral damages the basis of which is the claim
therefrom. that Te’s filing of a civil case against PWCC
Even in the absence of express or destroyed the company’s goodwill. The lower court
implied authority by ratification, the President as a ruled in favor Te.
general rule may bind the corporation by a ISSUE: Whether or not the ruling of the lower court
contract in the ordinary course of business, is correct.
provided the same is reasonable under the HELD: No. Te is what can be called as a self-
circumstances. These rules are basic but general dealing director – he deals business with the same
and flexible. Applies where the President is corporation in which he is a director. There is
dealing with third persons but different where a nothing wrong per se with that. However, Sec. 32
director is dealing with his own corporation. provides that:
The court herein held that the director SEC. 32. Dealings of directors, trustees or
holds a position of trust and as such he owes a duty officers with the corporation. —- A contract of the
of loyalty to his corporation and his contracts with corporation with one or more of its directors or
the corporation must always be at reasonable trustees or officers is voidable, at the option of such
terms otherwise the contract is void or voidable at corporation, unless all the following conditions are
the instance of the corporation. present:
NOTE: The President as the highest office 1. That the presence of such director or trustee in
of the corporation, by practice and jurisprudence
the board meeting in which the contract was Board sent a letter to Yao telling him that Prime
approved was not necessary to constitute a quorum White is instead willing to sell only 10,000 bags to
for such meeting; Yao Ka Sin and that he has ten days to reply; that if
2. That the vote of such director or trustee was not
no reply is made by Yao then they will consider it as
necessary for the approval of the contract;
3. That the contract is fair and reasonable under an acceptance and that thereafter Prime White shall
the circumstances; and deposit the P243k check in its account and then
4. That in the case of an officer, the contract with deliver the cements to Yao Ka Sin. Henry Yao never
the officer has been previously authorized by the replied.
Board of Directors.
In this particular case, the Supreme Court focused Later, Yao Ka Sin sued Prime White to compel the
on the fact that the contract between PWCC and Te latter to comply with what Yao Ka Sin considered as
through Falcon and Trazo was not reasonable. the true contract, i.e., 45,000 bags at P24.30 per
Hence, PWCC has all the rights to void the contract
bag. Prime White in its defense averred that
and look for someone else, which it did. The
contract is unreasonable because of the very low although Maglana is empowered to sign contracts in
selling price. The Price at that time was at least behalf of Prime White, such contracts are still
P13.00 per bag and the original contract only subject to approval by Prime White’s Board, and
stipulates P9.70. Also, the original contract was for then it still requires further approval by the National
6 years and there’s no clause in the contract which Investment and Development Corporation (NIDC), a
protects PWCC from inflation. As a director, Te in government owned and controlled corporation
this transaction should protect the corporation’s
interest more than his personal interest. His failure because Prime White is a subsidiary of NIDC.
to do so is disloyalty to the corporation.
Anent the issue of moral damages, there is no Henry Yao asserts that the letter from Maglana is a
question that PWCC’s goodwill and reputation had binding contract because it was made under the
been prejudiced due to the filing of this case. apparent authority of Maglana. The trial court ruled
However, there can be no award for moral damages in favor of Yao Ka Sin. The Court of Appeals
under Article 2217 of the Civil Code in favor of a reversed the trial court.
corporation.
Issue: Whether or not the president of a corporation
is clothed with apparent authority to enter into
binding contracts with third persons without the
G.R. No. L-53820 June 15, 1992 authority of the Board.

YAO KA SIN TRADING, owned and operated by Ruling: No. The Board may enter into contracts
YAO KA SIN, petitioner, vs. through the president. The president may only enter
into contracts upon authority of the Board. Hence,
HONORABLE COURT OF APPEALS and PRIME any agreement signed by the president is subject to
WHITE CEMENT CORPORATION, represented approval by the Board. Unlike a general manager
by its President-Chairman, CONSTANCIO B. (like the case of Francisco vs GSIS), the president
MALAGNA, respondents. has no apparent authority to enter into binding
contracts with third persons. Further, if indeed the
Facts: In 1973, Constancio Maglana, president of
by-laws of Prime White did provide Maglana with
Prime White Cement Corporation, sent an offer apparent authority, this was not proven by Yao Ka
letter to Yao Ka Sin Trading. The offer states that Sin.
Prime White is willing to sell 45,000 bags of cement
at P24.30 per bag. The offer letter was received by As a rule, apparent authority may result from (1) the
Yao Ka Sin’s manager, Henry Yao. Yao accepted general manner, by which the corporation holds out
the letter and pursuant to the letter, he sent a check an officer or agent as having power to act or, in
in the amount of P243,000.00 equivalent to the other words, the apparent authority with which it
value of 10,000 bags of cement. However, the clothes him to act in general or (2) acquiescence in
Board of Directors of Prime White rejected the offer his acts of a particular nature, with actual or
letter sent by Maglana but it considered Yao’s constructive knowledge thereof, whether within or
acceptance letter as a new contract offer hence the
Subsequently, petitioner repeatedly demanded payment for
without the scope of his ordinary powers. These are
not present in this case. her services but Ms. Lopez just ignored the demands. Ms.
Lopez curtly replied that she was not entitled to it because her
Also, the subsequent letter by Prime White to Yao designs did not conform to the banks policy of having a
Ka Sin is binding because Yao Ka Sin’s failure to standard design, and that there was no agreement between
her and the bank.
respond constitutes an acceptance, per stated in
the letter itself – which was not contested by Henry
Issue;
Yao during trial.
Whether or not there was a perfected contract between
petitioner Jazmin Soler and respondents COMBANK and Nida
ASMIN SOLER, vs. COURT OF APPEALS,Lopez, and whether or not Nida Lopez, the manager of the
bank branch, had authority to bind the bank in the transaction.
COMMERCIAL BANK OF MANILA, and
NIDA LOPEZ,
Held;
G.R. No. 123892
May 21, 2001 The discussions between petitioner and Ms. Lopez was to the
effect that she had authority to engage the services of
petitioner. During their meeting, she even gave petitioner
IX. CORPORATE POWERS, AUTHORITY AND ACTIVITIES specifications as to what was to be renovated in the branch
(ii) Theory of Apparent Authority (Art. 1883, Civil Code) premises and when petitioners requested for the blueprints of
the building, Ms. Lopez supplied the same.
Ms. Lopez was aware that petitioner hired the services of
people to help her come up with the designs for the
Facts; December, 1986 board meeting of the bank. Ms. Lopez even
insisted that the designs be rushed in time for presentation to
the bank. With all these discussion and transactions, it was
apparent to petitioner that Ms. Lopez indeed had authority to
Petitioner Jazmin Soler is a Fine Arts graduate of the engage the services of petitioner.
University of Sto. Tomas, Manila. She is a well known
licensed professional interior designer. In November 1986, In the case at bar, there was a perfected oral contract. When
Ms. Lopez and petitioner met in November 1986, and
her friend Rosario Pardo asked her to talk to Nida Lopez,
discussed the details of the work, the first stage of the
who was manager of the COMBANK (COMMERCIAL BANK contract commenced. When they agreed to the payment of
OF MANILA) Ermita Branch for they were planning to the ten thousand pesos (P10,000.00) as professional fees of
renovate the branch offices. petitioner and that she should give the designs before the
December 1986 board meeting of the bank, the second stage
During the November 1986 meeting between petitioner and of the contract proceeded, and when finally petitioner gave
Ms. Lopez, there were discussions as to what was to be the designs to Ms. Lopez, the contract was consummated.
renovated, which included a provision for a conference room, Petitioner believed that once she submitted the designs she
a change in the carpeting and wall paper, provisions for would be paid her professional fees. Ms. Lopez assured
bookshelves, a clerical area in the second floor, dressing up petitioner that she would be paid.
the kitchen, change of the ceiling and renovation of the tellers
booth. Ms. Lopez again assured petitioner that the bank It is familiar doctrine that if a corporation knowingly
would pay her fees. permits one of its officers, or any other agent, to act
within the scope of an apparent authority, it holds him
out to the public as possessing the power to do those
After a few days, petitioner requested for the blueprint of the
building so that the proper design, plans and specifications acts; and thus, the corporation will, as against anyone
could be given to Ms. Lopez in time for the board meeting in who has in good faith dealt with it through such agent, be
December 1986. Petitioner also did her research on the estopped from denying the agents authority.
designs and individual drawings of what the bank
wanted. She also hired the services of an electrical engineer,
architects and also contacted the suppliers of the wallpaper
and the sash makers for their quotation. So come December First Philippine International Bank VS. CA (1996)
1986, the lay out and the design were submitted to Ms.
Lopez. She even told petitioner that she liked the designs. Facts:
Petitioner First Philippine International Bank perpetrate a fraud upon his principal or some other
is a baking institution organized and existing under person, for his own ultimate benefit.
the laws of the Republic of the Philippines. Application of these principles is especially
Petitioner Mercurio Rivera was the Head Manager necessary because banks have a fiduciary
of the Property Management Department of the relationship with the public and their stability
petitioner bank. Respondent Carlos Ejercito is the depends on the confidence of the people in their
assignee of original plaintiffs-appellees Demetrio honesty and efficiency. Such faith will be eroded
Demetria and Jose Janolo. Petitioner bank acquired where banks do not exercise strict care in the
six parcels of land which was used to be owned by selection and supervision of its employees, resulting
BYME Investment and Development Corporation in prejudice to their depositors.”
which had them mortgaged with the bank as From the evidence found by respondent
collateral for a loan. Plaintiffs wanted to buy said Court, it is obvious that petitioner Rivera has
property and thus initiated negotiations for that apparent or implied authority to act for the Bank in
purpose. Plaintiffs then as suggested by BYME met the matter of selling its acquired assets.
with Rivera in which made plaintiff Janolo to create
a forma purchase offer to the bank. Several tenders
of payment were made but petitioner bank claimed
that Rivera has no authority to sell said property. Traders royal bank vs ca
Plaintiffs then filed a suit for specific performance
with damages against the bank and its manager. Facts:
The basis of the suit was that the transaction had
with the bank resulted in a perfected contract of Filriters Guaranty Assurance Corporation
sale. The defendants took the position that there (FGAC) is the owner of several Central Bank
was no such perfected sale because defendant Certificates of Indebtedness (CBCI). These
Rivera is not authorized to sell the property, and certificates are actually proof that FGAC has the
that there was no meeting of the minds as to the required reserve investment with the Central Bank
price. to operate as an insurer and to protect third persons
from whatever liabilities FGAC may incur. In 1979,
Issue: FGAC agreed to assign said CBCI to Philippine
W/N Rivera has no authority to act. Underwriters Finance Corporation (PUFC). Later,
PUFC sold said CBCI to petitioner Traders Royal
Ruling: Bank (TRB). Said sale with TRB comes with a right
Yes. to repurchase on a date certain. However, when the
The authority of a corporate officer in day to repurchase arrived, PUFC failed to
dealing with third persons may be actual or repurchase said CBCI hence TRB requested the
apparent. The doctrine of apparent authority, with Central Bank to have said CBCI be registered in
special reference to banks, was laid out in TRB’s name. Central Bank refused as it alleged that
Prudential Bank VS. Court of Appeals, where it was the CBCI are not negotiable; that as such, the
held that: transfer from FGAC to PUFC is not valid; that since
“Conformably, we have declared in it was invalid, PUFC acquired no valid title over the
countless decisions that the principal is liable for CBCI; that the subsequent transfer from PUFC to
obligations contracted by the agent. The agent’s TRB is likewise invalid.
apparent representation yields to the principal’s true
representation and the contract is considered as TRB then filed a petition for mandamus to
entered into between the principal and the third compel the Central Bank to register said CBCI in
person. TRB’s name. TRB averred that PUFC is the alter
A bank is liable for wrongful acts of its ego of FGAC; that PUFC owns 90% of FGAC; that
officers done in the interests of the bank or in the the two corporations have identical sets of directors;
course of dealings of the officers in their that payment of said CBCI to PUFC is like a
representative capacity but not for acts outside the payment to FGAC hence the sale between PUFC
scope of their authority. A bank holding out its and TRB is valid. In FGAC’s point it has Without any
officers and agents as trustworthy of confidence will consideration or benefit whatsoever to FGAC’s, in
not be permitted to profit by the frauds they may violation of law and the trust fund doctrine and to
accrue to the bank therefrom. Accordingly, a the prejudice of policyholders and to all who have
banking corporation is liable to innocent third present or future claim against policies issued by
persons where the representation is made in the FGAC, Alfredo Banaria, then Senior Vice-President-
course of its business by an agent acting within the Treasury of FGAC, without any board resolution,
general scope of his authority and attempting to knowledge or consent of the board of directors of
Filriters, and without any clearance or authorization
from the Insurance Commissioner, executed a
detached assignment purportedly assigning CBCI
No. 891 to PUFC. NYCO Sales Corp vs BA Finance Corp
Issue: G.R. No. 71694
August 16, 1991
Whether or not there was valid transfer of the CBCI
from FGAC to CBCI in order for TRB to acquire a FACTS:
valid ownership over the said CBCI. Nyco Sales whose president and general
manager Rufino Yao is engaged in the business
of selling construction materials.

Fernandezes acting on behalf of Sanshell


Corporation approached Yao for credit
Held: accommodation. They requested Nyco thru Yao
to garant Sanshell discounting priveleges which
Sec. 3. Assignment of Registered Certificates. —
Nyco had with BA Finance.
Assignment of registered certificates shall not be
valid unless made at the office where the same Fernandezes went to Yao for the purpose of
have been issued and registered or at the Securities discounting Sanshell’s BPI PDC’s on the aount
Servicing Department, Central Bank of the of Php 60,000 payable to Nyco. Nyco then thru
Philippines, and by the registered owner thereof, in Yao endorsed the check in favor of BA Finance.
person or by his representative, duly authorized in BA Finance then issued a check payable Nyco
writing. For this purpose, the transferee may be which endorsed it in favor of Sanshell which
designated as the representative of the registered made use of the negotiation. Nyco executed a
owner. Deed of Assignment in favor of BA Finance with
the conformity of Sanshell. Under the deed, the
Petitioner, being a commercial bank, cannot feign
subject of Assignment was the BPI PDC Check.
ignorance of Central Bank Circular 769, and its
requirements. An entity which deals with corporate Agreed that there will be a Continuing
agents within circumstances showing that the Suretyship Agreement whereby Fernandezes
agents are acting in excess of corporate authority, unconditionally guaranteed to BA Finance the
may not hold the corporation liable. 22 This is only compliance of all indebtness of Nyco. The
fair, as everyone must, in the exercise of his rights check was subsequently dishonored by the
and in the performance of his duties, act with drawee bank upon presentment for payment.
justice, give everyone his due, and observe honesty BA Finance reported the matter to the
and good faith. 23 Fernandezes and issued a substitute (Security
Bank) check with the same amount to BA
The transfer made by Filriters to Philfinance did not
Finance which was again subsequently
conform to the said. Central Bank Circular, which for
dishonored.
all intents, is considered part of the law. As found by
the courts a quo, Alfredo O. Banaria, who had Despite repeated demands, Nyco and the
signed the deed of assignment from Filriters to Fernandezes failed to pay the obligation. BA
Philfinance, purportedly for and in favor of Filriters, Finance then instituted an action to the court.
did not have the necessary written authorization Nyco and Fernandezes were considered in
from the Board of Directors of Filriters to act for the default. TC ruled in favor of BA Finance
latter. As it is, the sale from Filriters to Philfinance ordering the Fernandezes and Nycho solidarily
was fictitious, and therefore void and inexistent, as to pay the former. Nycho moved to set aside the
there was no consideration for the same. This is order and impleaded Sanshell. TC ruled in favor
fatal to the petitioner's cause, for then, Philfinance of BA Finance.
had no title over the subject certificate to convey the
Traders Royal Bank. Nemo potest nisi quod de jure With regards to the Fernandezes, the TC
potest — no man can do anything except what he denied the cross claim of Nycho because it
can do lawfully. seems that Fernadezes never received the
cross claim of Nycho and have not been was signed by Leonides P. Gonzales and Jesus J.
declared in default. Vergara, presidents of Inter-Asia and Asia
Industries, respectively. Under paragraph 7 of the
Upon appeal, affirmed TC decision with Agreement, Inter-Asia as seller made warranties
modification with respect to the running time of and representations. The Agreement was later
interest running from February 19, 1979 instead amended with respect to the "Closing Date,"
of February 1 1979. originally set up at 10:00 a.m. of 30 September
1978, which was moved to 31 October 1978, and to
Hence this appeal.
the mode of payment of the purchase price. The
NYCHO’S CONTENTIONS: Agreement, as amended, provided that pending
submission by SGV of FARMACOR's audited
o Discharged of liability when BA Finance failed financial statements as of 31 October 1978, Asia
to give a notice of dishonor. Industries may retain the sum of P7,500,000.00 out
o No novation when BA Finance accepted SBTC of the stipulated purchase price of P19,500,000.00;
check. that from this retained amount of P7,500,000.00,
o Yao as President is not authorized to enter into
credit assignment with BA Finance since there Asia Industries may deduct any shortfall on the
is no Board Resolution authorizing the same. Minimum Guaranteed Net Worth of P12,000,000.00;
and that if the amount retained is not sufficient to
ISSUE: make up for the deficiency in the Minimum
Guaranteed Net Worth, Inter-Asia shall pay the
Whether or not Nyco is liable for the acts of its difference within 5 days from date of receipt of the
president. audited financial statements.
RULING:

YES. Asia Industries paid Inter-Asia a total amount of


P12,000,000.00: P5,000,000.00 upon the signing of
The by-laws of Nyco expressly authorized its
the Agreement, and P7,000,000.00 on 2 November
President to enter into contracts, borrowing money ,
1978. From the STATEMENT OF INCOME AND
signing, indorsing checks in behalf of the compant.
DEFICIT attached to the financial report dated 28
Also, it appears that the same kind of transaction
November 1978 submitted by SGV, it appears that
already happened between Nyco and BA Finance.
FARMACOR had, for the 10 months ended 31
Hence, Nyco is placed from estopped from denying October 1978, a deficit of P11,244,225.00. Since
Yao’s authority because of its silence just to escape the stockholder's equity amounted to
liability. P10,000,000.00, FARMACOR had a net worth
deficiency of P1,244,225.00. The guaranteed net
worth shortfall thus amounted to P13,244,225.00
after adding the net worth deficiency of
Inter-Asia Investments Industries vs. Court of P1,244,225.00 to the Minimum Guaranteed Net
Appeals Worth of P12,000,000.00. The adjusted contract
price, therefore, amounted to P6,225,775.00 which
[GR 125778, 10 June 2003]
is the difference between the contract price of
P19,500,000.00 and the shortfall in the guaranteed
net worth of P13,224,225.00. Asia Industries having
Facts: On 1 September 1978, Inter-Asia Industries, already paid Inter-Asia P12,000,000.00, it was
Inc. (Inter-Asia), by a Stock Purchase Agreement entitled to a refund of P5,744,225.00. Inter-Asia
(the Agreement), sold to Asia Industries, Inc. (Asia thereafter proposed, by letter of 24 January 1980,
Industries) for and in consideration of the sum of signed by its president, that Asia Industries's claim
P19,500,000.00 all its right, title and interest in and for refund be reduced to P4,093,993.00, it promising
to all the outstanding shares of stock of to pay the cost of the Northern Cotabato Industries,
FARMACOR, INC. (FARMACOR). The Agreement Inc. (NOCOSII) superstructures in the amount of
P759,570.00. To the proposal respondent agreed. Section 23 of the Corporation Code of the
Inter-Asia, however, welched on its promise. Philippines. Under this provision, the power and
responsibility to decide whether the corporation
should enter into a contract that will bind the
corporation is lodged in the board, subject to the
Inter-Asia's total liability thus stood at
articles of incorporation, bylaws, or relevant
P4,853,503.00 (P4,093,993.00 plus P759,570.00)
provisions of law. However, just as a natural person
exclusive of interest. On 5 April 1983, Asia
may authorize another to do certain acts for and on
Industries filed a complaint against Inter-Asia with
his behalf, the board of directors may validly
the Regional Trial Court of Makati, one of two
delegate some of its functions and powers to
causes of action of which was for the recovery of
officers, committees or agents. The authority of
above-said amount of P4,853,503.00 17 plus
such individuals to bind the corporation is generally
interest. Denying Asia Industries's claim, Inter-Asia
derived from law, corporate bylaws or authorization
countered that Asia Industries failed to pay the
from the board, either expressly or impliedly by
balance of the purchase price and accordingly set
habit, custom or acquiescence in the general course
up a counterclaim. Finding for Asia Industries, the
of business, viz: "A corporate officer or agent may
trial court rendered on 27 November 1991 a
represent and bind the corporation in transactions
Decision, ordering Inter-Asia to pay Asia Industries
with third persons to the extent that [the] authority to
the sum of P4,853,503.00 plus interest thereon at
do so has been conferred upon him, and this
the legal rate from the filing of the complaint until
includes powers as, in the usual course of the
fully paid, the sum of P30,000.00 as attorney's fees
particular business, are incidental to, or may be
and the costs of suit; and (b) dismissing the
implied from, the powers intentionally conferred,
counterclaim. On appeal to the Court of Appeals,
powers added by custom and usage, as usually
and by Decision of 25 January 1996, the Court of
pertaining to the particular officer or agent, and such
Appeals affirmed the trial court's decision. Inter-
apparent powers as the corporation has caused
Asia's motion for reconsideration of the decision
person dealing with the officer or agent to believe
having been denied by the Court of Appeals by
that it has conferred.... [A]pparent authority is
Resolution of 11 July 1996, Inter-Asia filed the
derived not merely from practice. Its existence may
petition for review on certiorari.
be ascertained through (1) the general manner in
which the corporation holds out an officer or agent
as having the power to act or, in other words the
Issue: Whether the 24 January 1980 letter signed apparent authority to act in general, with which it
by Inter-Asia’s president is valid and binding. clothes him; or (2) the acquiescence in his acts of a
particular nature, with actual or constructive
knowledge thereof, within or beyond the scope of
his ordinary powers. It requires presentation of
Held: The 24 January 1980 letter signed by Inter-
evidence of similar acts executed either in its favor
Asia's president is valid and binding. As held in the
or in favor of other parties. It is not the quantity of
case of People's Aircargo and Warehousing Co.,
similar acts which establishes apparent authority,
Inc. v. Court of Appeals, the general rule is that, in
but the vesting of a corporate officer with the power
the absence of authority from the board of directors,
to bind the corporation." Hence, an officer of a
no person, not even its officers, can validly bind a
corporation who is authorized to purchase the stock
corporation. A corporation is a juridical person,
of another corporation has the implied power to
separate and distinct from its stockholders and
perform all other obligations arising therefrom, such
members, "having . . . powers, attributes and
as payment of the shares of stock. By allowing its
properties expressly authorized by law or incident to
president to sign the Agreement on its behalf, Inter-
its existence." Being a juridical entity, a corporation
Asia clothed him with apparent capacity to perform
may act through its board of directors, which
all acts which are expressly, impliedly and
exercises almost all corporate powers, lays down all
inherently stated therein.
corporate business policies and is responsible for
the efficiency of management, as provided in
LUNETA MOTOR COMPANY transaction of its lawful business may reasonably
vs. and necessarily require.
A.D. SANTOS, INC., ET AL. Petitioner’s corporate purposes are to carry
G.R. No. L-17716. July 31, 1962 on a general mercantile and commercial business,
etc., and that it is authorized in its articles of
FACTS: incorporation to operate and otherwise deal in and
concerning automobiles and automobile
On December 31, 1941, to secure payment accessories' business in all its multifarious
of a loan evidenced by a promissory note executed ramification and to operate, etc., and otherwise
by Nicolas Concepcion and guaranteed by one dispose of vessels and boats, etc., and to own and
Placido Esteban in favor of petitioner, Concepcion operate steamship and sailing ships and other
executed a chattel mortgage covering the above floating craft and deal in the same and engage in
mentioned certificate in favor of petitioner. the Philippine Islands and elsewhere in the
Thereafter, he constituted a second transportation of persons, merchandise and chattels
mortgage on the same certificate to secure payment by water; all this incidental to the transportation of
of a subsequent loan obtained by Concepcion from automobiles.
the Rehabilitation Finance Corporation (now The Court finds that Petitoner’s articles of
Development Bank of the Philippines). This second incorporation are precisely the best evidence that it
mortgage was approved by the respondent has no authority at all to engage in the business of
Commission, subject to the mortgage lien in favor of land transportation and operate a taxicab service.
petitioner. The certificate was later sold to Francisco That it may operate and otherwise deal in
Benitez, Jr., who resold it to Rodi Taxicab automobiles and automobile accessories; that it
Company. Both sales were made with assumption may engage in the transportation of persons by
of the mortgage in favor of the RFC, and were also water does not mean that it may engage in the
approved provisionally by the Commission, subject business of land transportation — an entirely
to petitioner's lien. different line of business. If it could not thus engage
On October 10, 1953 petitioner filed an in the line of business, it follows that it may not
action to foreclose the chattel mortgage executed in acquire a certificate of public convenience to
its favor by Concepcion. While the above case was operate a taxicab service, such as the one in
pending, the RFC also instituted foreclosure question, because such acquisition would be
proceedings on its second chattel mortgage, and as without purpose and would have no necessary
a result of the decision in its favor therein rendered, connection with petitioner's legitimate business.
the certificate of public convenience was sold at
public auction in favor of Amador D. Santos for
P24,010.00 on August 31, 1956. Santos
immediately applied with the Commission for the
approval of the sale, and the same was approved CENTRAL TEXTILE MILLS, INC., petitioner, vs.
on January 26, 1957, subject to the mortgage lien in NATIONAL WAGES AND PRODUCTIVITY
favor of petitioner. COMMISSION, REGIONAL TRIPARTITE WAGES
AND PRODUCTIVITY BOARD - NATIONAL
CAPITAL REGION, and UNITED CMC TEXTILE
ISSUE: WORKERS UNION, respondents.
DECISION
Whether or not the purpose for which ROMERO, J.:
petitioner was organized and the transaction of its On December 20, 1990, respondent Regional
lawful business reasonably and necessarily requires Tripartite Wages and Productivity Board - National
acquisition and holds the certificate and operates as Capital Region (the Board) issued Wage Order No.
a common carrier by land. NCR-02 (WO No. NCR-02), which took effect on
January 9, 1991. Said wage order mandated a
RULING: P12.00 increase in the minimum daily wage of all
employees and workers in the private sector in the
NO. NCR, but exempted from its application distressed
employers whose capital has been impaired by at
Under Section 13 (5) of the Corporation least twenty-five percent (25%) in the preceding
Law, a corporation created thereunder may year.
purchase, hold, etc., and otherwise deal in such real The Guidelines on Exemption from Compliance
and personal property is the purpose for which the With the Prescribed Wage/Cost of Living Allowance
corporation was formed may permit, and the Increase Granted by the Regional Tripartite Wage
and Productivity Boards, issued on February 25, petitioner included in its total paid-up capital
1991, defined capital as the paid-up capital at the payments on advance subscriptions, although the
end of the last full accounting period (in case of proposed increase in its capitalization had not yet
corporations). Under said guidelines, (a)n applicant been approved by, let alone presented for the
firm may be granted exemption from payment of the approval of, the SEC. As observed by the Board in
prescribed increase in wage/cost-of-living allowance its order of February 4, 1992, the aforementioned
for a period not to exceed one (1) year from (r)esolution (of August 15, 1990) has not been filed
effectivity of the Order x x x when accumulated by the corporation with the SEC, nor was a petition
losses at the end of the period under review have to amend its Articles of Incorporation by reason of
impaired by at least 25 percent the paid-up capital the increase in its capitalization filed by the same.
at the end of the last full accounting period It is undisputed that petitioner incurred a net loss of
preceding the application. P68,844,222.49 in 1990, and its authorized capital
By virtue of these provisions, petitioner filed on April stock as of that time stood at P128,000,000.00.[if
11, 1991 its application for exemption from !supportFootnotes][1][endif] On August 15, 1990, a Board

compliance with WO No. NCR-02 due to financial resolution increasing the capital stock of the
losses. corporation was affirmed by the requisite number of
In an order dated October 22, 1991, the Boards stockholders. Although no petition to that effect was
Vice-Chairman, Ernesto Gorospe, disapproved ever submitted to the SEC for its approval,
petitioners application for exemption after petitioner already started receiving subscriptions
concluding from the documents submitted that and payments on the proposed increase, which it
petitioner sustained an impairment of only 22.41%. allegedly held conditionally, that is, pending
On February 4, 1992, petitioners motion for approval of the same by the SEC. In its
reconsideration was dismissed by the Board for lack Memorandum, however, petitioner admitted, without
of merit. The Board, except for Vice-Chairman giving any reason therefor, that it indeed received
Gorospe who took no part in resolving the said subscriptions and payments to the said proposed
motion for reconsideration, opined that according to increase in capital stock, even in the absence of
the audited financial statements submitted by SEC approval of the increase as required by the
petitioner to them, to the Securities and Exchange Corporation Code.[if !supportFootnotes][2][endif] Thus, by the
Commission and to the Bureau of Internal Revenue, end of 1990, the corporation had a subscribed
petitioner had a total paid-up capital of capital stock of P482,748,900.00 and, after
P305,767,900.00 as of December 31, 1990, which deducting P176,981,000.00 in subscriptions
amount should be the basis for determining the receivables, a total paid-up capital of
capital impairment of petitioner, instead of the P305,767,900.00.[if !supportFootnotes][3][endif]

authorized capital stock of P128,000,000.00 which it P177,767,900.00 of this sum constituted the
insists should be the basis of computation. unauthorized increase in its subscribed capital
The Board also noted that petitioner did not file with stock, which are actually payments on future issues
the SEC the August 15, 1990 resolution of its Board of shares.
of Directors, concurred in by its stockholders These payments cannot as yet be deemed part of
representing at least two-thirds of its outstanding petitioners paid-up capital, technically speaking,
capital stock, approving an increase in petitioners because its capital stock has not yet been legally
authorized capital stock from P128,000,000.00 to increased. Thus, its authorized capital stock in the
P640,000,000.00. Neither did it file any petition to year when exemption from WO No. NCR-02 was
amend its Articles of Incorporation brought about by sought stood at P128,000,000.00, which was
such increase in its capitalization. impaired by losses of nearly 50%. Such payments
Petitioner maintains in the instant action that its constitute deposits on future subscriptions, money
authorized capital stock, not its unauthorized paid- which the corporation will hold in trust for the
up capital, should be used in arriving at its capital subscribers until it files a petition to increase its
impairment for 1990. Citing two SEC Opinions capitalization and a certificate of filing of increase of
dated August 10, 1971, and July 28, 1978, capital stock is approved and issued by the SEC.[if
interpreting Section 38 of the Corporation Code, it !supportFootnotes][4][endif] As a trust fund, this money is still

claims that the capital stock of a corporation withdrawable by any of the subscribers at any time
stand(s) increased or decreased only from and after before the issuance of the corresponding shares of
approval and the issuance of the certificate of filing stock, unless there is a pre-subscription agreement
of increase of capital stock. to the contrary, which apparently is not present in
We agree. the instant case. Consequently, if a certificate of
The guidelines on exemption specifically refer to increase has not yet been issued by the SEC, the
paid-up capital, not authorized capital stock, as the subscribers to the unauthorized issuance are not to
basis of capital impairment for exemption from WO. be deemed as stockholders possessed of such
No. NCR-02. The records reveal, however, that legal rights as the rights to vote and dividends. [if
!supportFootnotes][5][endif] 31, 1990. No pronouncement as to cost.
The Court observes that the subject wage order SO ORDERED.
exempts from its coverage employers whose capital
has been impaired by at least 25% because if
impairment is less than this percentage, the
employer can still absorb the wage increase. In the G.R. No. L-48237
case at hand, petitioners capital held answerable for
the additional wages would include funds it only
holds in trust, which to reiterate may not be deemed MADRIGAL & COMPANY, INC., petitioner,
par of its paid-up capital, the losses of which shall vs.
be the basis of the 25% referred to above. To HON. RONALDO B. ZAMORA, PRESIDENTIAL
include such funds in the paid-up capital would be ASSISTANT FOR LEGAL AFFAIRS, THE HON.
prejudicial to the corporation as an employer SECRETARY OF LABOR, and MADRIGAL
considering that the records clearly show that it is CENTRAL OFFICE EMPLOYEES UNION,
entitled to exemption, even as the anomaly was respondents.
brought about by an auditing error.
Another issue, raised late in the proceedings by FACTS:
respondents, is the alleged non-exhaustion of
administrative remedies by petitioner. They claim The petitioner was engaged in the management of
that the questioned order of the Board should have Rizal Cement Co., Inc. The petitioner and Rizal
first been appealed to the National Wages and Cement Co., Inc. are sister companies. Both are
Productivity Commission (the Commission), as owned by the same or practically the same
provided for under Section 9 of the Revised stockholders.
Guidelines on Exemption From Compliance With
the Prescribed Wage/Cost of Living Allowance On December 28, 1973, the Madrigal Central Office
Increases Granted by the Regional Tripartite Wages Employees Union, sought for the renewal of its
and Productivity Boards. collective bargaining agreement with the petitioner,
Petitioner explained that at the time it filed the which was due to expire on February 28, 1974.
instant petition for certiorari on March 6, 1992, the
procedure governing applications for exemption
Specifically, it proposed a wage increase of
from compliance with wage orders was the original
P200.00 a month, an allowance of P100.00 a
guidelines, which took effect on February 25, 1991.
month, and other economic benefits. The petitioner,
Under Section 6 of said guidelines, the denial by the
however, requested for a deferment in the
Board of a request for reconsideration shall be final
negotiations.
and immediately executory. Appeal to the
Commission as an optional remedy[if
!supportFootnotes][6][endif] was only made available after the On July 29, 1974, by an alleged resolution of its
issuance of the revised guidelines on September stockholders, the petitioner reduced its capital
25, 1992. Hence, petitioner cannot be faulted for not stock from 765,000 shares to 267,366 shares.
having first appealed the questioned orders. It must This was effected through the distribution of the
be added that since no order, resolution or decision marketable securities owned by the petitioner to its
of the Commission is being assailed in this petition, stockholders in exchange for their shares in an
it should be dropped as party respondent, as prayed equivalent amount in the corporation.
for in its manifestation and motion dated June 22,
1992.[if !supportFootnotes][7][endif] On August 22, 1975, by yet another alleged
In order to avoid any similar controversy, petitioner stockholders' action, the petitioner reduced its
is reminded to adopt a more systematic and precise authorized capitalization from 267,366 shares to
accounting procedure keeping in mind the various 110,085 shares, again, through the same scheme.
principles and nuances surrounding corporate
practice. After the petitioner's failure to sit down with the
WHEREFORE, the petition is hereby respondent union, the latter, on August 28, 1974,
GRANTED. The assailed orders of the Regional filed a complaint for unfair labor practice.
Tripartite Wages and Productivity Board National
Capital Region, dated October 22, 1991 and The petitioner alleged operational losses. Pending
February 4, 1992, are ANNULLED and SET ASIDE. the resolution of the case the petitioner informed the
Said Board is also hereby mandated to issue Secretary of Labor that "Rizal Cement Co., Inc. had
another order granting the application of petitioner ceased operating temporarily. Due to lack of
Central Textile Mills, Inc. for exemption from Wage business incentives and prospects and in order to
Order No. NCR-02 for the year ending December
prevent further losses it had to reduce its capital The petitioner would, however, have us believe that
stock on two occasions. As the situation, it in fact sustained losses. Whatever profits it
therefore, now stands, the Madrigal & Co., Inc. is earned, so it claims were in the nature of dividends
without substantial income to speak of, "declared on its shareholdings in other companies
necessitating a reorganization, by way of in the earning of which the employees had no
retrenchment, of its employees and operations." participation whatsoever."

The letter, however, was not verified and neither "Cash dividends," according to it, "are the absolute
was it accompanied by the proper supporting property of the stockholders and cannot be made
papers. For this reason, the Department of Labor available for disposition if only to meet the
took no action on the petitioner's request. employees' economic demands."

On January 19, 1976, the labor arbiter rendered a Dividends received by the company are corporate
decision against the petitioner. The arbiter earnings arising from corporate investment."
specifically found that the petitioner "had been Indeed, as found by the Commission, the petitioner
making substantial profits in its operation" since had entered such earnings in its financial
1972 through 1975. The petitioner appealed. statements as profits, which it would not have done
if they were not in fact profits.
Petitioner proceeded with the retrenchment program
and terminated the services of a number of Moreover, it is incorrect to say that such profits — in
employees. Respondent union filed a complaint for the form of dividends — are beyond the reach of the
illegal lockout. The Secretary of Labor found the petitioner's creditors since the petitioner had
dismissal to be contrary to law and ordered the received them as compensation for its management
petitioner to reinstate the employees. services in favor of the companies it managed as a
shareholder thereof. As such shareholder, the
The petitioner then moved for reconsideration, dividends paid to it were its own money, which may
which the Acting Labor Secretary, Amado Inciong, then be available for wage increments. It is not a
denied. case of a corporation distributing dividends in favor
of its stockholders, in which case, such dividends
would be the absolute property of the stockholders
On appeal, petitoner insists that it is incurring
losses; that as such, it has to reduce its and hence, out of reach by creditors of the
capitalization; that the profits it is earning are cash corporation. Here, the petitioner was acting as
stockholder itself, and in that case, the right to a
dividends from Rizal Cement Co.; that under the
share in such dividends, by way of salary increases,
law, dividends are the absolute property of a
may not be denied its employees.
stockholder like the petitioner and cannot be
compelled to share it with creditors (like the
employees). Accordingly, this court is convinced that the
petitioner's capital reduction efforts were, to begin
RULING: with, a subterfuge, a deception as it were, to
camouflage the fact that it had been making profits,
and consequently, to justify the mass layoff in its
What clearly emerges from the recorded facts is employee ranks, especially of union members. They
that the petitioner, awash with profits from its were nothing but a premature and plain distribution
business operations but confronted with the of corporate assets to obviate a just sharing to labor
demand of the union for wage increases, decided to of the vast profits obtained by its joint efforts with
evade its responsibility towards the employees by a capital through the years. Surely, we can neither
devised capital reduction. While the reduction in countenance nor condone this. It is an unfair labor
capital stock created an apparent need for practice.
retrenchment, it was, by all indications, just a mask
for the purge of union members, who, by then, had
agitated for wage increases. In the face of the
petitioner company's piling profits, the unionists had
the right to demand for such salary adjustments. Islamic Directorate of the Philippines vs. Court
of Appeals[GR117897,14May1997]
That the petitioner made quite handsome profits is
clear from the records Facts: Sometime in 1971, Islamic leaders of all
Muslim major tribal groups in the Philippines
headed by Dean Cesar Adib Majul organized and
incorporated the ISLAMIC DIRECTORATE OF THE the Deed of Absolute Sale signed by the Carpizo
PHILIPPINES (IDP), the primary purpose of which Group and the INC since the group of Engineer
is to establish an Islamic Center in Quezon City for, Carpizo was not the legitimate Board of Trustees of
the construction of a "Mosque (prayer place, the IDP.
Madrasah (Arabic School), and other religious
infrastructures" so as to facilitate the effective Meanwhile, INC, pursuant to the Deed of Absolute
practice of Islamic faith in the area. Towards this Sale executed in its favor, filed an action for Specific
end, that is, in the same year, the Libyan Performance with Damages against the vendor,
government donated money to the IDP to purchase Carpizo Group, before Branch 81 of the Regional
land at Culiat, Tandang Sora, Quezon City, to be Trial Court of Quezon City (Civil Case Q-90-6937)
used as a Center for the Islamic populace. The land to compel said group to clear the property of
were covered by two titles: TCTs RT-26520 squatters and deliver complete and full physical
(176616) and RT-26521 (170567), both registered possession thereof to INC. Likewise, INC filed a
in the name of IDP. motion in the same case to compel one Mrs. Leticia
P. Ligon to produce and surrender to the Register of
In 1972, after the purchase of the land by the Libyan Deeds of Quezon City the owner's duplicate copy of
government in the name of IDP, Martial Law was TCTs RT-26521 and RT-26520 covering the two
declared by the late President Ferdinand Marcos. parcels of land, so that the sale in INC's favor may
Most of the members of the 1971 Board of Trustees be registered and new titles issued in the name of
flew to the Middle East to escape political INC. Mrs. Ligon was alleged to be the mortgagee of
persecution. the two parcels of land executed in her favor by
certain Abdulrahman R.T. Linzag and Rowaida
Thereafter, two Muslim groups sprung, the Carpizo Busran-Sampaco claimed to be in behalf of the
Group, headed by Engineer Farouk Carpizo, and Carpizo Group.
the Abbas Group, led by Mrs. Zorayda Tamano and
Atty. Firdaussi Abbas. Both groups claimed to be Judge Celia Lipana-Reyes of Branch 81, Regional
the legitimate IDP. Trial Court of Quezon City, denied IDP's motion to
intervene on the ground of lack of juridical
Significantly, on 3 October 1986, the SEC, in a suit personality of the IDP- Tamano Group and that the
between these two contending groups, came out issues being raised by way of intervention are intra-
with a Decision in SEC Case 2687 declaring the corporate in nature, jurisdiction thereto properly
election of both the Carpizo Group and the Abbas pertaining to the SEC.
Group as IDP board members to be null and void.
Neither group, however, took the necessary steps Apprised of the pendency of SEC Case 4012
prescribed by the SEC in its 3 October 1986 involving the controverted status of the IDP-Carpizo
Decision, and no valid election of the members of Group but without waiting for the outcome of said
the Board of Trustees of IDP was ever called. case, Judge Reyes, on 12 September 1991,
Although the Carpizo Group attempted to submit a rendered Partial Judgment in Civil Case Q-90-6937
set of by-laws, the SEC found that, aside from that ordering the IDP-Carpizo Group to comply with its
Engineer Farouk Carpizo and Atty. Musib Buat, obligation under the Deed of Sale of clearing the
those who prepared and adopted the by-laws were subject lots of squatters and of delivering the actual
not bona fide members of the IDP, thus rendering possession thereof to INC. Thereupon Judge Reyes
the adoption of the by-laws likewise null and void. in another Order, dated 2 March 1992, pertaining
also to Civil Case Q-90-6937, treated INC as the
On 20 April 1989, without having been properly rightful owner of the real properties and disposed.
elected as new members of the Board of Trustees
of IDP, the Carpizo Group caused to be signed an On 6 April 1992, the Order was amended by Judge
alleged Board Resolution of the IDP, authorizing the Reyes directing Ligon "to deliver the owner's
sale of the subject two parcels of land to the Iglesia duplicate copies of TCT Nos. RT-26521 (170567)
ni Cristo (INC) for a consideration of and RT-26520 (176616) to the Register of Deeds of
P22,343,400.00, which sale was evidenced by a Quezon City for the purposes stated in the Order of
Deed of Absolute Sale 12 dated 20 April 1989. March 2, 1992."

On 30 May 1991, the 1971 IDP Board of Trustees Mortgagee Ligon went to the Court of Appeals, thru
headed by former Senator Mamintal Tamano, or the a petition for certiorari (CA-GR SP-27973), assailing
Tamano Group, filed a petition before the SEC the Orders of Judge Reyes. The appellate court
(SEC Case 4012) seeking to declare null and void dismissed her petition on 28 October 1992.
Undaunted, Ligon filed a petition for review before to comply with Section 40 of the Corporation Code
the Supreme Court (GR 107751). pertaining to the disposition of all or substantially all
assets of the corporation. The Tandang Sora
On 5 July 1993, finally came out with a Decision in property, it appears from the records, constitutes
SEC Case 4012, Declaring the by-laws submitted the only property of the IDP. Hence, its sale to a
by the IDP-Caprizo group as unauthorized, and third-party is a sale or disposition of all the
hence, null and void; declaring the sale of the two corporate property and assets of IDP falling
(2) parcels of land in Quezon City covered by the squarely within the contemplation of the foregoing
Deed of Absolute Sale entered into by Iglesia ni section. For the sale to be valid, the majority vote of
Kristo and the Islamic Directorate of the Philippines, the legitimate Board of Trustees, concurred in by
Inc. null and void; declaring the election of the the vote of at least 2/3 of the bona fide members of
Board of Directors 23 of the corporation from 1986 the corporation should have been obtained. These
to 1991 as null and void; and Declaring the twin requirements were not met as the Carpizo
acceptance of the respondents, except Farouk Group which voted to sell the Tandang Sora
Carpizo and Musnib Buat, as members of the IDP property was a fake Board of Trustees, and those
null and void. whose names and signatures were affixed by the
Carpizo Group together with the sham Board
The INC filed a Motion for Intervention but the same Resolution authorizing the negotiation for the sale
were, from all indications, not bona fide members of
was denied.
the IDP as they were made to appear to be.
Apparently, there are only 15 official members of
INC elevated SEC Case 4012 to the Court of the IDP including the 8 members of the Board of
Appeals by way of a special civil action for certiorari Trustees. All told, the disputed Deed of Absolute
and was granted. Sale executed by the fake Carpizo Board and INC
was intrinsically void ab initio.
Issue: Whether the Tandang Sora property was
legitimately sold to the INC.

Held: As far back as 3 October 1986, the SEC, in


ROSITA PEÑA
Case 2687, in a suit between the Carpizo Group
vs.
and the Abbas Group, already declared the election
THE COURT OF APPEALS, SPOUSES RISING T.
of the Carpizo Group (as well as the Abbas Group)
YAP and CATALINA YAP, PAMPANGA BUS CO.,
to the IDP Board as null and void for being violative
INC., JESUS DOMINGO, JOAQUIN BRIONES,
of the Articles of Incorporation. Nothing thus
SALVADOR BERNARDEZ, MARCELINO
becomes more settled than that the IDP-Carpizo
ENRIQUEZ and EDGARDO A. ZABAT
Group with whom INC contracted is a fake Board.
Premises considered, all acts carried out by the
Carpizo Board, particularly the sale of the Tandang G.R. No. 91478 ,February 7, 1991
Sora property, allegedly in the name of the IDP,
have to be struck down for having been done
without the consent of the IDP thru a legitimate IX. CORPORATE POWERS, AUTHORITY AND ACTIVITIES
Board of Trustees. Article 1318 of the New CivilSpecific (Express) Powers
Code lays down the essential requisites of
contracts, and where all these elements must be (e) Sell or Dispose of Assets (Sec. 40)
present to constitute a valid contract. For, where
even one is absent, the contract is void. Specifically,
consent is essential for the existence of a contract,
Facts;
and where it is wanting, the contract is non-existent.
Herein, the IDP, owner of the subject parcels of
land, never gave its consent, thru a legitimate Board
of Trustees, to the disputed Deed of Absolute Sale
executed in favor of INC. This is, therefore, a case Pampanga Bus Co.(PAMBUSCO), original owners of the lots
not only of vitiated consent, but one where consent in question under TCT Nos. 4314, 4315 and 4316, mortgaged
on the part of one of the supposed contracting the same to the Development Bank of the Philippines (DBP)
parties is totally wanting. Ineluctably, the subject
on January 3, 1962 in consideration of the amount of
sale is void and produces no effect whatsoever. The
Carpizo Group- INC sale is further deemed null and P935,000.00. This mortgage was foreclosed. In the
void ab initio because of the Carpizo Group's failureforeclosure sale under Act No. 3135 held on October 25,
1974, the said properties were awarded to Rosita Peña as In this case, neither petitioner nor respondents Yap spouses
highest bidder. A certificate of sale was issued in her favor by
are stockholders or officers of PAMBUSCO. Consequently,
the Senior Deputy Sheriff of Pampanga, Edgardo A. Zabat, the issue of the validity of the series of transactions resulting
upon payment of the sum of P128,000.00 to the Office of the in the subject properties being registered in the names of
Provincial Sheriff . The certificate of sale was registered on respondents Yap may be resolved only by the regular courts.
October 29, 1974.

In Teves vs. People's Homesite and Housing


Corporation, 7 this Court held:
On November 19, 1974, the board of directors of
PAMBUSCO, through three (3) out of its five (5) directors,
resolved to assign its right of redemption over the aforesaid
lots and authorized one of its members, Atty. Joaquin Briones
"to execute and sign a Deed of Assignment for and in behalf “that a person who is not a party obliged principally or
of PAMBUSCO in favor of any interested party . . .". subsidiarily in a contract may exercise an action for nullity of
Consequently, on March 18, 1975, Briones executed a Deed the contract if he is prejudiced in his rights with respect to one
of Assignment of PAMBUSCO's redemption right over the of the contracting parties, and can show the detriment which
subject lots in favor of Marcelino Enriquez . The latter then would positively result to him from the contract in which he
redeemed the said properties and a certificate of redemption
had no intervention,”
dated August 15, 1975 was issued in his favor by Sheriff
Zabat upon payment of the sum of one hundred forty
thousand, four hundred seventy four pesos P140,474.00) to
the Office of the Provincial Sheriff of Pampanga .
There can be no question in this case that the questioned
resolution and series of transactions resulting in the
A day after the aforesaid certificate was issued, Enriquez
executed a deed of absolute sale of the subject properties in registration of the properties in the name of respondent Yap
favor of plaintiffs-appellants, the spouses Rising T. Yap and spouses adversely affected the rights of petitioner to the said
Catalina Lugue, for the sum of P140,000.00. properties. Consequently, petitioner has the legal standing to
question the validity of said resolution and transactions.
On September 8, 1975, Peña wrote the Sheriff notifying him
that the redemption was not valid as it was made under a void
deed of assignment. She then requested the recall of the said
As to the question of validity of the board resolution of
redemption and a restraint on any registration or transaction
respondent PAMBUSCO adopted on November 19, 1974,
regarding the lots in question. She wrote the Sheriff asking for
Section 4, Article III of the amended by-laws of respondent
the execution of a deed of final sale in her favor on the ground
PAMBUSCO, provides as follows:
that "the one (1) year period of redemption has long elapsed
without any valid redemption having been exercised;" hence
she "will now refuse to receive the redemption money.
Sec. 4. Notices of regular and special meetings of the Board
of Directors shall be mailed to each Director not less than five
days before any such meeting, and notices of special meeting
Issue;
shall state the purpose or purposes thereof Notices of regular
meetings shall be sent by the Secretary and notices of special
meetings by the President or Directors issuing the call. No
1.) Legal standing of the petitioner (PEÑA) to institute the failure or irregularity of notice of meeting shall invalidate any
case at bar, and regular meeting or proceeding thereat; Provided a quorum of
2.The validity of the board resolution (Deed of Assignment) by the Board is present, nor of any special meeting; Provided at
the members of the board of directors of PAMBUSCO. least four Directors are present.

Held;
The by-laws of a corporation are its own private laws which
substantially have the same effect as the laws of the
corporation. They are in effect, written, into the charter. In this
sense they become part of the fundamental law of the stockholders holding shares in the corporation,
corporation with which the corporation and its directors and entitling them to atleast 2/3 of the voting power.
officers must comply. 11
Issue:
W/N the investment of corporate funds of
Ma-ao were in violation of corporation law.
Apparently, only three (3) out of five (5) members of the board
Ruling:
of directors of respondent PAMBUSCO convened on
No.
November 19, 1974 by virtue of a prior notice of a special Investment of corporate funds in another
meeting. There was no quorum to validly transact business corporation if done in pursuance of the corporate
since, under Section 4 of the amended by-laws hereinabove purpose, does not need the approval of the
reproduced, at least four (4) members must be present to stockholders, but where the purchase of the shares
constitute a quorum in a special meeting of the board of of another corporation is done solely for investment
directors of respondent PAMBUSCO. Under Section 25 of the and not to accomplish the purpose of its
incorporation, the vote of the approval of the
Corporation Code of the Philippines, the articles of
stockholders is necessary. The investment made in
incorporation or by-laws of the corporation may fix a greater Philippine Fiber was upheld by the SC. Philippine
number than the majority of the number of board members to Fiber was engaged in the manufacture of bags or
constitute the quorum necessary for the valid transaction of investments in another corporation engaged in the
business. Any number less than the number provided in the manufacture engaged in the manufacture of bags.
articles or by-laws therein cannot constitute a quorum and Since the sugar central is engaged in the
manufacture of sugars, sugar bags necessarily
any act therein would not bind the corporation; all that the
would come under the purview of its needs under
attending directors could do is to adjourn. the regular course of business.
Any corporation whatever its primary
purpose has a choice of placing such fund either in
savings or time deposit account or in money market
It is also undisputed that at the time of the passage of the placements, or treasury bills, or even in shares of
questioned resolution, respondent PAMBUSCO was insolvent stocks of other corporations which are traded in the
and its only remaining asset was its right of redemption over stock exchange. The exercise of such business
the subject properties. Since the disposition of said judgment on the part of the board in consistency
redemption right of respondent PAMBUSCO by virtue of the with the primary purpose, since it is expected even
questioned resolution was not approved by the required from the stockholders to believe, that it is within the
ordinary business discretion of the Board to place
number of stockholders under the law, the said resolution, as
the corporation’s investible fund in the form of
well as the subsequent assignment executed on March 8, investment that would yield the best possible return
1975 assigning to respondent Enriquez the said right of to the corporation and would not require the
redemption, should be struck down as null and void. ratification of the stockholders or members each
time.

De La Rama VS. Ma-ao Sugar Central Co. (1969)


Nielson vs lepanto
Facts:
De la Rama et. al. contend that Ma-ao Facts:
Sugar Central through its President, subscribed Php
300,000 worth of capital stock of Philippine Fiber An operating agreement was executed
Processing Co. Inc. They allege that the time of the before World War II (on 30 January 1937) between
first two payments were made there was no board Nielson & Co. Inc. and the Lepanto Consolidated
resolution authorizing the investment and that it was
Mining Co. whereby the former operated and
only before the third payment that the President was
so authorized by the Board of Directors. De la managed the mining properties owned by the latter
Rama also contends that even assuming arguendo, for a management fee of P2,500.00 a month and a
that the said Board Resolutions are valid, the 10% participation in the net profits resulting from the
transaction is still wanting in legality, no resolution operation of the mining properties, for a period of 5
having been approved by the affirmative vote of the years. In 1940, a dispute arose regarding the
computation of the 10% share of Nielson in the
profits. The Board of Directors of Lepanto, realizing the former in view of the refusal of the latter to
that the mechanics of the contract was unfair to comply with the terms of a management contract
Nielson, authorized its President to enter into an entered into between them on 30 January 1937,
including attorney's fees and costs. LEPANTO in its
agreement with Nielson modifying the pertinent
answer denied the material allegations of the
provision of the contract effective 1 January 1940 in complaint and set up certain special defenses,
such a way that Nielson shall receive (1) 10% of the among them, prescription and laches, as bars
dividends declared and paid, when and as paid, against the institution of the action.
during the period of the contract and at the end of
each year, (2) 10% of any depletion reserve that After trial, the court a quo rendered a decision
may be set up, and (3) 10% of any amount dismissing the complaint with costs. The court
stated that it did not find sufficient evidence to
expended during the year out of surplus earnings
establish LEPANTO's counterclaim and so it
for capital account. In the latter part of 1941, the likewise dismissed the same. NIELSON appealed.
parties agreed to renew the contract for another The Supreme Court reversed the decision of the
period of 5 years, but in the meantime, the Pacific trial court and enter in lieu thereof another, ordering
War broke out in December 1941. In January 1942 Lepanto to pay Nielson (1) 10% share of cash
operation of the mining properties was disrupted on dividends of December, 1941 in the amount of
account of the war. In February 1942, the mill, P17,500.00, with legal interest thereon from the
date of the filing of the complaint; (2) management
power plant, supplies on hand, equipment,
fee for January, 1942 in the amount of P2,500.00,
concentrates on hand and mines, were destroyed with legal interest thereon from the date of the filing
upon orders of the United States Army, to prevent of the complaint; (3) management fees for the sixty-
their utilization by the invading Japanese Army. month period of extension of the management
contract, amounting to P150,000.00, with legal
interest from the date of the filing of the complaint;
The Japanese forces thereafter occupied the mining (4) 10% share in the cash dividends during the
properties, operated the mines during the period of extension of the management contract,
continuance of the war, and who were ousted from amounting to P1,400,000.00, with legal interest
the mining properties only in August 1945. After the thereon from the date of the filing of the complaint;
mining properties were liberated from the Japanese (5) 10% of the depletion reserve set up during the
forces, LEPANTO took possession thereof and period of extension, amounting to P53,928.88, with
embarked in rebuilding and reconstructing the legal interest thereon from the date of the filing of
mines and mill; setting up new organization; the complaint; (6) 10% of the expenses for capital
clearing the mill site; repairing the mines; erecting account during the period of extension, amounting
staff quarters and bodegas and repairing existing to P694,364.76, with legal interest thereon from the
structures; installing new machinery and equipment; date of the filing of the complaint; (7) to issue and
repairing roads and maintaining the same; deliver to Nielson and Co. Inc. shares of stock of
salvaging equipment and storing the same within Lepanto Consolidated Mining Co. at par value
the bodegas; doing police work necessary to take equivalent to the total of Nielson's 10% share in the
care of the materials and equipment recovered; stock dividends declared on November 28, 1949
repairing and renewing the water system; and and August 22, 1950, together with all cash and
retimbering. The rehabilitation and reconstruction of stock dividends, if any, as may have been declared
the mine and mill was not completed until 1948. On and issued subsequent to November 28, 1949 and
26 June 1948 the mines resumed operation under August 22, 1950, as fruits that accrued to said
the exclusive management of LEPANTO. Shortly shares; provided that if sufficient shares of stock of
after the mines were liberated from the Japanese Lepanto's are not available to satisfy this judgment,
invaders in 1945, a disagreement arose between Lepanto shall pay Nielson an amount in cash
NIELSON and LEPANTO over the status of the equivalent to the market value of said shares at the
operating contract which as renewed expired in time of default, that is, all shares of stock that
1947. Under the terms thereof, the management should have been delivered to Nielson before the
contract shall remain in suspense in case fortuitous filing of the complaint must be paid at their market
event or force majeure, such as war or civil value as of the date of the filing of the complaint;
commotion, adversely affects the work of mining and all shares, if any, that should have been
and milling. On 6 February 1958, NIELSON brought delivered after the filing of the complaint at the
an action against LEPANTO before the Court of market value of the shares at the time Lepanto
First Instance of Manila to recover certain sums of disposed of all its available shares, for it is only then
money representing damages allegedly suffered by that Lepanto placed itself in condition of not being
able to perform its obligation; (8) the sum of limitation or lawful dissolution: Provided, That
P50,000.00 as attorney's fees; and (9) the costs. banking, savings and loan, and trust corporations
may receive deposits and issue certificates of
Lepanto seeks the reconsideration of the decision
deposit, checks, drafts, and bills of exchange, and
rendered on 17 December 1966.
the like in the transaction of the ordinary business of
banking, savings and loan, and trust corporations.
Issue: (As amended by Act No. 2792, and Act No. 3518;
Emphasis supplied.)
Whether or not Nielson is entitled to stock dividend

Held:
From the above-quoted provision of Section 16 of
Section 16 of the Corporation Law, in part, provides the Corporation Law, the consideration for which
as follows: shares of stock may be issued are: (1) cash; (2)
property; and (3) undistributed profits. Shares of
stock are given the special name "stock dividends"
No corporation organized under this Act shall create only if they are issued in lieu of undistributed profits.
or issue bills, notes or other evidence of debt, for If shares of stocks are issued in exchange of cash
circulation as money, and no corporation shall issue or property then those shares do not fall under the
stock or bonds except in exchange for actual cash category of "stock dividends". A corporation may
paid to the corporation or for: (1) property actually legally issue shares of stock in consideration of
received by it at a fair valuation equal to the par or services rendered to it by a person not a
issued value of the stock or bonds so issued; and in stockholder, or in payment of its indebtedness. A
case of disagreement as to their value, the same share of stock issued to pay for services rendered is
shall be presumed to be the assessed value or the equivalent to a stock issued in exchange of
value appearing in invoices or other commercial property, because services is equivalent to
documents, as the case may be; and the burden or property.14 Likewise a share of stock issued in
proof that the real present value of the property is payment of indebtedness is equivalent to issuing a
greater than the assessed value or value appearing stock in exchange for cash. But a share of stock
in invoices or other commercial documents, as the thus issued should be part of the original capital
case may be, shall be upon the corporation, or for stock of the corporation upon its organization, or
(2) profits earned by it but not distributed among its part of the stocks issued when the increase of the
stockholders or members; Provided, however, That capitalization of a corporation is properly authorized.
no stock or bond dividend shall be issued without In other words, it is the shares of stock that are
the approval of stockholders representing not less originally issued by the corporation and forming part
than two-thirds of all stock then outstanding and of the capital that can be exchanged for cash or
entitled to vote at a general meeting of the services rendered, or property; that is, if the
corporation or at a special meeting duly called for corporation has original shares of stock unsold or
the purpose. unsubscribed, either coming from the original
capitalization or from the increased capitalization.
Those shares of stock may be issued to a person
who is not a stockholder, or to a person already a
xxx xxx xxx stockholder in exchange for services rendered or for
cash or property. But a share of stock coming from
stock dividends declared cannot be issued to one
who is not a stockholder of a corporation.
No corporation shall make or declare any dividend
except from the surplus profits arising from its
business, or divide or distribute its capital stock or
property other than actual profits among its A "stock dividend" is any dividend payable in shares
members or stockholders until after the payment of of stock of the corporation declaring or authorizing
its debts and the termination of its existence by such dividend. It is, what the term itself implies, a
distribution of the shares of stock of the corporation can not be issued to a person who is not a
among the stockholders as dividends. A stock stockholder in payment of services rendered. And
dividend of a corporation is a dividend paid in so, in the case at bar Nielson can not be paid in
shares of stock instead of cash, and is properly shares of stock which form part of the stock
payable only out of surplus profits.15 So, a stock dividends of Lepanto for services it rendered under
dividend is actually two things: (1) a dividend, and the management contract. We sustain the
(2) the enforced use of the dividend money to contention of Lepanto that the understanding
purchase additional shares of stock at par.16 When between Lepanto and Nielson was simply to make
a corporation issues stock dividends, it shows that the cash value of the stock dividends declared as
the corporation's accumulated profits have been the basis for determining the amount of
capitalized instead of distributed to the stockholders compensation that should be paid to Nielson, in the
or retained as surplus available for distribution, in proportion of 10% of the cash value of the stock
money or kind, should opportunity offer. Far from dividends declared. And this conclusion of Ours
being a realization of profits for the stockholder, it finds support in the record.
tends rather to postpone said realization, in that the
fund represented by the new stock has been
transferred from surplus to assets and no longer
available for actual distribution.17 Thus, it is Jose Ricafort et al v. Hon Judge Felix Moya et al
apparent that stock dividends are issued only to
stockholders. This is so because only stockholders The case stemmed from a deed of absolute sale
executed on April 18 1978 between Daniel
are entitled to dividends. They are the only ones
Aguinaldo as vendor and Ricaforte and Calalang as
who have a right to a proportional share in that part vendees. Both had failed to fulfil their obligations in
of the surplus which is declared as dividends. A the deed which worsened as cases have been filed
stock dividend really adds nothing to the interest of against each party. After a number of suits filed,
the stockholder; the proportional interest of each Ricafort et al filed for Preliminary injunction against
stockholder remains the same.18If a stockholder is Aguinaldo to prohibit him from representing himself
deprived of his stock dividends - and this happens if as controlling stockholder of NADECOR and
attempting to sell that corporation’s so-called
the shares of stock forming part of the stock
Kingking Mining claims. This was granted by the
dividends are issued to a non-stockholder — then Court. Ricafort and Calalang believed that they and
the proportion of the stockholder's interest changes SAICOR are to be excluded by Aguinaldo and
radically. Stock dividends are civil fruits of the group from the management of NADECOR as
original investment, and to the owners of the shares Aguinaldo had refused to convoke the stockholders
belong the civil fruits.19 annual meeting for the year 1981 which should
have been called on third Monday of August, in the
by-laws.

The term "dividend" both in the technical sense and Stockholders elected as directors Calalang, Ricafort
its ordinary acceptation, is that part or portion of the and 5 others. The stockholders rejected the
profits of the enterprise which the corporation, by its aforesaid operating agreement in March 25 1981
between NADECOR as represented by Aguinaldo
governing agents, sets apart for ratable division
and the consortium of Black Mountain Corp et al
among the holders of the capital stock. It means the and instead approved the proposed operating
fund actually set aside, and declared by the agreement with Benguet Corporation. The
directors of the corporation as dividends and duly Certificate attesting to these events was filed with
ordered by the director, or by the stockholders at a SEC. NADECOR, represented by new officers
corporate meeting, to be divided or distributed entered into an Operating Agreement with Benguet
among the stockholders according to their Corporation for the operation by the latter of the
KINGKING MINES.
respective interests.20
ISSUE: Whether the Operating Agreement with the
Black Mountain Consortium of March 25, 1981 is
It is Our considered view, therefore, that under valid
Section 16 of the Corporation Law stock dividends
HELD: NO. in the 3 corporations subject of sale and the 9 Saug
lots as security for payment of balance price.
The Operating Agreement with the Black Mountain
Consortium was never ratified by the NADECOR August 18 1980- Shareholders of NADECOR
stockholders; indeed, it was explicitly rejected by elected Aguinaldo, Aytona, Calalang, Ricafort and 5
said stockholders. Considering that the Kingking others as directors. Aytona, Aguinaldo and Borsoto
Mines comprise all or substantially the assets of as Chairman, President and Secretary respectively
NADECOR, the operating agreement of March 25
1981 had to be ratified by the stockholders in order Sept 26 1980- Aguinaldo executed a deed of
to be valid and effective. This, in accordance with reconveyance of 9 Saug lots in favour of ADECOR
Section 44 of the Corporation Code. That no such as called for by the April 18 1978 stipulation which
ratification was ever given constitutes yet another was not complied with by Ricafort and Calalang (on
reason to invalidate such. the mortgaged lots) because the deed of
reconveyance of the Saug lots executed by
The agreement executed on March 25 1981 was Aguinaldo in favour of ADECOR was fatally
entered into in defiance of valid orders of a court of defective as it did not bear the signature of
competent jurisdiction and was in fact subsequently Aguinaldo’s wife, Helen Leontovich. No remedy
nullified by it; it was entered into against the wishes done to the omission until controversy between
of the majority of the stockholders and directors and parties has worsened
in truth, was not only not ratified by the majority of
the said stockholders as required by the FIRST CASE: Oct 6 1980
Corporation Code, but explicitly rejected and
disowned by them at a meeting duly convoked, said
Ricafort and Calalang filed before the CFI of Rizal
stockholders thereafter approving an operation
due to breach of contract of April 18 1978 by
agreement with Benguet Corporation; the
Aguinaldo’s failure to
agreement was sought to be vindicated and
enforced by individuals who no longer represented
the majority of the stockholders of NADECOR, over transfer the 9 Saug lots with prayer that Aguinaldo’s
the objection and against the wishes of the obligation to make conveyance be deemed waived
legitimate majority; the authority granted to the and that Ricafort and Calalang be discharged from
consortium to implement the agreement of March their obligationAguinaldo reacted by instructing a
25 1981 was rescinded and revoked by the Office of notary public, Neis to conduct Auction Sale of
the President and one of the companies in said pledged stock of DRACOR, ADECOR AND
consortium is now no longer capable on account of NADECOR
bankruptcy of complying with its contractual
commitments-it is impossible to accord the Ricafort and Calalang brought suit against
agreement any validity or effect whatsoever. Aguinaldo and Neis to be stopped from proceeding
with the auction sale and applied for preliminary
NOTA BENE: NOT ALL FACTS ARE STATED injunction
HEREIN
TRO issued by Judge Maddela enjoining the
auction sale.

Three more amendments were made by Ricafort.


On April 18, 1978, a deed of sale was executed by
Reformation of contract of sale to include all stocks
Daniel Aguinaldo and DR Aguinaldo Corporation
in NADECOR of Aguinaldo and DRACOR.
(DRACOR) as vendors, and Jose Ricafort and
Preliminary injunction against Aguinaldo to prohibit
Conrado Calalang as vendees. In the deed,
him from representing himself as controlling
Aguinaldo and DRACOR sold to Ricafort and
stockholder of NADECOR and attempting to sell
Calalang all their shares of stocks and subscriptions
that corporation’s so-called Kingking Mining
in three corporations: ADECOR (Aguinaldo Dev’t
claims.These amendments were allowed by court
Corp), MARBLECORP(Phil. Marble Corp) and
order. Third amendment added averments of fraud
NADECOR (Nationwide Devt Corp) Aguinaldo
relative to the transfer by Aguinaldo to himself of
bound himself to convey 9 parcels of rice land in
ADECOR shares in a foreign company.
Saug, Davao del Norte held in trust by him, to the
real or beneficial owner ADECOR. Ricafort and
Calalang pledged to Aguinaldo all shares of stocks Preliminary injunctions by Manila CFI. Injunctive
orders issued against Aguinaldo and his group by
the trial court as regards Kingking’s mining claims
and Operating Agreement involving Kingking Ricafort et al moved to dismiss the complaint for
between Aguinaldo in representation of NADECOR failure to state a cause of action, that NADECOR’s
and a consortium made up of Black Mountain Inc, agreement with Black Mountain Inc for the operation
Tetra Management Corp, and Energy Corporation. of the Kingking mining claims had never been
approved by the NADECOR stockholders owning
Court enjoined NADECOR in ratifying the Operating the majority of the capital stock. Rtc denied the
Management MTD.

Court stopped the auction sale which was


rescheduled by Neis and; Aguinaldo in representing
himself as controlling stockholder of NADECOR JOSE G. RICAFORT, CONRADO T. CALALANG,
offering its Kingking claims for sale NATIONWIDE DEVELOPMENT CORPORATION
and AGUINALDO DEVELOPMENT
Court order prohibiting Aguinaldo from selling the CORPORATION, petitioners,
ADECOR shares in Sawyer-Adecor International vs.
Corporation(SAICOR) which he had caused to be HON. FELIX L. MOYA, Judge, CFI, Davao, Br. II,
transferred in his name BLACK MOUNTAIN, INC., TETRA
MANAGEMENT CORPORATION and the
ENERGY CORPORATION, respondents.
Ricafort and Calalang believed that they and Sycip, Salazar, Hernandez & Gatmaitan for
SAICOR are to be excluded by Aguinaldo and petitioners.
 Reymundo P.G. Villarica and Siguion
group from the management of NADECOR as Reyna, Montecillo & Ongsiako for private
Aguinaldo had refused to convoke the stockholders respondents.
annual meeting for the year 1981 which should NARVASA, J.:
have been called on third Monday of August, in the With this judgment, the Court writes finis to a
by-laws. controversy principally involving two (2) groups of
individuals, which has given rise to no less than
Stockholders elected as directors Calalang, Ricafort eleven (11) actions and proceedings: three (3) in
and 5 others. The stockholders rejected the the Court of First Instance, two (2) in the Securities
aforesaid operating agreement in March 25 1981 and Exchange Commission, and six (6) in this
between NADECOR as represented by Aguinaldo Court.
and the consortium of Black Mountain Corp et al The roots of the controversy go as far back as 1978,
and instead approved the proposed operating to a deed of sale executed on April 18 of that year
agreement with Benguet Corporation. The by Daniel R. Aguinaldo and D.R. Aguinaldo
Certificate attesting to these events was filed with Corporation (DRACOR), as vendors, and Jose
SEC Ricafort and Conrado Calalang, as vendees. By that
deed:
NADECOR, represented by new officers entered 1) Aguinaldo and DRACOR sold to Jose Ricafort
into an Operating Agreement with Benguet and Conrado Calalang all their shares of stock and
Corporation for the operation by the latter of the subscriptions in three (3) corporations, namely:
KINGKING MINES. a) ADECOR (Aguinaldo Development Corporation ),
b) MARBLECORP (Philippine Marble Corporation),
SEC CASE: NADECOR, represented by its newly and
elected directors and officers filed against Aguinaldo c) NADECOR (Nationwide Development
and group for continuingly fraudulently representing Corporation);
themselves as the legitimate officers of NADECOR. 2) Aguinaldo bound himself to convey nine (9)
parcels of rice land in Saug, Davao del Norte, held
in trust by him, to the real or beneficial owner,
CIVIL CASE no. 143: the consortium of Black
ADECOR;
Mountain Inc et al filed a complaint against Benguet
3) As security for payment of the balance of the
Corporation and
price (a down payment having been made on
execution of the deed of sale) Ricafort and Calalang
NADECOR and the directors of NADECOR bound themselves:
(Ricafort and Calalang included) enjoining them 1) To pledge to Aguinaldo all the shares of stock in
from interfering with Black Mountain’s possession of the three (3) corporations, subject of the sale; and
NADECOR’s Kingking Mines and recover damages. 2) To mortgage to Aguinaldo the nine (9) "Saug
lots."
The pledge of the stock certificates was effected on
the same day, April 18, 1978. 38117 of the Court of First Instance at Pasig, supra.
On August 18, 1980, at the stockholders' meeting of Case No. 38117 thus became functus officio.
NADECOR, Daniel R. Aguinaldo, Dominador Consequently, Ricafort and Calalang caused its
Aytona, Conrado Calalang, Jose G. Ricafort, and dismissal by filing with the Pasig Court a notice to
five (5) others were elected directors. These that effect dated November 6, 1980, in accordance
directors later elected Aytona, Aguinaldo, and with Section 1, Rule 17 of the Rules of Court. 1
Romeo H. Borsoto as, respectively, Chairman, Temporary Restraining Order
President and Secretary. On October 8, 1980, a temporary restraining order
A month later, or on September 26, 1980, Aguinaldo was issued by Judge Tomas Maddela, to whose
executed the Deed of Reconveyance of the nine sala Civil Case No. 135262 had been raffled,
"Saug lots" in favor of ADECOR, as called for by the enjoining the auction sale of the pledged stock
Deed of Sale of April 18, 1978, supra. But the subject of the contract of sale of April 18, 1978.
related stipulation—that Ricafort and Calalang Three more amendments of the complaint were
cause the mortgage of those lots in Aguinaldo's thereafter sought by Ricafort and Calalang through
favor as security for the payment of the balance of separate motions.
the price fixed in the sale of April 18, 1978—was not The first amendment sought to add a plea:
complied with. Ricafort and Calalang refused to a) for reformation of the contract of sale of April 18,
fulfill that prestation because, according to them, the 1978 to include all of the shares of stock in
deed of reconveyance of the "Saug lots" executed NADECOR of Aguinaldo, DRACOR and all their
by Aguinaldo in favor of ADECOR dated September nominees (totaling 67% of the outstanding stock in
26, 1980, was fatally defective as it did not bear the NADECOR); and
signature of Aguinaldo's wife, Helen Leontovich. For b) for a preliminary injunction against Aguinaldo to
some undisclosed reason, the latter never saw fit to prohibit him from representing himself as the
remedy the omission until very, very much later, controlling stockholder of NADECOR and
after the controversy between the parties had attempting to sell that corporation's so-called
worsened and spawned bitter litigation in various "Kingking Mining Claims" in Pantukan, Davao del
courts, as will now be briefly narrated. Norte.
CIVIL CASE No. 38117, CFI, Rizal The amendment was allowed by Order dated April
The first case was commenced on October 6, 1980. 20, 1981.
On that day Civil Case No. 38117 was instituted by The second amendment impleaded NADECOR as
Ricafort and Calalang in the then Court of First additional defendant, and prayed that it also be
Instance of Rizal. The stated cause: breach of the enjoined from offering the Kingking Mining Claims
contract of sale of April 18, 1978 by Aguinaldo's for sale. The amendment was admitted by Order
failure to make a valid transfer of the nine "Saug issued on June 25, 1981.
lots;" the prayer: that Aguinaldo's obligation to make These mining claims, by the way, are embraced in
the conveyance be deemed waived and nine (9) Lode Lease Contracts docketed as LLC-V-
correspondingly, that Ricafort and Calalang, as 908 to V-910; V-935, V-948, V-949, V-966, V-1074
vendees, be deemed discharged from their own and V-1075;2 and there is evidence on record that
obligation to pay the balance of the price, and the said claims constitute practically all the assets of
pledge of the stock purchased by them be NADECOR.
considered discharged and released. The third amendment added averments of fraud
Aguinaldo reacted by instructing a Notary Public, relative to the transfer by Aguinaldo to himself of
Wilfred Neis, to conduct the auction sale of the ADECOR shares in a foreign company, Sawyer-
pledged stock (of DRACOR; ADECOR and Adecor International, Inc. (SAICOR). This
NADECOR, supra). Atty. Neis scheduled the sale amendment was approved by Order dated February
on October 10, 1980. This gave rise to the second 5, 1982.
litigation between the parties. Preliminary Injunctions by Manila CFI (CC 135262)
CIVIL CASE NO. 135262, Manila Re Kingking Mining Claims, etc.
On October 8, 1980, Ricafort and Calalang brought Several injunctive orders were thereafter issued
suit against Aguinaldo and Notary Public Neis in the against Aguinaldo and his group by the Trial Court
Court of First Instance of Manila, which was mainly as regards the Kingking Mining Claims in
docketed as Civil Case No. 135262. They asked Davao del Norte. These resulted mainly from an
that the latter be stopped from proceeding with the Operating Agreement involving said Kingking
auction sale of the stock in question on October 10, Claims executed on March 25, 1981 between
1980, or at any other time thereafter. They also Aguinaldo, in representation of NADECOR, on the
applied for a preliminary injunction pending one hand, and a consortium made up of Black
determination of the merits of the action. Mountain, Inc., Tetra Management Corporation, and
Ricafort and Calalang later amended the complaint Energy Corporation, on the other. On March 30,
to incorporate their cause of action in Civil Case No. 1981 the Court enjoined the NADECOR Board from
ratifying that Operating Agreement of March 25, Energy Corporation, and Tetra Management
1981. Corporation. The stockholders instead approved the
On April 20, 1981, the Court stopped (a) the auction proposed operating agreement with Benguet
sale of the pledged shares of stock which had been Corporation. The directors-elect then organized the
re-scheduled by Notary Public Wilfred Neis at NADECOR Board: they elected the corporate
Aguinaldo's instance, and (b) Aguinaldo from officers headed by Calalang as chairman and
representing himself as the controlling stockholder president. The Secretary's Certificate attesting to
of NADECOR and offering its Kingking Claims for these events was in due course filed with the
sale. Securities and Exchange Commission.
On June 29, 1981, the Court issued another Order On that day, too, NADECOR, represented by the
(a) declaring Aguinaldo and the NADECOR new officers, entered into an Operating Agreement
directors guilty of contempt of court for having, with Benguet Corporation for the operation by the
despite the injunction of March 30, 1981, approved latter of the company's KINGKING MINES. This
and confirmed the Operating Agreement involving was the second such agreement involving the
the Kingking Claims entered into by NADECOR with Kingking Mines . 4
Black Mountain, etc., administering an admonition Litigation in Securities & Exchange Commission
on them, and (b) NULLIFYING said Operating The area of conflict now widened, to include
Agreement. litigation in the Securities and Exchange
And on September 15, 1981 yet another Order was Commission. Two (2) cases were instituted in that
rendered by the Trial Court, prohibiting Aguinaldo quasi-judicial agency: SEC Case No. 2143, in 1981
from voting or selling the ADECOR shares in and SEC Case No. 2878, in 1984.
Sawyer-Adecor International Corporation SEC Case No. 2143
(SAICOR), which he had caused to be transferred SEC Case No. 2143 was commenced by complaint
to his name. dated September 24, 1981 of NADECOR,
At about this time, Ricafort and Calalang perceived represented by its newly elected directors and
what they believed to be a plot by the Aguinaldo- officers (the Ricafort-Calalang Group), against
Aytona group to exclude them and SAICOR from Aguinaldo, Aytona and a certain Romeo H. Borsoto.
the management of NADECOR. It appears that The complaint alleged that despite the election of
Aguinaldo and his group had refused to convoke the the new officers at the stockholders' meeting of
stockholders of NADECOR for the annual meeting August 17, 1981, the defendants continued to
for the year 1981 which, under the by-laws 3 should fraudulently represent themselves as the legitimate
have been called on the third Monday of August. So officers of NADECOR and to exclude said officers-
on August 17, 1981, the Ricafort Group, and the elect from the exercise of their rights. The complaint
President of SAICOR, Carol Garvice, who was in also adverted to the defendants' allegedly malicious
the country at the time, went to the main offices of refusal to perform their ministerial duty to issue
NADECOR, and proceeded to hold a meeting for notices of the annual stockholders' meeting, supra.
the declared purpose of electing the directors for the Acting on the application for preliminary injunction
incoming year, and otherwise transacting corporate contained in the complaint, an order was issued on
business. Dominador Aytona, Daniel Aguinaldo's September 28, 1981 by SEC Director and Hearing
colleague and counsel, moved to postpone the Officer Sixto Villanueva (a) prohibiting the
meeting, drawing attention to a temporary defendants "from acting and representing
restraining order supposedly issued by the Superior themselves as officers of NADECOR until further
Court of California dated August 14, 1981. The orders," and (b) setting the hearing on the
majority of the stockholders then present however preliminary injunction on October 7, 1981.
voted against the postponement, opining inter alia On October 7, 1981 and various dates thereafter,
that the restraining order of the California Court was the complainants presented evidence in
not enforceable in the Philippines. Aytona then substantiation of their plea for a preliminary
questioned Garvice's qualification to take part in the injunction. Defendants refused to adduce proof of
stockholders' meeting. He was reminded that as their own. They contented themselves with
early as May, 1981, he already knew Garvice to be presenting their Answer to the Complaint, and an
the President of SAICOR. When the majority of the opposition to the application for preliminary
stockholders expressed their firm wish to continue injunction. And instead of submitting countervailing
with the meeting, Aguinaldo and Aytona walked out. evidence, they filed various motions to lift the
The stockholders then elected as directors, temporary restraining order of September 28, 1981.
Conrado Calalang, Jose Ricafort and five (5) others. They also attempted to present Calalang and
The stockholders also rejected the aforesaid Ricafort as hostile witnesses at the injunction
operating agreement of March 25, 1981 between hearing and caused issuance of subpoena towards
NADECOR, represented by the Aguinaldo-Aytona this end. Those processes were however quashed
Group, and the consortium of Black Mountain, Inc., by the Hearing Officer who ordered said defendants
instead to submit a proposal for a stipulation or a proceedings in the Supreme Court.
request for admission of the facts as to which PROCEEDINGS INSTITUTED IN SUPREME
examination of Calalang and Ricafort was being COURT
sought. The defendants ignored the order and G.R. No. 60376 (Aguinaldo, et al. v. Hon. T.P.
renewed their attempts to have Calalang and Maddela, et al. [RTC No. 135262])
Ricafort testify as their hostile witnesses. On November 20, 1981, Aguinaldo, Neis and
Then, without awaiting resolution of the application NADECOR filed with this Court a petition for
for preliminary injunction by the Hearing Officer certiorari and prohibition with application for
(who was still waiting for the defendants to submit preliminary injunction, assailing Manila CFI Judge
their evidence), said defendants brought the matter Maddela's alleged failure to act on Civil Case No.
up to the Securities and Exchange Commission en 135262. This was docketed as G.R. No. 60376.
banc (by filing with that body a petition for certiorari Petitioners Aguinaldo, et al., later filed on June 15,
and mandamus, with application for preliminary 1982, a supplemental petition:
injunction). The Hearing Officer, Director Villanueva, (1) to annul and enjoin enforcement of Judge
thereafter denied the defendants' motions to lift the Maddela's temporary restraining order dated March
restraining order of September 28, 1981. The 30, 1981—stopping the NADECOR Board (then
defendants thereupon filed a supplemental petition controlled by the Aguinaldo-Aytona Group) from
with the Commission en banc, asking that the approving the Operating Agreement with Black
Healing Officer's own restraining order of Mountain, Inc. involving the Kingking Mines; and
September 28, 1981 be itself restrained. They later (2) to nullify and set aside other adverse orders,
moved to be allowed to continue exercising dated April 20, 1981, June 25, 1981, June 26, 1981,
functions as officers of NADECOR. and Sept. 15, 1981.
CIVIL CASE NO. 143, CFI, DAVAO G.R. No. 59114 (Ricafort, et al. v. Hon. Felix Moya,
Still another action was begun at about this time. On et al. [Davao RTC CC 143])
November 6, 1981, the consortium of Black For their part, Ricafort and Calalang, together with
Mountain, Inc., Energy Corporation and Tetra NADECOR and ADECOR, filed with this Court on
Management Corporation filed a complaint in the December 17, 1981 a petition for certiorari, against
Court of First Instance of Davao, which was Judge Felix L. Moya of the Davao CFI (Branch II),
docketed as Civil Case No. 143. Named defendants Black Mountain, Inc., Tetra Management
were Benguet Corporation, NADECOR, and the Corporation and Energy Corporation. The petition
directors of NADECOR, including Ricafort and was later amended on January 26, 1982. The
Calalang. The complaint sought to enjoin them from petitioners sought annulment of three orders of
interfering with Black Mountain's possession of Judge Moya, to wit:
NADECOR's Kingking mines and recover damages. (1) the temporary restraining order issued on
The Trial Court issued a temporary restraining November 11, 1981;
Order to this effect on November 11, 1981. (2) the order of December l5, 1981, denying the
Ricafort and his co-defendants moved to dismiss petitioners' motion to dismiss the action (on the
the complaint for failure to state a cause of action. ground that the Operating Agreement of Black
They argued that the complaint contained no Mountain, Inc., et al. with NADECOR had not been
averment—in any case, it was an uncontroverted approved by stockholders holding the majority of the
fact—that NADECOR's agreement with Black capital stock as required by the Corporation Code,
Mountain, Inc., et al. for the operation of the supra); and
Kingking Mining Claims had never been approved (3) the order of January 20, 1982, denying their
by the NADECOR stockholders owning the majority motion to quash the contempt charges against
of the capital stock, although such approval was them.
required by Section 44 of the Corporation Code for Due Course to G.R. Nos. 60376 & 59114
any contract, such as the one in question, "whereby Both petitions in G.R. Nos. 60376 and 59114 were
a corporation undertakes to manage or operate all given due course in virtue of a Resolution of this
or substantially all of the business of another Court dated August 23, 1982.
corporation, whether such contracts are called After Black Mountain, Inc., filed on September 9,
service contracts, operating agreements or 1982, a motion alleging chiefly that G.R. No. 59114
otherwise . . .," and it appearing that the agreement had been rendered moot by Letter of Instructions
with Black Mountain embraced 93.5% of the total No. 1349 issued by former President Marcos on
number and area of NADECOR's mining claims and August 1, 1983, both "Kingking Claims Cases," G.R.
NADECOR had no other mining properties or No. 59114 and G.R. No. 60376, lay fallow for some
business. three years.
The Trial Court denied the motion to dismiss in an A word about said Letter of Instructions No. 1349. It
Order dated December 15,1981. advised the consortium of Black Mountain, Inc.,
All the foregoing actions in turn generated Energy Corporation, and Tetra Management
Corporation to implement the operating agreement and Benguet Corporation. The suit was chiefly
involving the NADECOR's Kingking mining property grounded on an order of preliminary injunction of
in Pantukan, Davao. It superseded Letter of the Superior Court of California enjoining several
Instructions Nos. 1210, dated March 9, 1982, specified stockholders of NADECOR from voting a
directing that said property be immediately put into large bloc of NADECOR shares owned by SAICOR,
production and that a company called "North Davao pending judicial determination of ownership of 1.2
Mining Corporation" undertake the development of million SAICOR shares of stock, registered in the
said Kingking mining property. 5 name of the late D.R. Aguinaldo. The Quezon City
G.R. No. 61377 (Aguinaldo, Aytona, Borsoto v. Court issued a temporary restraining order on
SEC, Ricafort, et al. [RE SEC Case. No. 2143]) August 16, 1985 enjoining defendants from voting
A third certiorari action involving substantially the the SAICOR shares at the annual stockholders'
same parties was commenced in this Court on meeting set on August 19, 1985, and all other
August 13, 1982. That action, docketed as G.R. No. meetings.
61377, concerned SEC Case No. 2143 in which, it The defendants, the Ricafort-Calalang Group,
will be recalled, the Aguinaldo-Aytona Group had promptly moved to dismiss the action on the ground
presented to the Commission en banc a petition and of (1) lack of jurisdiction of the Court over of the
supplemental petition impugning certain actuations subject-matter; (2) non-enforceability of the foreign
of the Hearing Officer including the latter's order of injunction, hence, failure of the complaint
temporary restraining Order. In this Court the founded thereon to state a cause of action; and (3)
Aguinaldo-Aytona Group complained of the alleged failure of plaintiffs counsel to indicate his IBP
inaction of the Securities and Exchange number. The motion was denied by Order dated
Commission en banc on their petition to nullify the October 25, 1985; and a writ of preliminary
same which had resulted to their disadvantage, and injunction subsequently issued against the
prayed for invalidation of said Hearing Officer's defendants on November 5, 1975. Ricafort and his
restraining order. co-defendants challenged that order of denial
G.R. Nos. 88895 and 88095 before the Court of Appeals by way of a petition for
Still another proceeding in connection with SEC certiorari and prohibition. SAICO filed a separate
Case No. 2143 was instituted in this Court by the petition for certiorari and prohibition against the
Aytona Group, although much, much later, in 1988. same order also before the Court of Appeals. The
That stemmed from a motion submitted by the cases were consolidated; and judgment was
Aytona Group in said SEC Case No. 2143 for a rendered thereon in due course on November 28,
preliminary injunction to stop the Ricafort-Calalang 1986, annulling and setting aside the Order of the
Group from calling and holding the annual Court a quo of October 25, 1985 and the order for a
stockholders' meeting of NADECOR on August 15, preliminary injunction of November 5, 1985.
1988 pursuant to the By-Laws. When that motion Aytona's motion for reconsideration was later
was denied, Aytona went to the Court of Appeals denied by the Court of Appeals, in a Resolution
which, however, sustained the SEC's denial of the dated January 23, 1987.
preliminary injunction. Aytona then appealed to this Aytona appealed to this Court. His appeal was
Court, his petitions being docketed here as G.R. docketed as G.R. Nos. 77274-75 (Dominador R.
Nos. 88895 and 88095. Later, Aytona moved to Aytona, as Executor of the Estate of Deceased
withdraw and dismiss that appeal, but the appeal Daniel R. Aguinaldo vs. Conrado T. Calalang, Jose
was instead denied by this Court for failure to G. Ricafort, Salvador O. Rivera, Benjamin V. Aritao,
comply with the requisites of the Rules of Court. Edgar de Castro, and Sawyer- Adecor International,
G.R. Nos. 77274-75 (Dominador R. Aytona, as Inc.)
Executor of the Estate of Deceased Daniel R. G.R. No. 75098 (Ricafort, et al. v. Hon. T. Maddela,
Aguinaldo vs. Conrado T. Calalang, et al. [RE CC et al. [Case No. 135262])
Q-45704]) One more proceeding in this Court has to be
One more proceeding on appeal involving the same mentioned: G.R. No. 75098, which was an appeal
parties was added to this Court's docket sometime by certiorari by Ricafort and Calalang against the
in 1987. This was G.R. Nos. 77274-75. It had its Order of Judge Maddela of June 10, 1986 in Civil
origin in yet another action filed by Aytona, this time Case No. 135262. Said order was issued by Judge
with the Regional Trial Court at Quezon City, on Maddela in connection with hearings that this Court
August 15, 1985. He filed the case as a stockholder instructed him to conduct, on a motion of the Aytona
of NADECOR and as executor of the estate of Group that the auction sale of the pledged stock
Daniel R. Aguinaldo, who had died in the meantime. should proceed in view of certain recent
Named as defendants were Conrado T. Calalang, developments. The instruction was contained in a
Jose G. Ricafort, Salvador O. Rivera, Benjamin V. Resolution dated July 8, 1985 in G.R. No. 60376.
Aritao, Edgar de Castro, (as officers of NADECOR), G.R. No. 75098 was later consolidated with G.R.
and Sawyer-Adecor International, Inc. (SAICOR) No. 60376 by this Court's Resolution of November
5, 1986. Benjamin V. Aritao, Edgar de Castro, and Sawyer-
Intervention by the Office of the President Adecor International, Inc.) on June 20, 1988. 7 This
It appears that the Office of the President of the Court ruled that QC RTC Case Q-45704 involved a
Philippines took some interest in the Kingking controversy arising out of intra-corporate relations
mining property of' NADECOR and issued two between and among stockholders and thus fell
letters of instructions concerning it. The first, LOI within the original and exclusive jurisdiction of the
No. 1210, dated March 9, 1982, directed that the Securities and Exchange Commission; moreover,
mining property be immediately put into production there was as yet no foreign judgment to be enforced
and that a firm known as "North Davao Mining by Philippine Courts, the petitioners' action in the
Corporation" undertake its development. The Regional Trial Court being founded on a mere
second, LOI No. 1349, dated August 1, 1983, interlocutory order.
advised Black Mountain, Inc., the Energy G.R. No. 61377
Corporation, and the Tetra Management Decision was also promulgated by the Court in G.R.
Corporation to implement their operating agreement No. 61377 on June 30, 1988, dismissing the
involving said Kingking mining property in Pantukan, petition. 8 It held that the challenged temporary
Davao. 6 restraining order of the Hearing Officer in SEC Case
FINAL DISPOSITION OF SUPREME COURT AND No. 2143, like those issued by Regional Trial
SEC CASES Courts, lapsed after twenty (20) days; that the
G.R. NOS. 60376 AND 75058 (Judgment on Securities and Exchange Commission still had
Compromise) jurisdiction and indeed the obligation to proceed
A compromise was arrived at by the parties in G.R. with the hearing on the merits of Case No. 2143 and
Nos. 60376 and 75098. It was embodied and issue appropriate orders pursuant thereto, subject
approved in a Resolution dated March 16, 1988, to review by the Court of Appeals and the Supreme
dictated at a hearing presided over by the Chairman Court; and that since petitioners (defendants) had
of the First Division, in the presence of the parties not yet finished presenting evidence on such
and their counsel. Basically the compromise matters affecting the corporate affairs of NADECOR
provided for the consummation of the deed of sale as the validity of proxy votes, alleged usurpation of
of April 18, 1978 through the compliance by the corporate powers, claims of majority status,
parties with their yet unfulfilled prestations as well issuance of notices—evidentiary issues the
as the obligations thereto related. The Resolution resolution of which is primarily lodged with the
approving said compromise declared that those Securities and Exchange Commission, the latter
contractual and related commitments should be ought to continue to hear and then decide the
fulfilled on March 21, 1988, and that thereupon, respective lights of parties in NADECOR.
"G.R. Nos. 60376 and 75098, as well as the case SEC Case No. 2143
thereto related, Civil Case No. 135262 of the On July 31, 1989, a Joint Manifestation was
Regional Trial Court of Manila, shall be ordered presented by the Ricafort Group and the Aytona
dismissed, closed, and terminated." Group in SEC Case No. 2143, briefly to the
On March 23, 1988, this Court promulgated another following effect:
Resolution declaring the compromise approved by 1) the Aytona Group will no longer question the
the Resolution of March 16, 1988 to have been fully 1989 NADECOR annual stockholders' meeting and
executed to all the parties' satisfaction. It also the election of directors on that occasion, as well as
ordered the Register of Deeds of Davao to cancel the organizational meeting of the board of directors;
the transfer certificates of title covering the nine (9) 2) both groups waive any and all claims for
"Saug lots" still in the name of Daniel R. Aguinaldo damages they may have against each other in the
and to issue new ones in the name of ADECOR. case;
Finally, the Resolution declared "G.R. No. 75098 3) the Aytona Group will not move for
DISMISSED, CLOSED AND TERMINATED, and reconsideration, appeal, or in any way question the
G.R. No. 60376 and Civil Case No. 135262 decision which may be rendered in the case
DISMISSED only insofar as they involve the subject pursuant to the joint manifestation.
matter of this compromise agreement." Later, Acting thereon, Hearing Officer Alberto P. Atas
however, the parties having reached a complete rendered an Order on August 9, 1989, ruling that
settlement, the case was ordered dismissed by this 1. (The Commission had decided to) . . . recognize
Court, by Resolution of August 14, 1989. (Rollo, and affirm the validity of the annual stockholders'
G.R. Nos. 60376 and 75098, p. 2298) meeting of Nationwide Development Corporation
G.R. Nos. 77274-75 held on August 17, 1981, the election of petitioners
Decision was rendered in G.R. Nos. 77274-75 as directors of the Corporation in that meeting, and
(Dominador R. Aytona, as Executor of the Estate of the validity of the organizational meeting of such
Deceased Daniel R. Aguinaldo vs. Conrado T. board electing the officers of the corporation; . . .
Calalang, Jose G. Ricafort, Salvador O. Rivera, 2. . . . all claims for damages of the parties against
each other related to this action are hereby declaration of the invalidity of said Agreement of
considered waived," and March 25, 1981.
3. Case No. 2143 was "considered CLOSED." 1. On March 30, 1981, in Civil Case No. 135262,
SEC Case No. 2878 the Manila Trial Court enjoined the NADECOR
To complete the picture, mention must be made of Board (controlled by the Aguinaldo-Aytona Group)
one other action in the Securities and Exchange from ratifying this Operating Agreement.
Commission between the same parties, SEC. Case 2. On April 20, 1981, the same Court inter alia
No. 2878. That case was filed by the Ricafort- stopped Aguinaldo from representing himself as the
Calalang Group to enjoin the Aytona Group from controlling stockholder of NADECOR and offering
continuing to act and represent themselves as its Kingking Claims for sale.
Directors and/or officers of NADECOR. The Ricafort 3. On June 29, 1981, the Court issued another
Group alleged that they were duly elected as Order (a) declaring Aguinaldo and the NADECOR
directors of NADECOR at the annual stockholders' directors guilty of contempt of court for having,
meeting on August 20, 1984, and later, elected as despite the injunction of March 30, 1981, approved
officers of the firm by the directors; that the meeting and confirmed the Operating Agreement involving
was adjourned by Calalang, as Chairman, in view of the Kingking Claims entered into by NADECOR with
the objections by Aytona (presenting TRO by RTC, Black Mountain, etc., administering an admonition
QC) to the voting of SAICO's 7,000 shares at the on them, and (b) NULLIFYING said Operating
election of directors; that after adjournment, Agreement.
however, Aytona and his group elected themselves 4. The Ricafort-Calalang Group validly elected
officers of NADECOR. A temporary restraining directors at the annual stockholders meeting of
order was issued by Hearing Officer Emmanuel NADECOR on August 17, 1981; and said directors
Sison, followed, after presentation of evidence, by a thereafter validly elected the officers of the
preliminary injunction, against the Aytona Group. corporation at the organizational meeting of the
Aytona moved for reconsideration of the Order of board.
injunction, dated October 14, 1985, but his motion 5. The same group (Ricafort-Calalang) had been
was denied by order dated January 17, 1986. validly re-elected since then, in 1985, 1986, 1987.
On May 25, 1989, the decision in said SEC Case An attempt of the Aguinaldo-Aytona group to have
No. 2878 was handed down by Hearing Officer its members elected as directors at the
Felipe S. Tongco. Tongco ruled that the only issue stockholders' meeting of August 19, 1985 was
concerned the validity of the adjournment of the declared null and void.
meeting of August 20, 1984 by Calalang; that as to 6. At the annual meeting of August 17, 1981, too,
the question regarding the election of Aytona, et al. the NADECOR stockholders rejected the operating
as directors after said adjournment, the same had agreement executed on March 25, 1981 by
been mooted by the subsequent and indisputably NADECOR, then acting through the Aguinaldo-
valid election of Calalang and his group in 1986 and Aytona Group, and the Black Mountain Consortium,
1987; that the validity of the acts of the Aytona supra. The stockholders also approved the
Group as pseudo directors and officers had to be proposed Agreement with Benguet Corporation for
determined; that the evidence sufficiently the operation by the latter of the company's
established that the annual stockholders' meeting of KINGKING MINES. The agreement with Benguet
August 20, 1984 had been validly adjourned by Corporation was subsequently signed and
Calalang; that the election of the Aytona Group as executed.
directors following the adjournment was therefore 7. On January 22, 1987, President Corazon C.
void ab initio; and that group's acts as directors and Aquino issued Memorandum Order No. 69, entitled
officers of NADECOR were also null and void. "RESCINDING LETTER OF INSTRUCTION NOS.
Remaining Proceedings and Resolution Thereof 1210 AND 1349, DATED MARCH 9, 1982 AND
Thus, the only proceedings left undecided are Civil AUGUST 1, 1983," treating directly of the "approved
Case No. 143 of the Regional Trial Court of Davao, operating agreement involving the Kingking mining
and G.R. No. 59114, related to Case No. 143. property in Pantukan, Davao" of the consortium
The main issue in these two (2) cases is the validity composed of Black Mountain, Inc., Energy
of the Operating Agreement relative to the Kingking Corporation, and Tetra Management Corporation . 9
Mines entered into on March 25, 1981 between The memorandum reads as follows:
NADECOR, then represented by the Aguinaldo- Letter of Instructions Nos. 1210, dated March 9,
Aytona Group, and the consortium composed of 1982, directing that the Kingking mining property in
Black Mountain, Inc., Tetra Management Pantukan, Davao, covered by mining lease
Corporation, and Energy Corporation. contracts issued in the name of the Nationwide
The facts and considerations hereunder Development Corporation be immediately put into
summarized, developed beyond dispute in the production and that the North Davao Mining
various legal proceedings above surveyed, dictate a Corporation undertake the development of the
Kingking mining property; and Letter of Instructions but explicitly rejected and disowned by them at a
No. 1349, dated August 1, 1983, advising the Black meeting duly convoked, said stockholders thereafter
Mountain, Inc., the Energy Corporation, and the approving an operating agreement with Benguet
Tetra Management Corporation to implement the Corporation; the agreement was sought to be
approved operating agreement involving the vindicated and enforced by individuals who no
Kingking mining property in Pantukan, Davao, are longer represented the majority of the stockholders
hereby rescinded and revoked. of NADECOR, over the objection and against the
This Memorandum Order takes effect immediately. wishes of the legitimate majority; the authority
This memorandum Order was sent on February 18, granted to the consortium (Black Mountain, Inc.,
1987, by Presidential Staff Director Melquiades T. Energy Corporation, and Tetra Management
de la Cruz to Director Benjamin Gonzales of the Corporation) to implement the agreement of March
Bureau of Mines and Geo-Sciences, and Carlos G. 25, 1981 was rescinded and revoked by the Office
Dominguez, Secretary of Natural Resources, of the President of the Philippines; and one of the
evidently for implementation. These acts, according companies in said consortium is now, admittedly, no
to Ricafort and Calalang , 10 "rectified the "great longer capable on account of bankruptcy of
prejudice" caused to . . . (them) by the 'unjust complying with its contractual commitments—it is
awards by then President Marcos of the operation impossible to accord the agreement any validity or
of the Kingking mines to "crony" North Davao effect whatsoever.
Mining Corporation which had no operating It thus clearly appears, not only that upon purely
agreement from the claim owners and then to legal considerations, the operating agreement of
"cronies" Black Mountain, Inc., et al. which had a March 25, 1981 is, if not outrightly void,
spurious operating agreement from Aguinaldo with unenforceable for want of requisite valid ratification
a court-cancelled board of directors' ratification and and conferred upon private respondents no
no stockholders' approval . . ." actionable, vindicable rights, but also that, from a
It was evidently on account of Memorandum Order practical standpoint, any issue about said
No. 69 of President Aquino that NADECOR (the respondents' rights under the agreement has been
Ricafort-Calalang Group) finally succeeded in mooted by supervening events effectively
getting possession of the mines; this, sometime in precluding their exercise in any case.
December, 1989, NADECOR was granted authority WHEREFORE, the petition is GRANTED. Civil
by the Secretary of Natural Resources "to enter the Case No. 143 of the Regional Trial Court of Davao
area" and "proceed with exploration and is DISMISSED, and the restraining order of
development activities" subject to certain specified November 11, 1981 issued therein, if still extant, is
conditions, one of which was that NADECOR itself DISSOLVED. Costs against private respondents.
shall conduct said exploration and development SO ORDERED.
activities and not contract said activities to an
operator . . ."
8. As admitted by the respondents, Black Mountain,
Inc. "ceased operations and became bankrupt years G.R. No. 129459 September 29, 1998
before Marcos was booted out of office."
9. The Operating Agreement with the Black SAN JUAN STRUCTURAL AND STEEL
Mountain Consortium of March 25, 1981 was never FABRICATORS, INC., petitioner, vs.
ratified by the NADECOR stockholders; indeed, it
was explicitly rejected by said stockholders. COURT OF APPEALS, MOTORICH SALES
Considering that the Kingking Mines comprise all or CORPORATION, NENITA LEE GRUENBERG,
substantially all the assets of NADECOR, the
ACL DEVELOPMENT CORP. and JNM REALTY
operating agreement of March 25, 1981 had to be
ratified by the stockholders in order to be valid and AND DEVELOPMENT CORP., respondents.
effective. This, in accordance with Section 44 of the
Corporation Code. 11 That no such ratification was FACTS: In 1989, San Juan Structural and Steel
ever given constitutes yet another reason to Fabricators, Inc. (San Juan) alleged that it entered
invalidate the same. into a contract of sale with Motorich Sales
Under these circumstances—the agreement Corporation (Motorich) through the latter’s treasurer,
executed on March 25, 1981 was entered into in Nenita Gruenberg. The subject of the sale was a
defiance of valid orders of a court of competent
parcel of land owned by Motorich. San Juan
jurisdiction and was in fact subsequently nullified by
it; it was entered into against the wishes of the advanced P100k to Nenita as earnest money.
majority of the stockholders and directors and in
truth, was not only not ratified by the majority of said On the day agreed upon on which Nenita was
stockholders as required by the Corporation Code, supposed to deliver the title of the land to Motorich,
Nenita did not show up. Nenita and Motorich did not generally has been described as "to receive and
heed the subsequent demand of San Juan to keep the funds of the corporation, and to disburse
comply with the contract hence San Juan sued them in accordance with the authority given him by
Motorich. Motorich, in its defense, argued that it is the board or the properly authorized officers."
not bound by the acts of its treasurer, Nenita, since
her act in contracting with San Juan was not Neither was such real estate sale shown to be a
authorized by the corporate board. normal business activity of Motorich. The primary
purpose of Motorich is marketing, distribution,
San Juan raised the issue that Nenita was actually export and import in relation to a general
the wife of the President of Motorich; that Nenita merchandising business. Unmistakably, its treasurer
and her husband owns 98% of the corporation’s is not cloaked with actual or apparent authority to
capital stocks; that as such, it is a close corporation buy or sell real property, an activity which falls way
and that makes Nenita and the President as beyond the scope of her general authority.
principal stockholders who do not need any
authorization from the corporate board; that in this
case, the corporate veil may be properly pierced.
AF Realty & Development Inc. vs. Dieselman
ISSUE: Whether or not the contract Gruenberg Freight Services Co.
signed is binding on the two corporations? [GR 111448, 16 January 2002]
Third Division, Sandoval-Gutierrez (J): 4 concur
HELD: No. Unless duly authorized, a treasurer,
whose powers are limited, cannot bind the Facts: On 10 May 1988, Manuel C. Cruz, Jr., a
corporation in a sale of its assets. Furthermore, the member of the board of directors of Dieselman
Freight Services Co., issued a letter denominated
Court has also recognized the rule that "persons
as "Authority To Sell Real Estate" to Cristeta N.
dealing with an assumed agent, whether the Polintan, a real estate broker of the CNP Real
assumed agency be a general or special one bound Estate Brokerage. Cruz, Jr. authorized Polintan "to
at their peril, if they would hold the principal liable, look for a buyer/buyers and negotiate the sale" of
to ascertain not only the fact of agency but also the the lot at P3,000.00 per square meter, or a total of
nature and extent of authority, and in case either is P6,282,000.00. Cruz, Jr. has no written authority
controverted, the burden of proof is upon them to from Dieselman to sell the lot. In turn, Cristeta
Polintan, through a letter dated 19 May 1988,
establish it (Harry Keeler v. Rodriguez, 4 Phil. 19)."
authorized Felicisima ("Mimi") Noble to sell the
same lot. Felicisima Noble then offered for sale the
In the case at bar, Respondent Motorich
property to AF Realty & Development, Inc. (AF
categorically denies that it ever authorized Nenita Realty) at P2,500.00 per square meter. Zenaida
Gruenberg, its treasurer, to sell the subject parcel of Ranullo, board member and vice-president of AF
land. Consequently, petitioner had the burden of Realty, accepted the offer and issued a check in the
proving that Nenita Gruenberg was in fact amount of P300,000.00 payable to the order of
authorized to represent and bind Motorich in the Dieselman. Polintan received the check and signed
an "Acknowledgment Receipt" indicating that the
transaction. Petitioner failed to discharge this
amount of P300,000.00 represents the partial
burden. Its offer of evidence before the trial court payment of the property but refundable within two
contained no proof of such authority. It has not weeks should AF Realty disapprove Ranullo's
shown any provision of said respondent's articles of action on the matter. On 29 June 1988, AF Realty
incorporation, bylaws or board resolution to prove confirmed its intention to buy the lot. Hence, Ranullo
that Nenita Gruenberg possessed such power. asked Polintan for the board resolution of
Dieselman authorizing the sale of the property.
That Nenita Gruenberg is the treasurer of Motorich However, Polintan could only give Ranullo the
does not free petitioner from the responsibility of original copy of TCT 39849, the tax declaration and
tax receipt for the lot, and a photocopy of the
ascertaining the extent of her authority to represent
Articles of Incorporation of Dieselman. On 2 August
the corporation. Petitioner cannot assume that she, 1988, Manuel F. Cruz, Sr., president of Dieselman,
by virtue of her position, was authorized to sell the acknowledged receipt of the said P300,000.00 as
property of the corporation. Selling is obviously "earnest money" but required AF Realty to finalize
foreign to a corporate treasurer's function, which the sale at P4,000.00 per square meter. AF Realty
replied that it has paid an initial down payment of between Dieselman and Midas is valid, there being
P300,000.00 and is willing to pay the balance. no bad faith on the part of the latter. The Court of
However, on 13 August 1988, Mr. Cruz, Sr. Appeals then declared Dieselman and Cruz, Jr.
terminated the offer and demanded from AF Realty jointly and severally liable to AF Realty for
the return of the title of the lot earlier delivered by P100,000.00 as moral damages; P100,000.00 as
Polintan. Claiming that there was a perfected exemplary damages; and P100,000.00 as attorney's
contract of sale between them, AF Realty filed with fees. On 5 August 1993, the Court of Appeals, upon
the Regional Trial Court, Branch 160, Pasig City a motions for reconsideration filed by the parties,
complaint for specific performance (Civil Case promulgated an Amending Decision, in the sense
56278) against Dieselman and Cruz, Jr.. The that only Cruz, should be made liable to pay AF
complaint prayed that Dieselman be ordered to Realty the damages and attorney's fees awarded
execute and deliver a final deed of sale in favor of therein, plus the amount of P300,000.00 unless, in
AF Realty." In its amended complaint, AF Realty the case of the said P300,000.00, the same is still
asked for payment of P1,500,000.00 as deposited with the Court which should be restituted
compensatory damages; P400,000.00 as attorney's to AF Realty. AF Realty filed the petition for review
fees; and P500,000.00 as exemplary damages. In on certiorari.
its answer, Dieselman alleged that there was no
meeting of the minds between the parties in the sale Issue: Whether there was a perfected contract of
of the property and that it did not authorize any sale involving the Dieselman real property in favor
person to enter into such transaction on its behalf. of AF Realty.
Meanwhile, on 30 July 1988, Dieselman and Midas
Development Corporation (Midas) executed a Deed
Held: Section 23 of the Corporation Code expressly
of Absolute Sale of the same property. The agreed provides that the corporate powers of all
price was P2,800.00 per square meter. Midas
corporations shall be exercised by the board of
delivered to Dieselman P500,000.00 as down
directors. Just as a natural person may authorize
payment and deposited the balance of
another to do certain acts in his behalf, so may the
P5,300,000.00 in escrow account with the PCIBank.
board of directors of a corporation validly delegate
Constrained to protect its interest in the property, some of its functions to individual officers or agents
Midas filed on 3 April 1989 a Motion for Leave to appointed by it. Thus, contracts or acts of a
Intervene in Civil Case 56278. Midas alleged that it
corporation must be made either by the board of
has purchased the property and took possession
directors or by a corporate agent duly authorized by
thereof, hence Dieselman cannot be compelled to
the board. Absent such valid
sell and convey it to AF Realty. The trial court
delegation/authorization, the rule is that the
granted Midas' motion. After trial, the lower court declarations of an individual director relating to the
rendered the Decision holding that the acts of Cruz, affairs of the corporation, but not in the course of, or
Jr. bound Dieselman in the sale of the lot to AF
connected with, the performance of authorized
Realty. Consequently, the perfected contract of sale
duties of such director, are held not binding on the
between Dieselman and AF Realty bars Midas'
corporation. Herein, Cruz has no written authority
intervention. The trial court also held that Midas
from the board of directors of Dieselman to sell or to
acted in bad faith when it initially paid Dieselman negotiate the sale of the lot, much less to appoint
P500,000.00 even without seeing the latter's title to other persons for the same purpose. Cruz's lack of
the property. Moreover, the notarial report of the
such authority precludes him from conferring any
sale was not submitted to the Clerk of Court of the
authority to Polintan involving the subject realty.
Quezon City RTC and the balance of P5,300,000.00
Necessarily, neither could Polintan authorize
purportedly deposited in escrow by Midas with a Felicisima Noble. Clearly, the collective acts of
bank was not established. Dieselman was ordered Cruz, Polintan and Noble cannot bind Dieselman in
to execute and deliver to AF Realty the final deed of
the purported contract of sale.
sale of the property covered by TCT 39849 of the
Registry of Deed of Rizal, Metro Manila District II,
including the improvements thereon, and ordering
Dieselman to pay AF Realty attorney's fees in the
amount of P50,000.00 and to pay the costs. Atrium Management Corporation vs. Court of
Dissatisfied, all the parties appealed to the Court of Appeals
Appeals. In its Decision dated 10 December 1992, [GR 109491, 28 February 2001], also De Leon vs.
the Court of Appeals reversed the judgment of the Court of Appeals [GR 121894]
trial court holding that since Cruz, Jr. was not First Division, Pardo (J): 4 concur
authorized in writing by Dieselman to sell the
subject property to AF Realty, the sale was not
perfected; and that the Deed of Absolute Sale
Facts: Hi-Cement Corporation through its corporate it was for securing a loan to finance the activities of
signatories, Lourdes M. de Leon, treasurer, and the the corporation, hence, not an ultra vires act. An
late Antonio de las Alas, Chairman, issued checks ultra vires act is one committed outside the object
in favor of E.T. Henry and Co. Inc., as payee. E.T. for which a corporation is created as defined by the
Henry and Co., Inc., in turn, endorsed the four law of its organization and therefore beyond the
checks to Atrium Management Corporation for power conferred upon it by law" The term "ultra
valuable consideration. Upon presentment for vires" is "distinguished from an illegal act for the
payment, the drawee bank dishonored all four former is merely voidable which may be enforced by
checks for the common reason "payment stopped". performance, ratification, or estoppel, while the
On 3 January 1983, Atrium Management latter is void and cannot be validated.
Corporation filed with the Regional Trial Court,
Manila an action for collection of the proceeds of Issue [2]: Whether Lourdes M. de Leon and
four postdated checks in the total amount of P2 Antonio de las Alas were personally liable for the
million, after its demand for payment of the value of checks issued as corporate officers and authorized
the checks was denied. After due proceedings, on signatories of the check.
20 July 1989, the trial court rendered a decision
ordering Lourdes M. de Leon, her husband Rafael Held [2]: Personal liability of a corporate director,
de Leon, E.T. Henry and Co., Inc. and Hi-Cement trustee or officer along (although not necessarily)
Corporation to pay Atrium jointly and severally, the
with the corporation may so validly attach, as a rule,
amount of P2 million corresponding to the value of
only when: (1) He assents (a) to a patently unlawful
the four checks, plus interest and attorney's fees.
act of the corporation, or (b) for bad faith or gross
On appeal to the Court of Appeals, on 17 March
negligence in directing its affairs, or (c) for conflict of
1993, the Court of Appeals promulgated its decision interest, resulting in damages to the corporation, its
modifying the decision of the trial court, absolving
stockholders or other persons; (2) He consents to
Hi-Cement Corporation from liability and dismissing
the issuance of watered down stocks or who, having
the complaint as against it. The appellate court
knowledge thereof, does not forthwith file with the
ruled that: (1) Lourdes M. de Leon was not
corporate secretary his written objection thereto; (3)
authorized to issue the subject checks in favor of He agrees to hold himself personally and solidarily
E.T. Henry, Inc.; (2) The issuance of the subject liable with the corporation; or (4) He is made, by a
checks by Lourdes M. de Leon and the late Antonio
specific provision of law, to personally answer for
de las Alas constituted ultra vires acts; and (3) The
his corporate action." Herein, Lourdes M. de Leon
subject checks were not issued for valuable
and Antonio de las Alas as treasurer and Chairman
consideration. Hence, Atrium filed the petition.
of Hi-Cement were authorized to issue the checks.
However, Ms. de Leon was negligent when she
Issue [1]: Whether the issuance of the checks was signed the confirmation letter requested by Mr. Yap
an ultra vires act. of Atrium and Mr. Henry of E.T. Henry for the
rediscounting of the crossed checks issued in favor
Held [1]: The record reveals that Hi-Cement of E.T. Henry. She was aware that the checks were
Corporation issued the four (4) checks to extend strictly endorsed for deposit only to the payee's
financial assistance to E.T. Henry, not as payment account and not to be further negotiated. What is
of the balance of the P30 million pesos cost of more, the confirmation letter contained a clause that
hydro oil delivered by E.T. Henry to Hi-Cement. was not true, that is, "that the checks issued to E.T.
Why else would petitioner de Leon ask for Henry were in payment of Hydro oil bought by Hi-
counterpart checks from E.T. Henry if the checks Cement from E.T. Henry". Her negligence resulted
were in payment for hydro oil delivered by E.T. in damage to the corporation. Hence, Ms. de Leon
Henry to Hi-Cement? Hi-Cement, however, may be held personally liable therefor.
maintains that the checks were not issued for
consideration and that Lourdes and E.T. Henry
engaged in a "kiting operation" to raise funds for
E.T. Henry, who admittedly was in need of financial
assistance. There was no sufficient evidence to China Banking Corporation vs. Court of Appeals
show that such is the case. Lourdes M. de Leon is [GR 117604, 26 March 1997]
the treasurer of the corporation and is authorized to First Division, Kapunan (J): 4 concur
sign checks for the corporation. At the time of the
issuance of the checks, there were sufficient funds Facts: On 21 August 1974, Galicano Calapatia, Jr.,
in the bank to cover payment of the amount of P2 a stockholder of Valley Golf & Country Club, Inc.
million pesos. Thus, the act of issuing the checks (VGCCI), pledged his Stock Certificate 1219 to
was well within the ambit of a valid corporate act, for China Banking Corporation (CBC). On 16
September 1974, CBC wrote VGCCI requesting that 1990, CBC filed a complaint with the Securities and
the pledge agreement be recorded in its books. In a Exchange Commission (SEC) for the nullification of
letter dated 27 September 1974, VGCCI replied that the sale of Calapatia's stock by VGCCI; the
the deed of pledge executed by Calapatia in CBC's cancellation of any new stock certificate issued
favor was duly noted in its corporate books. On 3 pursuant thereto; for the issuance of a new
August 1983, Calapatia obtained a loan of certificate in petitioner's name; and for damages,
P20,000.00 from CBC, payment of which was attorney's fees and costs of litigation. On 3 January
secured by the pledge agreement still existing 1992, SEC Hearing Officer Manuel P. Perea
between Calapatia and CBC. Due to Calapatia's rendered a decision in favor of VGCCI, stating in the
failure to pay his obligation, CBC, on 12 April 1985, main that considering that the said share is
filed a petition for extrajudicial foreclosure before delinquent, VGCCI had valid reason not to transfer
Notary Public Antonio T. de Vera of Manila, the share in the name of CBC in the books of
requesting the latter to conduct a public auction sale VGCCI until liquidation of delinquency.
of the pledged stock. On 14 May 1985, CBC Consequently, the case was dismissed. On 14 April
informed VGCCI of the foreclosure proceedings and 1992, Hearing Officer Perea denied CBC's motion
requested that the pledged stock be transferred to for reconsideration. CBC appealed to the SEC en
its name and the same be recorded in the corporate banc and on 4 June 1993, the Commission issued
books. However, on 15 July 1985, VGCCI wrote an order reversing the decision of its hearing officer;
CBC expressing its inability to accede to CBC's holding that CBC has a prior right over the pledged
request in view of Calapatia's unsettled accounts share and because of pledgor's failure to pay the
with the club. Despite the foregoing, Notary Public principal debt upon maturity, CBC can proceed with
de Vera held a public auction on 17 September the foreclosure of the pledged share; declaring that
1985 and CBC emerged as the highest bidder at the auction sale conducted by VGCCI on 10
P20,000.00 for the pledged stock. Consequently, December 1986 is declared NULL and VOID; and
CBC was issued the corresponding certificate of ordering VGCCI to issue another membership
sale. On 21 November 1985, VGCCI sent Calapatia certificate in the name of CBC. VGCCI sought
a notice demanding full payment of his overdue reconsideration of the order. However, the SEC
account in the amount of P18,783.24. Said notice denied the same in its resolution dated 7 December
was followed by a demand letter dated 12 1993. The sudden turn of events sent VGCCI to
December 1985 for the same amount and another seek redress from the Court of Appeals. On 15
notice dated 22 November 1986 for P23,483.24. On August 1994, the Court of Appeals rendered its
4 December 1986, VGCCI caused to be published decision nullifying and setting aside the orders of
in the newspaper Daily Express a notice of auction the SEC and its hearing officer on ground of lack of
sale of a number of its stock certificates, to be held jurisdiction over the subject matter and,
on 10 December 1986 at 10:00 a.m. Included consequently, dismissed CBC's original complaint.
therein was Calapatia's own share of stock (Stock The Court of Appeals declared that the controversy
Certificate 1219). Through a letter dated 15 between CBC and VGCCI is not intra-corporate;
December 1986, VGCCI informed Calapatia of the nullifying the SEC orders and dismissing CBC’s
termination of his membership due to the sale of his complaint. CBC moved for reconsideration but the
share of stock in the 10 December 1986 auction. On same was denied by the Court of Appeals in its
5 May 1989, CBC advised VGCCI that it is the new resolution dated 5 October 1994. CBC filed the
owner of Calapatia's Stock Certificate 1219 by petition for review on certiorari.
virtue of being the highest bidder in the 17
September 1985 auction and requested that a new Issue: Whether CBC is bound by VGCCI's by-laws.
certificate of stock be issued in its name. On 2
March 1990, VGCCI replied that "for reason of Held: In order to be bound, the third party must
delinquency" Calapatia's stock was sold at the
have acquired knowledge of the pertinent by-laws at
public auction held on 10 December 1986 for
the time the transaction or agreement between said
P25,000.00. On 9 March 1990, CBC protested the
third party and the shareholder was entered into.
sale by VGCCI of the subject share of stock and
Herein, at the time the pledge agreement was
thereafter filed a case with the Regional Trial Court executed. VGCCI could have easily informed CBC
of Makati for the nullification of the 10 December of its by-laws when it sent notice formally
1986 auction and for the issuance of a new stock
recognizing CBC as pledgee of one of its shares
certificate in its name. On 18 June 1990, the
registered in Calapatia's name. CBC's belated
Regional Trial Court of Makati dismissed the
notice of said by-laws at the time of foreclosure will
complaint for lack of jurisdiction over the subject
not suffice. By-laws signifies the rules and
matter on the theory that it involves an intra- regulations or private laws enacted by the
corporate dispute and on 27 August 1990 denied corporation to regulate, govern and control its own
CBC's motion for reconsideration. On 20 September
actions, affairs and concerns and its stockholders or Inc., (Concord for brevity), a domestic corporation,
members and directors and officers with relation filed an affidavit-complaint with the Quezon City
thereto and among themselves in their relation to it. Prosecutors Office, charging Vic Ang Siong with
In other words, by-laws are the relatively permanent violation of B.P. Blg. 22. Docketed by the prosecutor
and continuing rules of action adopted by the as I.S. No. 93-15886, the complaint alleged that a
corporation for its own government and that of the check for the amount of P83,550,000.00, issued by
individuals composing it and having the direction, Vic Ang Siong in favor of Concord, was dishonored
management and control of its affairs, in whole or in when presented for encashment.
part, in the management and control of its affairs Vic Ang Siong sought the dismissal of the
and activities. The purpose of a by-law is to regulate case on two grounds: First, that petitioner had no
the conduct and define the duties of the members authority to file the case on behalf of Concord, the
towards the corporation and among themselves. payee of the dishonored check, since the firms
They are self-imposed and, although adopted board of directors had not empowered him to act on
pursuant to statutory authority, have no status as its behalf. Second, he and Concord had already
public law. Therefore, it is the generally accepted agreed to amicably settle the issue after he made a
rule that third persons are not bound by by-laws, partial payment of P19,000,000.00 on the
except when they have knowledge of the provisions dishonored check.
either actually or constructively. For the exception to The City Prosecutor dismissed the said
the general accepted rule that third persons are not case since petitioner lacked the requisite authority
bound by by-laws to be applicable and binding upon to initiate the criminal complaint for and on
the pledgee, knowledge of the provisions of the Concords behalf and that Concord and Vic Ang
VGCCI By-laws must be acquired at the time the Siong had already agreed upon the payment of the
pledge agreement was contracted. Knowledge of latter’s balance on the dishonored check. Petitioner
said provisions, either actual or constructive, at the appealed the dismissal of his complaint by the City
time of foreclosure will not affect pledgee's right Prosecutor to the Chief State Prosecutor. The Chief
over the pledged share. Article 2087 of the Civil State Prosecutor dismissed the appeal for having
Code provides that it is also of the essence of these been filed out of time. Petitioner then filed the Civil
contracts that when the principal obligation Case for mandamus with the Regional Trial Court of
becomes due, the things in which the pledge or Quezon City to compel the Chief State Prosecutor
mortgage consists maybe alienated for the payment to file or cause the filing of an information charging
to the creditor. Further, VGCCI's contention that Vic Ang Siong with violation of B.P. Blg. 22. Said
CBC is duty-bound to know its by-laws because of mandamus was later on denied. Hence, the instant
Article 2099 of the Civil Code which stipulates that petition.
the creditor must take care of the thing pledged with
the diligence of a good father of a family, fails to Issue:
convince. CBC was never informed of Calapatia's W/N Respondent Chief State Prosecutor in
unpaid accounts and the restrictive provisions in refusing to order the filing of an information for
VGCCI's by-laws. Furthermore, Section 63 of the violation of B.P. Blg. 22 against Vic Ang Siong acted
Corporation Code which provides that "no shares of without or in excess of jurisdiction or with grave
stock against which the corporation holds any abuse of discretion.
unpaid claim shall be transferable in the books of
the corporation" cannot be utilized by VGCCI. The Ruling:
term "unpaid claim" refers to "any unpaid claim No.
arising from unpaid subscription, and not to any A minority stockholder and member of the
indebtedness which a subscriber or stockholder board has no power or authority to sue on the
may owe the corporation arising from any other corporation’s behalf. Nor can we uphold this as a
transaction." Herein, the subscription for the share derivative suit, since it is required that the minority
in question has been fully paid as evidenced by the stockholder suing for and on behalf of the
issuance of Membership Certificate 1219. What corporation must allege in his complaint that he is
Calapatia owed the corporation were merely the suing on a derivative cause of action on behalf of
monthly dues. Hence, Section 63 does not apply. the corporation and all other stockholders similarly
situated who may wish to join him in the suit. There
is now showing that petitioner has complied with the
foregoing requisites.
Tam Wing Tak VS. Makasiar It is not disputed in the instant case that
Facts: Concord, a domestic corporation, was the payee of
On November 11, 1992, petitioner, in his the bum check, not petitioner. Therefore, it is
capacity as director of Concord-World Properties, Concord, as payee of the bounced check, which is
the injured party. Since petitioner was neither a
payee nor a holder of the bad check, he had neither of action. The trial court denied said petitioner’s
the personality to sue nor a cause of action against motion in which made petitioner to file a petition for
Vic Ang Siong. Under Section 36 of the Corporation certiorari and prohibition with the CA. The CA
Code, read in relation to Section 23, it is clear that however dismissed said petition on the ground that
where a corporation is an injured party, its power to the verification and certification in the petition, under
sue is lodged with its board of directors or trustees. the signature of Lorenzo Balbin, Jr., was made
Note that petitioner failed to show any proof that he without authority. Hence, the instant petition.
was authorized or deputized or granted specific
powers by Concords board of director to sue Victor Issue:
Ang Siong for and on behalf of the firm. Clearly, W/N an authorization from petitioner’s
petitioner as a minority stockholder and member of Board of Directors is still required in order for its
the board of directors had no such power or resident manager to institute or commence a legal
authority to sue on Concords behalf. Nor can we action for and in behalf of the corporation.
uphold his act as a derivative suit. For a derivative
suit to prosper, it is required that the minority Ruling:
stockholder suing for and on behalf of the Yes, however in this case the merits of
corporation must allege in his complaint that he is petitioner’s case should be considered special
suing on a derivative cause of action on behalf of circumstances or compelling reasons that justify
the corporation and all other stockholders similarly tempering the requirement in regard to the
situated who may wish to join him in the suit. There certificate of non-forum shopping. In Loyala,
is no showing that petitioner has complied with the Roadway, and Uy, the Court excused non-
foregoing requisites. It is obvious that petitioner has compliance with the requirement as to the certificate
not shown any clear legal right which would warrant of non-forum shopping. With more reason should
the overturning of the decision of public we allow the instant petition since petitioner herein
respondents to dismiss the complaint against Vic did submit a certification on non-forum shopping,
Ang Siong. A public prosecutor, by the nature of his failing only to show proof that the signatory was
office, is under no compulsion to file a criminal authorized to do so. That petitioner subsequently
information where no clear legal justification has submitted a secretary’s certificate attesting Balbin
been shown, and no sufficient evidence of guilt nor was authorized to file an action on behalf of
prima facie case has been presented by the petitioner likewise mitigates this oversight.
petitioner. No reversible error may be attributed to A corporation, such as petitioner, has no
the court a quo when it dismissed petitioners special power except those expressly conferred on it by the
civil action for mandamus. Corporation Code and those that are implied or
incidental to its existence. In turn, a corporation
exercises said powers through its board of directors
and / or its duly authorized officers and agents.
Shipside Inc. VS. CA (2001) Thus, it has been observed that the power of a
corporation to sue and be sued in any court is
Facts: lodged with the board of directors that exercises its
Certificates of Title were issued in favor of corporate powers. In turn, physical acts of the
Rafael Galvez, over four parcels of land. Lots 1 and corporation, like the signing if documents, can be
4 were conveyed in favor of Mamaril et. al. which performed only by natural persons duly authorized
subsequently were sold to Lepanto Consolidated for the purpose by corporate by-laws or by a
Mining Company. Meanwhile, CFI of Bulacan specific act of the board of directors.
ordered the cancellation of the registration of said
lands and held that the titles were null and void.
Lepanto Consolidated Mining however sold Lots 1 SSS vs Coa
and 4 to herein petitioner, Shipside Inc. Thereafter,
the CA affirmed the said order and its decision Facts:
became final and executory. A writ of execution was
subsequently served but after 24 long years, the A collective negotiation agreement (CNA)
Office of the Solicitor General received a letter form executed on 10 July 1996 between the Social
Victor Floresca, Vice President of John Hay Poro Security Commission (SSC) in behalf of the SSS
Point Development Corp., stating that the said writ
and the Alert and Concerned Employees for Better
have not been executed by the Register of Deeds.
The OSG then filed a complaint for revival of SSS (ACCESS), the sole and exclusive negotiating
judgment. Petitioner then file its Motion to Dismiss agent for employees of the SSS.
alleging inter alia that the complaint stated no cause
As a gesture of good will and benevolence, which has the power to approve, confirm, pass upon
the Management agrees that once the Collective or review the action of the SSS to sue in court.
Negotiation Agreement is approved and signed by Moreover, the appearance of the internal legal staff
the parties, Management shall grant each official of the SSS as counsel in the present proceedings is
and employee of the SYSTEM the amount of similarly questionable because under both RA 1161
P5,000.00 as contract signing bonus. and RA 8282 it is the Department of Justice (DoJ)
that has the authority to act as counsel of the SSS.
However the Department of Budget and It is well settled that the legality of the
Management (DBM) declared as illegal the contract representation of an unauthorized counsel may be
signing bonus which the CNA authorized to be raised at any stage of the proceedings and that
distributed among the personnel of the SSS. The such illicit representation produces no legal effect.
SSS Corporate Auditor disallowed fund releases for Since nothing in the case at bar shows that the
the signing bonus since it was an allowance in the approval or ratification of the SSC has been
form of additional compensation prohibited by the undertaken in the manner prescribed by law and
Constitution. that the DoJ has not delegated the authority to act
as counsel and appear herein, the instant petition
ACCESS appealed the disallowance to the
must necessarily fail. These procedural deficiencies
Commission on Audit (COA). On 5 July 2001
are serious matters which this Court cannot take
despite the delay in the filing of the appeal, a
lightly and simply ignore since the SSS is in reality
procedural matter which COA considered to be
confessing judgment to charge expenditure against
inconsequential, COA affirmed the disallowance
the trust fund under its custodianship.
and ruled that the grant of the signing bonus was
improper. It held that the provision on the signing In Premium Marble Resources v. Court of Appeals
bonus in the CNA had no legal basis since Sec. 16 we held that no person, not even its officers, could
of RA 7658 (1989) had repealed the authority of the validly sue in behalf of a corporation in the absence
SSC to fix the compensation of its personnel. Hence of any resolution from the governing body
the instant petition which, curiously, was filed in the authorizing the filing of such suit. Moreover, where
name of the Social Security System (and not the corporate officers power as an agent of the
ACCESS) by authority of the officer-in-charge for corporation did not derive from such resolution, it
the SSS through its legal staff. would nonetheless be necessary to show a clear
source of authority from the charter, the by-laws or
Issue:
the implied acts of the governing body.
Whether or not ACCESS can sue in the Unfortunately there is no palpable evidence in the
name of SSS records to show that the officer-in-charge could all
by himself order the filing of the instant petition
without the intervention of the SSC, nor that the
legal staff of SSS could act as its counsel and
appear therein without the intervention of the DoJ.
The power of attorney supposedly authorizing this
Held:
suit as well as the signature of the legal counsel
We find no legitimate and compelling appearing on the signing page of the instant petition
reason to reverse the COA. To begin with, the is therefore ineffectual.
instant petition is fatally defective. It was filed in the
name of the SSS although no directive from the
SSC authorized the instant suit and only the officer- SANTIAGO ESLABAN, JR., in his capacity as
in-charge in behalf of petitioner executed the Project Manager of the National Irrigation
purported directive. Clearly, this is irregular since Administration, petitioner, vs. CLARITA VDA. DE
under Sec. 4, par. 10, in relation to par. 7, RA 1161 ONORIO, respondent.
as amended by RA 8282 (The Social Security Act of DECISION
MENDOZA, J.:
1997, which was already effective when the instant
This is a petition for review of the decision[if
petition was filed), it is the SSC as a collegiate body !supportFootnotes][1][endif] of the Court of Appeals which
affirmed the decision of the Regional Trial Court, meters; and (3) respondent was not entitled to
Branch 26, Surallah, South Cotabato, ordering the compensation for the taking of her property
National Irrigation Administration (NIA for brevity) to considering that she secured title over the property
pay respondent the amount of P107,517.60 as just by virtue of a homestead patent under C.A. No. 141.
compensation for the taking of the latters property. At the pre-trial conference, the following facts
The facts are as follows: were stipulated upon: (1) that the area taken was
Respondent Clarita Vda. de Enorio is the 24,660 square meters; (2) that it was a portion of
owner of a lot in Barangay M. Roxas, Sto. Nio, the land covered by TCT No. T-22121 in the name
South Cotabato with an area of 39,512 square of respondent and her late husband (Exh. A); and
meters. The lot, known as Lot 1210-A-Pad-11- (3) that this area had been taken by the NIA for the
000586, is covered by TCT No. T-22121 of the construction of an irrigation canal.[if
Registry of Deeds, South Cotabato. On October 6, !supportFootnotes][2][endif]

1981, Santiago Eslaban, Jr., Project Manager of the On October 18, 1993, the trial court rendered
NIA, approved the construction of the main irrigation a decision, the dispositive portion of which reads:
canal of the NIA on the said lot, affecting a 24,660 In view of the foregoing, decision is hereby
square meter portion thereof. Respondents rendered in favor of plaintiff and against the
husband agreed to the construction of the NIA canal defendant ordering the defendant, National
provided that they be paid by the government for Irrigation Administration, to pay to plaintiff the sum
the area taken after the processing of documents by of One Hundred Seven Thousand Five Hundred
the Commission on Audit. Seventeen Pesos and Sixty Centavos
Sometime in 1983, a Right-of-Way (P107,517.60) as just compensation for the
agreement was executed between respondent and questioned area of 24,660 square meters of land
the NIA (Exh. 1). The NIA then paid respondent the owned by plaintiff and taken by said defendant NIA
amount of P4,180.00 as Right-of-Way damages. which used it for its main canal plus costs.[if
Respondent subsequently executed an Affidavit of !supportFootnotes][3][endif]

Waiver of Rights and Fees whereby she waived any On November 15, 1993, petitioner appealed
compensation for damages to crops and to the Court of Appeals which, on October 31, 2000,
improvements which she suffered as a result of the affirmed the decision of the Regional Trial Court.
construction of a right-of-way on her property (Exh. Hence this petition.
2). The same year, petitioner offered respondent the The issues in this case are:
sum of P35,000.00 by way of amicable settlement 1. WHETHER OR NOT THE PETITION IS
pursuant to Executive Order No. 1035, 18, which DISMISSIBLE FOR FAILURE TO COMPLY WITH
provides in part that THE PROVISIONS OF SECTION 5, RULE 7 OF
Financial assistance may also be given to owners of THE REVISED RULES OF CIVIL PROCEDURE.
lands acquired under C.A. 141, as amended, for the 2. WHETHER OR NOT LAND GRANTED BY
area or portion subject to the reservation under VIRTUE OF A HOMESTEAD PATENT AND
Section 12 thereof in such amounts as may be SUBSEQUENTLY REGISTERED UNDER
determined by the implementing PRESIDENTIAL DECREE 1529 CEASES TO BE
agency/instrumentality concerned in consultation PART OF THE PUBLIC DOMAIN.
with the Commission on Audit and the assessors 3. WHETHER OR NOT THE VALUE OF JUST
office concerned. COMPENSATION SHALL BE DETERMINED
Respondent demanded payment for the FROM THE TIME OF THE TAKING OR FROM THE
taking of her property, but petitioner refused to pay. TIME OF THE FINALITY OF THE DECISION.
Accordingly, respondent filed on December 10, 4. WHETHER THE AFFIDAVIT OF WAIVER OF
1990 a complaint against petitioner before the RIGHTS AND FEES EXECUTED BY
Regional Trial Court, praying that petitioner be RESPONDENT EXEMPTS PETITIONER FROM
ordered to pay the sum of P111,299.55 as MAKING PAYMENT TO THE FORMER.
compensation for the portion of her property used in We shall deal with these issues in the order
the construction of the canal constructed by the they are stated.
NIA, litigation expenses, and the costs. First. Rule 7, 5 of the 1997 Revised Rules on Civil
Petitioner, through the Office of the Solicitor- Procedure provides
General, filed an Answer, in which he admitted that Certification against forum shopping. The plaintiff or
NIA constructed an irrigation canal over the property principal party shall certify under oath in the
of the plaintiff and that NIA paid a certain landowner complaint or other initiatory pleading asserting a
whose property had been taken for irrigation claim for relief, or in a sworn certification annexed
purposes, but petitioner interposed the defense that: thereto and simultaneously filed therewith: (a) that
(1) the government had not consented to be sued; he has not theretofore commenced any action or
(2) the total area used by the NIA for its irrigation filed any claim involving the same issues in any
canal was only 2.27 hectares, not 24,600 square court, tribunal or quasi-judicial agency and, to the
best of his knowledge, no such other action or claim or instrument of conveyance [sales patent]
is pending therein; (b) if there is such other pending registered with the Register of Deeds and the
action or claim, a complete statement of the present corresponding certificate and owners duplicate of
status thereof; and (c) if he should thereafter learn title issued, such lands are deemed registered lands
that the same or similar action or claim has been under the Torrens System and the certificate of title
filed or is pending, he shall report the fact within five thus issued is as conclusive and indefeasible as any
(5) days therefrom to the court wherein his other certificate of title issued to private lands in
aforesaid complaint or initiatory pleading has been ordinary or cadastral registration proceedings.[if
filed. !supportFootnotes][5][endif]

Failure to comply with the foregoing requirements The Solicitor-General contends, however,
shall not be curable by mere amendment of the that an encumbrance is imposed on the land in
complaint or other initiatory pleading but shall be question in view of 39 of the Land Registration Act
cause for the dismissal of the case without (now P.D. No. 1529, 44) which provides:
prejudice, unless otherwise provided, upon motion Every person receiving a certificate of title in
and after hearing . . . . pursuance of a decree of registration, and every
By reason of Rule 45, 4 of the 1997 Revised subsequent purchaser of registered land who takes
Rules on Civil Procedure, in relation to Rule 42, 2 a certificate of title for value in good faith shall hold
thereof, the requirement of a certificate of non-forum the same free from all encumbrances except those
shopping applies to the filing of petitions for review noted on said certificate, and any of the following
on certiorari of the decisions of the Court of encumbrances which may be subsisting, namely:
Appeals, such as the one filed by petitioner. ....
As provided in Rule 45, 5, The failure of the Third. Any public highway, way, private way
petitioner to comply with any of the foregoing established by law, or any government irrigation
requirements regarding . . . the contents of the canal or lateral thereof, where the certificate of title
document which should accompany the petition does not state that the boundaries of such highway,
shall be sufficient ground for the dismissal thereof. way, irrigation canal or lateral thereof, have been
The requirement in Rule 7, 5 that the determined.
certification should be executed by the plaintiff or As this provision says, however, the only
the principal means that counsel cannot sign the servitude which a private property owner is required
certificate against forum-shopping. The reason for to recognize in favor of the government is the
this is that the plaintiff or principal knows better than easement of a public highway, way, private way
anyone else whether a petition has previously been established by law, or any government canal or
filed involving the same case or substantially the lateral thereof where the certificate of title does not
same issues. Hence, a certification signed by state that the boundaries thereof have been pre-
counsel alone is defective and constitutes a valid determined. This implies that the same should have
cause for dismissal of the petition.[if been pre-existing at the time of the registration of
!supportFootnotes][4][endif] the land in order that the registered owner may be
In this case, the petition for review was filed compelled to respect it. Conversely, where the
by Santiago Eslaban, Jr., in his capacity as Project easement is not pre-existing and is sought to be
Manager of the NIA. However, the verification and imposed only after the land has been registered
certification against forum-shopping were signed by under the Land Registration Act, proper
Cesar E. Gonzales, the administrator of the agency. expropriation proceedings should be had, and just
The real party-in-interest is the NIA, which is a body compensation paid to the registered owner thereof.[if
corporate. Without being duly authorized by !supportFootnotes][6][endif]

resolution of the board of the corporation, neither In this case, the irrigation canal constructed
Santiago Eslaban, Jr. nor Cesar E. Gonzales could by the NIA on the contested property was built only
sign the certificate against forum-shopping on October 6, 1981, several years after the property
accompanying the petition for review. Hence, on had been registered on May 13, 1976. Accordingly,
this ground alone, the petition should be dismissed. prior expropriation proceedings should have been
Second. Coming to the merits of the case, the land filed and just compensation paid to the owner
under litigation, as already stated, is covered by a thereof before it could be taken for public use.
transfer certificate of title registered in the Registry Indeed, the rule is that where private property
Office of Koronadal, South Cotabato on May 13, is needed for conversion to some public use, the
1976. This land was originally covered by Original first thing obviously that the government should do
Certificate of Title No. (P-25592) P-9800 which was is to offer to buy it.[if !supportFootnotes][7][endif] If the owner
issued pursuant to a homestead patent granted on is willing to sell and the parties can agree on the
February 18, 1960. We have held: price and the other conditions of the sale, a
Whenever public lands are alienated, granted or voluntary transaction can then be concluded and
conveyed to applicants thereof, and the deed grant the transfer effected without the necessity of a
judicial action. Otherwise, the government will use proceeding except on such terms as the court
its power of eminent domain, subject to the payment deems just and equitable. (Emphasis added)
of just compensation, to acquire private property in Thus, the value of the property must be
order to devote it to public use. determined either as of the date of the taking of the
Third. With respect to the compensation which the property or the filing of the complaint, whichever
owner of the condemned property is entitled to came first. Even before the new rule, however, it
receive, it is likewise settled that it is the market was already held in Commissioner of Public
value which should be paid or that sum of money Highways v. Burgos[if !supportFootnotes][11][endif] that the
which a person, desirous but not compelled to buy, price of the land at the time of taking, not its value
and an owner, willing but not compelled to sell, after the passage of time, represents the true value
would agree on as a price to be given and received to be paid as just compensation. It was, therefore,
therefor.[if !supportFootnotes][8][endif] Further, just error for the Court of Appeals to rule that the just
compensation means not only the correct amount to compensation to be paid to respondent should be
be paid to the owner of the land but also the determined as of the filing of the complaint in 1990,
payment of the land within a reasonable time from and not the time of its taking by the NIA in 1981,
its taking. Without prompt payment, compensation because petitioner was allegedly remiss in its
cannot be considered just for then the property obligation to pay respondent, and it was respondent
owner is made to suffer the consequence of being who filed the complaint. In the case of Burgos,[if
immediately deprived of his land while being made !supportFootnotes][12][endif] it was also the property owner

to wait for a decade or more before actually who brought the action for compensation against
receiving the amount necessary to cope with his the government after 25 years since the taking of
loss.[if !supportFootnotes][9][endif] Nevertheless, as noted in his property for the construction of a road.
Ansaldo v. Tantuico, Jr.,[if !supportFootnotes][10][endif] there Indeed, the value of the land may be affected
are instances where the expropriating agency takes by many factors. It may be enhanced on account of
over the property prior to the expropriation suit, in its taking for public use, just as it may depreciate.
which case just compensation shall be determined As observed in Republic v. Lara:[if
as of the time of taking, not as of the time of filing of !supportFootnotes][13][endif]

the action of eminent domain. [W]here property is taken ahead of the filing of the
Before its amendment in 1997, Rule 67, 4 condemnation proceedings, the value thereof may
provided: be enhanced by the public purpose for which it is
Order of condemnation. When such a motion is taken; the entry by the plaintiff upon the property
overruled or when any party fails to defend as may have depreciated its value thereby; or there
required by this rule, the court may enter an order of may have been a natural increase in the value of
condemnation declaring that the plaintiff has a the property from the time it is taken to the time the
lawful right to take the property sought to be complaint is filed, due to general economic
condemned, for the public use or purpose described conditions. The owner of private property should be
in the complaint upon the payment of just compensated only for what he actually loses; it is
compensation to be determined as of the date of the not intended that his compensation shall extend
filing of the complaint. . . . beyond his loss or injury. And what he loses is only
It is now provided that- the actual value of his property at the time it is
SEC. 4. Order of expropriation. If the objections to taken. This is the only way that compensation to be
and the defense against the right of the plaintiff to paid can be truly just, i.e., just not only to the
expropriate the property are overruled, or when no individual whose property is taken, but to the public,
party appears to defend as required by this Rule, which is to pay for it . . . .
the court may issue an order of expropriation In this case, the proper valuation for the
declaring that the plaintiff has a lawful right to take property in question is P16,047.61 per hectare, the
the property sought to be expropriated, for the price level for 1982, based on the appraisal report
public use or purpose described in the complaint, submitted by the commission (composed of the
upon the payment of just compensation to be provincial treasurer, assessor, and auditor of South
determined as of the date of the taking of the Cotabato) constituted by the trial court to make an
property or the filing of the complaint, whichever assessment of the expropriated land and fix the
came first. price thereof on a per hectare basis.[if
A final order sustaining the right to expropriate the !supportFootnotes][14][endif]

property may be appealed by any party aggrieved Fourth. Petitioner finally contends that it is exempt
thereby. Such appeal, however, shall not prevent from paying any amount to respondent because the
the court from determining the just compensation to latter executed an Affidavit of Waiver of Rights and
be paid. Fees of any compensation due in favor of the
After the rendition of such an order, the plaintiff shall Municipal Treasurer of Barangay Sto. Nio, South
not be permitted to dismiss or discontinue the Cotabato. However, as the Court of Appeals
correctly held: sq meters that have been used for the construction
[I]f NIA intended to bind the appellee to said of the canal. The Court of Appeals also affirmed the
affidavit, it would not even have bothered to give her decision of the RTC.
any amount for damages caused on the
improvements/crops within the appellees property.
This, apparently was not the case, as can be
gleaned from the disbursement voucher in the Lopez Realty Inc. v. Fontecha G.R. No.
amount of P4,180.00 (page 10 of the Folder of 76801 August 11, 1995
Exhibits in Civil Case 396) issued on September 17,
1983 in favor of the appellee, and the letter from the
Office of the Solicitor General recommending the Facts:
giving of financial assistance in the amount of Lopez Realty, Inc. (LRI) and Asuncion Lopez-
P35,000.00 to the appellee. Gonzalez initiated a “Complaint for annulment of
Thus, We are inclined to give more credence to the sale, cancellation of title, reconveyance and
appellees explanation that the waiver of rights and damages with prayer for the issuance of temporary
fees pertains only to improvements and crops and restraining order (TRO) and/or writ of preliminary
not to the value of the land utilized by NIA for its injunction against the spouses Tanjangco, Arturo
main canal.[if !supportFootnotes][15][endif] and the Registrar of Deeds of Manila.”
WHEREFORE, premises considered, the assailed
decision of the Court of Appeals is hereby Previously, LRI and Dr. Jose Tanjangco (Jose)
AFFIRMED with MODIFICATION to the extent that “were the registered co-owners of three parcels of
the just compensation for the contested property be land and the building erected thereon known as the
paid to respondent in the amount of P16,047.61 per ‘Trade Center Building’… Jose’s one-half share in
hectare, with interest at the legal rate of six percent the subject properties were later transferred and
(6%) per annum from the time of taking until full registered in the name of his son Reynaldo
payment is made. Costs against petitioner. Tanjangco and daughter-in-law, Maria Luisa
SO ORDERED. Arguelles (spouses Tanjangco).”

ESLABAN VS. ONORIO [360 SCRA 230; G.R. NO. These were the stockholders of record of LRI at the
146062; 28 JUN 2001] time material to this case:

Saturday, January 31, 2009 Posted by Coffeeholic 1. Asuncion Lopez-Gonzalez (Asuncion,


Writes Director & Corporate Secretary) – 7,831
Labels: Case Digests, Political Law shares;
2. Arturo F. Lopez (Arturo) – 7,830 shares;
Facts: Clarita Vda. De Onorio is the owner of the 3. Teresita Lopez-Marquez (Teresita) – 7,830
land in Barangay M. Roxas, Sto. Nino, South shares;
Cotabato. Such land is the subject for the 4. Rosendo de Leon (Rosendo, Director) – 5
construction of an irrigation canal of the National shares
Irrigation Administration (NIA). Mr. Santiago 5. Benjamin Bernardino (Benjamin, Director) –
Eslaban Jr. is the project manager of NIA. The 1 share;
parties agreed to the construction of the canal 6. Augusto de Leon (Augusto, Director) – 1
provided that the government will pay for the area share; and
that has been taken. A right-of-way agreement was 7. Leo Rivera (Leo, Director) – 1 share
entered into by the parties in which respondent was
During a special stockholders’ meeting held on 27
paid the amount of P4, 180.00 as right of way
July 1981, the sale of 1/2 share of LRI in the Trade
damages. Subsequently, respondent executed an
Center Building was taken up. While the selling
Affidavit of Waiver of Rights and Fees which waives
price was at P4 M, the Tanjancos offered P3.8 M.
her rights for the damage to the crops due to
To this, Asuncion countered with P5 M which was
construction of the right of way. After which,
not accepted by the Tanjancos. Thus, the board
respondent demands that petitioner pay P111,
agreed to give Asuncion the priority to equal the
299.55 for taking her property but the petitioner
Tanjanco offer and the same to be exercised within
refused. Petitioner states that the government had
ten (10) days. Otherwise, the Tanjanco offer will be
not consented to be sued and that the respondent is
deemed accepted. Just a day after, Teresita died
not entitled for compensation by virtue of the
(her estate’s executor Juanito L. Santos
homestead patent under CA no. 141. The RTC held
represented her afterwards).
that the NIA should pay respondent the amount of
P107, 517.60 as just compensation for the 24,660
As Asuncion failed to exercise her option to meeting. The Court takes into account that majority
purchase the subject properties, and while she was of the board of directors except for Asuncion, had
abroad, “the remaining directors: Rosendo, already approved of the sale to the spouses
Benjamin and Leo convened in a special meeting” Tanjangco prior to this meeting. As a consequence,
passing and approving the 17 August 1981 the power to ratify the previous resolutions and
Resolution authorizing Arturo to negotiate and actions of the board of directors in this case lies in
“carry out the complete termination of the sale terms the stockholders, not in the board of directors. It
and conditions as embodied in the Resolution of would be absurd to require the board of directors to
July 27, 1981”, among others. Subsequently, the ratify their own acts—acts which the same director s
sale was perfected with payments subsequently already approved of beforehand. Hence, Juanito, as
made. the administrator of Teresita’s estate even though
not a director, is entitled to vote on behalf of
After learning of the sale, Asuncion filed this Teresita’s estate as the administrator thereof.”
complaint challenging the validity of the 17 August
1982 Resolution on the ground that she was not Citing jurisprudence, in stock corporations,
notified of the meeting. “shareholders may generally transfer their shares.
Issue: Thus, on the death of a shareholder, the executor or
administrator duly appointed by the Court is vested
with the legal title to the stock and entitled to vote it.
Whether the sale was valid binding the stockholders Until a settlement and division of the estate is
to the acts of its board of directors. effected, the stocks of the decedent are held by the
administrator or executor.”
Held:
As there exists no corporate secretary’s certification
of the minutes of the meeting, “only Juanito,
The sale was valid. The 17 August 1981 Board Benjamin and Roseno, whose signature appeared
Resolution did not give Arturo the authority to act as on the minutes, could be considered as to have
LRI’s representative in the sale “as the meeting of ratified the sale to the spouses Tanjangco.” As Leo
the board of directors where such was passed was owns only 1 share, the results are the same against
conducted without giving any notice to Asuncion.” the overwhelming shares who voted in favor of
This is in violation of Section 53 of the Corporation ratification.
Code which requires sending of notices for regular
or special meetings to every director. “In sum, whatever defect there was on the sale to
the spouses Tanjangco pursuant to the August 17,
As a result, “a meeting of the board of directors is 1981 Board Resolution, the same was cured
legally infirm if there is failure to comply with the through its ratification in the July 30, 1982 Board
requirements or formalities of the law or the Resolution. It is of no moment whether Arturo was
corporation’s by laws and any action taken on such authorized to merely negotiate or to enter into a
meeting may be challenged as a consequence.” contract of sale on behalf of LRI as all his actions in
connection to the sale were expressly ratified by the
Notwithstanding, “the actions taken in such a
stockholders holding 67% of the outstanding capital
meeting by the directors or trustees may be ratified
stock.”
expressly or impliedly.” In the case of ratification, it
means that “the principal voluntarily adopts, Citing jurisprudence, “the Court held that by virtue of
confirms and gives sanction to some unauthorized ratification, the acts of the board of directors
act of its agent on its behalf.” become the acts of the stockholders themselves,
even if those acts were, at the outset,
Here, “the ratification was expressed through the
unauthorized.”
July 30, 1982 Board Resolution.” Regarding
Asuncion’s claims that the 30 July 1982 Board
Resolution did not ratify the 17 August 1981 TUASON VS. BOLANOS
Resolution due to Juanito’s disqualification and
Leo’s negative vote. “Asuncion assails the authority Facts:
of Juanito to vote because he was not a director
and he did not own any share of stock which would Plaintiff’s complaint against defendant was
qualify him to be one. On the contrary, Juanito to recover possession of a registered land. In the
defends his right to vote as the representative of complaint, the plaintiff is represented by its
Teresita’s estate. Upon examination of the July 30, Managing Partner, Gregorio Araneta, Inc., another
1982 minutes of the meeting, it can be deduced that corporation. Defendant, in his answer, sets up
the meeting is a joint stockholders and directors’ prescription and title in himself thru "open,
continuous, exclusive and public and notorious Facts:
possession under claim of ownership, adverse to
the entire world by defendant and his predecessors Petitioners were sued by the private respondents to
in interest" from "time immemorial". After trial, the nullify the issuance of 823 shares of stock of the
lower court rendered judgment for plaintiff, declaring Inocentes de la Rama, Inc..
defendant to be without any right to the land in
question and ordering him to restore possession
The Lopue’s, are the owners of 1,328 shares of
thereof to plaintiff and to pay the latter a monthly
stock of the Inocentes de la Rama, Inc., a domestic
rent. Defendant appealed directly to the Supreme
corporation, with an authorized capital stock of
Court and contended, among others, that Gregorio
3,000 shares, with a par value of P100.00 per
Araneta, Inc. can not act as managing partner for share, 2,177 of which were subscribed and issued,
plaintiff on the theory that it is illegal for two thus leaving 823 shares unissued;
corporations to enter into a partnership

Issue: The Lopue’s acquired of the shares of stock held by


Rafael Ledesma and Jose Sicangco, Jr., then
Whether or not a corporation may enter into President and Vice-President of the corporation.
a joint venture with another corporation.
The petitioners Mercedes R. Borromeo, Honorio de
Ruling: la Rama, and Ricardo Gamboa, remaining
members of the board of directors of the
It is true that the complaint states that the corporation, to forestall the takeover by the Lopue’s
plaintiff is "represented herein by its Managing of the corporation, surreptitiously met and elected
Partner Gregorio Araneta, Inc.", another Ricardo L. Gamboa and Honorio de la Rama as
corporation, but there is nothing against one president and vice-president of the corporation, and
corporation being represented by another person, thereafter passed a resolution authorizing the sale
natural or juridical, in a suit in court. The contention of the 823 unissued shares of the corporation to the
that Gregorio Araneta, Inc. cannot act as managing Ricardo L. Gamboa, Lydia R. Gamboa, Honorio de
partner for plaintiff on the theory that it is illegal for la Rama, Ramon de la Rama, Paz R. Battistuzzi
two corporations to enter into a partnership is Eduardo de la Rama, and Mercedes R. Borromeo,
without merit, for the true rule is that "though a at par value, Honorio de la Rama, Lydia de la
corporation has no power to enter into a Rama-Gamboa, and Enzo Battistuzzi were elected
partnership, it may nevertheless enter into a joint to the board of directors of the corporation;
venture with another where the nature of that
venture is in line with the business authorized by its Private respondents alleged that the sale of the
charter." (Wyoming-Indiana Oil Gas Co. vs. Weston, unissued 823 shares of stock of the corporation was
80 A. L. R., 1043, citing 2. Fletcher Cyc. of Corp., in violation of the plaintiffs' and pre-emptive rights
1082.). There is nothing in the record to indicate and made without the approval of the board of
that the venture in which plaintiff is represented by directors representing 2/3 of the outstanding capital
Gregorio Araneta, Inc. as "its managing partner" is stock, and is in disregard of the strictest relation of
not in line with the corporate business of either of trust existing between the defendants, as
them. stockholders thereof; and that the defendants Lydia
de la Rama-Gamboa, Honorio de la Rama, and
Enzo Battistuzzi were not legally elected to the
board of directors of the said corporation and has
unlawfully usurped or intruded into said office to the
G.R. No. L-40620 May 5, 1979 prejudice of the plaintiffs.

RICARDO L. GAMBOA, LYDIA R. GAMBOA, -They prayed that a writ of preliminary injunction be
HONORIO DE 1A RAMA, EDUARDO DE LA issued restraining the petitioners from committing,
RAMA, and the HEIRS OF MERCEDES DE LA or continuing the performance of an act tending to
RAMA-BORROMEO, petitioners, prejudice, diminish or otherwise injure the plaintiffs'
vs. rights in the corporate properties and funds of the
HON. OSCAR R. VICTORIANO as Presiding corporation, and from disposing, transferring,
Judge of the Court of First Instance of Negros selling, or otherwise impairing the value of the 823
Occidental, Branch II, BENJAMIN LOPUE, SR., shares of stock illegally issued by the defendants;
BENJAMIN LOPUE, JR., LEONITO LOPUE, and that a receiver be appointed to preserve and
LUISA U. DACLES respondents.
administer the property and funds of the The defendants also filed a motion to declare the
corporation; defendants Ramon L. de la Rama, Paz de la Rama
Battistuzzi and Enzo Battistuzzi in contempt of
-that defendants Lydia de la Rama-Gamboa, court, for having violated the writ of preliminary
Honorio de la Rama, and Enzo Battistuzzi be injunction when they entered into the aforesaid
declared as usurpers or intruders into the office of compromise agreement with the plaintiffs, but the
director in the corporation and, consequently, respondent judge denied the said motion for lack of
ousting them therefrom and declare Luisa U. Dacles merit.
as a legally elected director of the corporation; that
the sale of 823 shares of stock of the corporation be The defendants filed a motion for the
declared null and void; and that the defendants be reconsideration of the order denying their motion to
ordered to pay damages and attorney's fees, as dismiss the complaint' and subsequently, an
well as the costs of suit . Addendum thereto, claiming that the respondent
court has no jurisdiction to interfere with the
The respondent judge issued the corresponding writ management of the corporation by the board of
of preliminary injunction restraining the defendants directors, and the enactment of a resolution by the
and/or their representatives, agents, or persons defendants, as members of the board of directors of
acting in their behalf from the commission or the corporation, allowing the sale of the 823 shares
continuance of any act tending in any way to of stock to the defendants was purely a
prejudice, diminish or otherwise injure plaintiffs' management concern which the courts could not
rights in the corporate properties and funds of the interfere with.
corporation Inocentes de la Rama, Inc.' and from
disposing, transferring, selling or otherwise When the trial court denied said motion and its
impairing the value of the certificates of stock addendum, the defendants filed the instant petition
allegedly issued illegally in their names and ordering for certiorari for the review of said orders.
them to deposit with the Clerk of Court the
corresponding certificates of stock for the 823 Issue:
shares issued to said defendants, upon plaintiffs'
posting a bond in the sum of P50,000.00, to answer
W/N the respondent court has jurisdictionto interfere
for any damages and costs that may be sustained
with the management of the corporation and the
by the defendants by reason of the issuance of the
enactment of resolution.
writ, copy of the bond to be furnished to the
defendants. "
Ruling: As a general rule. No.
The defendants deposited with the clerk of court the
corporation's certificates of stock Nos. 80 to 86 The well-known rule is that courts cannot undertake
representing the disputed 823 shares of stock of the to control the discretion of the board of directors
corporation. 3 about administrative matters as to which they have
legitimate power of, 10 action and contracts intra
vires entered into by the board of directors are
The private respondents, entered into a
binding upon the corporation and courts will not
compromise agreement with Ramon de la Rama,
interfere unless such contracts are so
Paz de la Rama Battistuzzi and Enzo Battistuzzi unconscionable and oppressive as to amount to a
, 4 whereby the contracting parties withdrew their
wanton destruction of the rights of the minority.
respective claims against each other and the
petitioners waived and transferred their rights and
interests over the questioned 823 shares of stock in In the instant case, the plaintiffs aver that the
favor of the plaintiffs. defendants have concluded a transaction among
themselves as will result to serious injury to the
interests of the plaintiffs, so that the trial court has
The compromise agreement was approved by the
jurisdiction over the case.
trial court. The defendants filed a motion to dismiss
the complaint upon the grounds: (1) that the
plaintiffs' cause of action had been waived or The claim of the petitioners, in their Addendum to
abandoned; and (2) that they were estopped from the motion for reconsideration of the order denying
further prosecuting the case since they have, in the motion to dismiss the complaint, questioning the
effect, acknowledged the validity of the issuance of trial court's jurisdiction on matters affecting the
the disputed 823 shares of stock. The motion was management of the corporation, is without merit.
denied.
The petitioners further contend that the proper Board Members in behalf of the ASSA Law
remedy of the plaintiffs would be to institute a and Associates.
derivative suit against the petitioners in the name of a. As a partner in the said law firm, he
the corporation in order to secure a binding relief only filed a “Manifestation of
after exhausting all the possible remedies available Extreme Urgency” in the case with
within the corporation. the Ombudsman and entered into
the retainer contract with the
An individual stockholder is permitted to institute a PPSTA Board in representation of
derivative suit on behalf of the corporation wherein the ASSA Law Firm.
he holds stock in order to protect or vindicate b. For the SEC case, Ernesto alleged
corporate rights, whenever the officials of the that the same was being handled
corporation refuse to sue, or are the ones to be by the law firm of Atty. Eduardo de
sued or hold the control of the corporation. In such Mesa, and not ASSA.
actions, the suing stockholder is regarded as a 2. After investigation, IBP CBD recommended
nominal party, with the corporation as the real party that Ernesto be suspended from the
in interest. 12 practice of law for six months.

Issue: WON a lawyer, engaged by a corporation,


In the case at bar, however, the plaintiffs are
can defend members of the board of the same
alleging and vindicating their own individual
corporation in a derivative suit?
interests or prejudice, and not that of the
corporation. At any rate, it is yet too early in the
Ruling:
proceedings since the issues have not been joined.
Besides, misjoinder of parties is not a ground to 1. Definition/test for conflict of interest:
dismiss an action. 13  Definition: There is conflict of
interest when a lawyer represents
inconsistent interests of two or
more opposing parties.
 Test:
Hornilla vs. Salunat
o Whether or not in behalf of
one client, it is the lawyer’s
Facts:
duty to fight for an issue or
1. Hornilla and Ricafort filed an administrative
claim, but it is his duty to
complaint with the IBP against Atty. Ernesto
oppose it for the other
S. Salunat for illegal and unethical practice
client.
and conflict of interest alleging that Salunat
o If the acceptance of the
is a member of the ASSA Law and
new retainer will require the
Associates, the retained counsel of the
atty to perform an act w/c
PPSTA.
will injuriously affect his 1st
2. Ernesto’s brother, Aurelio S. Salunat, was a
client in any matter in w/c
member of the PPSTA Board which
he represents him and also
approved Ernesto’s engagement as
whether he will be called
retained counsel of PPSTA.
upon in his new relation to
3. Complainants, members of the Philippine
use against his 1st client
Public School Teachers Association
any knowledge acquired
(PPSTA), filed an intra-corporate case
through their connection.
against its members of the Board of
o Whether the acceptance of
Directors before the Securities and
a new relation will prevent
Exchange Commission(SEC) and a
an atty from the full
complaint before the Ombudsman for
discharge of his duty of
unlawful spending and the undervalued sale
undivided fidelity and
of real property of the PPSTA.
loyalty to his client of invite
4. Atty. Ernesto S. Salunat entered his
suspicion of unfaithfulness
appearance as counsel for the PPSTA
or double dealing.
Board members in the said cases.
2. A corporation’s board of directors is
Defense:
separate and distinct from the corporate
1. Ernesto stressed that he entered his
entity itself.
appearance as counsel for the PPSTA
 A stockholder may sue on behalf of Benjamin A. Tango over his house and lot in
himself and other stockholders and Quezon City. The mortgages were third party or
for the benefit of the corporation, to accommodation mortgages on behalf of the
bring about a redress of the wrong spouses Bernardino and Carmelita Ibarra who were
done directly to the corporation and
dealers of SMC products in Aparri, Cagayan. Other
indirectly to the stockholders.
 This is what is known as a defendants in the case were Francisco A.G. De
derivative suit where the Liano and Alberto O. Villa-Abrille, Jr., who are
corporation is the real party in senior executives of petitioner SMC.
interest while the stockholder filing
suit for the corporation’s behalf is SMC, De Liano and Abrille appealed the aforesaid
only nominal party. decision to the Court of Appeals. In due time, their
 A lawyer engaged as counsel for a counsel, Atty. Edgar B. Afable, filed an Appellants'
corporation cannot represent Brief which failed to comply with Section 13, Rule
members of the same corporation’s
44 of the Rules of Court. The appellee (herein
board of directors in a derivative
suit brought against them. private respondent) was quick to notice these
 To do so would be tantamount to deficiencies, and accordingly filed a "Motion to
representing conflicting interests Dismiss Appeal” dated March 8, 1999.
which is prohibited by the Code of
Professional Responsibility. The appellants averred that their brief had
substantially complied with the contents as set forth
3. Ernesto admits that ASSA Law Firm, of in the rules. They proffered the excuse that the
which he is the Managing Partner, was the omissions were only the result of oversight or
retained counsel of PPSTA.
inadvertence and as such could be considered
 By filing the pleading in the
Ombudsman case, he necessarily "harmless" errors. They prayed for liberality in the
entered his appearance therein. application of technical rules, adding that they have
 This constituted conflict of interests, a meritorious defense.
considering that the complaint in
the Ombudsman, although in the The Court of Appeals dismissed the appeal and
name of the individual members of ruled that the Brief does not contain a Subject Index
the PPSTA, was brought in behalf nor a Table of Cases and Authorities, with page
of and to protect the interest of the references. Moreover, the Statement of the Case,
corporation.
Statement of Facts, and Arguments in the Brief has
 Therefore, Ernesto is guilty of
no page reference to the record. These procedural
representing conflicting
interests. Since first offense - lapses justify the dismissal of the appeal, pursuant
Admonish to Section 1 (f), Rule 50 of 1997 Rules of Civil
Procedure.

From the denial of their motion for reconsideration,


only petitioner SMC interposed the instant petition.
G.R. No. 142316 November 22, 2001
ISSUE:
FRANCISCO A.G. DE LIANO, ALBERTO O.
VILLA-ABRILLE, JR., and SAN MIGUEL Whether or not the Court of Appeals erred in
CORPORATION, petitioners, dismissing SMC’s appeal.
vs.
HON. COURT OF APPEALS and BENJAMIN A. RULING:
TANGO, respondents.
The petition has no merit.
FACTS:
The premise that underlies all appeals is that they
This case involves the cancellation of two (2) real are merely rights which arise from statute; therefore,
estate mortgages in favor of petitioner San Miguel they must be exercised in the manner prescribed by
Corporation (SMC) executed by private respondent law. It is to this end that rules governing pleadings
and practice before appellate courts were imposed. Parañaque Rice Mill, Inc. and the other
These rules were designed to assist the appellate stockholders who may desire to join, plaintiffs-
court in the accomplishment of its tasks, and appellees, vs.
overall, to enhance the orderly administration of
justice. TEODORICO B. SANTOS, ESTANISLAO
MAYUGA, APOLONIO PASCUAL, and BASILISA
Petitioner's contention that the appellate court RODRIGUEZ, defendant-appellants.
should have considered the substance of the appeal
prior to dismissing it due to technicalities does not Facts: The plaintiff and the defendant are all
gain our favor.
stockholders and member of the board of directors
of the "Parañaque Rice Mill, Inc., "a corporation
Generally, the negligence of counsel binds his
client. Actually, Atty. Afable is also an employee organized for the purpose of operating a rice mill. A
of petitioner San Miguel Corporation. Yet even complaint was filed with the Court.
this detail will not operate in petitioner's favor. A
corporation, it should be recalled, is an artificial The complaint avers subtantially the following: (a)
being whose juridical personality is only a That the plaintiffs are stockholders and constitute
fiction created by law. It can only exercise its the minority and the defendants are also
powers and transact its business through the stockholers and constitute the majority of the board
instrumentalities of its board of directors, and
of directors of the Parañaque Rice Mill, Inc.;
through its officers and agents, when authorized
by resolution or its by-laws.
(b) that the stockholders appointed an investigation
committee of which the plaintiff Jose de Lara was
Moreover, a corporate officer or agent
chairman and the stockholers Dionisio Tomas and
may represent and bind the corporation
in transactions with third persons to the Aguedo Bernabe were members, to investigate and
extent that authority to do so has been determine the properties, operations, and losses of
conferred upon him, and this includes the corporation as shown in the auditor's report, but
powers which have been intentionally the defendants, particularly Teodorico B. Santos,
conferred, and also such powers as, in who was the president of the corporation, denied
the usual course of the particular
access to the properties, books and record of the
business, are incidental to, or may be
implied from, the powers intentionally corporation which were in their possession
conferred, powers added by custom and
usage, as usually pertaining to the (c) That the defendant Teodorico B. Santos, in
particular officer or agent, and such violation of the by-laws of the corporation, had taken
apparent powers as the corporation has possession of the books, vouchers, and corporate
caused persons dealing with the officer records as well as of the funds and income of the
or agent to believe that it has conferred. Parañaque Rice Mill, Inc., all of which, according to
the by-laws, should be under the exclusive control
That Atty. Afable was clothed with sufficient and possession of the secretary-treasurer, the
authority to bind petitioner SMC is undisputable.
plaintiff Aguedo Bernabe;
Petitioner SMC's board resolution of May 5, 1999
attests to that. Coupled with the provision of law that
(d) That the said Teodorico B. Santos, had
a lawyer has authority to bind his client in taking
appeals and in all matters of ordinary judicial appropriated to his own benefit properties, funds,
procedure, a fortiori then, petitioner SMC must be and income of the corporation; (e) that Teodoro B.
held bound by the actuations of its counsel of Santos, for the purpose of illegally controlling the
record, Atty. Afable. affairs of the corporation, refuse to sign and issue
the corresponding certificate of stock for the 600
fully paid-up share of the plaintiff, Higinio Angeles,

G.R. No. L-43413 August 31, 1937 ( f ) that notwithstanding written requests made in
conformity with the by-laws of the corporation of
HIGINIO ANGELES, JOSE E. LARA and three members of the board of directors who are
AGUEDO BERNABE, as stockholders for an in holders of more than one-third of the subscribed
behalf and for the benefit of the corporation, capital stock of the corporation, the defendant
Teodorico B. Santos as president of the corporation directors, for and in behalf of the corporation, to
refuse to call a meeting of the board of directors and prevent waste and dissipation and the commission
of the stockholers; of illegal acts and otherwise redress the injuries of
the minority stockholders against the wrongdoing of
(g) that in violation of the by-laws of the corporation, the majority. The action in such a case is said to be
the defendant who constitute the majority of the brought derivatively in behalf of the corporation to
board of directors refused to hold ordinary monthly protect the rights of the minority stockholders
meetings; thereof (7 R. C. L., pars. 293 and 294, and authority
therein cited; 13 Fletcher, Cyc. of Corp., pars. 593,
(h) that Teodorico B. Santos as president of the
et seq., an authorities therein cite).
corporation, in connivance with his co-defendants,
was disposing of the properties and records of the It is well settled in this jurisdiction that where
corporation without authority from the board of corporate directors are guilty of a breach of trust —
directors or the stockholders of the corporation and not of mere error of judgment or abuse of discretion
without making any report of his acts to the said — and intracorporate remedy is futile or useless, a
board of directors or to any other officer of the stockholder may institute a suit in behalf of himself
corporation, and that, to prevent any interferrence and other stockholders and for the benefit of the
with or examination of his arbitrary acts, he corporation, to bring about a redress of the wrong
arbitrarily suspended plaintiff Jose de Lara from the inflicted directly upon the corporation and indirectly
office of general manager to which office the latter upon the stockholers. An illustration of a suit of this
had been lawfully elected by the stockholders; kind is found in the case of Pascual vs. Del Sanz
Orozco (19 Phil., 82), decided by this court as early
The court issued an ex parte order of receivership
as 1911. In that case, the Banco Español-Filipino
appointing Melchor de Lara as receiver of the
suffered heavy losses due to fraudulent connivance
corporation upon the filling of a bond by the
between a depositor and an employee of the bank,
plaintiffs-appellees.
which losses, it was contened, could have been
Issue: Whether or not minority stockholders may file avoided if the president and directors has been
a suit for and in behalf of the corporation? more vigilant in the administration of the affairs of
the bank. The stockholers constituting the minority
Ruling: Yes, There is ample evidence in the present brought a suit in behalf of the bank against the
case to show that the defendants have been guilty directors to recover damages, and this over the
of breach of trust as directors of the corporation and objection of the majority of the stockholers and the
the lower court so found. The board of directors of a directors. This court held that the suit properly be
corporation is a creation of the stockholders and maintained.
controls and directs the affairs of the corporation by
allegation of the stockholders. But the board of
directors, or the majority thereof, in drawing to
BOARD OF LIQUIDATORS
themselves the power of the corporation, occupies a vs.
position of trusteeship in relation to the minority of HEIRS OF MAXIMO KALAW
the stock in the sense that the board should GR L-18805, 14 August 1967
exercise good faith, care and diligence in the
administration of the affairs of the corporation and FACTS:
should protect not only the interest of the majority
National Coconut Corporation (NACOCO) is
but also those of the minority of the stock. Where a
with Maximo Kalaw as its General Manager and
majority of the board of directors wastes or Chairman of the BOD. Under his tenure NACOCO
dissipates the funds of the corporation or entered into different contracts involving the trade of
fraudulently disposes of its properties, or performs coconuts. It failed, however, due to natural
ultra vires acts, the court, in the exercise of its calamities that greatly affected the production of
equity jurisdiction, and upon showing that coconuts. This led to some customers of NACOCO
intracorporate remedy is unavailing, will entertain a suing the corporation for undelivered coconuts due
to them under the contracts that they signed. This
suit filed by the minority members of the board of
was settled by NACOCO by paying the customers.
Thereafter, NACOCO seeks to recover the Orientalist Co. and Fernandez (2001)
above sum of P1,343,274.52 from general manager
and board chairman Maximo M. Kalaw, and Facts:
directors Juan Bocar, Casimiro Garcia and Leonor The Orientalist Company is a corporation,
Moll. It charges Kalaw with negligence under Article duly organized under the laws of the Phil. Islands,
1902 of the old Civil Code (now Article 2176, new and was engaged in business of maintaining and
Civil Code); and defendant board members, conducting a theatre in the City of Manila for the
including Kalaw, with bad faith and/or breach trust exhibition of cinematographic films. The plaintiff J.F.
for having approved the contracts. Ramirez was engaged in the business of marketing
films for a manufacturer, and in the production of
ISSUE: distribution of cinematographic material. Certain of
the directors of the Orientalist Company became
Whether or not Kalaw may be held liable by apprised of the fact that the plaintiff had control of
NACOCO for the debts the corporation incurred the agencies for two different marks of films, namely
under his administration. the Éclair Films and the Milano Films. Negotiation
have begun with said officials of the company with
RULING: Ramirez. The defendant, Fernandez, one of the
board of directors, and treasurer, was chiefly active
NO. on this matter, believing that the securing of said
films was necessary to the success of the
They were done with implied authority from corporation. Ramirez placed in the hands of
the BOD. These previous contracts, it should be Fernandez an offer in which the latter accepted for
stressed, were signed by Kalaw without prior the exclusive agencies of said two films. The
authority from the board. Said contracts were known communications indicating the acceptance were
all along to the board members. Nothing was said signed by Ramirez as an individual and in his
by them. The aforesaid contracts stand to prove one capacity of Treasurer of the Orientalist.
thing. Obviously NACOCO board met the difficulties
attendant to forward sales by leaving the adoption Issue:
of means to end, to the sound discretion of W/N Fernandez, as treasurer, had authority
NACOCO's general manager Maximo M. Kalaw. to bind the company by signing it name to the letters
Settled jurisprudence has it that where in question.
similar acts have been approved by the directors as
a matter of general practice, custom, and policy, the Ruling:
general manager may bind the company without No.
formal authorization of the board of directors. In A Director-Treasurer has no power to bind
varying language, existence of such authority is the company even in transactions that are pursuant
established, by proof of the course of business, the to the primary purpose of its corporation, especially
usages and practices of the company and by the when the by-laws specifically provided that the acts
knowledge which the board of directors has, or must entered into can only be done by the Board of
be presumed to have, of acts and doings of its Directors.
subordinates in and about the affairs of the It is declared in Sec. 28 of the Corporation
corporation. Law that corporate power shall be exercised, and all
Authorities, great in number, are one in the corporate business conducted by the board of
idea that "ratification by a corporation of an directors; and this principle is recognized in the by-
unauthorized act or contract by its officers or others laws of the corporation in question which contain a
relates back to the time of the act or contract provision declaring that the power to make contracts
ratified, and is equivalent to original authority;" and shall be vested in the board of directors.
that "[t]he corporation and the other party to the
transaction are in precisely the same position as if
the act or contract had been authorized at the time." Ramirez v. Orientalist Co. and Fernandez (1918) –
The language of one case is expressive: "The Street, J.
adoption or ratification of a contract by a corporation
is nothing more nor less than the making of an Concept: Control and Management of Corporation
original contract. The theory of corporate ratification
is predicated on the right of a corporation to
contract, and any ratification or adoption is
equivalent to a grant of prior authority. FACTS:
Orientalist Company (Orientalist for brevity) or by Hernandez when they fell due; and it was
exhibited films in a theatre in Manila. Plaintiff JF finally necessary for Ramirez to take them up as
Ramirez, a resident of Paris and represented in dishonoured by non-payment.
Manila by his son Jose Ramirez, was engaged in
business of marketing films for manufacturers and
in the production or distribution of cinematographic
material. In 1913, there were negotiations between Ramirez instituted an action against Orientalist and
the officials of Orientalist and Jose Ramirez, as RJ Fernandez. Upon application of Ramirez, the
agent of JF Ramirez, for the exclusive agency of films were sold and the amount realized from the
two films in the hands of Orientalist. Jose Ramirez sale was applied to the satisfaction of the plaintiff’s
placed a formal offer stating in detail the terms upon claim. Judgment was given for the balance due to
which Ramirez would undertake to supply from Ramiez. Orientalist was declared to be a principal
Paris the films. The board of directors approved and debtor and Fernandez was declared to be
accepted the offer. The most important portion of subsidiarily liable as guarantor. Defendants
the two letters of acceptance written by Fernandez appealed. The Court noted that the action is
to Ramirez is in the following terms: “These primarily founded upon the liability created by the
communications were signed in the following form, two acceptance letters.
in which it will be noted the separate signature of RJ
Fernandez, as an individual, is placed somewhat
below and to the left of the signature of the
ISSUES:
Orientalist Company, as signed by RJ Fernandez, in
the capacity of treasurer: 1. WON Fernandez’s actions bound the
company .
2. WON the company is still liable, assuming
that the company was able to deny the
authority of Fernandez.
THE ORIENTALIST COMPANY, 3. What is the character of liability
assumed by Fernandez?
By RJ Fernandez
HELD:
Treasurer.
1. YES. The corporation was not able to
RJ Fernandez” deny the genuineness and due execution of the
contracts in question and the authority of
Fernandez to bind the Orientalist Company.
The record showed that JF Ramirez himself Sec. 103 of the Code of Civil Procedure
procured the films upon his own responsibility. requires that the Answer setting up the defense
Thus, the only contracting parties in this case are JF of lack of authority of an officer of a corporation
Ramirez (first party), and Orientalist with RJ to bind it by a contract should be verified and
Fernandez (second party). The films arrived in the denial contemplated must be specific. In this
Manila but Orientalist had no funds to meet its case, the failure of the corporation to make any
obligations. Hence, the first few drafts were issue in its answer with regard to the authority
accepted in the name of Orientalist by its president of Fernandez to bind it, and particularly its
B Hernandez, and were taken up by him with his failure to deny specifically under oath the
own funds. As the drafts had been paid by genuineness and due execution of the contracts
Hernandez, he treated the films as his own sued upon, have the effect of eliminating the
property, and they never came into the actual question of his authority from the case.
possession of Orientalist as owner at all. Hernandez
rented the films to Orientalist and they were
exhibited by it in the Oriental Theater under an Whether a particular officer actually possesses
arrangement made between him and the theater’s the authority which he assumes to exercise is
manager. Several remittances of films from Paris frequently known to very, very few and the proof
arrived. All of the drafts accompanying these films of it usually is not readily accessible to the
were drawn upon the Orientalist Company; and all stranger who deals with the corporation on the
were accepted in the name of Orientalist by its faith of the ostensible authority exercised by
president, B Hernandez, except the last which was some of the corporate officers.
accepted by Hernandez individually. None of the
drafts thus accepted were taken up by the drawee
2. YES. If a corporation knowingly permits one of Acuña entered into a contract with Batac
its officers, or any other agent, to do acts within wherein he agreed to advance P20,000.00 to the
the scope of an apparent authority and thus company for its tobacco planting and drying,
holds him out to the public as possessing power provided that he shall be assigned as the
to do those acts, the corporation will, as against company’s representative in Manila and supervise
anyone who has in good faith dealt with the the transport and delivery of the goods in the said
corporation through such agent, be estopped place. Batac’s Board of Directors are amenable with
from denying his authority; and where it is said the idea and thereafter issued a resolution
“if the corporation permits” this means the same authorizing Manager Leon Verano to enter into the
as “if the thing is permitted by the directing agreement on behalf of the corporation.
power of the corporation.” The necessary contract between Acuña and
Verano was entered into, with some of the Board of
Directors acting as witness. Acuña then inquired if
The stockholders adopted a resolution to the the contract needs to be ratified by the Board, in
effect that the agencies of the two films should which the counsel for Batac answered in the
be accepted if the corporation could obtain the negative. Acuña thereafter proceeded to perform his
money with which to meet the expenditure part of the contrac, including the advancement of
involved, and to this end appointed a committee the amount promised, which was accepted by
to apply to the bank for a credit. An attempt to Batac.
obtain credit was made, but failed. Another Batac’s BoD, however, disapproved the
special meeting of stockholders was held and a contract. Acuña insisted on its performance, but the
resolution was passed to the effect that the corporation refused, stating that the contract is not
company should pay to Hernandez, Fernandez, binding by reason that it was not ratified by the
Monroy and Papa an amount equal to 10% of board.
their outlay in importing the films, said payment
to be made in shares of the company. At the ISSUE:
time this meeting was held three shipments of
the film had already been received in Manila. Whether or not the contract between Acuña
Therefore, the body was then cognizant that the and Verano is binding with the corporation.
offer had already been accepted in the name of
Orientalist Company and that the films which RULING:
were then expected to arrive were being
imported by virtue of such acceptance. YES.

A perusal of the complaint reveals that it


contains sufficient allegations indicating such
3. In affixing his signature to the contracts, approval or at least subsequent ratification. On the
Fernandez was a guarantor. From the first point note the following averments: that on May
testimony of both Ramirez and Fernandez, the 9th the plaintiff met with each and all of the
Court was convinced that the responsibility of individual defendants (who constituted the entire
the later was that of a guarantor. Fernandez Board of Directors) and discussed with them
said that his name was signed as a guaranty extensively the tentative agreement and he was
that the contract would be approved by the made to understand that it was acceptable to them,
corporation, while Ramirez said that the name except as to plaintiff's remuneration; that it was
was put on the contract for the purpose of finally agreed between plaintiff and all said Directors
guaranteeing its performance. The Court that his remuneration would be P0.30 per kilo (of
believed that the latter was the real intention of tobacco); and that after the agreement was formally
the parties. executed he was assured by said Directors that
there would be no need of formal approval by the
Board. It should be noted in this connection that
although the contract required such approval it did
not specify just in what manner the same should be
EMILIANO ACUÑA given.
vs. On the question of ratification the complaint
BATAC PRODUCERS alleges that plaintiff delivered to the defendant
GR L-20333, 30 June 1963 corporation the sum of P20,000.00 as called for in
the contract; that he rendered the services he was
FACTS: required to do; that he furnished said defendant
3,000 sacks at a cost of P6,000.00 and advanced to
it the further sum of P5,000.00; and that he did all of In 1953, the appellants initiated the present action,
these things with the full knowledge, acquiescence contending that three Negros sugar centrals (La
and consent of each and all of the individual Carlota, Binalbagan-Isabela and San Carlos), with a
defendants who constitute the Board of Directors of total annual production exceeding one-third of the
the defendant corporation. There is abundant production of all the sugar central mills in the
authority in support of the proposition that province, had already granted increased
ratification may be expressed or implied, and that participation (of 62.5%) to their planters, and that
implied ratification may take diverse forms, such as under paragraph 9 of the resolution, the appellee
by silence or acquiescence; by acts showing had become obligated to grant similar concessions
approval or adoption of the contract; or by to the plaintiffs (appellants herein).
acceptance and retention of benefits flowing
therefrom. The appellee Bacolod-Murcia Milling Co., inc.,
resisted the claim, and defended by urging that the
stipulations contained in the resolution were made
without consideration; that the resolution in question
was, therefore, null and void ab initio, being in effect
G.R. No. L-15092 May 18, 1962 a donation that was ultra vires and beyond the
powers of the corporate directors to adopt.
ALFREDO MONTELIBANO, ET AL., plaintiffs-
appellants, The court rendered judgment upholding the stand of
vs. the defendant Milling company, and dismissed the
BACOLOD-MURCIA MILLING CO., complaint. Plaintiffs appealed to this Court.
INC., defendant-appellee.

Facts:
Issue:
Plaintiffs-appellants, Alfredo Montelibano, Alejandro
Montelibano, and the Limited co-partnership W/N Bacolod-Murcia Milling is obligated to grant
Gonzaga and Company, had been and are sugar similar concessions to the plaintiffs (appellants
planters adhered to the defendant-appellee's sugar herein).
central mill under identical milling contracts.

Said contracts were stipulated to be in force for 30


years starting with the 1920-21 crop, and that the
Ruling: Yes.
resulting product should be divided in the ratio of
45% for the mill and 55% for the planters.
There can be no doubt that the directors of the
appellee company had authority to modify the
Sometime in 1936, it was proposed to execute
proposed terms of the Amended Milling Contract for
amended milling contracts, increasing the planters'
the purpose of making its terms more acceptable to
share to 60% of the manufactured sugar and the other contracting parties. The rule is that —
resulting molasses, besides other concessions, but
extending the operation of the milling contract from
the original 30 years to 45 years. To this effect, a It is a question, therefore, in each case of
printed Amended Milling Contract form was drawn the logical relation of the act to the
up. corporate purpose expressed in the charter.
If that act is one which is lawful in itself, and
not otherwise prohibited, is done for the
The Board of Directors of the appellee Bacolod-
purpose of serving corporate ends, and is
Murcia Milling Co., Inc., adopted a resolution
reasonably tributary to the promotion of
granting further concessions to the planters over
those ends, in a substantial, and not in a
and above those contained in the printed Amended
remote and fanciful sense, it may fairly be
Milling Contract. considered within charter powers. The test
to be applied is whether the act in question
Appellants signed and executed the printed is in direct and immediate furtherance of the
Amended Milling Contract but a copy of the corporation's business, fairly incident to the
resolution signed by the Central's General Manager, express powers and reasonably necessary
was not attached to the printed contract. to their exercise. If so, the corporation has
the power to do it; otherwise, not.
As the resolution in question was passed in good are barred from intruding into business judgements
faith by the board of directors, it is valid and binding, of corporations, when the same are made in good
and whether or not it will cause losses or decrease faith. The same rule precludes the reversal of the
the profits of the central, the court has no authority decision of the PSE, to which PALI had previously
to review them. agreed to comply, the PSE retains the discretion to
accept of reject applications for listing. Thus, even if
They hold such office charged with the duty to act an issuer has complied with the PSE listing rules
for the corporation according to their best judgment, and requirements, PSE retains the discretion to
accept or reject the issuer’s listing application if the
and in so doing they cannot be controlled in the
PSE determines that the listing shall not serve the
reasonable exercise and performance of such duty. interests of the investing public.
Whether the business of a corporation should be It is undeniable that the petitioner PSE is not an
operated at a loss during depression, or close down ordinary corporation, in that although it is clothed
at a smaller loss, is a purely business and economic with the markings of a corporate entity, it functions
problem to be determined by the directors of the as the primary channel through which the vessels of
corporation and not by the court. It is a well-known capital trade ply. The PSEi’s relevance to the
continued operation and filtration of the securities
rule of law that questions of policy or of transaction in the country gives it a distinct color of
management are left solely to the honest decision of importance such that government intervention in its
officers and directors of a corporation, and the court affairs becomes justified, if not necessarily. Indeed,
is without authority to substitute its judgment of the as the only operational stock exchange in the
board of directors; the board is the business country today, the PSE enjoys monopoly of
manager of the corporation, and so long as it acts in securities transactions, and as such it yields a
monopoly of securities transactions, and as such, it
good faith its orders are not reviewable by the
yields an immerse influence upon the country’s
courts. economy.
The SEC’s power to look into the subject ruling of
the PSE, therefore, may be implied from or be
Philippine Stock Exchange Inc. vs Court of considered as necessary or incidental to the
Appeals carrying out of the SEC’s express power to insure
281 SCRA 232 [GR No. 125469 October 27, 1997] fair dealing in securities traded upon a stock
Facts: The Puerto Azul Land Inc. (PALI), a exchange or to ensure the fair administration of
domestic real estate corporation, had sought to offer such exchange. It is likewise, observed that the
its shares to the public in order to raise funds principal function of the SEC is the supervision and
allegedly to develop its properties and pay its loans control over corporations, partnerships and
with several banking institutions. In January, 1995, associations with the end in view that investment in
PALI was issued a permit to sell its shares to the these entities may be encouraged and protected
public by the Securities and Exchange Commission and their activities for the promotion of economic
(SEC). To facilitate the trading of its shares among development.
investors, PALI sought to course the trading of its A corporation is but an association of individuals,
shares through the Philippine Stock Exchange Inc. allowed to transact under an assumed corporate
(PSEi), for which purpose it filed with the said stock name, and with a distinct legal personality. In
exchange an application to list its shares, with organizing itself as a collective body, it waives no
supporting documents attached pending the constitutional immunities and requisites appropriate
approval of the PALI’s listing application, a letter to such a body as to its corporate and management
was received by PSE from the heirs of Ferdinand decisions, therefore, the state will generally not
Marcos to which the latter claims to be the legal and interfere with the same. Questions of policy and
beneficial owner of some of the properties forming management are left to the honest decision of the
part of PALI’s assets. As a result, PSE denied officers and directors of a corporation, and the
PALI’s application which caused the latter to file a courts are without authority to substitute their
complaint before the SEC. The SEC issued an judgements for the judgement of the board of
order to PSE to grant listing application of PALI on directors. The board is the business manager of the
the ground that PALI have certificate of title over its corporation and so long as it acts in good faith, its
assets and properties and that PALI have complied orders are not reviewable by the courts.
with all the requirements to enlist with PSE. In matters of application for listing in the market the
Issue: Whether or not the denial of PALI’s SEC may exercise such power only if the PSE’s
application is proper. judgement is attended by bad faith.
Held: Yes. This is in accord with the “Business The petitioner was in the right when it refused
Judgement Rule” whereby the SEC and the courts application of PALI, for a contrary ruling was not to
the best interest of the general public. The business harmony between the Ongs and the
Tius in FLADC, however, was shortlived because
G.R. No. 144476 April 8, 2003 the Tius, on February 23, 1996, rescinded the Pre-
Subscription Agreement. The Tius accused the
ONG YONG, JUANITA TAN ONG, WILSON T. Ongs of:
ONG, ANNA L. ONG, WILLIAM T. ONG, WILLIE T.
ONG, and JULIE ONG ALONZO, petitioners, (1) refusing to credit to them the FLADC
vs. shares covering their real property
DAVID S. TIU, CELY Y. TIU, MOLY YU GAW, contributions;
BELEN SEE YU, D. TERENCE Y. TIU, JOHN YU,
LOURDES C. TIU, INTRALAND RESOURCES (2) preventing David S. Tiu and Cely Y. Tiu
DEVELOPMENT CORP., MASAGANA from assuming the positions of and
TELAMART, INC., REGISTER OF DEEDS OF performing their duties as Vice-President
PASAY CITY, and the SECURITIES AND and Treasurer, respectively, and
EXCHANGE COMMISSION, respondents.
(3) refusing to give them the office spaces
x-----------------------------x agreed upon.

G.R. No. 144629 April 8, 2003 In their defense, the Ongs said that David S. Tiu
and Cely Y. Tiu had in fact assumed the positions of
DAVID S. TIU, CELY Y. TIU, MOLY YU GAW, Vice-President and Treasurer of FLADC but that it
BELEN SEE YU, D. TERENCE Y. TIU, JOHN YU, was they who refused to comply with the corporate
LOURDES C. TIU, and INTRALAND RESOURCES duties assigned to them.
DEVELOPMENT CORP., petitioners,
vs. This case was commenced by the Tius on February
ONG YONG, JUANITA TAN ONG, WILSON T. 27, 1996 at the SEC, seeking confirmation of their
ONG, ANNA L. ONG, WILLIAM T. ONG, WILLIE T. rescission of the Pre-Subscription Agreement. After
ONG, and JULIA ONG ALONZO, respondents. hearing, the SEC issued a decision confirming the
rescission sought by the Tius.
FACTS:
SEC en banc affirmed and confirmed the rescission
of the Pre-Subscription Agreement.
In 1994, the construction of the Masagana Citimall
in Pasay City was threatened with stoppage and On appeal, the Court of Appeals likewise affirmed
incompletion when its owner, the First Landlink the SEC decision. However, the CA concluded that
Asia Development Corporation (FLADC), which both the Ongs and the Tius were in pari delicto
was owned by the Tius, encountered dire financial (which would not have legally entitled them to
difficulties. It was heavily indebted to the Philippine rescission) but, "for practical considerations," that is,
National Bank (PNB) for P190 million. their inability to work together, it was best to
separate the two groups by rescinding the Pre-
To stave off foreclosure of the mortgage on the two Subscription Agreement, returning the original
lots where the mall was being built, the Tius invited investment of the Ongs and awarding practically
the Ongs to invest in FLADC. everything else to the Tius.

Under the Pre-Subscription Agreement they entered Both parties filed separate petitions for review
into, the Ongs and the Tius agreed to maintain before the Supreme Court.
equal shareholdings in FLADC.
This Court affirmed the fact that both the Ongs and
Furthermore, they agreed that the Tius were entitled the Tius violated their respective obligations under
to nominate the Vice-President and the Treasurer the Pre-Subscription Agreement. The Ongs
plus five directors while the Ongs were entitled to prevented the Tius from assuming the positions of
nominate the President, the Secretary and six Vice-President and Treasurer of the corporation. On
directors (including the chairman) to the board of the other hand, the Decision established that the
directors of FLADC. Tius failed to turn over FLADC funds to the Ongs
and that the Tius diverted rentals due to FLADC to
their MATTERCO account. Consequently, it held
Moreover, the Ongs were given the right to manage
that rescission was not possible since both parties
and operate the mall.
were in pari delicto.
stock and (2) the SEC to approve said decrease.
However, this Court agreed with the Court of This new argument has no merit.
Appeals that the remedy of specific performance, as
espoused by the Ongs, was not practical and sound The Tius' case for rescission cannot validly be
either and would only lead to further "squabbles and deemed a petition to decrease capital stock
numerous litigations" between the parties. because such action never complied with the formal
requirements for decrease of capital stock under
ISSUE: Section 33 of the Corporation Code. No majority
vote of the board of directors was ever taken.
Whether or not the Tius could legally rescind the Neither was there any stockholders meeting at
Pre-Subscription Agreement. which the approval of stockholders owning at least
two-thirds of the outstanding capital stock was
RULING: secured. There was no revised treasurer's affidavit
and no proof that said decrease will not prejudice
In the instant case, the rescission of the Pre- the creditors' rights. On the contrary, all their
Subscription Agreement will effectively result in the pleadings contained were alleged acts of violations
unauthorized distribution of the capital assets and by the Ongs to justify an order of rescission.
property of the corporation, thereby violating the
Trust Fund Doctrine and the Corporation Code, Furthermore, it is an improper judicial intrusion
since rescission of a subscription agreement is not into the internal affairs of the corporation to
one of the instances when distribution of capital compel FLADC to file at the SEC a petition for
assets and property of the corporation is allowed. the issuance of a certificate of decrease of
stock. Decreasing a corporation's authorized
Contrary to the Tius' allegation, rescission will, in capital stock is an amendment of the Articles of
the final analysis, result in the premature liquidation Incorporation. It is a decision that only the
of the corporation without the benefit of prior stockholders and the directors can make,
dissolution in accordance with Sections 117, 118, considering that they are the contracting parties
119 and 120 of the Corporation Code. thereto. In this case, the Tius are actually not just
asking for a review of the legality and fairness of a
The Tius maintain that rescinding the subscription corporate decision. They want this Court to make a
contract is not synonymous to corporate liquidation corporate decision for FLADC. We decline to
because all rescission will entail would be the intervene and order corporate structural changes
simple restoration of the status quo ante and a not voluntarily agreed upon by its stockholders and
return to the two groups of their cash and property directors.
contributions. We wish it were that simple. Very
noticeable is the fact that the Tius do not explain Truth to tell, a judicial order to decrease capital
why rescission in the instant case will not effectively stock without the assent of FLADC's directors and
result in liquidation. The Tius merely refer in cavalier stockholders is a violation of the "BUSINESS
fashion to the end-result of rescission (which JUDGMENT RULE" which states that:
incidentally is 100% favorable to them) but turn a
blind eye to its unfair, inequitable and disastrous xxx xxx xxx (C)ontracts intra vires entered
effect on the corporation, its creditors and the Ongs.
into by the board of directors are binding
upon the corporation and courts will not
The Tius claim that rescission of the agreement will interfere unless such contracts are so
not result in an unauthorized liquidation of the unconscionable and oppressive as to
corporation because their case is actually a petition amount to wanton destruction to the rights
to decrease capital stock pursuant to Section 38 of the minority, as when plaintiffs aver that
of the Corporation Code. Section 122 of the law the defendants (members of the board),
provides that "(e)xcept by decrease of capital have concluded a transaction among
stock…, no corporation shall distribute any of its themselves as will result in serious injury to
assets or property except upon lawful dissolution the plaintiffs stockholders.29
and after payment of all its debts and liabilities." The
Tius claim that their case for rescission, being a
The reason behind the rule is aptly explained by
petition to decrease capital stock, does not violate Dean Cesar L. Villanueva, an esteemed author in
the liquidation procedures under our laws. All that corporate law, thus:
needs to be done, according to them, is for this
Court to order (1) FLADC to file with the SEC a
petition to issue a certificate of decrease of capital
Courts and other tribunals are wont to received Audit Memorandum issued by the
override the business judgment of the board Commission on Audit ("COA"). This Memorandum
mainly because, courts are not in the noted that cash advances received by officers and
business of business, and the laissez faire
employees of petitioner Beneco had been virtually
rule or the free enterprise system prevailing
in our social and economic set-up dictates written off in the books of Beneco. In the Audit
that it is better for the State and its organs Memorandum, the COA directed petitioner Beneco
to leave business to the businessmen; to secure the approval of the National Electrification
especially so, when courts are ill-equipped Administration ("NEA") before writing off or
to make business decisions. More condoning those cash advances, and
importantly, the social contract in the recommended the adoption of remedial measures.
corporate family to decide the course of the
corporate business has been vested in the Having been made aware of the serious financial
board and not with courts.30
condition of Beneco and what appeared to be
mismanagement, respondent Cosalan initiated
Apparently, the Tius do not realize the illegal
consequences of seeking rescission and control of implementation of the remedial measures
the corporation to the exclusion of the Ongs. Such recommended by the COA. The respondent
an act infringes on the law on reduction of capital members of the Board of Beneco reacted by
stock. Ordering the return and distribution of the adopting a series of resolutions. These Board
Ongs' capital contribution without dissolving the Resolutions abolished the housing allowance of
corporation or decreasing its authorized capital respondent Cosalan; reduced his salary and his
stock is not only against the law but is also
representation and commutable allowances;
prejudicial to corporate creditors who enjoy absolute
priority of payment over and above any individual directed him to hold in abeyance all pending
stockholder thereof. personnel disciplinary actions; and struck his name
out as a principal signatory to transactions of
Stripped to its barest essentials, the issue of petitioner Beneco.
rescission in this case is not difficult to understand.
If rescission is denied, will injustice be inflicted on The respondent Beneco Board members adopted
any of the parties? The answer is no because the another series of resolutions which resulted in the
financial interests of both the Tius and the Ongs will ouster of respondent Cosalan as General Manager
remain intact and safe within FLADC. On the other of Beneco and his exclusion from performance of
hand, if rescission is granted, will any of the parties
his regular duties as such, as well as the
suffer an injustice? Definitely yes because the Ongs
will find themselves out in the streets with nothing withholding of his salary and allowances.
but the money they had in 1994 while the Tius will
not only enjoy a windfall estimated to be anywhere Cosalan then filed a complaint for illegal dismissal
from P450 million to P900 million but will also take against the BENECO Board Members, he later
over an extremely profitable business without much impleaded BENECO itself. The Labor Arbiter (LA)
effort at all. ruled in favor of Cosalan. The National Labor
Relations Commission (NLRC) affirmed the decision
of the LA but modified it so as to absolve the Board
Members from liability as it held that the Board
G.R. No. 89070 May 18, 1992
Members merely acted in their official capacity.
BENGUET ELECTRlC COOPERATIVE, INC., BENECO, being the only party adjudged to be
petitioner, vs. liable, then appealed said decision.

NATIONAL LABOR RELATIONS COMMISSION, Issue: Whether or not the National Labor Relations
PETER COSALAN and BOARD OF DIRECTORS Commission is correct?
OF BENGUET ELECTRIC COOPERATIVE, INC., *
Ruling: No. The act of the Board Members is ultra
respondents.
vires. There was no legal basis for them to suspend
Facts: Private respondent Peter Cosalan was the Cosalan indefinitely for under the Implementing
General Manager of Petitioner Benguet Electric Rules of the Labor Code the maximum period form
Cooperative, Inc. ("Beneco"). respondent Cosalan preventive suspension should not go beyond 30
days. Further, it was found that Cosalan was never contended that the Board acted without authority
informed of the charges against him nor was he and in usurpation of the power of the stockholders.
afforded the opportunity to present his case. He was Petitioner averred that the membership of the Board
of Directors had changed since the authority was
deprived of due process. Nor was Cosalan’s
given in 1961, there being six (6) new directors.
suspension approved by the NEA, which is also It was claimed that prior to the questioned
required for due process purposes. March 13, 1961 amendment, petitioner had all the
qualifications to be a director of respondent
These acts by the Board Members are tainted with corporation, being a substantial stockholder thereof;
bad faith. A very strong presumption arises that the that as a stockholder, petitioner had acquired rights
Board Members are acting in reprisal against the inherent in stock ownership, such as the rights to
reforms sought to be introduced by Cosalan in order vote and to be voted upon in the election of
to address the irregularities within BENECO. The directors; and that in amending the by-laws,
respondents purposely provided for petitioner's
Board Members are therefore liable for damages
disqualification and deprived him of his vested right
under Section 31 of the Corporation Code. And as afore-mentioned, hence the amended by-laws
even though BENECO is a cooperative, it is still are null and void.
covered by the Corporation Code because under
PD 269, cooperatives are considered as ISSUE:
corporations.
Whether or not the disqualification of
Gokongwei Jr. to run for directorship of the
corporation valid, as such was only provided in the
The Supreme Court ruled that BENECO and the amended by-laws of the corporation.
BENECO Board Members are liable for the
RULING:
damages caused against Cosalan. However
BENECO can seek reimbursement from the Board YES.
Members so as not to unduly penalize the innocent
members of BENECO. It is recognized by all authorities that 'every
corporation has the inherent power to adopt by-laws
'for its internal government, and to regulate the
conduct and prescribe the rights and duties of its
members towards itself and among themselves in
reference to the management of its affairs.'" At
JOHN GOKONGWEI, JR., petitioner common law, the rule was "that the power to make
vs. and adopt by-laws was inherent in every corporation
SECURITIES AND EXCHANGE COMMISSION, as one of its necessary and inseparable legal
ANDRES M. SORIANO et.al. respondents. incidents.
G.R. No. L-45911 April 11, 1979. Any person "who buys stock in a
corporation does so with the knowledge that its
FACTS: affairs are dominated by a majority of the
stockholders and that he impliedly contracts that the
Petitioner alleged that on September 18, will of the majority shall govern in all matters within
1976, individual respondents amended the by-laws the limits of the act of incorporation and lawfully
of the corporation, basing their authority to do so on enacted by-laws and not forbidden by law."
a resolution of the stockholders adopted on March Under section 22 of the same law, the
13, 1961. It was contended that according to section owners of the majority of the subscribed capital
22 of the Corporation Law and Article VIII of the by- stock may amend or repeal any by-law or adopt
laws of the corporation, the power to amend, new by-laws. It cannot be said, therefore, that
modify, repeal or adopt new by-laws may be petitioner has a vested right to be elected director,
delegated to the Board of Directors only by the in the face of the fact that the law at the time such
affirmative vote of stockholders representing not right as stockholder was acquired contained the
less than 2/3 of the subscribed and paid up capital prescription that the corporate charter and the by-
stock of the corporation, which 2/3 should have law shall be subject to amendment, alteration and
been computed on the basis of the capitalization at modification.
the time of the amendment. Since the amendment
was based on the 1961 authorization, petitioner
Detective & Protective Bureau INC. vs Cloribel
Facts: be maintained. Petitioner contended that
respondent Alberto had arrogated to himself the
Detective and Protective Bureau, Inc., therein power of the Board of Directors of the corporation
plaintiff (petitioner herein) against Fausto S. Alberto, because he refused to vacate the office and
therein defendant (respondent herein), for surrender the same to Jose de la Rosa who had
accounting with preliminary injunction and been elected managing director by the Board to
receivership. succeed him. This assertion, however, was disputed
by respondent Alberto who stated that Jose de la
plaintiff was a corporation duly organized and
Rosa could not be elected managing director
existing under the laws of the Philippines; that
because he did not own any stock in the
defendant was managing director of plaintiff
corporation.
corporation from 1952 until January 14, 1964; that
in June, 1963, defendant illegally seized and took Issue:
control of all the assets as well as the books,
records, vouchers and receipts of the corporation Whether or not a person maybe elected as a
from the accountant-cashier, concealed them managing director even if he has no does not own
illegally and refused to allow any member of the any stock in the corporation
corporation to see and examine the same; that on
January 14, 1964, the stockholders, in a meeting, Held:
removed defendant as managing director and
There is in the record no showing that Jose de la
elected Jose de la Rosa in his stead; that defendant
Rosa owned a share of stock in the corporation. If
not only had refused to vacate his office and to
he did not own any share of stock, certainly he
deliver the assets and books to Jose de la Rosa,
could not be a director pursuant to the mandatory
but also continued to perform unauthorized acts for
provision of Section 30 of the Corporation Law,
and in behalf of plaintiff corporation; that defendant
which in part provides:
had been required to submit a financial statement
and to render an accounting of his administration Sec. 30. Every director must own in his own right at
from 1952 but defendant has failed to do so; that least one share of the capital stock of the stock
defendant, contrary to a corporation of which he is a director, which stock
shall stand in his name on the books of the
resolution adopted by the Board of Directors on
corporations....
November 24, 1963, had been illegally disposing of
corporate funds; that defendant, unless immediately If he could not be a director, he could also not be a
restrained ex-parte, would continue discharging the managing director of the corporation, pursuant to
functions of managing director; and that it was Article V, Section 3 of the By-Laws of the
necessary to appoint a receiver to take charge of Corporation which provides that:
the assets and receive the income of the
corporation. Plaintiff prayed that a preliminary The manager shall be elected by the Board of
injunction ex-parte be issued restraining defendant Directors from among its members.... (Record, p.
from exercising the functions of managing director 48)
and from disbursing and disposing of its funds; that
Jose M. Barredo be appointed receiver; that, after If the managing director-elect was not qualified to
judgment, the injunction be made permanent and become managing director, respondent Fausto
defendant be ordered to render an accounting. Alberto could not be compelled to vacate his office
and cede the same to the managing director-elect
The preliminary injunction was not approved by the because the bylaws of the corporation provides in
respondent judge because respondent alberto filed Article IV, Section 1 that "Directors shall serve until
a counter-bond. Which prompt plaintiff file a the election and qualification of their duly qualified
certiorari case alleging grave abuse of discretion on successor."
the part of the judge and public interest demanded
that the writ enjoining respondent Fausto Alberto
from exercising the functions of managing director RAMON C. LEE and ANTONIO DM. LACDAO
vs. ALFA which was the case before the execution of
THE HON. COURT OF APPEALS, SACOBA the subject voting trust agreement. There appears
MANUFACTURING CORP., PABLO GONZALES, to be no dispute from the records that DBP has
JR. and THOMAS GONZALES taken over full control and management of the firm.
G.R. No. 93695 February 4, 1992

FACTS: PREMIUM MARBLE RESOURCES, INC.


vs.
In 1985, a complaint for sum of money was THE COURT OF APPEALS and
filed by the International Corporate Bank, Inc. INTERNATIONAL CORPORATE BANK
against the private respondents who, in turn, filed a 1996 Nov 4, G.R. No. 96551
third party complaint against Alfa Integrated Textile
Mills (ALFA) and the petitionersRamon C. Lee and FACTS:
Antonio Dm. Lacdao who were officers of ALFA.
Meanwhile, in 1988, the trial court issued an order Herein petitioner filed a case for damages
requiring the issuance of an alias summons upon against respondent for allowing clearance of checks
ALFA through the DBP as a consequence of the by unauthorized officers of the former, to the
petitioners' letter informing the court that the former’s prejudice. However this case was opposed
summons for ALFA was erroneously served upon by some members of the petitioner on the ground
them considering that the management of ALFA that the filing of the complaint was not authorized by
had been transferred to the Development Bank of the Board. Hence, a resolution of this case was
the Philippines (DBP). necessary to litigate the claim of the petitioner for
In a manifestation, the DBP claimed that it damages against the respondent bank.
was not authorized to receive summons on behalf of
ALFA since the DBP had not taken over the ISSUE:
company which has a separate and distinct
corporate personality and existence. Whether or not the filing was authorized by
a duly constituted Board of Directors of the
ISSUE: petitioner corporation.

Whether or not despite the execution of the RULING:


Voting Trust Agreement, the summons be served
upon the petitioners who were officers and directors NO.
of ALFA (the trustor).
The petitioners asserted that the Board
RULING: authorized such filing. However, from the records of
the case as well as that of the corporation, no
NO. evidence was seen and shown that the results of
the election where the supposed members of the
There is no dispute as to the most Board who allegedly authorized the filing were filed
immediate effect of a voting trust agreement on the with the Securities and Exchange Commission. The
status of a stockholder who is a party to its Corporation Code mandates that within thirty (30)
execution from legal titleholder or owner of the days after the election of the directors, trustees and
shares subject of the voting trust agreement, he officers of the corporation, the secretary, or any
becomes the equitable or beneficial owner. other officer of the corporation, shall submit to the
Note that in order to be eligible as a Securities and Exchange Commission, the names,
director, what is material is the legal title to, not nationalities and residences of the directors,
beneficial ownership of, the stock as appearing on trustees and officers elected. Failure to comply with
the books of the corporation such requirement, the elected members cannot be
Considering that the voting trust agreement considered as the duly constituted and elected
between ALFA and the DBP transferred legal members of the Board. Hence, being not duly
ownership of the stocks covered by the agreement constituted, the filing of the case was not authorized
to the DBP as trustee, the latter became the by the Board.
stockholder of record with respect to the said shares
of stocks. In the absence of a showing that the DBP
had caused to be transferred in their names one
share of stock for the purpose of qualifying as
directors of ALFA, the petitioners can no longer be
deemed to have retained their status as officers of
Grace Christian High School vs. Court of past elections should be reexamined." Following
Appeals this advice, notices were sent to the members of the
[GR 108905, 23 October 1997] association that the provision on election of
Second Division, Mendoza (J): 2 concur, 1 on leave directors of the 1968 by-laws of the association
would be observed. The school requested the
Facts: Grace Christian High School is an chairman of the election committee to change the
educational institution offering preparatory, notice of election by following the procedure in
kindergarten and secondary courses at the Grace previous elections, claiming that the notice issued
Village in Quezon City. Grace Village Association, for the 1990 elections ran "counter to the practice in
Inc., on the other hand, is an organization of lot previous years" and was "in violation of the by-laws
and/or building owners, lessees and residents at (of 1975)" and "unlawfully deprive[d] Grace
Grace Village, while Alejandro G. Beltran and Christian High School of its vested right [to] a
Ernesto L. Go were its president and chairman of permanent seat in the board." As the association
the committee on election, respectively, in 1990, denied its request, the school brought suit for
when this suit was brought. As adopted in 1968, the mandamus in the Home Insurance and Guaranty
by-laws of the association provided in Article IV, that Corporation to compel the board of directors of the
"the annual meeting of the members of the association to recognize its right to a permanent
Association shall be held on the first Sunday of seat in the board. The school based its claim on the
January in each calendar year at the principal office following portion of the proposed amendment which,
of the Association at 2:00 P.M. where they shall it contended, had become part of the by-laws of the
elect by plurality vote and by secret balloting, the association as Article VI, paragraph 2. It appears
Board of Directors, composed of 11 members to that the opinion of the Securities and Exchange
serve for one year until their successors are duly Commission on the validity of this provision was
elected and have qualified." It appears, that on 20 sought by the association and that in reply to the
December 1975, a committee of the board of query, the SEC rendered an opinion to the effect
directors prepared a draft of an amendment to the that the practice of allowing unelected members in
by-laws, providing that "The Annual Meeting of the the board was contrary to the existing by-laws of the
members of the Association shall be held on the association and to §92 of the Corporation Code.
second Thursday of January of each year. Each The association cited the SEC opinion, among
Charter or Associate Member of the Association is others, in its answer. A preliminary conference was
entitled to vote. He shall be entitled to as many held on 29 March 1990 but nothing substantial was
votes as he has acquired thru his monthly agreed upon. A subsequent meeting was held on 17
membership fees only computed on a ratio of TEN April 1990, but the parties failed to reach an
(P10.00) PESOS for one vote. The Charter and agreement. Instead, the board adopted a resolution
Associate Members shall elect the Directors of the declaring the 1975 provision null and void for lack of
Association. The candidates receiving the first 14 approval by members of the association and the
highest number of votes shall be declared and 1968 by-laws to be effective. On 20 June 1990, the
proclaimed elected until their successors are hearing officer of the HIGC rendered a decision
elected and qualified. GRACE CHRISTIAN HIGH dismissing the school's action. The appeals board
SCHOOL representative is a permanent Director of of the HIGC affirmed the decision of the hearing
the ASSOCIATION." This draft was never officer in its resolution dated 13 September 1990.
presented to the general membership for approval. Petitioner appealed to the Court of Appeals but the
Nevertheless, from 1975, after it was presumably school again lost as the appellate court on 9
submitted to the board, up to 1990, Grace Christian February 1993, affirmed the decision of the HIGC.
High School was given a permanent seat in the The school filed the petition for review.
board of directors of the association. On 13
February 1990, the association's committee on Issue: Whether the school's representative should
election in a letter informed James Tan, principal of be elected to have the right to sit in the board of
the school, that "it was the sentiment that all directors of Grace Village Association, Inc. as a
directors should be elected by members of the member thereof.
association" because "to make a person or entity a
permanent Director would deprive the right of voters Held: It is actually §§28 and 29 of the Corporation
to vote for 15 members of the Board," and "it is Law — § 23 of the present law; not §92 of the
undemocratic for a person or entity to hold office in present law or §29 of the former one — which
perpetuity." For this reason, Tan was told that "the require members of the boards of directors of
proposal to make the Grace Christian High School corporations to be elected. The board of directors of
representative as a permanent director of the corporations must be elected from among the
association, although previously tolerated in the stockholders or members. There may be
corporations in which there are unelected members 1906. El Hogar Filipino, organized in 1911 under the
in the board but it is clear that in the examples cited laws of the Philippine Islands, was the first
by the school, the unelected members sit as ex corporation organized under Sec. 171-190 Act No.
officio members, i.e., by virtue of and for as long as
1459, devoted to the subject of building and loan
they hold a particular office. But in the case of the
school itself, there is no reason at all for its associations, their organization and administration.
representative to be given a seat in the board. Nor In the said law, the capital of the corporation was
does the school claim a right to such seat by virtue not permitted to exceed P3M, but Act No. 2092
of an office held. In fact it was not given such seat in amended the statute, permitting capitalization to the
the beginning. It was only in 1975 that a proposed amount of ten millions.
amendment to the by-laws sought to give it one.
Since the provision in question is contrary to law,
the fact that for 15 years it has not been questioned
or challenged but, on the contrary, appears to have El Hogar took advantage of the amendment of Act
been implemented by the members of the No. 1459 and amended its AOI as a result thereof,
association cannot forestall a later challenge to its
stating that the amount of capital must not exceed
validity. Neither can it attain validity through
acquiescence because, if it is contrary to law, it is what has been stated in Act No. 2092. This resulted
beyond the power of the members of the to El Hogar having 5,826 shareholders, 125,750
association to waive its invalidity. For that matter the shares with paid-up value of P8.7M. The
members of the association may have formally corporation paid P7.16M to its withdrawing
adopted the provision in question, but their action stockholders.
would be of no avail because no provision of the by-
laws can be adopted if it is contrary to law. It is
probable that, in allowing the school's
representative to sit on the board, the members of The Government of the Philippine Islands filed an
the association were not aware that this was
action against El Hogar due to the alleged illegal
contrary to law. It should be noted that they did not
actually implement the provision in question except holding title to real property for a period exceeding
perhaps insofar as it increased the number of five (5) years after the same was bought in a
directors from 11 to 15, but certainly not the foreclosure sale. Sec. 13(5) of the Corporation Law
allowance of the school's representative as an states that corporations must dispose of real estate
unelected member of the board of directors. It is obtained within 5 years from receiving the title. The
more accurate to say that the members merely Philippine Government also prays that El Hogar be
tolerated the school's representative and tolerance
excluded from all corporate rights and privileges
cannot be considered ratification. Nor can the
school claim a vested right to sit in the board on the and effecting a final dissolution of said corporation.
basis of "practice." Practice, no matter how long
continued, cannot give rise to any vested right if it is
contrary to law. Even less tenable is the school's
claim that its right is "coterminus with the existence It appears from the records that El Hogar was the
of the association." holder of a recorded mortgage on the San Clemente
land as security for a P24K loan to El Hogar.
However, shareholders and borrowers defaulted in
payment so El Hogar foreclosed the mortgage and
The Government of the Philippine Islands vs. El purchased the land during the auction sale. A deed
Hogar Filipino of conveyance in favor of El Hogar was executed
and sent to the Register of Deeds of Tralac with a
G.R. No. L-26649 July 13, 1917
request that the certificate of title be cancelled and a
new one be issued in favor of El Hogar from the
Register of Deeds of Tarlac. However, no reply was
Facts: received.

The Philippine Commission enacted Act No. 1459, El Hogar filed a complaint with the Chief of the
also known as the Corporation Law, on March 1, General Land Registration Office. The certificate of
title to the San Clemente land was received by El Art. 61 of El Hogar’s by-laws which states that
Hogar and a board resolution authorizing Benzon to “attendance in person or by proxy by shareholders
find a buyer was issued. Alcantara, the buyer of the owning one-half plus one of the shareholders shall
land, was given extension of time to make payment be necessary to constitute a quorum for the election
but defaulted so the contract treated rescinded. of directors” is contrary to Sec. 31 of the Corpo Law
Efforts were made to find another buyer. which provides that owners of the majority of the
Respondent acquired title in December 1920 until subscribed capital stock entitled to vote must be
the property was finally sold to Felipa Alberto in July present either in person or by proxy at all elections
1926. The interval exceeded 5 years but the period of directors.
did not commence to run until May 7, 1921 when
the register of deeds delivered the new certificate of Unless the law or the charter of a corporation
title. It has been held that a purchaser of land expressly provides that an office shall become
registered under the Torrens system cannot acquire vacant at the expiration of the term of office for
the status of an innocent purchaser for value unless which the officer was elected, the general rule is to
the vendor is able to place the owner’s duplicate in allow the officer to holdover until his successor is
his hands showing the title to be in the vendor. duly qualified. Mere failure of a corporation to elect
During the period before May 1921, El Hogar was officers does not terminate the terms of existing
not in a position to pass an indefeasible title to any officers nor dissolve the corporation.
purchaser. Therefore, El Hogar cannot be held
accountable for this delay which was not due to its The doctrine above stated finds expressions in
fault. Likewise, the period from March 25, 1926 to article 66 of the by-laws of the respondent which
April 20, 1926 must not be part of the five-year declares in so many words that directors shall hold
period because this was the period where office "for the term of one year on until their
respondent was under the obligation to sell the successors shall have been elected and taken
property to Alcantara prior to the contract’s possession of their offices." No fault can be imputed
rescission due to Alcantara’s non-payment. to the corporation on account of the failure of the
shareholders to attend the annual meetings and
their non-attendance in meetings is doubtless to be
interpreted in part as expressing their satisfaction of
Another circumstance causing the delay is the fact
the way in which things have been conducted.
that El Hogar purchased the property in the full
amount of the loan made by the former owner which
is nearly P24K when it was subsequently found that It result that the practice of the directorate of filling
the property was not salable and later sold for P6K vacancies by the action of the directors themselves
notwithstanding El Hogar’s efforts to find a is valid. Nor can any exception be taken to then
purchaser upon better terms. personality of the individuals chosen by the
directors to fill vacancies in the body. Certainly it is
no fair criticism to say that they have chosen
competent businessmen of financial responsibility
ISSUE: instead of electing poor persons to so responsible a
position. The possession of means does not
disqualify a man for filling positions of responsibility
W/N the practice of the directorate of filling in corporate affairs.
vacancies by the action of the directors themselves
is valid
DOMINGO PONCE AND BUHAY PONCE
vs.
Ruling: Yes. DEMETRIO B. ENCARNACION AND
POTENCIANO GAPOL
G.R. NO. L-5883 NOVEMBER 28, 1953

FACTS:
may issue an order to any stockholder or member of
Daguhoy Enterprises, Inc., was duly a corporation, directing him to call a meeting of the
registered as such on 24 June 1948. On 16 April corporation by giving the proper notice required by
1951 at a meeting duly called, the voluntary this Act or the by-laws; and if there be no person
dissolution of the corporation and the appointment legally authorized to preside at such meeting, the
of Gapol as receiver were agreed upon and to that judge of the Court of First Instance may direct the
end a petition for voluntary dissolution was drafted person calling the meeting to preside at the same
which was sent to, and signed by, the petitioner until a majority of the members or stockholders
Domingo Ponce. Instead of filing the petition for representing a majority of the stock present and
voluntary dissolution of the corporation as agreed permitted by law to be voted have chosen one of
upon, Gapol, who is the largest stockholder, their number to act as presiding officer for the
changed his mind and filed a complaint in the CFI of purposes of the meeting.
Manila to compel the petitioners to render an Petitioners were not deprived of their right
accounting of the funds and assets of the without due process of law. They had no right to
corporation, to reimburse it, jointly and severally, a continue as directors of the corporation unless
total sum of P18,690, plus interest, which have reelected by the stockholders in a meeting called for
been converted by the petitioner Domingo Ponce to that purpose every even year.
his own use and benefit.
On 18 May 1951 Gapol filed a motion
praying that the petitioners be removed as members
of the board of directors which was denied by the G.R. No. L-26555 November 16, 1926
court. On 3 January 1952 Gapol filed a petition
praying for an order directing him to call a meeting
of the stockholders of the corporation and to preside BALDOMERO ROXAS, ENRIQUE ECHAUS and
at such meeting in accordance with section 26 of ROMAN J. LACSON, petitioners,
the Corporation Law. Two-days later, without notice vs.
to the petitioners and to the other members of the Honorable MARIANO DE LA ROSA, Auxiliary
board of directors and in violation of the Rules of Judge of First Instance of Occidental Negros,
Court which require that the adverse parties be AGUSTIN CORUNA, MAURO LEDESMA and
notified of the hearing of the motion three days in BINALBAGAN ESTATE, INC., respondents.
advance, the respondent court issued the order as
prayed for. DOCTRINE: CONTROL AND MANAGEMENT OF
CORPORATION
ISSUE:
Removal of Directors: Under the law the directors of
Whether or not under and pursuant to a corporation can only be removed from office by a
section 26 of the Corporation Law, the respondent vote of the stockholders representing at least two-
court may issue the order complained of. thirds of the subscribed capital stock entitled to vote
(Act No. 1459, sec. 34); while vacancies in the
RULING: board, when they exist, can be filled by mere
majority vote, (Act No. 1459, sec. 25).
NO.
Moreover, the law requires that when action is to be
Article 9 of the by-laws of the Daguhoy taken at a special meeting to remove the directors,
Enterprises, Inc., provides: The Board of Directors such purpose shall be indicated in the call (Act No.
shall compose of five (5) members who shall be 1459, sec. 34)
elected by the stockholders in a general meeting
called for that purpose which shall be held every SUMMARY:
even year during the month of January. Article 22 of Representatives of the voting trust, holding majority
the by-laws provides: The Chairman shall have the of the shares, calls for a shareholders meeting with
right to fix the date, the time and the place where the purpose of electing the members of the board of
the general meeting shall be held, either special or directors notwithstanding the fact that all the
general. positions in the board are occupied by the members
Section 26 of the Corporation Code elected in a previous shareholders meeting. A civil
provides: - Whenever, from any cause, there is no action was filed to enjoin such meeting and the
person authorized to call a meeting, or when the petitioners filed a certiorari proceeding for the
officer authorized to do so refuses, fails, or neglects issuance of the CFI judge of a restraining order to
to call a meeting, any judge of a Court of First enjoin the meeting. SC held that the restraining
Instance, on the showing of good cause therefor, order was valid because in order to remove the
current members of the BOD, a vote of at least 2/3 Echaus, and Lacson presumably constitute its
of the shareholders is necessary. membership.

FACTS: The current members of the voting trust (petitioners)


wanted to oust the current officers/directors of the
Binalbagan Estate, Inc. (BEI), is a corporation corporation, even though it was the previous
having its principal plant in Occidental Negros representative of the voting trust (Heilbronn) who
where it is engaged in the manufacture of raw sugar elected them. Thus, the petitioners in their character
from canes grown upon farms accessible to its as members of the voting trust, on August 2, 1926,
central. caused the secretary of the Binalbagan Estate, Inc.,
to issue to the shareholders a notice calling for a
In July, 1924, the possessors of a majority of the special general meeting of shareholders to be
shares of the Binalbagan Estate, Inc., formed a held at 10 a. m., on August 16, 1926, "for the
voting trust composed of three members, namely, election of the board of directors, for the
Salvador Laguna, Segunda Monteblanco, and amendment of the By-Laws, and for any other
Arthur F. Fisher, as trustee. business that can be dealt with in said meeting."

By the document constituting this voting trust, the Respondents Coruna and Ledesma, as director and
trustees were authorized to represent and vote the shareholder of the corporation respectively, filed a
shares pertaining to their constituents, and to this civil action before CFI to enjoin the meeting to be
end the shareholders undertook to assign their held on Aug. 16, 1926. Respondent judge De La
shares to the trustees on the books of the company. Rosa issued a restraining order or preliminary
injunction to enjoin the meeting which gave rise to
The total number of outstanding shares of the the present certiorari proceeding filed by petitioners.
corporation is somewhat over 5,500, while the
number of shares controlled by the voting trust ISSUE:
is less than 3,000.
Whether or not it was within the judicial powers of
On 26 Feb 1926, BEI held its General Annual Judge De La Rosa to issue the restraining order or
Shareholders Meeting at which Mr. J. P. Heilbronn preliminary injunction? (YES)
appeared as representative of the voting trust,
his authority being recognized by the holders of all MAIN ISSUE: W/N the petitioners can hold another
the other shares present at this meeting. shareholders meeting for the election of board of
directors even though no vacancies have occurred
Heilbronn having the control of the majority of to justify such election? (NO)
the shares (the case didn’t say how that happened
– maybe he owned several shares plus the shares RULING:
of the voting trust he was representing to make up
the majority – it’s just an inference) was able to Vacancies in the Board of Directors occur either due
nominate and elect a board of directors to his to death, resignation, removal, or otherwise. The
own liking, without opposition from the law requires that for a director to be removed, a
minority. vote of at least two-thirds of the subscribed capital
stock is necessary. In this case, the voting trust only
After the board of directors had been thus elected has the majority of the shares. Majority is not
and had qualified, they chose a set of officers equivalent to two-thirds.
constituting of Jose M. Yusay, president, Timoteo
Unson, vice-president, Jose G. Montalvo, secretary- It must be noted that there are no vacancies in the
treasurer, and H. W. Corp and Agustin Coruna, as board of directors. Therefore, a call for an election
members. Said officials immediately entered upon of the board of directors made by the petitioners is
the discharged of their duties and have continued in tantamount to an ousting of the current members of
possession of their respective offices until the the board. The present board of directors are de
present time. facto incumbents of the office whose acts will be
valid until they shall be lawfully removed from the
Since the creation of the voting trust there have office or cease from the discharge of their functions.
been a number of vacancies caused by resignation In this case it is not necessary for us to agitate
or the absence of members from the Philippine ourselves over the question whether the respondent
Islands, with the result that various substitutions judge properly exercised his judicial discretion in
have been made in the personnel of the voting trust. granting the order complained of. If suffices to know
At the present time the petitioners Roxas,
that in making the order he was acting within the appointment, addressed to the Board, from a
limits of his judicial powers. majority of the faculty and from a number of alumni
Dr. Blanco's appointment had lapsed.
On May 26, 1970, President Lopez
Now, upon examining into the number of shares extended another ad interim appointment to her,
controlled by the voting trust, it will be seen that, effective from May 26, 1970 to April 30, 1971, with
while the trust controls a majority of the stock, it the same conditions as the first.However, such ad
does not have a clear two-thirds majority. It was interim appointment had not been confirmed by the
therefore impolitic for the petitioners, in forcing the Board of Regents. Due to the following votes: 5-yes,
call for the meeting of August 16, to come out 3-no and 4-abstain.
frankly and say in the notice that one of the purpose On August 18, 1970 Dr. Blanco wrote the
of the meeting was to removed the directors of the President of the University, protesting the
corporation from office. Instead, the call was limited appointment of Oseas A. del Rosario as Officer-in-
to the election of the board of directors, it being the Charge of the College of Education. Neither
evident intention of the voting trust to elect a new communication having elicited any official reply, Dr.
board as if the directorate had been then vacant. Blanco went to the Court of First Instance of
Quezon City on a petition for certiorari and
But the complaint in civil No. 3840 directly asserts prohibition with preliminary injunction.
that the members of the present directorate were
ISSUE:
regularly elected at the general annual meeting held
in February, 1926; and if that assertion be true, the
proposal to elect, another directorate, as per the call Whether or not respondent Dr. Consuelo S.
of August 2, if carried into effect, would result in the Blanco was duly elected Dean of the College of
election of a rival set of directors, who would Education, University of the Philippines, in the
probably need the assistance of judgment of court meeting of the Board of Regents on July 9, 1970.
in an independent action of quo warranto to get
RULING:
them installed into office, even supposing that their
title to the office could be maintained. That the trial
judge had jurisdiction to forestall that step and NO.
enjoin the contemplated election is a matter about
which there cannot be the slightest doubt. The law The votes of abstention, viewed in their
contemplates and intends that there will be one of setting, can in no way be construed as votes for
directors at a time and that new directors shall be confirmation of the appointment. There can be no
elected only as vacancies occur in the directorate doubt whatsoever as to the decision and
by death, resignation, removal, or otherwise. recommendation of the three members of the
Personnel Committee: it was for rejection of the
appointment. No inference can be drawn from this
that the members of the Personnel Committee, by
SALVADOR P. LOPEZ their abstention, intended to acquiesce in the action
vs. taken by those who voted affirmatively. Neither, for
ERICTA that matter, can such inference be drawn from the
G.R. No. L-32991 June 29, 1972 abstention that he was abstaining because he was
not then ready to make a decision.
FACTS: Dr. Blanco was clearly not the choice of a
majority of the members of the Board of Regents,
The first such appointment was extended as unequivocally demonstrated by the transcript of
on April 27, 1970, "effective May 1, 1970 until April the proceedings. This fact cannot be ignored simply
30, 1971, unless sooner terminated and subject to because the Chairman, in submitting the question to
the appproval of the Board of Regents and to the actual vote, did not frame it as accurately as the
pertinent University regulations." Pursuant thereto preceding discussion called for, such that two of the
Dr. Blanco assumed office as ad interim Dean on Regents present (Silva and Kalaw) had to make
May 1, 1970. some kind of clarification.
The Board of Regents met on May 26,
1970, and President Lopez submitted to it the ad
Western Institute of Technology Inc. vs. Salas
interim appointment of Dr. Blanco for
[GR 113032, 21 August 1997]
reconsideration. The minutes of that meeting
disclose that "the Board voted to defer action on the First Division, Hermosisima Jr. (J): 4 concur
matter in view of the objections cited by Regent
Kalaw based on the petition against the
Facts: Ricardo T. Salas, Salvador T. Salas, against the accused therein. Villasis, et. al. filed a
Soledad Salas-Tubilleja, Antonio S. Salas, and Motion for Reconsideration of the civil aspect of the
Richard S. Salas, belonging to the same family, are RTC Decision which was, however, denied in an
the majority and controlling members of the Board Order dated 23 November 1993. Villasis, et. al. filed
of Trustees of Western Institute of Technology, Inc. the petition for review on certiorari. Significantly on
(WIT), a stock corporation engaged in the operation, 8 December 1994, a Motion for Intervention, dated
among others, of an educational institution. 2 December 1994, was filed before this Court by
According to Homero L. Villasis, Dimas Enriquez, Western Institute of Technology, Inc., disowning its
peston F. Villasis, and Reginald F. Villasis, the inclusion in the petition and submitting that Atty.
minority stockholders of WIT, sometime on 1 June Tranquilino R. Gale, counsel for Villasis, et. al., had
1986 in the principal office of WIT at La Paz, Iloilo no authority whatsoever to represent the
City, a Special Board Meeting was held. In corporation in filing the petition. Intervenor likewise
attendance were other members of the Board prayed for the dismissal of the petition for being
including Reginald Villasis. Prior to said Special utterly without merit. The Motion for Intervention
Board Meeting, copies of notice thereof, dated 24 was granted on 16 January 1995.
May 1986, were distributed to all Board Members.
The notice allegedly indicated that the meeting to be Issue: Whether the grant of compensation to Salas,
held on 1 June 1986 included Item 6 which states et. al. is proscribed under Section 30 of the
that "Possible implementation of Art. III, Sec. 6 of Corporation Code.
the Amended By-Laws of Western Institute of
Technology, Inc. on compensation of all officers of
Held: Directors or trustees, as the case may be, are
the corporation." In said meeting, the Board of
not entitled to salary or other compensation when
Trustees passed Resolution 48, series 1986, they perform nothing more than the usual and
granting monthly compensation to Salas, et. al. as
ordinary duties of their office. This rule is founded
corporate officers retroactive 1 June 1985, in the
upon a presumption that directors/trustees render
following amounts: “Chairman 9,000.00/month, Vice
service gratuitously, and that the return upon their
Chairman P3,500.00/month, Corporate Treasurer
shares adequately furnishes the motives for service,
P3,500.00/month and Corporate Secretary without compensation. Under Section 30 of the
P3,500.00/month, retroactive June 1, 1985 and the Corporation Code, there are only two (2) ways by
ten percentum of the net profits shall be distributed
which members of the board can be granted
equally among the ten members of the Board of
compensation apart from reasonable per diems: (1)
Trustees. This shall amend and supercede any
when there is a provision in the by-laws fixing their
previous resolution.” A few years later, or on 13
compensation; and (2) when the stockholders
March 1991, Homero Villasis, Preston Villasis, representing a majority of the outstanding capital
Reginald Villasis and Dimas Enriquez filed an stock at a regular or special stockholders' meeting
affidavit-complaint against Salas, et. al. before the
agree to give it to them. Also, the proscription,
Office of the City Prosecutor of Iloilo, as a result of
however, against granting compensation to
which 2 separate criminal informations, one for
director/trustees of a corporation is not a sweeping
falsification of a public document under Article 171
rule. Worthy of note is the clear phraseology of
of the Revised Penal Code and the other for estafa Section 30 which state: "[T]he directors shall not
under Article 315, par. 1(b) of the RPC, were filed receive any compensation, as such directors." The
before Branch 33 of the Regional Trial Court of Iloilo
phrase as such directors is not without significance
City. The charge for falsification of public document
for it delimits the scope of the prohibition to
was anchored on Salas, et. al.'s submission of
compensation given to them for services performed
WIT's income statement for the fiscal year 1985- purely in their capacity as directors or trustees. The
1986 with the Securities and Exchange Commission unambiguous implication is that members of the
(SEC) reflecting therein the disbursement of
board may receive compensation, in addition to
corporate funds for the compensation of Salas, et.
reasonable per diems, when they render services to
al. based on Resolution 4, series of 1986, making it
the corporation in a capacity other than as
appear that the same was passed by the board on
directors/trustees. Herein, resolution 48, s. 1986
30 March 1986, when in truth, the same was granted monthly compensation to Salas, et. al. not
actually passed on 1 June 1986, a date not covered in their capacity as members of the board, but
by the corporation's fiscal year 1985-1986
rather as officers of the corporation, more
(beginning May 1, 1995 and ending April 30, 1986).
particularly as Chairman, Vice-Chairman, Treasurer
Thereafter, trial for the two criminal cases (Criminal
and Secretary of Western Institute of Technology.
Cases 37097 and 37098), was consolidated. After a
Clearly, therefore, the prohibition with respect to
full-blown hearing, Judge Porfirio Parian handed granting compensation to corporate
down a verdict of acquittal on both counts dated 6 directors/trustees as such under Section 30 is not
September 1993 without imposing any civil liability
violated in this particular case. Consequently, the The privilege to utilize, exploit, and develop
last sentence of Section 30 which provides that "In the natural resources of this country was granted,
no case shall the total yearly compensation of by Article III of the Constitution, to Filipino citizens
directors, as such directors, exceed ten (10%) or to corporations or associations 60% of the capital
percent of the net income before income tax of the of which is owned by such citizens. With the Parity
corporation during the preceding year" does not Amendment to the Constitution, the same right was
likewise find application in this case since the extended to citizens of the United States and
compensation is being given to Salas, et. al. in their business enterprises owned or controlled, directly or
capacity as officers of WIT and not as board indirectly, by citizens of the United States.
members. There could be no serious doubt as to the
meaning of the word "citizens" used in the
aforementioned provisions of the Constitution. The
right was granted to 2 types of persons: natural
PALTING persons (Filipino or American citizens) and juridical
vs. persons (corporations 60% of which capital is
SAN JOSE PETROLEUM INC. owned by Filipinos and business enterprises owned
G.R. No.L-14441. December 17, 1966 or controlled directly or indirectly, by citizens of the
United States). In American law, "citizen" has been
FACTS: defined as "one who, under the constitution and
laws of the United States, has a right to vote for
SAN JOSE OIL, is a domestic mining representatives in congress and other public
corporation, 90% of the outstanding capital stock of officers, and who is qualified to fill offices in the gift
which is owned by respondent SAN JOSE of the people."
PETROLEUM, a foreign (Panamanian) corporation, SAN JOSE PETROLEUM an American
the majority interest of which is owned by OIL business is not entitled to parity rights in the
INVESTMENTS, INC., another foreign Philippines. In the circumstances, we have to hold
(Panamanian) company. that the respondent SAN JOSE PETROLEUM, as
This latter corporation in turn is wholly presently constituted, is not a business enterprise
(100%) owned by PANTEPEC OIL COMPANY, C. that is authorized to exercise the parity privileges
A., and PANCOASTAL PETROLEUM COMPANY, under the Parity Ordinance, the Laurel-Langley
C. A., both organized and existing under the laws of Agreement and the Petroleum Law. Its tie-up with
Venezuela. As of September 30, 1956, there were SAN JOSE OIL is, consequently, illegal.
9,979 stockholders of PANCOASTAL PETROLEUM
found in 49 American states and U.S. territories, Palting VS. San Jose Petroleum, Inc. (1966)
holding 3,476,988 shares of stock; whereas, as of
November 30, 1956, PANTEPEC OIL COMPANY Facts:
was said to have 3,077,916 shares held by 12,373 San Jose Petroleum filed with the SEC a
stockholders scattered in 49 American states. sworn registration statement for the registration
In the two lists of stockholders, there is no and licensing for sale in the Philippine voting trust
indication of the citizenship of these stockholders, or certificate representing 2 million shares of its
of the total number of authorized stocks of each capital stock of a par value of $0.35/share at
corporation for the purpose of determining the P1/share. It was alleged that the proceeds thereof
corresponding percentage of these listed will be used to finance the operations of San Jose
stockholders in relation to the respective capital Oil Co. which has 14 petroleum exploration
stock of said corporation. concessions in various provinces. It was
expressly conditioned that instead of stock
ISSUE: certificates, registered or bearer-voting trust
certificates from voting trustees (Americans) will
Whether or not the "tie-up" between the two be given. San Jose Petroleum amended the
corporations is violative of the Constitution, the application from P2M to P5M a reduced offering at
Laurel-Langley Agreement, the Petroleum Act of P0.70/share.
1949, and the Corporation Law. Palting, et.al filed with the SEC an
opposition to said registration on the following
RULING: grounds: (1) the tie-up between SJP, a
Panamanian corporation and SJO, a domestic
YES. corporation violates the Constitution, the Corp. Law
and the Petroleum Act of 1949 (2) the issuer is not
licensed to transact business in the Philippines (3)
the sale of shares is fraudulent (4) the issuer is entered into which this company either for their own
based on unsound business principles (sic). benefit, or for the benefit of any person, firm,
Some of the provisions of the Articles of association or corporation in which they may be
Incorporation of respondent SAN JOSE interested.
PETROLEUM are noteworthy; viz: The impact of these provisions upon the
(1) the directors of the Company need not traditional judiciary relationship between the
be shareholders; directors and the stockholders of a corporation is
(2) that in the meetings of the board of too obvious to escape notice by those who are
directors, any director may be represented and may called upon to protect the interest of investors. The
vote through a proxy who also need not be a directors and officers of the company can do
director or stockholder; and anything, short of actual fraud, with the affairs of the
(3) that no contract or transaction between corporation even to benefit themselves directly or
the corporation and any other association or other persons or entities in which they are
partnership will be affected, except in case of fraud, interested, and with immunity because of the
by the fact that any of the directors or officers of the advance condonation or relief from responsibility by
corporation is interested in, or is a director or officer reason of such acts. This and the other provision
of, such other association or partnership, and that which authorizes the election of non-stockholders
no such contract or transaction of the corporation as directors, completely disassociate the
with any other person or persons, firm, association stockholders from the government and
or partnership shall be affected by the fact that any management of the business in which they have
director or officer of the corporation is a party to or invested.
has an interest in, such contract or transaction, or
has in anyway connected with such other person or Prime White Cement Corporation
persons, firm, association or partnership; and finally, vs.
that all and any of the persons who may become Intermediate Appellate Court
director or officer of the corporation shall be relieved GR 68555, 19 March 1993
from all responsibility for which they may otherwise
be liable by reason of any contract entered into with FACTS:
the corporation, whether it be for his benefit or for
the benefit of any other person, firm, association or On or about 16 July 1969, Alejandro Te and
partnership in which he may be interested. Prime White Cement Corporation (PWCC) thru its
President, Mr. Zosimo Falcon and Justo C. Trazo,
Issue: as Chairman of the Board, entered into a dealership
W/N said provisions are contrary to the agreement whereby Te was obligated to act as the
corporation law. exclusive dealer and/or distributor of PWCC of its
cement products in the entire Mindanao area for a
Ruling: term of 5 years.
Yes, these provisions are in direct Right after Te entered into the dealership
opposition to our corporation law and corporate agreement, he placed an advertisement in a
practices in this country. These provisions alone national, circulating newspaper the fact of his being
would outlaw any corporation locally organized or the exclusive dealer of PWWC's white cement
doing business in this jurisdiction. Consider the products in Mindanao area, more particularly, in the
unique and unusual provision that no contract or Manila Chronicle dated 16 August 1969 and was
transaction between the company and any other even congratulated by his business associates, so
association or corporation shall be affected except much so, he was asked by some of his
in case of fraud, by the fact that any of the directors businessmen friends and close associates if they
or officers of the company may be interested in or can be his sub-dealer in the Mindanao area.
are directors or officers of such other association or
corporation; and that none of such contracts or ISSUE:
transactions of this company with any person or
persons, firms, associations or corporations shall be Whether the "dealership agreement"
affected by the fact that any director or officer of this referred by the President and Chairman of the
company is a party to or has an interest in such Board of PWCC is a valid and enforceable contract.
contract or transaction or has any connection with
such person or persons, firms associations or RULING:
corporations; and that any and all persons who may
become directors or officers of this company are NO.
hereby relieved of all responsibility which they
would otherwise incur by reason of any contract
The “dealership agreement” is not valid and financial condition, in contemplation of an
unenforceable. Under the Corporation Law, which insolvency and dissolution. That on September 11,
was then in force at the time the case arose, as well 1923, when the petition was filed for its dissolution
as under the present Corporation Code, all upon the ground that it was insolvent, its accounts
corporate powers shall be exercised by the Board of payable amounted to P9,241.19, and its accounts
Directors, except as otherwise provided by law. receivable P12,512.47, or an apparent asset of
Although it cannot completely abdicate its power P3,271.28 over and above its liabilities.
and responsibility to act for the juridical entity, the
Board may expressly delegate specific powers to its ISSUE:
President or any of its officers.
In the absence of such express delegation, Whether or not the Corporation acted in bad
a contract entered into by its President, on behalf of faith in acquiring its own shares of stocks.
the corporation, may still bind the corporation if the
board should ratify the same expressly or impliedly. RULING:
Implied ratification may take various forms — like
silence or acquiescence; by acts showing approval YES.
or adoption of the contract; or by acceptance and
retention of benefits flowing therefrom. Furthermore, There is no stipulation or finding of facts as
even in the absence of express or implied authority to what was the actual cash value of its accounts
by ratification, the President as such may, as a receivable. Neither is there any stipulation that
general rule, bind the corporation by a contract in those accounts or any part of them ever have been
the ordinary course of business, provided the same or will be collected, and it does appear that after his
is reasonable under the circumstances. These rules appointment on February 28, 1924, the receiver
are basic, but are all general and thus quite flexible. made a diligent effort to collect them, and that he
They apply where the President or other officer, was unable to do so, and it also appears from the
purportedly acting for the corporations, is dealing minutes of the board of directors that the president
with a third person, i.e., a person outside the and manager "recommended that P3,000 — out of
corporation. The situation is quite different where a the surplus account to be set aside for dividends
director or officer is dealing with his own payable, and that payments be made in installments
corporation. Herein, Te was not an ordinary so as not to effect the financial condition of the
stockholder; he was a member of the Board of corporation."
Directors and Auditor of the corporation as well. He It is very apparent that on June 24, 1922,
was what is often referred to as a "self-dealing" the board of directors acted on assumption that,
director. because it appeared from the books of the
corporation that it had accounts receivable of the
face value of P19,126.02, therefore it had a surplus
C. H. STEINBERG, as Receiver of the Sibugueyover
Trading Company,
and above Incorporated
its debts and liabilities. Thus, in the
vs. purchase of its own stock to the amount of P3,300
GREGORIO VELASCO, ET AL. and in declaring the dividends to the amount of
G.R. No. L-30460. March 12, 1929 P3,000, the real assets of the corporation were
diminished P6,300. The corporation did not then
FACTS: have an actual bona fide surplus from which the
dividends could be paid, and that the payment of
Plaintiff is the receiver of the Sibuguey them in full at the time would "affect the financial
Trading Company, a domestic corporation. The condition of the corporation."
defendants are residents of the Philippine Islands. It Creditors of a corporation have the right to
is alleged that the defendants, Gregorio Velasco, as assume so long as there are outstanding debts and
president, Felix del Castillo, as vice-president, liabilities, the board of directors will not use the
Andres L. Navallo, as secretary-treasurer, and assets of the corporation to purchase its own stock,
Rufino Manuel, as director of Trading Company, at and that it will not declare dividends to stockholders
a meeting of the board of directors, approved and when the corporation is insolvent.
authorized various lawful purchases already made
of a large portion of the capital stock of the
company from its various stockholders with total CHARLES W. MEAD
amount of the capital stock unlawfully purchased vs.
was P3,300. At the time of such purchase, the E.C. MCCULLOUGH, et. al.
corporation had accounts payable amounting to GR 6217, 26 December 2011
P13,807.50, most of which were unpaid at the time
petition for the dissolution of the corporation was its FACTS:
their interest; and if they are themselves creditors
On March 15, 1902, the plaintiff (Mead will while the insolvent corporation is under their
be referred to as the plaintiff in this opinion unless it management, they will not be permitted to secure to
is otherwise stated) and the defendant organized themselves by purchasing the corporate property or
the "Philippine Engineering and Construction otherwise any personal advantage over the other
Company. creditors. Nevertheless, a director or officer may in
Shortly after the organization, the directors good faith and for an adequate consideration
held a meeting and elected the plaintiff as general purchase from a majority of the directors or
manager. The plaintiff held this position with the stockholders the property even of an insolvent
company for nine months, when he resigned to corporation, and a sale thus made to him is valid
accept the position of engineer of the Canton and and binding upon the minority.
Shanghai Railway Company.
The contract and work undertaken by the
company during the management of Mead were the JOHN GOKONGWEI
wrecking contract with the Navy Department at vs.
Cavite for the raising of the Spanish ships sunk by SEC, ANDRES SORIANO, et al.
Admiral Dewey; the contract for the construction of GR L-45911, 11 April 1979
certain warehouses for the quartermaster
department; the construction of a wharf at Fort FACTS:
McKinley for the Government; The supervision of
the construction of the Pacific Oriental Trading Gokonwei alleged that on September 18,
Company's warehouse; and some other odd jobs 1976, individual respondents amended by bylaws of
not specifically set out in the record. San Miguel Corporation, basing their authority to do
Shortly after the plaintiff left the Philippine so on a resolution of the stockholders adopted on
Islands for China, the other directors, the March 13, 1961, when the outstanding capital stock
defendants in this case, held a meeting on of respondent corporation was only
December 24, 1903, for the purpose of discussing P70,139.740.00, divided into 5,513,974 common
the condition of the company at that time and shares at P10.00 per share and 150,000 preferred
determining what course to pursue. shares at P100.00 per share. At the time of the
The assignees of the wrecking contract, amendment, the outstanding and paid up shares
including McCullough, formed was not known as the totalled 30,127,043, with a total par value of
"Manila Salvage Association." This association paid P301,270,430.00. It was contended that according
to McCullough $15,000 Mexican Currency cash for to section 22 of the Corporation Law and Article VIII
the assignment of said contract. In addition to this of the by-laws of the corporation, the power to
payment, McCullough retained a one-sixth interest amend, modify, repeal or adopt new by-laws may
in the new company or association. be delegated to the Board of Directors only by the
affirmative vote of stockholders representing not
ISSUE: less than 2/3 of the subscribed and paid up capital
stock of the corporation, which 2/3 should have
Whether or not the respondents are self- been computed on the basis of the capitalization at
dealing directors. the time of the amendment. Since the amendment
was based on the 1961 authorization, petitioner
RULING: contended that the Board acted without authority
and in usurpation of the power of the stockholders.
NO. It was claimed that prior to the questioned
amendment, petitioner had all the qualifications to
While a corporation remains solvent, there be a director of respondent corporation, being a
is no reason why a director or officer, by the substantial stockholder thereof; that as a
authority of a majority of the stockholders or board stockholder, petitioner had acquired rights inherent
of managers, may not deal with the corporation, in stock ownership, such as the rights to vote and to
loan it money or buy property from it, in like manner be voted upon in the election of directors; and that
as a stranger. So long as a purely private in amending the by-laws, respondents purposely
corporation remains solvent, its directors are agents provided for petitioner's disqualification and
or trustees for the stockholders. They owe no duties deprived him of his vested right as afore-mentioned,
or obligations to others. But the moment such a hence the amended by-laws are null and void.
corporation becomes insolvent, its directors are
trustees of all the creditors, whether they are ISSUE:
members of the corporation or not, and must
manage its property and assets with strict regard to
Whether or not SMC’s BoD acted in bad possession of F. Stuart Jones, as her agent. Instead
faith in making the amendment which disqualified of seeing Jones, who had an office next door,
Gokongwei from being elected as Director. Repide employed one Kauffman. Kaufmann, in turn,
employed Mr. Sloan, a broker, to purchase the
RULING: stock for him. Kauffman told Sloan that the stock to
be purchased was for a member of his wife’s family.
NO. This action by Repide was due to the negotiations
initiated by the government where the latter will
SMC is merely protecting its interest from purchase the company’s lands (together with other
Gokongwei, who owns companies in direct friar lands) at a price which greatly enhance the
competition with SMC’s business. Although in the value of the stock.
strict and technical sense, directors of a private As a result of the negotiations, Jones,
corporation are not regarded as trustees, there assuming he had the power and without consulting
cannot be any doubt that their character is that of a Strong, sold the 800 shares. Strong filed a case to
fiduciary insofar as the corporation and the recover the shares from Repide on the ground that
stockholders as a body are concerned. As agents the shares had been sold and delivered by Strong’s
entrusted with the management of the corporation agent without authority to do so and on the ground
for the collective benefit of the stockholders, they that Repide fraudulently concealed from Strong’s
occupy a fiduciary relation, and in this sense the agent the facts affecting the value of the stock so
relation is one of trust. It springs from the fact that sold and delivered.
directors have the control and guidance of corporate
affairs and property; hence of the property interests ISSUE:
of the stockholders. Equity recognizes that
stockholders are the proprietors of the corporate Whether or notRepide, acting in good faith,
interests and are ultimately the only beneficiaries has the duty to disclose to the agent of Strong the
thereof facts bearing upon or which might affect the value of
It is obviously to prevent the creation of an the stocks.
opportunity for an officer or director of San Miguel
Corporation, who is also the officer or owner of a RULING:
competing corporation, from taking advantage of the
information which he acquires as director to YES.
promote his individual or corporate interests to the
prejudice of San Miguel Corporation and its The Court ruled that there is no relationship
stockholders, that the questioned amendment of the of a fiduaciary nature exists between a director and
by-laws was made. a shareholder in a business corporation. There are
Certainly, where two corporations are cases, however, where, by reason of special facts,
competitive in a substantial sense, it would seem such duty of a director to disclose to a shareholder
improbable, if not impossible, for the director, if he the knowledge which he may possess regarding the
were to discharge effectively his duty, to satisfy his value of the shares of the company before he
loyalty to both corporations and place the purchases any from a shareholder. Some special
performance of his corporation duties above his facts are present in this case such as the fact the
personal concerns. Repide is not only a director of the corporation but
an owner of three-fourths shares of its stock. He
was the chief negotiator for the sale of all the lands
ELEANOR ERICA STRONG, ET AL. and was acting substantially as the agent of the
vs. shareholders by reason of his ownership of the
FRANCISCO GUTIERREZ REPIDE shares. Thus, a purchase of stock in a corporation
G.R. No. L-7154. February 21, 1912 by a director and owner of three-fourths of the entire
capital stock, who was also administrator general of
FACTS: the company and engaged in the negotiations which
finally led to the sale of company’s lands to the
Eleanor Strong was the owner of 800 government at a price which greatly enhanced the
shares of the capital stock of Philippine Sugar value of the stock, was fraudulent as procured by
Estate Development Company. Gutierrez Rapide, insidious machination where he employed an agent
owner of three-fourths shares of the company’s to make the purchase, concealing both his identity
stock , 1 of the 5 directors of the company and was as purchaser and his knowledge of the state of the
elected by the board as administrator general of negotiations and their probable successful result
such company, took steps to purchase the 800
shares owned by Strong, which he knew were in the
G.R. No. L-17131 June 30, 1922 until 12 o'clock, she could do as she pleased.
Interpreting the phrase to mean that the plaintiffs
SING JUCO and SING BENGCO, plaintiffs- waived their option to buy, Maria Gay closed the
appellees, vs. sale of the estate in favor of the defendant Antonio
Sunyantong.
ANTONIO SUNYANTONG and his wife VICENTA
LLORENTE DE SUNYANTONG, defendants- Issue: Whether or not the defendant guilty of
appellants. infidelity?

Facts: the plaintiffs obtained from Maria Gay a Ruling: Yes. The action of the defendant
written option to purchase an estate known as "San Sunyantong in intervening in the negotiations in the
Antonio Estate,” The term of the option expired, but manner in which he did does not make him innocent
the plaintiffs had it extended verbally. of infidelity in view of the fact that he was an
employee of the plaintiffs to whom he owed loyalty
The defendant Antonio Sunyantong was at the time
and faithfullnes.
an employee of the plaintiffs.
Even though it be concede that when he closed the
The defendant advised the plaintiffs to let some
contract of sale with Maria Gay the plaintiffs' option
days elapse before accepting the terms of the
had expired, but the fact cannot be denied that he
transfer as proposed by Maria Gay, in order that the
was the cause of the option having precipitously
latter might not think that they were coveting said
come to such an end. His disloyalty to his
property. Defendant met Alipio de los Santos after
employers was responsible for Maria Gay not
the latter's return to Iloilo, sent by the plaintiffs to
accepting the terms proposed by the plaintiffs,
examine the estate and satisfy himself of its
because of being certain of another less exigent
condition, and Alipio de los Santos told him of his
buyer. Without such intervention on the part of the
favorable impression of the estate, he advised De
defendant it is presumed, taking into account all the
los Santos not to report the estate to the plaintiffs as
circumstances of the case, that the sale of the
being so highly valuable, for if it proved failure they
estate in question would have been consummated
might blame him, De los Santos. One becomes
between Maria Gay and the plaintiffs, perhaps with
more strongly convinced that this defendant has
such advantages to the plaintiffs, as they expected
been unfaithful to his principals, the plaintiffs, when
to obtain by prolonging negotiations.
on the midday of which the term of plaintiff's option
to purchase was to expire, defendant Antonio Such an act of infidelity committed by a trusted
Sunyantong called Mari Gay, and offered to buy the employee calculated to redound to his own benefit
estate on the same terms proposed by her not yet and to the detriment of his employers cannot pass
accepted by the plaintiffs, making the offer to buy without legal sanction. Nemo debet aliena jactura
not for the benefit of the plaintiff's, but for own wife, locupletari; nemo ex suo delicto meliorem suam
his codefendant Vicenta Llorente de Sunyatong. conditionem facere potest. It is an illicit act
committed with culpa and, therefore, its agent is
Maria Gay communicated by telephone with Manuel
liable (art. 1089, Civil Code), for the damage caused
Sotelo, who was acting as broker for the plaintiffs in
(art. 1902, ibidem). Not identical, but similar, to this
these transactions, and told him that another buyer
infidelity is the abuse of the confidence sanctioned
of the estate had presented himself who would
in our Penal Code as a generic circumstances, nay
accept the terms proposed by her and that she
as specific aggravating one, and even as an
would like to know immediately what decision had
essential element of certain crimes.
been reached by the plaintiffs on the matter. In view
of Maria Gay's insistence that the plaintiff give a Such principle, however, in case of this nature is
categorical answer, Sing Bengco, one of the generally recognized in our laws, since the case of
plaintiffs who happened to be present at the time commercial agents (factories) it is expressly
the telephone conversation between Maria Gay and established. Undoubtedly, formerly under the
Manuel Sotelo took place, instructed Sotelo to circumstances then prevailing such sanction was
inform her at the time that if she did not care to wait not necessary in the field of civil law, because is
sphere of action is the general relations of society; Remington asserted that Marinduque Mining, PNB,
but event then it was deemed necessary expressly DBP, Nonoc Mining, Maricalum Mining and Island
to protect with such sanction the commercial Cement must be treated in law as one and the
same entity by disregarding the veil of corporate
relations wherein the question of gain was involved,
fiction since the personnel, key officers and rank-
which is sometimes so imperatives as to ignore and-file workers and employees of co-defendants
everything, even the very principles of loyalty, NMIC, Maricalum and Island Cement creations of
honesty, and fidelity. co-defendants PNB and DBP were the personnel of
co-defendant MMIC such that practically there has
This specific relief, however, has already come to only been a change of name for all legal purpose
be applied in this jurisdiction in similar cases, and intents.
among which can be cited that of Camacho vs.
Municipality of Baliuag (28 Phil., 466.) ISSUE:

And in the North American law such sanction is Whether or not the takeover of PNB and
DBP over Marinduque Mining is in bad faith.
expressly recognized, and the transaction of this
nature might be regarded as an "equitable trust" by RULING:
virtue of which the things acquired by an employee
is deemed not to have been acquired for his own NO.
benefit or that of any other person but for his
principal, and held in trust for the latter (21 R. C. L., Their actions are mandated under the law.
Where the corporations have directors and officers
825; 2 Corpus Juris, 353).
in common, there may be circumstances under
which their interest as officers in one company may
disqualify them in equity from representing both
corporations in transactions between the two. Thus,
DEVELOPMENT BANK OF THE PHILIPPINES
where one corporation was ‘insolvent and indebted
vs.
to another, it has been held that the directors of the
COURT OF APPEALS, REMINGTON
creditor corporation were disqualified, by reason of
INDUSTRIAL SALES
self-interest, from acting as directors of the debtor
GR 126200, 16 August 2001
corporation in the authorization of a mortgage or
deed of trust to the former to secure such
FACTS:
indebtedness In the same manner that when the
corporation is insolvent, its directors who are its
Between July 1981 and April 1984,
creditors cannot secure to themselves any
Marinduque Mining entered into 3 mortgage
advantage or preference over other creditors. They
agreements with PNB and DBP involving its real
cannot thus take advantage of their fiduciary
properties located in Surigao del Norte, Negros
relation and deal directly with themselves, to the
Occidental, and Rizal, as well as its equipment
injury of others in equal right. If they do, equity will
located therein. Marinduque failed to pay its loans,
set aside the transaction at the suit of creditors of
causing the foreclosure of the said mortgages. PNB
the corporation or their representatives, without
and DBP thereafter gained control of the said
reference to the question of any actual fraudulent
properties.
intent on the part of the directors, for the right of the
In the meantime, between July 16, 1982 to
creditors does not depend upon fraud in fact, but
October 4, 1983, Marinduque Mining purchased and
upon the violation of the fiduciary relation to the
caused to be delivered construction materials and
directors.
other merchandise from Remington Industrial Sales
Directors of insolvent corporation, who are
Corporation. The purchases remained unpaid as of
creditors of the company, can not secure to
August 1, 1984 when Remington filed a complaint
themselves any preference or advantage over other
for a sum of money and damages against
creditors in the payment of their claims. It is not
Marinduque Mining for the value of the unpaid
good morals or good law. The governing body of
construction materials and other merchandise
officers thereof are charged with the duty of
purchased by Marinduque Mining, as well as
conducting its affairs strictly in the interest of its
interest, attorney’s fees and the costs of suit.
existing creditors, and it would be a breach of such
Remington’s original complaint was
trust for them to undertake to give any one of its
amended to include PNB, DBP, Maricalum Mining
members any advantage over any other creditors in
Corporation (Maricalum Mining) and Island Cement
securing the payment of his debts in preference to
Corporation (Island Cement) as co-defendants.
all others.
appointed manager by the board of directors and as
Gurrea vs. Lezama such does not have the character of an officer, the
conclusion is inescapable that he can be suspended
or removed by said board of directors under such
terms as it may see fit and not as provided for in the
Facts: by-laws. Evidently, the power to appoint carries with
it the power to remove, and it would be incongruous
Plaintiff instituted this action in the Court of First to hold that having been appointed by the board of
Instance of Iloilo to have Resolution No. 65 of the directors he could only be removed by the
Board of Directors of the La Paz Ice Plant and Cold stockholders.
Storage Co., Inc., removing him from his position of
manager of said corporation declared null and void Therefore hold that plaintiff has been properly
and to recover damages incident thereto. The action removed when the board of directors of the instant
is predicated on the ground that said resolution was corporation approved its Resolution No. 65 on June
adopted in contravention of the provisions of the by- 3, 1948.
laws of the corporation, of the Corporation Law and
of the understanding, intention and agreement MITA PARDO DE TAVERA V. PHILIPPINE
reached among its stockholders. TUBERCULOSIS SOCIETY G.R.
NO. L-48928
Defendant answered the complaint setting up as
defense that plaintiff had been removed by virtue of Facts:
a valid resolution.
Plaintiff is a doctor of Medicine by profession and a
Issue: recognized specialist in the treatment of
tuberculosis. She is a member of the Board of
Whether or not managers are considered officers of Directors of the defendant Society, in representation
the corporation. of the PCSO. She was duly appointed as Executive
Secretary of the Society. On May 29, 1974, the past
Ruling: Board of Directors removed her summarily from her
position, the lawful cause of which she was not
No.
informed, through the simple expedient of declaring
Section 33 of the Corporation Law provides: her position vacant. Defendant Romulo was
"Immediately after the election, the directors of a appointed to the position and defendants Pardo,
corporation must organize by the election of a Nubla, Garcia and Adil, not being members of
president, who must be one of their number, a defendant Society were elevated as members of the
secretary or clerk who shall be a resident of the Board of Directors. Not being qualified, petitioner
Philippines . . . and such other officers as may be
alleged said acts to be null and void. The court a
provided for in the by-laws." The by-laws of the
instant corporation in turn provide that in the board quo rendered a decision holding that the present
of directors there shall be a president, a vice- suit being one for quo warranto it should be filed
president, a secretary and a treasurer. These are within one year from plaintiff's ouster from office;
the only ones mentioned therein as officers of the that nevertheless, plaintiff was not illegally removed
corporation. The manager is not included although from her position as Executive Secretary in The
the latter is mentioned as the person in whom the Society since plaintiff was holding an appointment
administration of the corporation is vested, and with
at the pleasure of the appointing power and hence
the exception of the president, the by-laws provide
that the officers of the corporation may be removed temporary.
or suspended by the affirmative vote of 2/3 of the
corporation. Issue:

Whether or not petitioner was illegally removed from


From the above the following conclusion is clear:
that we can only regard as officers of a corporation her position as Executive Secretary in violation of
those who are given that character either by the Code of By-Laws of the society.
Corporation Law or by its by-laws. The rest can be
considered merely as employees or subordinate Ruling:
officials. And considering that plaintiff has been
NO. Defendant Romulo was appointed to the position
and defendants Pardo, Nubla, Garcia and Adil, not
Although the minutes of the organizational meeting being members of defendant Society were elevated
show that the Chairman mentioned the need of as members of the Board of Directors. Not being
appointing a “permanent” Executive Secretary, such qualified, petitioner alleged said acts to be null and
void. The court a quo rendered a decision holding
statement alone cannot characterize the
that the present suit being one for quo warranto it
appointment of petitioner without a contract of should be filed within one year from plaintiff's ouster
employment definitely fixing her term because of the from office; that nevertheless, plaintiff was not
specific provision of Section 7.02 of the Code of By- illegally removed from her position as Executive
Laws that: “The Executive Secretary, the Auditor, Secretary in The Society since plaintiff was holding
and all other officers and employees of the Society an appointment at the pleasure of the appointing
shall hold office at the pleasure of the Board of power and hence temporary.
Directors, unless their term of employment shall
have been fixed in their contract of employment.” MITA PARDO DE TAVERA, Plaintiff-Appellant, v.
Besides the word permanent” could have been used PHILIPPINE TUBERCULOSIS SOCIETY, INC.,
to distinguish the appointment from acting capacity”. FRANCISCO ORTIGAS, JR., MIGUEL
CAÑIZARES, BERNARDO P. PARDO, RALPH
The absence of a fixed term in the letter addressed NUBLA, MIDPANTAO ADIL, ENRIQUE GARCIA,
to petitioner informing her of her appointment as ALBERTO G. ROMULO, and THE PRESENT
Executive Secretary is very significant. This could BOARD OF DIRECTORS, PHILIPPINE
TUBERCULOSIS SOCIETY, INC., Defendants-
have no other implication than that petitioner held
Appellees.
an appointment at the pleasure of the appointing
power. Juan T. David, for Plaintiff-Appellant.

An appointment held at the pleasure of the Mauricio Nubla for defendant-appellee Ralph
appointing power is in essence temporary in nature. Nubla.
It is co-extensive with the desire of the Board of
Directors. Hence, when the Board opts to replace Ramon Gonzales for defendant-appellee Adil.
the incumbent, technically there is no removal but
Delfino Salazar for defendant-appellee E. Garcia.
only an expiration of term and in an expiration of
term, there is no need of prior notice, due hearing or Camilo D. Quiason for the other defendants-
sufficient grounds before the incumbent can be appellees.
separated from office. The protection afforded by
Section 7.04 of the Code of By-Laws on Removal of SYNOPSIS
Officers and Employees, therefore, cannot be
claimed by petitioner. Appellant was Executive Secretary of the Philippine
Tuberculosis Society (Society for short) until the
past Board of Directors declared her position vacant
DE TAVERA VS. PHIL. TUBERCULOSIS SOCIETY on May 29, 1974, and seven of the directors
appointed Alberto Romulo to the position. More than
DE TAVERA VS. PHIL. TUBERCULOSIS one year after her removal, appellant filed a
SOCIETY complaint with the trial court against the said
GR. No. L-48928 February 25, 1982 appointing directors, the Society, the new Board of
GUERRERO, J.: Directors, and Alberto Romulo, questioning the
FACTS: Plaintiff is a doctor of Medicine by legality of her summary cutter from her office and
profession and a recognized specialist in the seeking reinstatement thereto; contending That the
treatment of tuberculosis. She is a member of the action of the past Board was a nullity since four of
Board of Directors of the defendant Society, in the directors were not qualified to be elevated to the
representation of the PCSO. She was duly position because they were not members of the
appointed as Executive Secretary of the Society. On Society; and claiming that the removal was in
May 29, 1974, the past Board of Directors removed violation of her rights under the By-Laws of the
her summarily from her position, the lawful cause of Society, the New Civil Code, and the New
which she was not informed, through the simple Constitution, which thereby rendered the individuals
expedient of declaring her position vacant. responsible therefor, countable for damages. The
trial court rendered a decision holding that the suit 2. ID.; ID.; ID.; ID.; COMPLAINT IN CASE AT BAR
was one for quo warranto and has thus prescribed; NOT QUO WARRANTO ALTHOUGH IT
that, nevertheless, appellant had not been illegally QUESTIONS REMOVAL OF PETITIONER FROM
removed because she was holding an appointment POSITION. — While it is true that the complaint
at the pleasure of the Board, temporary in nature, questions petitioner’s removal from the position of
and terminable at any time; and, that the Executive Secretary and seeks her reinstatement
qualifications of the members of the Board could not thereto, the nature of the suit is one involving a
be attacked collaterally. Appellant appealed to the violation of the rights of the plaintiff under the By-
Court of Appeals, but the same was certified to the Laws of the Society, the Civil Code and the
Supreme Court as only questions of law were Constitution, which allegedly renders the individuals
involved. responsible therefor, accountable for damages as
maybe gleaned from the allegations in the complaint
The Supreme Court held, that even if the complaint as constituting the plaintiff’s cause of action.
questions appellant’s removal from her position and Further, the action is not only against Alberto
seeks her reinstatement thereto, the suit is not Romulo, the person appointed in her stead, but
necessarily one of quo warranto since the primarily against the Society and the past members
allegations in the complaint constituting her cause of the Board who are responsible for her removal.
of action show that the case is for damages and the
defendants-appellees, except one, are not actually 3. ID.; ID.; PRESCRIPTION OF ACTIONS; FOR
holding the office in question; that appellant is not QUO WARRANTO AND INJURY TO RIGHTS OF
entitled to damages because she has not been PLAINTIFF. — Since the suit could not be one for
illegally ousted since pursuant to the Society’s Code quo warranto the one-year period fixed in Section
of By-Laws, the Executive Secretary holds office at 16, Rule 66 of the Revised Rules of Court within
the pleasure of the Board of Directors unless the which a petition for quo warranto should be filed,
term of employment has been fixed in the contract counted from date of ouster, does not apply to the
of employment, which in the case of appellant has case at bar. The action must be brought within four
not been so fixed; and that appellant’s right to her (4) years, in accordance with Valencia v. Cebu
office, being specifically limited by the Society’s By- Portland Cement Co., Et Al., L-13715, December
Laws, she may not invoke in relation thereto the 23, 1959, 106 Phil. 732, a case involving a plaintiff
general provisions of the New Civil Code on Human separated from his employment for alleged
Relations and the fundamental principles of the New unjustifiable causes, where this Court held that the
Constitution on preservation of human dignity which action is one for "injury to the rights of the plaintiff,
are merely guides for human conduct in the and must be brought within 4 years under Article
absence of specific legal provisions and definite 1146 of the New Civil Code.’’
contractual stipulations.
4. LABOR LAWS: NATURE OF APPOINTMENT;
Appealed decision affirmed. CHARACTERIZED BY CODE OF BY-LAWS OF
SOCIETY IN CASE AT BAR. — The statement in
the minutes of the organizational meeting showing
SYLLABUS that the Chairman mentioned the need of appointing
a "permanent" Executive Secretary cannot
characterize the appointment of petitioner without a
1. REMEDIAL LAW; CIVIL PROCEDURE; contract of employment definitely fixing her term
COMPLAINT; NATURE AND PURPOSE OF because of the specific provision of Section 7.02 of
ACTION DETERMINED BY ALLEGATIONS the Code of By-Laws that; "The Executive
THEREIN. — The nature of an action filed in court Secretary, the Auditor, and all other officers and
is determined by the facts alleged in the complaint employees of the Society shall hold office at the
as constituting the cause of action, and not those pleasure of the Board of Directors, unless their term
averred as a defense in the defendants’ answer. of employment shall have been fixed in their
The theory adopted by the plaintiff in his complaint contract of employment ." Besides, the word
is one thing; that by the defendant in his answer "permanent" could have been used to distinguish
another. The purpose of an action or suit and the the appointment from "acting capacity." The
law to govern it, including the period of prescription, absence of a fixed term in the letter addressed to
is to be determined not by the claim of the party petitioner informing her of her appointment as
filing the action, made in his argument or brief, but Executive Secretary could have no other implication
rather by the complaint itself, its allegations and than that petitioner held an appointment at the
prayer for relief (Rone, Et. Al. vs.Claro, Et Al., L- pleasure of the appointing power.
4472, May 8, 1952, 91 Phil. 250).
5. ID.; ID.; TEMPORARY APPOINTMENT;
APPOINTMENT HELD AT PLEASURE OF Directors of the defendant Society, in representation
APPOINTING POWER TERMINABLE AT ANY of the Philippine Charity Sweeptakes Office; that
TIME AND WITHOUT CAUSE. — An appointment she was duly appointed on April 27, 1973 as
held at the pleasure of the appointing power is in Executive Secretary of the Society; that on May 29,
essence temporary. It is co-extensive with the 1974, the past Board of Directors removed her
desire of the Board of Directors. Hence, when the summarily from her position, the lawful cause of
Board opts to replace the incumbent, technically which she was not informed, through the simple
there is no removal but only an expiration of term expedient of declaring her position vacant; that
and in an expiration of term, there is no need of immediately thereafter, defendant Alberto Romulo
prior notice, due hearing or sufficient grounds was appointed to the position by an affirmative vote
before the incumbent can be separated from office. of seven directors, with two abstentions and one
objection; and that defendants Pardo, Nubla, Garcia
6. ID.; ID.; ID.; SPECIFIC PROVISION IN and Adil, not being members of defendant Society
EMPLOYER’S BY-LAWS REGARDING TERM OF when they were elevated to the position of members
OFFICE PREVAILS OVER GENERAL of the Board of Directors, are not qualified to be
PROVISIONS OF NEW CIVIL CODE AND elected as such and hence, all their acts in said
FUNDAMENTAL PRINCIPLES OF NEW meeting of May 29, 1974 are null and void.
CONSTITUTION; CASE AT BAR. — Petitioner
cannot seek relief from the general provisions of the The defendants filed their answer on May 12, 1976,
New Civil Code on Human Relations nor from the specifically denying that plaintiff was illegally
fundamental principles of the New Constitution on removed from her position as Executive Secretary
preservation of human dignity. While these and averring that under the Code of By-Laws of the
provisions present some basic principles that are to Society, said position is held at the pleasure of the
be observed for the rightful relationship between Board of Directors and when the pleasure is
human beings and the stability of social order, these exercised, it only means that the incumbent has to
are merely guides for human conduct in the vacate the same because her term has expired; that
absence of specific legal provisions and definite defendants Pardo, Nubla, Adil and Garcia were, at
contractual stipulations In the case at bar, the Code the time of their election, members of the defendant
of By-Laws of the Society contains a specific Society and qualified to be elected as members of
provision governing the term of office of petitioner the Board; that assuming that said defendants were
The same necessarily limits her rights under the not members of defendant Society at the time of
New Civil Code and the New Constitution upon their election, the question of qualification of the
acceptance of the appointment. members of the Board of Directors should have
been raised at the time of their election; that
assuming that the qualification of members of the
DECISION Board of Directors can be questioned after their
assumption of their offices as directors, such
contest cannot be done in a collateral action; that an
GUERRERO, J.: action to question the qualifications of the Directors
must be brought within one year from their election;
and that a Director elected without necessary
On March 23, 1976, plaintiff-appellant Mita Pardo qualification becomes at least a de facto director,
de Tavera filed with the Court of First Instance of whose acts are as valid and binding as a de jure
Rizal a complaint against the Philippine director. Further, defendants disputed the timeliness
Tuberculosis Society, Inc. (hereinafter referred to as of the filing of the action stating that an action to
the Society), Miguel Cañizares, Ralph Nubla, question one’s ouster from a corporate office must
Bernardo Pardo, Enrique Garcia, Midpantao Adil, be filed within one year from said ouster.
Alberto Romulo, and the present Board of Directors
of the Philippine Tuberculosis Society, Inc. On the same date, defendant Adil filed a Motion to
Dismiss on the ground that the complaint states no
On April 12, 1976, plaintiff-appellant filed an cause of action, or if it does, the same has
amended complaint impleading Francisco Ortigas, prescribed. Inasmuch as plaintiff seeks
Jr. as party defendant. reinstatement, he argued that the complaint is an
action for quo warranto and hence, the same should
In substance, the complaint alleged that plaintiff is a be commenced within one year from May 29, 1974
doctor of Medicine by profession and a recognized when the plaintiff was ousted from her position.
specialist in the treatment of tuberculosis, having
been in the continuous practice of her profession Plaintiff filed an Opposition to Motion to Dismiss on
since 1945; that she is a member of the Board of May 28, 1976, stating that the complaint is a suit for
damages filed under the authority of Section 6, I. The lower court erred in holding that the present
Article Il of the present Constitution in relation to case isone for quo warranto and not an action for
Articles 21 and 32(6) of the New Civil Code, and her damages.
constitutional right to equal protection of the law, as
guaranteed by Section 1, Article IV of the present II. In deciding the case, the lower court erred in not
Constitution. upholding the Society’s By-Laws, the applicable
laws, and the pertinent provisions of the
On June 2, 1976, defendant Adil filed a Reply to Constitution.
Plaintiff’s Opposition to Motion to Dismiss arguing
that since there is an averment of plaintiff’s right to III. The lower court erred in holding that the plaintiff-
office, and that defendant Romulo is unlawfully in appellant is not in the civil service, and therefore,
possession thereof, then, it is indeed, a case for quo not entitled to the guaranty against removal from
warranto; and that assuming that it is merely a suit office except for cause and after due process of law.
for damages, then, the same is premature, pursuant
to Section 16, Rule 66 of the Rules of Court. The naure of an action filed in court is determined
by the facts alleged in the complaint as constituting
On September 3, 1976, the court a quo rendered a the cause of action, and not those averred as a
decision holding that the present suit being one for defense in the defendant’s answer. The theory
quo warranto, it should be filed within one year from adopted by the plaintiff in his complaint is one thing,
plaintiff’s ouster from office; that nevertheless, that by the defendant in his answer another. The
plaintiff was not illegally removed or ousted from her purpose of an action or suit and the law to govern it,
position as Executive Secretary in the Society since including the period of prescription, is to be
plaintiff was holding an appointment at the pleasure determined not by the claim of the party filing the
of the appointing power and hence her appointment action, made in his argument or brief, but rather by
in essence was temporary in nature, terminable at a the complaint itself, its allegations and prayer for
moment’s notice without need to show that the relief. (Rone, Et. Al. v. Claro, Et Al., L-4472, May 8,
termination was for cause; and that plaintiff’s ouster 1952, 91 Phil. 250) In Baguioro v. Barrios, Et Al., 77
from office may not be challenged on the ground Phil. 120, the Supreme Court held that if the relief
that the acts of defendants Pardo, Adil, Nubla and demanded is not the proper one which may be
Garcia are null and void, they being not qualified to granted under the law, it does not characterize or
be elected members of the Board of Directors which determine the nature of plaintiff’s action, and the
removed plaintiff from office may not be the subject relief to which plaintiff is entitled based on the facts
of a collateral attack in the present suit for quo alleged by him in his complaint, although it is not the
warranto affecting title to the office of Executive relief demanded, is what determines the nature of
Secretary. the action.

On October 13, 1976, plaintiff filed a Motion for While it is true that the complaint questions
Reconsideration to which defendants filed an petitioner’s removal from the position of Executive
Opposition. On November 25, 1976, the court a quo Secretary and seeks her reinstatement thereto, the
denied the Motion for Reconsideration. nature of the suit is not necessarily one of quo
warranto. The nature of the instant suit is one
Dissatisfied with the decision and the order denying involving a violation of the rights of the plaintiff
the motion for reconsideration, plaintiff filed a Notice under the By-Laws of the Society, the Civil Code
of Appeal and an Urgent Motion for Extension of and the Constitution, which allegedly renders the
Time to File Record on Appeal, which was granted individuals responsible therefore, accountable for
in an order dated December 15, 1976. However, on damages, as may be gleaned from the following
December 20, 1976, the court a quo issued an allegations in the complaint as constituting the
amended order where it qualified the action as plaintiff’s causes of action, to
principally one for quo warranto and hence, wit:jgc:chanrobles.com.ph
dispensed with the filing of a record on appeal as
the original records of the case are required to be "20. That, as a consequence of the unfair and
elevated to the Court of Appeals. malicious removal of plaintiff from her office, which
the plaintiff maintains to be contrary to morals, good
On August 8, 1978, the Court of Appeals issued a customs, public policy, the pertinent provisions of
resolution certifying this case to this Court said By-Laws of the Society, the laws, and the
considering that the appeal raises no factual issues guaranties of the Constitution, by defendants
and involves only issues of law, as may be gleaned Cañizares, Ortigas Jr., Pardo, Adil, Nubla and
from the following assignments of errors:. Garcia, the plaintiff suffered not only material
damages, but serious damage to her priceless
properties, consisting of her honor and reputation, restoring the plaintiff to her position as Executive
which were maliciously and unlawfully besmirched, Secretary of the defendant Society, payment of
thereby entitling her to compensation for material salaries and other benefits, corresponding to the
and moral damages, from said defendants, jointly period of her illegal and unconstitutional removal
and severally, under Article 21, in relation to Article from office."cralaw virtua1aw library
32(6) of the New Civil Code;
Further, it must be noted that the action is not only
x x x against Alberto Romulo, the person appointed in her
stead, but also against the Society and the past and
present members of the Board. In fact, Romulo is
"24. That as a consequence of the inordinate use sued as present occupant of the office and not to
and abuse of power by defendants, Cañizares, hold him accountable for damages because he did
Ortigas Jr., Pardo, Adil, Nubla and Garcia, in not participate in the alleged illegal and
arbitrarily, illegally, and unjustly removing the unconstitutional removal of plaintiff-appellant. The
plaintiff from office, without due process of law and action is primarily against the Society and the past
in denying to her the enjoyment of the guaranty of members of the Board who are responsible for her
the Constitution to equal protection of the law, the removal. The present Board of Directors has been
plaintiff suffered material and moral damages as a impleaded as party defendant for the purpose
result of the debasement of her dignity, both as an merely of enabling it to act, "to reinstate the plaintiff
individual and as a professional (physician) of good to her position as Executive Secretary of the
standing, therefore, defendant Cañizares, Ortigas defendant Society" being one of the reliefs prayed
Jr., Pardo, Adil, Nubla and Garcia should be for in the prayer of the complaint.
ordered to pay her moral damages, jointly and
severally; Hence, We hold that where the respondents, except
for one, namely, Alberto Romulo, are not actually
x x x holding the office in question, the suit could not be
one for quo warranto.

"26. That the acts of the defendants Cañizares, Corollarily, the one-year period fixed in Section 16,
Ortigas Jr., Pardo, Adil, Nubla and Garcia, in Rule 66 of the Revised Rules of Court within which
illegally removing the plaintiff from her position as a petition for quo warranto should be filed, counted
Executive Secretary of defendant Society, which from the date of ouster, does not apply to the case
plaintiff was then holding under a valid appointment at bar. The action must be brought within four (4)
and thereafter, immediately appointing defendant years, in accordance with Valencia v. Cebu Portland
Alberto Romulo to the position, is most unfair, unjust Cement Co., Et Al., L-13715, December 23, 1959,
and malicious, because it is contrary to good 106 Phil. 732, a case involving a plaintiff separated
morals, good customs, public policy, the pertinent from his employment for alleged unjustifiable
provisions of the Code of By-Laws of the defendant causes, where this Court held that the action is one
Society, the laws and the aforementioned for "injury to the rights of the plaintiff, and must be
guaranties of the Constitution; that the plaintiff brought within 4 years under Article 1146 of the
maintains that the said defendants are legally New Civil Code."cralaw virtua1aw library
obligated to compensate her, in concept of
exemplary damages, in order to restrain persons in Nonetheless, although the action is not barred by
authority from committing similar illegal and the statute of limitations, We rule that it will not
unconstitutional acts which debase human dignity prosper. Contrary to her claim, petitioner was not
and inflict injuries to their fellowmen; illegally removed or ousted from her position as
Executive Secretary in violation of the Code of By-
x x x Laws of the Society, the New Civil Code and the
pertinent provisions of the Constitution.

"31. That, as a consequence of the said unjustified Petitioner claims and the respondents do not
refusal of the defendant, present Board of Directors dispute that the Executive Secretary is an officer of
of the defendant Society, to resolve the complaint of the Society pursuant to this provision in the Code of
the plaintiff and extend to her the reliefs to which By-Laws:chanroblesvirtualawlibrary
she is entitled under the law and the Constitution, it
is respectfully submitted that said defendant Board "Section 7.01. — Officers of the Society — The
is under legal obligation to correct the illegal and executive officers of the Society shall be the
unconstitutional act of defendants Cañizares, President, a Vice-President, a Treasurer, who shall
Ortigas Jr., Pardo, Nubla, Adil and Garcia, by be elected by the Board of Directors, an Executive
Secretary, and an Auditor, who shall be appointed while on terminal leave, Dr. Mita Pardo de Tavera
by the Board of Directors, all of whom shall exercise was appointed Acting Executive Secretary. In view
the functions, powers and prerogatives generally thereof, Don Francisco Ortigas, Jr. moved, duly
vested upon such officers, the functions hereinafter seconded, that Dr. Mita Pardo de Tavera be
set out for their respective offices and such other appointed Executive Secretary of the Philippine
duties as from time to time may be prescribed by Tuberculosis Society, Inc. The motion was
the Board of Directors. One person may hold more unanimously approved.
than one office except when the functions thereof
are incompatible with each other."cralaw virtua1aw On April 27, 1973, petitioner was informed in writing
library of the said appointment, to
wit:jgc:chanrobles.com.ph
It is petitioner’s contention that she is subject to
removal pursuant to Section 7.04 of the Code of By- "Dr. Mita Pardo de Tavera
Laws which respondents correctly dispute citing
Section 7.02 of the same Code. The Philippine Tuberculosis Society, Inc.
aforementioned provisions state as
follows:jgc:chanrobles.com.ph Manila

"Section 7.02. — Tenure of Office — All executive Madam:chanrob1es virtual 1aw library
officers of the Society except the Executive
Secretary and the Auditor, shall be elected by the I am pleased to inform you that at the meeting of the
Board of Directors, for a term of one year, and shall Board of Directors held on April 25, 1973, you were
hold office until their successors are elected and appointed Executive Secretary, Philippine
have qualified. The Executive Secretary, the Auditor Tuberculosis Society, Inc. with such compensation
and all other officers and employees of the Society and allowances as are provided for in the Budget of
shall hold office at the pleasure of the Board of the Society, effective immediately, vice Dr. Jose Y.
Directors, unless their term of employment shall Buktaw, retired.
have been fixed in their contract of employment.
Congratulations.
x x x
Very truly yours,

"Section 7.04. — Removal of Officers and For the Board of Directors:chanrob1es virtual 1aw
Employees. — All officers and employees shall be library
subject to suspension or removal for a sufficient
cause at any time by affirmative vote of a majority of (Sgd) Miguel Cañizares, M.D.
all the members of the Board of Directors, except
that employees appointed by the President alone or MIGUEL CANIZARES, M.D.
by the other officers alone at the pleasure of the
officer appointing him."cralaw virtua1aw library President"

It appears from the records, specifically the minutes Although the minutes of the organizational meeting
of the special meeting of the Society on August 3, show that the Chairman mentioned the need of
1972, that petitioner was designated as Acting appointing a "permanent" Executive Secretary, such
Executive Secretary with an honorarium of P200.00 statement alone cannot characterize the
monthly in view of the application of Dr. Jose Y. appointment of petitioner without a contract of
Buktaw for leave effective September 1, 1972 for employment definitely fixing her term because of the
300 working days. This designation was formalized specific provision of Section 7.02 of the Code of By-
in Special Order No. 110, s. 1972 wherein it was Laws that: "The Executive Secretary, the Auditor,
indicated that: "This designation shall take effect on and all other officers and employees of the Society
September 1, 1972 and shall remain until further shall hold office at the pleasure of the Board of
advice."cralaw virtua1aw library Directors, unless their term of employment shall
have been fixed in their contract of employment."
In the organizational meeting of the Society on April Besides the word "permanent" could have been
25, 1973, the minutes of the meeting reveal that the used to distinguish the appointment from "acting
Chairman mentioned the need of appointing a capacity."cralaw virtua1aw library
permanent Executive Secretary and stated that the
former Executive Secretary, Dr. Jose Y. Buktaw, The absence of a fixed term in the letter addressed
tendered his application for optional retirement, and to petitioner informing her of her appointment as
Executive Secretary is very significant. This could of social order, these are merely guides for human
have no other implication than that petitioner held conduct in the absence of specific legal provisions
an appointment at the pleasure of the appointing and definite contractual stipulations. In the case at
power. bar, the Code of By-Laws of the Society contains a
specific provision governing the term of office of
An appointment held at the pleasure of the petitioner. The same necessarily limits her rights
appointing power is in essence temporary in nature. under the New Civil Code and the New Constitution
It is co-extensive with the desire of the Board of upon acceptance of the appointment.
Directors. Hence, when the Board opts to replace
the incumbent, technically there is no removal but Moreover, the act of the Board in declaring her
only an expiration of term and in an expiration of position as vacant is not only in accordance with the
term, there is no need of prior notice, due hearing or Code of By-Laws of the Society but also meets the
sufficient grounds before the incumbent can be exacting standards of honesty and good faith. The
separated from office. The protection afforded by meeting of May 29, 1974, at which petitioner’s
Section 7.04 of the Code of By-Laws on Removal of position was declared vacant, was called
Officers and Employees, therefore, cannot be specifically to take up the unfinished business of the
claimed by petitioner.chanrobles virtual lawlibrary Reorganizational Meeting of the Board of April 30,
1974. Hence, said act cannot be said to impart a
Thus, in the case of Mojilla v. Mariño, 13 SCRA dishonest purpose or some moral-obliquity and
293, where the appointment contains the following conscious doing to wrong but rather emanates from
proviso: that it may be terminated at anytime without the desire of the Board to reorganize itself.
any proceedings, at the pleasure of the President of
the Philippines, this Court held: "It may, therefore, Finally, We find it unnecessary to resolve the third
be said that, though not technically a temporary assignment of error. The proscription against
appointment, as this term is used in Section 24(b) of removal without just cause and due process of law
the Civil Service Act of 1959, petitioner’s under the Civil Service Law does not have a bearing
appointment in essence is temporary because of its on the case at bar for the reason, as We have
character that it is terminable at the pleasure of the explained, that there was no removal, in her case
appointing power. Being temporary in nature, the but merely an expiration of term pursuant to Section
appointment can be terminated at a moment’s 7.02 of the Code of By-Laws. Hence, whether or not
notice without need to show cause as required in the petitioner falls within the protective mantle of the
appointments that belong to the classified Civil Service Law is immaterial and definitely
service."cralaw virtua1aw library unnecessary to resolve this
case.chanroblesvirtualawlibrary
In Paragas v. Bernal, 17 SCRA 150, this Court
distinguished between removal and expiration of WHEREFORE, premises considered, the decision
term:jgc:chanrobles.com.ph of the lower court holding that petitioner was not
illegally removed or ousted from her position as
"In the case at bar there has been, however, no Executive Secretary of the Philippine Tuberculosis
removal from office, Pursuant to the charter of Society, Inc., is hereby AFFIRMED.
Dagupan City, the Chief of Police thereof holds
office at the pleasure of the President. SO ORDERED.
Consequently, the term of office of the Chief of
Police expires at any time that the President may so
declare. This is not removal, inasmuch as the latter
entails the ouster of an incumbent before the
expiration of his term. In the present case,
petitioner’s term merely expired upon receipt by him
of the communication of respondent Assistant
Executive Secretary of the President, dated
September 14, 1962."cralaw virtua1aw library

Petitioner cannot likewise seek relief from the


general provisions of the New Civil Code on Human
Relations nor from the fundamental principles of the PSBA
New Constitution on preservation of human dignity. vs.
While these provisions present some basic LEANO
principles that are to be observed for the rightful GR L- 58468 February 24, 1984
relationship between human beings and the stability
relations between and among stockholders, and
FACTS: between the stockholders and the corporation. It
also has exclusive jurisdiction over controversies
Tan is one of the stockholders of PSBA. He involving the election and appointment of officers,
was a director and Executive Vice- President directors, trustees or managers of such corporation.
enjoying salaries and allowances. During a regular The case is not a case of dismissal. The
meeting, the Board of Directors declared all case is that of a corporate office having been
corporate positions vacant except those of the declared vacant and of Tan’s not having been re-
president and chairman and at the same time elected thereafter. The matter of whom to elect is a
elected new set of officers. Tan was not re elected prerogative that belongs to the Board and involves
for which he filed for illegal dismissal before the the exercise of deliberate choice and the faculty of
NLRC. He also instituted a one million peso discriminative selection. Generally speaking, the
damage suit before the Court of First Instance for relationship of a person to the corporation, whether
the illegal and oppressive removal. He lodged as an officer or as agent or employee, is not
another complaint with the SEC questioning the determined by the services performed but by the
validity of the elections and his ouster. The SEC incidents of the relationship as they actually exist.
issued a subpoena duces tecum commanding the
production of all corporate documents, records,
books. The Labor Arbiter also issued a subpoena G.R. No. L-68544 October 27, 1986
duces tecum for the production of the same records
and documents. Petitioners moved for the dismissal LORENZO C. DY, ZOSIMO DY, SR., WILLIAM
of the complaint before the NLRC invoking the IBERO, RICARDO GARCIA AND RURAL BANK
principle against split jurisdiction. OF AYUNGON, INC., petitioners,
vs.
ISSUE:
NATIONAL LABOR RELATIONS COMMISSION
AND EXECUTIVE LABOR ARBITER ALBERTO L.
Whether or not the NLRC has jurisdiction DALMACION, AND CARLITO H.
over the case. VAILOCES, respondents.
RULING:
Facts:
NO.
Private respondent, Carlito H. Vailoces, was the
PSBA is a domestic corporation duly manager of the Rural Bank of Ayungon (Negros
organized and existing under our laws. General Oriental), a banking institution duly organized under
management is vested in a Board of seven elected Philippine laws. He was also a director and
annually by stockholders entitled to vote, who serve stockholder of the bank.
until the election and qualification of their
successors. Any vacancy in the board of directors is A special stockholders' meeting was called for the
filled up by a majority vote of the subscribed capital purpose of electing the members of the bank's
stock entitled to vote at a meeting generally called Board of Directors. Immediately after the election
for the purpose, and the directors so chosen shall the new Board proceeded to elect the bank's
hold office for the unexpired term. Corporate officers executive officers.
are provided for, among them, the Executive Vice-
President, who is elected by the board from their Petitioners Lorenzo Dy, William Ibero and Ricardo
own number. The officers receive such salaries as Garcia were elected president, vice-president and
the board may fix. The by-laws likewise provide that corporate secretary, respectively. Vailoces was not
should the office be rendered vacant by reason of re-elected as bank manager, 3 Because of this
death, resignation, disqualification or otherwise, the development, the Board passed Resolution No. 5,
board, by a majority vote may choose a successor series of 1983, relieving him as bank manager.
who shall hold office for the unexpired term of the
predecessor.
Vailoces filed a complaint for illegal dismissal and
The controversy is intra-corporate in nature.
damages with the Ministry of Labor and
It revolves around the election of directors, officers
Employment against Lorenzo Dy and Zosimo Dy,
and managers of PSBA, the relation between and
Sr. The complaint was amended to include
among its stockholders, and between them and the
additional respondents-William Ibero, Ricardo
corporation. PD 902-A vests in the SEC the original
Garcia and the Rural Bank of Ayungon, and
and exclusive jurisdiction to hear and decide cases
involving controversies arising out of intra-corporate
additional causes of action for underpayment of In this Court, petitioners assail said ruling as an
salary and non-payment of living allowance. arbitrary deprivation of their right to appeal through
unreasonable adherence to procedural technicality.
Vailoces asserted that Lorenzo Dy, after obtaining
control of the majority stock of the bank by buying Issue:
the shares of Marcelino Maximo, called an illegal
stockholders' meeting and elected a Board of W/N the election of directors and the election of
Directors controlled by him; that after its illegal officers which resulted in Vailoces failure to be re-
constitution, said Board convened and passed a elected, were validly held.
resolution dismissing him as manager, without
giving him the opportunity to be heard first; that his Ruling: Yes.
dismissal was motivated by Lorenzo Dy's desire to
take over the management and control of the bank,
not to mention the fact that he (Dy) harbored ill This is not a case of dismissal. The situation is that
feelings against Vailoces. of a corporate office having been declared vacant,
and of Vailoces not having been elected thereafter.
The matter of whom to elect is a prerogative that
Lorenzo Dy, et al. denied the charge of illegal
belongs to the Board, and involves the exercise of
dismissal. They pointed out that Vailoces' position
deliberate choice and the faculty of discriminative
was an elective one, and he was not re-elected as
selection. Generally speaking, the relationship of a
bank manager because of the Board's loss of
person to corporation, whether as officer or as
confidence in him brought about by his absenteeism agent or employee, is not determined by the nature
and negligence in the performance of his duties; of the services performed, but by the incidents of
and that the Board's action was taken to protect the
the relationship as they actually exist.
interest of the bank and was "designed as an
internal control measure to secure the check and
balance of authority within the organization." 5 The questioned decision of the Labor Arbiter and
the Resolution of the NLRC dismissing petitioners'
appeal from said decision are hereby set aside
The Executive Labor Arbiter found that Vailoces
because rendered without jurisdiction.
was Illegally dismissed because of the resentment
of petitioners against Vailoces which arose from the
latter's filing of the cases for recognition as natural
child against Zosimo Dy, Sr. and for violation of the
corporation code against Lorenzo Dy; Dy vs. National Labor Relations Commission

No. L-68544. October 27, 1986


and second, because he was not afforded the due
process of law when he was dismissed during the
Narvasa, J.
Board meeting the validity of which is seriously
doubted;

Lorenzo Dy, et al. appealed to the NLRC, assigning Doctrine: It is the Securities and Exchange
error to the decision of the Labor Arbiter on the
Commission (SEC) and not the National Labor
ground that Vailoces was not entitled to notice of
the Board meeting which decreed his relief because Relations Commission (NLRC) that has jurisdiction
he was no longer a member of the Board on said over a dispute involving the termination of a bank
date; that he nonetheless had the opportunity to manager as a result of his non-reelection, thereto,
refute the charges against him and seek a formal as prescribed in the Bank’s by-laws.
investigation because he received a copy of the
minutes of said meeting while he was still the bank
manager, instead of which he simply abandoned the
work he was supposed to perform up to the It is no hindrance to SEC jurisdiction that a
effective date of his relief; and that the matter of his person raises in his complaint the issues that he
relief was within the adjudicatory powers of the was illegally dismissed and asks for remuneration
Securities and Exchange Commission. 7
where complainant is not a mere employee but a
stockholder and officer of the corporation.
The NLRC, however bypassed the issues raised
and simply dismissed the appeal for having been
filed late.
FACTS: Petitioners Lorenzo C. Dy, Zosimo Dy, Sr., the issues and dismissed the appeal for having
William Ibero, Ricardo Garcia and Rural Bank of been filed late. Hence, this petition.
Ayungon, Inc. assail in this Court the resolution of
public respondent NLRC dismissing their appeal
from the decision of the Executive Labor Arbiter in
ISSUE: Whether or not the SEC, and not
Cebu City which found private respondent Carlito H.
respondent NLRC, has jurisdiction over the dispute.
Vailoces to have been illegally dismissed by them.

Private respondent Vailoces was the


manager of the Rural Bank of Ayungon (Negros RULING: YES. While the comment of Vailoces
Oriental), a banking institution duly organized under traverses the averments of the petition that of the
Philippine laws. He was also a director and Solicitor General on behalf of public respondents
stockholder of the bank. perceives the matter as an intra-corporate
controversy of the class described in Section 5, par.
On June 4, 1983, a special stockholders’
(c) of Presidential Decree No. 902-A, namely:
meeting was called for the purpose of electing the
members of the bank’s Board of Directors.
Immediately after the election, the new Board
proceeded to elect the bank’s executive officers.

Pursuant to Article 4 of the bank’s by-laws,


providing for the election by the entire membership
of the Board of the executive officers of the bank, ‘Original and exclusive
i.e., the president, vice president, secretary, cashier jurisdiction to hear and decide
and bank manager, in that board meeting of June 4, cases involving:
1983, petitioners Lorenzo Dy, William Ibero and
Xxxx
Ricardo Garcia were elected president, vice
president, and corporate secretary, respectively. Xxxx
Private respondent Vailoces was not re-elected as
bank manager. Because of this, the Board passed a
Resolution relieving him as bank manager.
“(c) Controversies in the
Subsequently, Vailoces filed a complaint for election or appointments of
illegal dismissal and damages with the Ministry of directors, trustees, officers or
Labor and Employment against herein petitioners, managers of such corporations,
asserting that an illegal stockholders’ meeting was partnerships or associations.”’
held. In their answer, petitioners denied the charge
of illegal dismissal. The Executive Labor Arbiter
found that Vailoces was illegally dismissed due to
explicitly declared to be within the original
the resentment of petitioners against Vailoces and
jurisdiction of the SEC, and recommends that the
consequently ordered the individual petitioners
questioned resolution of the NLRC as well as the
Lorenzo Dy and Zosimo Dy, Sr. to pay Vailoces
decision of the Labor Arbiter be set aside as null
jointly and severally the sum of P111,480.60 and
and void.
reinstate the latter to his position as bank manager,
with additional backwages. The judgment of the Labor Arbiter and the
resolution of the NLRC are void for lack of
Petitioner Lorenzo Dy appealed to the
jurisdiction. It is of no moment that Vailoces, in his
NLRC, assigning error to the decision of the Labor
amended complaint, seeks other relief which would
Arbiter, one being that the matter of Vailoces’ relief
seemingly fall under the jurisdiction of the Labor
was within the adjudicatory powers of the Securities
Arbiter, because underpayment of salary and non-
and Exchange Commission. The NLRC bypassed
payment of living allowance show that they are
actually part of the perquisites of his elective
position, hence, intimately linked with his relations corporation.
with the corporation. The question of remuneration Upon organization in 1978, respondent corporation
involving a person who is not a mere employee but immediately opened a Japanese establishment,
known as Fujiyama Hotel & Restaurant, located at
a stockholder and officer of a corporation is not a
1413 M. Adriatico St., Ermita, Manila. In order to
simple labor problem but a matter that comes within fully offer an authentic Japanese cuisine and
the area of corporate affairs and management, and traditional Japanese style of service, private
is in fact a corporate controversy in contemplation of respondent hired the services of Isamu Akasako as
the Corporate Code. its chef and restaurant supervisor. (Private
respondent's memorandum, p. 4).
In June, 1980, Lourdes Jureidini and Milagros
Tsuchiya, allegedly pretending to be stockholders of
Wherefore, the questioned decision of the the corporation, filed a case with the then Court of
Labor Arbiter and the Resolution of the NLRC First Instance of Manila, Branch XXXVI against
dismissing petitioners’ appeal are hereby set aside Rivera and Akasako to wrest control over the
establishment. In June, 1981, the said court issued
for being rendered without jurisdiction.
a writ of preliminary mandatory injunction
transferring possession of all the assets of the
company and the management thereof to Jureidini
and Tsuchiya. The stockholders and directors of the
LUZVIMINDA VISAYAN, BENJAMIN BORJA, corporation were thereby excluded from the
PABLO AJERO, LORETO DEDOYCO, NESTOR
management and operation of the restaurant.
GORGOLLO, DOMINGO METRAN, LITO
Upon assuming management, Jureidini and
MONTERON, ROMEO OMAGBON, BOMBOM Tsuchiya replaced almost all of the existing
PAUSAMOS, CIRILO RAMOS, MARCOS SISON, employees with new ones, majority of whom are the
ERIC BONDOLO, REY ZAMORA, TERESA present petitioners in the instant case. Apparently,
ANAVISO, EVELYN BACULINAO, MARIBEL the new employees were extended probationary
BASAG, VIOLETA DAGUISA, ADELAIDA
appointments for six (6) months from December 15,
CANALDA, LAILA DIMLA, MACHAELA LUCERO,
1981 to June 1 5, 1982.
DIVINA MARIANO, EPIFANIA OBLIGADO, In the meantime, Rivera and the rest of the
RAQUEL PONCIANO, ELLEN SACRAMENTO, stockholders elevated the civil case to the Supreme
GRACE SULLETA FELY TAPAY, SUSAN Court through a petition for certiorari assailing the
VILLAMOR, ANAINO AMPLAYO, MARIO ground for the issuance of the writ of preliminary
CHIONG NESTOR ESTARES, ALELI ALEJO,
mandatory injunction by the said Court of First
ELVIE BAUTISTA, JANINA ESTARES NORMA
Instance, which case was entitled Aquilino Rivera,
MENDOZA, LIGAYA SYDUA and JANETTE et al. vs. Hon. Alfredo C. Florendo, et al., docketed
VILLAREAL, petitioners, as G.R. No. 57586. On motion of Rivera, et al. in
vs. the said case, this Court on August 21, 1981 issued
NATIONAL LABOR RELATIONS COMMISSION
a writ of preliminary injunction to enjoin enforcement
(THIRD DIVISION) and FUJIYAMA RESTAURANT
of the June 23, 1981 writ of preliminary mandatory
AND HOTEL, INC. and its
injunction issued by the said Court of First Instance.
MANAGER/OPERATOR, respondents. Since Jureidini and Tsuchiya disregarded the writ
Danilo S. Lorredo for petitioners.
 King, Capuchino, We had previously issued, We issued another
Banico & Associates for private respondent. resolution on May 26, 1982 directing both Jureidini
and Tsuchiya to strictly and immediately comply
PARAS, J.:
with the Court's injunction. Thus, this Court ordered
Assailed in the instant petition is the Resolution of Jureidini and Tsuchiya, "their agents,
public respondent National Labor Relations representatives, and/or any person or persons
Commission (NLRC, for brevity) promulgated acting upon their orders or in their place or stead to
January 15, 1985 for being contrary to law and refrain from further managing and/or interfering with
jurisprudence and arrived at in grave abuse of the management of the business and assets of
discretion amounting to lack or in excess of petitioner corporation and . . . . to turn over all
jurisdiction. assets and the management of petitioner
The facts are briefly stated as follows: corporation, Fujiyama Hotel & Restaurant, Inc., to
Private respondent Fujiyama Hotel & Restaurant, Aquilino Rivera and Isamu Akasako." (NLRC,
Inc. was formally organized in April, 1978 with Resolution, p. 4; Rollo, p. 116).
Aquilino Rivera holding a majority interest in the Pursuant to the above-quoted resolution, Rivera
corporation. The rest of the four (4) incorporators and Akasako regained control and management of
composed the minority stockholders of respondent Fujiyama Hotel & Restaurant, Inc. Immediately upon
assumption of the management of the corporation, complainants hired by them while operating the
Rivera et al., refused to recognize as employees of Fujiyama Restaurant & Hotel, Inc. Consequently,
the corporation all persons that were hired by the establishment and its present operator, Isamu
Jureidini and Tsuchiya during the one-year period Akasako, is absolved of any liability to them but the
that the latter had operated the company and entire record is remanded for further appropriate
reinstated the employees previously hired by them. proceedings to determine who are the complainants
This gave rise to the filing of the present case by the hired by said Jureidini and Tsuchiya.
dismissed employees hired by Jureidini and SO ORDERED.
Tsuchiya (some of whom had allegedly been hired (NLRC Resolution, pp. 19-20; Rollo, p. 7-8)
by Rivera and Akasako even before Jureidini and The legal issues in the instant case are: (1) whether
Tsuchiya assumed management of the corporation) or not there is privity of contract between petitioners
against Fujiyama Hotel & Restaurant, Inc. for illegal and private respondent as to establish an employer-
dismissal, which case was docketed as NLRC-NCR employee relationship between the parties, and (2)
Case No. 6-4110-82. On motion of private whether or not the respondent NLRC erred in giving
respondent corporation, the Labor Arbiter included due course to private respondent's appeal and in
Jureidini and Tsuchiya as third-party respondents reversing the September 21, 1982 decision of the
therein. Thereafter, the parties, except Jureidini and Labor Arbiter.
Tsuchiya, submitted their respective position papers Section 23 of B.P. 68, otherwise known as the
and affidavits in support of their contentions. On the "Corporation Code of the Philippines," expressly
basis of said position papers and affidavits, the provides as follows:
Labor Arbiter rendered a decision on September 21, Unless otherwise provided in this Code, the
1982 ordering respondent company and/or corporate powers of all corporations formed under
Akasako, Jureidini and Tsuchiya to reinstate all the this Code shall be exercised, all business
complainants to their former positions plus conducted and all property of such corporations
backwages and to pay jointly and severally the controlled and held by the board of directors or
complainants their unpaid wages plus their share in trustees to be elected from among the holders of
the service charges. (NLRC Decision, pp. 4-5; stocks, or where there is no stock, from among the
Rollo, pp. 25-26). members of the corporation, who shall hold office
On October 12, 1982, the aforesaid decision of the for one (1) year and until their successors are
Labor Arbiter was received by private respondent's elected and qualified. . . .
counsel. Ten (10) days thereafter, or on October 22, It is clear from the above-quoted provision that a
1982, said counsel filed a notice of appeal with an corporation can act only through its board of
accompanying supersedeas bond in the sum of directors. "The law is settled that contracts between
P80,000.00 as fixed by the Labor Arbiter. Notably, a corporation and third persons must be made by or
the memorandum of appeal was not filed until under the authority of its board of directors and not
November 24, 1982 when the attention of private by its stockholders. Hence, the action of the
respondent's counsel was called by the filing on stockholders in such matters is only advisory and
November 19, 1982 of a motion for execution of the not in any wise binding on the corporation." (De
September 21, 1982 decision by the complainants. Leon, The Corporation Code of the Philippines,
Thus, upon motion of private respondent, the NLRC 1989 edition, p. 168, citing the case of Barreto vs.
temporarily stayed execution and directed the Labor La Previsora Filipina, 57 Phil. 649).
Arbiter to transmit the entire record of the case to A corporation, like a natural person who may
the NLRC for appropriate action. authorize another to do certain acts for and in his
On December 28, 1983, the NLRC resolved to deny behalf, through its board of directors, may legally
the appeal of private respondent for having been delegate some of its functions and powers to its
filed out of time.1âwphi1 Subsequently, a motion for officers, committees or agents appointed by it.
reconsideration was seasonably filed by private (Campos & Campos, The Corporation Code-
respondent which became the basis of another Comments, Notes, and Selected cases, 1981 ed., p.
resolution dated January 15, 1985 issued by the 253). In the absence of an authority from the board
NLRC setting aside its previous resolution of of directors, no person, not even the officers of the
December 28, 1983 as well as the Labor Arbiter's corporation, can validly bind the corporation. Thus,
decision dated September 21, 1982. The decretal the Supreme Court, has made the following
portion of the January 15, 1982 NLRC Resolution is pronouncement in the case of Vicente vs. Geraldez,
quoted, thus: L-32473, 53 SCRA 210:
WHEREFORE, the Resolution sought to be . . . Whatever authority the officers or agents of a
reconsidered and the Decision appealed from are corporation may have is derived from the board of
hereby SET ASIDE and a new Decision is entered, directors or other governing body, unless conferred
declaring respondents Lourdes Jureidini and Mila by the charter of the corporation. A corporate
Tsuchiya as the previous employer of the officer's power as an agent of the corporation must
therefore be sought from the statute, the charter, regaining control and management of the latter on
the by-laws, or in a delegation of authority to such May 31, 1982, having been made by the
officer, from the acts of the board of directors, corporation's board of directors, was valid. Even if
formally expressed or implied from a habit or Jureidini and Tsuchiya took over the management
custom of doing business. In the case at bar no and control of respondent corporation, the
provision of the charter and by-laws of the employer-employee relationship between the
corporation or any resolution or any other act of the corporation and its original employees has not been
board of directors has been cited from which we severed for lack of authority on the part of Jureidini
could reasonably infer that the administration trative and Tsuchiya to dismiss said employees.
manager had been granted expressly or impliedly Consequently, petitioners' claim of illegal dismissal
the power to bind the corporation or the authority to is entirely mistaken as they were not hired by
compromise the case. The signature of Atty. respondent corporation or its duly authorized
Cardenas on the Agreement would therefore be officers or agents, hence, no employer-employee
legally ineffectual". (Vicente vs. Geraldez, L-32473, relationship ever existed between them. Jureidini
52 SCRA 210, p. 227). (Respondent's and Tsuchiya, the persons who hired petitioners'
Memorandum, p. 11) services, are to be considered their employer, and
Applying the aforesaid doctrines in the case at bar, not the private respondents.
We hold that all acts done solely by Jureidini and Neither may petitioners claim good faith or
Tsuchiya allegedly, for and in behalf of private ignorance of the lack of authority on the part of
respondent during the period from June, 1981 up to Jureidini and Tsuchiya to legally hire them and bind
May 31, 1982 were not binding upon respondent the corporation because they were all informed by
corporation. Isamu Tatewaki respondent corporation's Assistant
It is not denied by both parties that the operation Manager, of such fact at the time they were hired.
and management of the Fujiyama Hotel & (Reply Brief of Isamu Tatewaki Annex "10").
Restaurant Corporation, including the control and Besides, it was clearly shown that the appointments
possession of all its assets, were forcibly taken by of the petitioners were on a probationary basis.
Jureidini and Tsuchiya from the owners thereof by Further, it will be recalled that on August 21, 1981,
virtue of a writ of preliminary mandatory injunction this Court issued a writ of preliminary injunction in
issued by then Court of First Instance of Manila, the case of Rivera, et al. vs. Judge Alfredo C.
Branch XXXVI These owners, the Rivera-Akasako Florendo, et al., G.R. No. 57586, promulgated
group, composed the board of directors of October 8, 1986, enjoining the enforcement of the
respondent corporation during the one (1) year writ of preliminary mandatory injunction issued by
period that Jureidini and Tsuchiya controlled the respondent judge therein. Despite the issuance of
respondent corporation, the former managed and said writ, Jureidini and Tsuchiya refused to return
operated the latter apparently without any authority the management of the corporation but continued
from the latter's board of directors. As alleged by managing and operating respondent corporation
Rivera, et al., Jureidini and Tsuchiya were not even and in fact terminated the original employees of
officers of respondent corporation as to be respondent corporation and hired new ones in place
considered its agents, which act prompted this of those dismissed. The appointment papers of
tribunal to order said persons, under pain of these new employees would show that they were
contempt, to turn over the management and assets hired only in one day, i.e., December 15, 1981, and
of respondent corporation to Rivera et al., as shown that they were hired on a probationary basis. It
by this Court's resolution of May 26, 1982. Thus, all follows that only Jureidini and Tsuchiya, being the
acts done by Jureidini and Tsuchiya for and in ones who hired the petitioners, should be the ones
behalf of respondent corporation, having been responsible for the petitioners' claims.
made without the requisite authority from the board Since it would be most unfair and unjust to hold the
of directors, were not binding upon the said respondent corporation liable for the claims of
corporation. One of these unauthorized acts was petitioners, even if respondent corporation's
the unwarranted termination of the original memorandum was filed beyond the 10 day
employees of respondent corporation who were reglementary period (note that the notice of appeal
validly hired by its board of directors, vis-a-vis, the had been filed on time), We rule that the NLRC did
hiring of new employees, the petitioners in the case not commit grave abuse of discretion in giving due
at bar, to replace the said original employees. Since course to respondent corporation's appeal and in
said acts were not binding upon the corporation, no reversing the Labor Arbiter's decision dated
employer-employee existed between the Fujiyama September 21, 1982.
Hotel & Restaurant, Inc. and the herein petitioners. The NLRC is vested with broad powers by the
We agree with private respondent that the act of the Labor Code, particularly Art. 218 thereof, to correct,
Rivera-Akasako group in admitting the original amend or waive any error, injustice, defect or
employees of respondent corporation after irregularity whether in substance or in form; and in
adjudicating all cases brought before it, the NLRC is peculiar factual background of the instant case,
likewise empowered to use every and all private respondent's defense of lack of privity of
reasonable means to ascertain the facts in each contract with petitioners merits greater
case expeditiously and objectively without regard to consideration in the interest of substantial justice.
procedural technicalities. Thus, Art. 221 of the It will be recalled that the Labor Arbiter's finding of
Labor Code provides as follows: illegal dismissal and order of reinstatement were
In any proceeding before the Commission or any of anchored on an erroneous premise that Jureidini
the Labor Arbiters, the rules of evidence prevailing and Tsuchiya were duly authorized and legitimate
in Courts of Law or equity shall not be controlling officers of the corporation. The enforcement of said
and it is the spirit and intention of this Code that the erroneous ruling will cause serious injustice, not
Commission and its members and the Labor only upon respondent corporation but also upon the
Arbiters shall use every and all reasonable means corporation's original employees who were taken
to ascertain the facts in each case speedily and back by the Aquilino Rivera group when they
objectively and without regard to technicalities of regained possession and management of the
law or procedure, all in the interest of due process. corporation. If petitioners are reinstated, that would
In any proceeding before the Commission or any result in an absurd situation wherein the corporation
Labor Arbiter to exercise complete control of the will have employees very much more in excess of
proceedings at all stages. what the business would require.
The factual circumstances and substantial merits of Besides, it is quite evident that private respondent
the instant case justify the NLRC's exercise of its seriously intended to appeal the Labor Arbiter's
reserve powers granted by the aforequoted decision and We hereby quote a portion of the
provision. Private respondent's appeal should be herein assailed NLRC Resolution:
granted and entertained in order to prevent a . . . In fact, it even filed an urgent petition for
manifest injustice upon said respondent. reduction of supersedeas bond, praying that it be
While it is true that an appeal within the meaning of allowed to file a P50,000.00 bond but it was fixed at
the Labor Code must include the assignments of P80,000.00 by the Labor Arbiter which it filed with
error, memorandum of arguments in support thereof its notice of appeal. In the conference on 15
and the reliefs prayed for such that a mere notice of October 1982 called by the Labor Arbiter issuing his
appeal will not toll the running of the period for decision for the purpose of settling the case
perfecting an appeal, and the general rule is that amicably, the respondent again manifested after no
after a judgment has become final the appellate settlement was arrived at that it will file its appeal.
court loses jurisdiction to entertain the appeal, the With these in mind, We are convinced that
aforementioned rules admit of exceptions too, respondent's failure to file its memorandum on
because it is also well-settled that such rules of appeal with its notice of appeal was through
procedure are used only to help secure and not excusable mistake only on the part of the
override substantial justice. messenger-clerk. Otherwise, it would not have gone
Litigations should, as much as possible, be decided through the burden of going through the rigors of
on their merits and not on technicality, and under having the supersedeas bond reduced and abiding
the circumstances obtaining in this case, We are with the amount fixed which entailed expenses.
reminded of what We said in the case of Gregorio Consequently, in the interest of substantial justice
vs. CA, 72 SCRA 120, –– "Dismissal of appeals and in line with the repeated rulings of the Supreme
purely on technical grounds is frowned upon where Court lately which abhors dismissal of cases based
the policy of the courts is to encourage hearings of solely on technicalities, We set aside the Resolution
appeals on their merits and the rules of procedure sought to be reconsidered and give due course to
ought not to be applied in a very rigid, technical the appeal. (pp. 15-16, Rollo)
sense; rules of procedure are used only to help Finally' it is clear that petitioners were not
secure, not override substantial justice. If a abandoned by the NLRC as the latter ordered that
technical and rigid enforcement of the rules is the case be remanded to the Arbitration Branch for
made, their aim would be defeated. (American further proceedings to determine who among the
Home Insurance Co. vs. Court of Appeals, 109 petitioners were really hired by respondent
SCRA 180) corporation or by Jureidini, et al., in order to
In the case at bar, the finding of the Labor Arbiter ultimately determine who is responsible for the
that there is an employer-employee relationship settlement of petitioners' claims. Thus, petitioners
existing between petitioners and private respondent are not without recourse relative to their claims.
counter-acts the provisions of the Corporation Code ACCORDINGLY, the instant petition is hereby
such that to strictly apply the procedural rules on DISMISSED for lack of merit and the assailed
appeal under the Labor Code would obviously result decision of the National Labor Relations
in patent and gross injustice upon private Commission dated January 15, 1985 is AFFIRMED
respondent's substantive rights. In relation to the in toto.
SO ORDERED. WHETHER OR NOT THE NATIONAL LABOR
RELATIONS COMMISSION COMMITTED GRAVE
ABUSE OF DISCRETION AMOUNTING TO LACK
OF JURISDICTION OR ACTED IN EXCESS OF
G.R. No. 108710 September 14, 1999 ITS JURISDICTION IN HOLDING THAT THE
SECURITIES AND EXCHANGE COMMISSION
ARMANDO T. DE ROSSI, petitioner, HAS JURISDICTION OVER THE COMPLAINT
vs.
FOR ILLEGAL DISMISSAL FILED BY
NATIONAL LABOR RELATIONS COMMISSION
(First Division), MATLING INDUSTRIAL AND PETITIONER.
COMMERCIAL CORPORATION AND RICHARD
K. SPENCER, respondents. RULING:

FACTS: In a string of cases this Court has consistently held


that the SEC, and not the NLRC, has original and
An Italian citizen, petitioner was the Executive Vice- exclusive jurisdiction over cases involving the
President and General Manager of private removal of corporate officers. Section 5, paragraph
(c) of P.D. 902-A unequivocally provides that SEC
respondent, Matling Industrial and Commercial
has jurisdiction over intra-corporate affairs regarding
Corporation (MICC). He started work on July 1, the election or appointment of officers of a
1985. On August 10, 1988, MICC terminated his corporation.
employment.
We have earlier pronounced that an "office" is
Aggrieved, petitioner filed with the NLRC, National created by the charter of the corporation under
Capital Region on September 21, 1989, a complaint which a corporation is organized, and the officer is
for illegal dismissal with corresponding damages. elected by the directors or stockholders. In the
present case, private respondents aver that the
MICC based petitioner's dismissal on the ground officers and their terms of office are prescribed by
that the petitioner failed to secure his employment the corporation's by-laws, which provide as follows:
permit, grossly mismanaged the business affairs of
the company, and misused corporate funds. BY-LAW NO. III Directors and Officers
However, petitioner argued that it was the duty of
xxx xxx xxx
the company to secure his work permit during the
term of his office, and that his termination was
The officers of the corporation shall be the
illegal for lack of just cause.
President, Executive Vice President,
Secretary and Treasurer, each of whom may
Labor Arbiter Asuncion rendered a decision in favor
hold his office until his successor is elected
of petitioner. and qualified, unless sooner removed by the
Board of Directors; Provided, That for the
Private respondents contended that the position of convenience of the corporation the office of
executive vice-president is an elective post, the Secretary and Treasurer may be held by
specifically provided by the corporate's by-laws. one and the same person. Officers shall be
Thus, the dismissal of the petitioner was an intra- designated by the stockholders' meeting at
corporate matter within the jurisdiction of the the time they elect the members of the Board
of Directors. Any vacancy occurring among
Securities and Exchange Commission (SEC) and
the officers of the Corporation on account of
neither with the Labor Arbiter nor the NLRC. They removal or resignation shall be filled by a
argued that the SEC and not the NLRC has original stockholder's meeting. Stockholders holding
and exclusive jurisdiction over the subject matter one half, or more of the subscribed capital
which involves the removal of a corporate officer. stock of the corporation may demand and
compel the resignation of any officer at any
NLRC rendered its decision recognizing the SEC's time. 10
jurisdiction over the case.
The by-laws being in force, clearly petitioner is
ISSUE: considered an officer of MICC, elected and/or
designated by its board of directors. Following
Section 5(c) of P.D. No. 902-A, the SEC exercises complaint. The LA granted the motion to dismiss,
exclusive jurisdiction over controversies regarding which decision was reversed by the NLRC.
the election and/or designation of directors,
trustees, officers or managers of a corporation, ISSUE:
partnership or association. This provision is
indubitably applicable to the petitioner's case. Whether or not respondent was a corporate
Jurisdiction here is not with the Labor Arbiter nor the officer.
NLRC, but with the SEC.
RULING:
Note that a corporate officer's removal from his
office is a corporate act. If such removal occasions YES.
an intra-corporate controversy, its nature is not
altered by the reason or wisdom, or lack thereof, Private respondent is an officer of Petitioner
with which the Board of Directors might have in Corporation and not its mere employee. The by-
taking such action. When petitioner, as Executive laws of the Galeria de Magallanes Condominium
Vice-President allegedly diverted company funds for Association specifically include the
his personal use resulting in heavy financial losses Superintendent/Administrator in its roster of
to the company, this matter would amount to fraud. corporate officers. He was appointed directly by the
Such fraud would be detrimental to the interest not Board of Directors not by any managing officer of
only of the corporation but also of its members. This the corporation and his salary was, likewise, set by
type of fraud encompasses controversies in a the same Board. Having thus determined, his
relationship within the corporation covered by SEC dismissal or non-appointment is clearly an intra-
jurisdiction. Perforce, the matter would come within corporate matter and jurisdiction, therefore, properly
the area of corporate affairs and management, and belongs to the SEC and not the NLRC. Despite not
such a corporate controversy would call for the being elected, P.D. 902-A Sec. 5(c) expressly
adjudicative expertise of the SEC, not the Labor covers both election and appointment of corporate
Arbiter or the NLRC. directors, trustees, officers and managers.

PURIFICACION G. TABANG
vs.
BIENVENIDO ONGKINGCO, as President and
NATIONAL LABOR RELATIONS COMMISSION
GALERIA DE MAGALLANES CONDOMINIUM
G.R. No. 121143 January 21, 1997
ASSOCIATION, INC.
vs.
FACTS:
NATIONAL LABOR RELATIONS COMMISSION
and FEDERICO B. GUILAS
Purificacion Tabang was a founding
G.R. No. 119877, March 31, 1997
member, a member of the Board of Trustees, and
the corporate secretary of private respondent
FACTS:
Pamana Golden Care Medical Center Foundation,
Inc., a non-stock corporation engaged in extending
Petitioner Galeria de Magallanes
medical and surgical services. Medical Director and
Condominium Association, Inc. is a non-stock, non-
Hospital Administrator of private respondent's
profit corporation with a primary purpose of holding
Pamana Golden Care Medical Center in Calamba,
title to the common areas of the Galeria de
Laguna. Although the memorandum was silent as to
Magallanes Condominium Project and to manage
the amount of remuneration for the position,
and administer the same for the use and
petitioner claims that she received a monthly
convenience of the residents and/or owners.
retainer fee of five thousand pesos (P5,000.00) from
Petitioner Bienvenido Ongkingco was the president
private respondent, but the payment thereof was
of Galeria at the time private respondent filed his
allegedly stopped in November, 1991.
complaint. Subsequently, Galeria's Board of
As medical director and hospital
Directors appointed private respondent Federico B.
administrator, petitioner was tasked to run the
Guilas as Administrator/Superintendent.
affairs of the aforesaid medical center and perform
Respondent, however, was no longer re-appointed
all acts of administration relative to its daily
as Administrator; hence he filed a case for illegal
operations.Petitioner was allegedly informed
dismissal. Petitioners filed a motion to dismiss
personally by Dr. Ernesto Naval that in a special
alleging that it is the SEC, and not the labor arbiter,
meeting held on April 30, 1993, the Board of
which has jurisdiction over the subject matter of the
Trustees passed a resolution relieving her of her
position as Medical Director and Hospital
Administrator, and appointing the latter and Dr.
Benjamin Donasco as acting Medical Director and
acting Hospital Administrator, respectively. DILY DANY NACPIL
Petitioner averred that she thereafter received a vs.
copy of said board resolution. Petitioner then filed a INTERNATIONAL BROADCASTING
complaint for illegal dismissal and non-payment of CORPORATION
wages, allowances and 13th month pay before the G.R. No. 144767. March 21, 2002
labor arbiter but the complaint was dismissed for
lack of jurisdiction. FACTS:

ISSUE: Petitioner was Assistant General Manager


for Finance/Administration and Comptroller of
Whether or not the NLRC has jurisdiction private respondent Intercontinental Broadcasting
over the case. Corporation (IBC) from 1996 until April 1997.
According to petitioner, when Emiliano Templo was
RULING: appointed to replace IBC President Tomas Gomez
III sometime in March 1997, the former told the
NO. Board of Directors that as soon as he assumes the
IBC presidency, he would terminate the services of
It is the SEC which has jurisdiction over the petitioner. Apparently, Templo blamed petitioner,
case. The charges against private respondent along with a certain Mr. Basilio and Mr. Gomez, for
partake of the nature of an intra-corporate the prior mismanagement of IBC. Upon his
controversy. Similarly, the determination of the assumption of the IBC presidency, Templo allegedly
rights of petitioner and the concomitant liability of harassed, insulted, humiliated and pressured
private respondent arising from her ouster as a petitioner into resigning until the latter was forced to
medical director and/or hospital administrator, which retire. However, Templo refused to pay him his
are corporate offices, is an intra-corporate retirement benefits, allegedly because he had not
controversy subject to the jurisdiction of the SEC. yet secured the clearances from the Presidential
Contrary to the contention of petitioner, a medical Commission on Good Government and the
director and a hospital administrator are considered Commission on Audit.
as corporate officers under the by-laws of IBC filed a motion to dismiss contending
respondent corporation. that petitioner was a corporate officer who was duly
The president, vice-president, secretary and elected by the Board of Directors of IBC; hence, the
treasurer are commonly regarded as the principal or case qualifies as an intra-corporate dispute falling
executive officers of a corporation and modern within the jurisdiction of the SEC.
corporation statutes usually designate them as the On the other hand, petitioner argues that he
officers of the corporation. However, other offices is not a corporate officer of IBC but an employee
are sometimes created by the charter or by-laws of thereof since he had not been elected nor appointed
a corporation or the board of directors may be as Comptroller and Assistant Manager by the IBC's
empowered under the by-laws of a corporation to Board of Directors but by an IBC General Manager.
create additional offices as may be necessary. This is also because the IBC's By-Laws do not even
In the case at bar, considering that herein include the position of comptroller in its roster of
petitioner, unlike an ordinary employee, was corporate officers.He therefore contends that his
appointed by Respondent Corporation’s Board of dismissal is a controversy falling within the
Trustees in its memorandum, she is deemed an jurisdiction of the labor courts.
officer of the corporation. Section 5(c) of
Presidential Decree No. 902-A, provides that the ISSUE:
SEC exercises exclusive jurisdiction over
controversies in the election appointment of Whether or not petitioner is a corporate
directors, trustees, officers or managers of officer although the position of comptroller is not
corporations, partnerships or associations, applies expressly mentioned in the by-laws.
in the present dispute. Accordingly, jurisdiction over
the same is vested in the SEC, and not in the Labor RULING:
Arbiter or the NLRC.
A corporate officer's dismissal is always a NO.
corporate act, or an intra-corporate controversy, and
the nature is not altered by the reason or wisdom The fact that the position of Comptroller is
with which the Board of Directors may have in not expressly mentioned among the officers of the
taking such action. IBC in the By-Laws is of no moment, because the
IBC's Board of Directors is empowered under price rate of P15.00 per square meter.
Section 25 of the Corporation Code and under the
corporation's By-Laws to appoint such other officers
as it may deem necessary.
The by-laws may and usually do provide for One of the lawyers of HI Cement, Atty.
such other officers," and that where a corporate Francisco Ventura, then notified the Board of
office is not specifically indicated in the roster of Directors of HI Cement for the approval of the
corporate offices in the by-laws of a corporation, the
compromise agreement. But the Board disapproved
board of directors may also be empowered under
the by-lawsto create additional officers as may be the compromise agreement hence Atty. Ventura
necessary. Furthermore, as petitioner's appointment filed a motion with the court to disregard the
as comptroller required the approval and formal compromise agreement. Vicente et al naturally
action of the IBC's Board of Directors to become assailed the motion. Vicente et al insisted that the
valid, it is clear therefore holds that petitioner is a compromise agreement is binding because prior to
corporate officer whose dismissal may be the entering into the compromise agreement, the three
subject of a controversy cognizable by the SEC
lawyers of HI Cement declared in open court that
under Section 5(c) of P.D. 902-A which includes
controversies involving both election and they are authorized to enter into a compromise
appointmentof corporate directors, trustees, officers, agreement for HI Cement; that one of the lawyers of
and managers. Had petitioner been an ordinary HI Cement, Atty. Florentino Cardenas, is an
employee, such board action would not have been executive official of HI Cement; that Cardenas even
required. nominated one of the commissioners; that such act
ratified the compromise agreement even if it was
VICENTE VS GERALDEZ
not approved by the Board. HI Cement, in its
defense, averred that the lawyers were not
authorized and that in fact there was no special
Facts: power of attorney executed in their favor for the
purpose of entering into a compromise agreement.
Judge Ambrosio Geraldez ruled in favor of HI
Cement.
In 1967, HI Cement Corporation was
granted authority to operate mining facilities in
Bulacan. However, the areas allowed for it to
explore cover areas which were also being explored ISSUE: Whether or not a compromise agreement
by Ignacio Vicente, Juan Bernabe, and Moises entered into by a lawyer purportedly in behalf of the
Angeles. And so a dispute arose between the three corporation is valid without a written authority.
and HI Cement as neither side wanted to give up
their mining claims over the disputed areas.
Eventually, HI Cement filed a civil case against the
HELD: No. Corporations may compromise only in
three. During pre-trial, the possibility of an amicable
the form and with the requisites which may be
settlement was explored where HI Cement offered
necessary to alienate their property. Under the
to purchase the areas of claims of Vicente et al at
corporation law the power to compromise or settle
the rate of P0.90 per square meter. Vicente et al
claims in favor of or against the corporation is
however wanted P10.00 per square meter.
ordinarily and primarily committed to the Board of
Directors but such power may be delegated. The
delegation must be clearly shown for as a general
In 1969, the lawyers of HI Cement agreed rule an officer or agent of the corporation has no
to enter into a compromise agreement with the power to compromise or settle a claim by or against
three whereby commissioners shall be assigned by the corporation, except to the extent that such
the court for the purpose of assessing the value of power is given to him either expressly or by
the disputed areas of claim. An assessment was reasonable implication from the circumstances. In
subsequently made pursuant to the compromise the case at bar, there was no special power of
agreement and the commissioners recommended a attorney authorizing the three lawyers to enter into a
compromise agreement. This is even if the lawyers the tennis court located within the premises of the
declared in open court that they are authorized to Hidden Valley Springs Resort at Limao, Calauan,
do so by the corporation (in this case, the transcript Laguna;
b) Pay the plaintiff the amount of P300.00 per
of stenographic notes does not show that the
month from September 10, 1983, for his occupancy
lawyers indeed declare such in open court). of the said residential house until the same is
vacated; and
The fact that Cardenas, an officer of HI Cement, c) Pay the costs. (Rollo, p. 36)
acted in effecting the compromise agreement, i.e. In two (2) separate complaints for recovery of
nominating a commissioner, does not ratify the possession filed with the Regional Trial Court of
compromise agreement. There is no showing that Laguna against petitioners Rebecca Boyer-Roxas
Cardenas’ act binds HI Cement; no proof that he is and Guillermo Roxas respectively, respondent
authorized by the Board; no proof that there is a corporation, Heirs of Eugenia V. Roxas, Inc., prayed
for the ejectment of the petitioners from buildings
provision in the articles of incorporation of HI
inside the Hidden Valley Springs Resort located at
Cement that he can bind the corporation. Limao, Calauan, Laguna allegedly owned by the
respondent corporation.
In the case of petitioner Rebecca Boyer-Roxas
(Civil Case No-802-84-C), the respondent
REBECCA BOYER-ROXAS and GUILLERMO corporation alleged that Rebecca is in possession of
ROXAS, petitioners, two (2) houses, one of which is still under
vs. construction, built at the expense of the respondent
HON. COURT OF APPEALS and HEIRS OF corporation; and that her occupancy on the two (2)
EUGENIA V. ROXAS, INC., respondents. houses was only upon the tolerance of the
respondent corporation.
GUTIERREZ, JR., J.: In the case of petitioner Guillermo Roxas (Civil
This is a petition to review the decision and Case No. 803-84-C), the respondent corporation
resolution of the Court of Appeals in CA-G.R. No. alleged that Guillermo occupies a house which was
14530 affirming the earlier decision of the Regional built at the expense of the former during the time
Trial Court of Laguna, Branch 37, at Calamba, in when Guillermo's father, Eriberto Roxas, was still
the consolidated RTC Civil Case Nos. 802-84-C and living and was the general manager of the
803-84-C entitled "Heirs of Eugenia V. Roxas, Inc. respondent corporation; that the house was
v. Rebecca Boyer-Roxas" and Heirs of Eugenia V. originally intended as a recreation hall but was
Roxas, Inc. v. Guillermo Roxas," the dispositive converted for the residential use of Guillermo; and
portion of which reads: that Guillermo's possession over the house and lot
IN VIEW OF THE FOREGOING, judgment is was only upon the tolerance of the respondent
hereby rendered in favor of the plaintiff and against corporation.
the defendants, by ordering as it is hereby ordered In both cases, the respondent corporation alleged
that: that the petitioners never paid rentals for the use of
1) In RTC Civil Case No. 802-84-C: Rebecca Boyer- the buildings and the lots and that they ignored the
Roxas and all persons claiming under her to: demand letters for them to vacate the buildings.
a) Immediately vacate the residential house near In their separate answers, the petitioners traversed
the Balugbugan pool located inside the premises of the allegations in the complaint by stating that they
the Hidden Valley Springs Resort at Limao, are heirs of Eugenia V. Roxas and therefore, co-
Calauan, Laguna; owners of the Hidden Valley Springs Resort; and as
b) Pay the plaintiff the amount of P300.00 per co-owners of the property, they have the right to
month from September 10, 1983, for her occupancy stay within its premises.
of the residential house until the same is vacated; The cases were consolidated and tried jointly.
c) Remove the unfinished building erected on the At the pre-trial, the parties limited the issues as
land of the plaintiff within ninety (90) days from follows:
receipt of this decision; 1) whether plaintiff is entitled to recover the
d) Pay the plaintiff the amount of P100.00 per questioned premises;
month from September 10, 1983, until the said 2) whether plaintiff is entitled to reasonable rental
unfinished building is removed from the land of the for occupancy of the premises in question;
plaintiff; and 3) whether the defendant is legally authorized to
e) Pay the costs. pierce the veil of corporate fiction and interpose the
2) In RTC Civil Case No. 803-84-C: Guillermo same as a defense in an accion publiciana;
Roxas and all persons claiming under him to: 4) whether the defendants are truly builders in good
a) Immediately vacate the residential house near faith, entitled to occupy the questioned premises;
5) whether plaintiff is entitled to damages and pavilions; that the house near the Balugbugan Pool
reasonable compensation for the use of the (Exh. "B-l") being occupied by Rebecca B. Roxas
questioned premises; was originally intended as staff house but later used
6) whether the defendants are entitled to their as the residence of Eriberto Roxas, deceased
counterclaim to recover moral and exemplary husband of the defendant Rebecca Boyer-Roxas
damages as well as attorney's fees in the two and father of Guillermo Roxas; that this house
cases; presently being occupied by Rebecca B. Roxas was
7) whether the presence and occupancy by the built from corporate funds; that the construction of
defendants on the premises in questioned (sic) the unfinished house (Exh. "B-2") was started by the
hampers, deters or impairs plaintiff's operation of defendant Rebecca Boyer-Roxas and her husband
Hidden Valley Springs Resort; and Eriberto Roxas; that the third building (Exh. "B-3")
8) whether or not a unilateral and sudden presently being occupied by Guillermo Roxas was
withdrawal of plaintiffs tolerance allowing originally intended as a recreation hall but later
defendants' occupancy of the premises in converted as a residential house; that this house
questioned (sic) is unjust enrichment. (Original was built also from corporate funds; that the said
Records, 486) house occupied by Guillermo Roxas when it was
Upon motion of the plaintiff respondent corporation, being built had nipa roofing but was later changed
Presiding Judge Francisco Ma. Guerrero of Branch to galvanized iron sheets; that at the beginning, it
34 issued an Order dated April 25, 1986 inhibiting had no partition downstairs and the second floor
himself from further trying the case. The cases were was an open space; that the conversion from a
re-raffled to Branch 37 presided by Judge Odilon recreation hall to a residential house was with the
Bautista. Judge Bautista continued the hearing of knowledge of Eufrocino Roxas and was not
the cases. objected to by any of the Board of Directors of the
For failure of the petitioners (defendants below) and plaintiff; that most of the materials used in
their counsel to attend the October 22, 1986 hearing converting the building into a residential house
despite notice, and upon motion of the respondent came from the materials left by Coppola, a film
corporation, the court issued on the same day, producer, who filmed the movie "Apocalypse Now";
October 22, 1986, an Order considering the cases that Coppola left the materials as part of his
submitted for decision. At this stage of the payment for rents of the rooms that he occupied in
proceedings, the petitioners had not yet presented the resort; that after the said recreation hall was
their evidence while the respondent corporation had converted into a residential house, defendant
completed the presentation of its evidence. Guillermo Roxas moved in and occupied the same
The evidence of the respondent corporation upon together with his family sometime in 1977 or 1978;
which the lower court based its decision is as that during the time Eufrocino Roxas was still alive,
follows: Eriberto Roxas was the general manager of the
To support the complaints, the plaintiff offered the corporation and there was seldom any board
testimonies of Maria Milagros Roxas and that of meeting; that Eufrocino Roxas together with
Victoria Roxas Villarta as well as Exhibits "A" to "M- Eriberto Roxas were (sic) the ones who were
3". running the corporation; that during this time,
The evidence of the plaintiff established the Eriberto Roxas was the restaurant and wine
following: that the plaintiff, Heirs of Eugenia V concessionaire of the resort; that after the death of
Roxas, Incorporated, was incorporated on Eufrocino Roxas, Eriberto Roxas continued as the
December 4, 1962 (Exh. "C") with the primary general manager until his death in 1980; that after
purpose of engaging in agriculture to develop the the death of Eriberto Roxas in 1980, the defendants
properties inherited from Eugenia V. Roxas and that Rebecca B. Roxas and Guillermo Roxas, committed
of y Eufrocino Roxas; that the Articles of acts that impeded the plaintiff's expansion and
Incorporation of the plaintiff, in 1971, was amended normal operation of the resort; that the plaintiff could
to allow it to engage in the resort business (Exh. not even use its own pavilions, kitchen and other
"C-1"); that the incorporators as original members of facilities because of the acts of the defendants
the board of directors of the plaintiff were all which led to the filing of criminal cases in court; that
members of the same family, with Eufrocino Roxas cases were even filed before the Ministry of
having the biggest share; that accordingly, the Tourism, Bureau of Domestic Trade and the Office
plaintiff put up a resort known as Hidden Valley of the President by the parties herein; that the
Springs Resort on a portion of its land located at Bo. defendants violated the resolution and orders of the
Limao, Calauan, Laguna, and covered by TCT No. Ministry of Tourism dated July 28, 1983, August 3,
32639 (Exhs. "A" and "A-l"); that improvements 1983 and November 26, 1984 (Exhs. "G", "H" and
were introduced in the resort by the plaintiff and "H-l") which ordered them or the corporation they
among them were cottages, houses or buildings, represent to desist from and to turn over
swimming pools, tennis court, restaurant and open immediately to the plaintiff the management and
operation of the restaurant and wine outlets of the the lower court issued an Order dated July 15, 1986
said resort (Exh. "G-l"); that the defendants also cancelling the July 21, 1986 hearing and resetting
violated the decision of the Bureau of Domestic the hearing to August 11, 1986. (Original records,
Trade dated October 23, 1983 (Exh. "C"); that on 262-263) Three separate copies of the order were
August 27, 1983, because of the acts of the sent and received by the petitioners and their
defendants, the Board of Directors of the plaintiff counsel. (Original Records, pp. 268, 269, 271)
adopted Resolution No. 83-12 series of 1983 (Exh. A motion to cancel and re-schedule the August 11,
"F") authorizing the ejectment of the defendants 1986 hearing filed by the respondent corporation's
from the premises occupied by them; that on counsel was denied in an Order dated August 8,
September 1, 1983, demand letters were sent to 1986. Again separate copies of the Order were sent
Rebecca Boyer-Roxas and Guillermo Roxas (Exhs. and received by the petitioners and their counsel.
"D" and "D-1") demanding that they vacate the (Original Records, pp. 276-279)
respective premises they occupy; and that the At the hearing held on August 11, 1986, only Atty.
dispute between the plaintiff and the defendants Benito P. Fabie, counsel for the respondent
was brought before the barangay level and the corporation appeared. Neither the petitioners nor
same was not settled (Exhs. "E" and "E-l"). (Original their counsel appeared despite notice of hearing.
Records, pp. 454-456) The lower court then issued an Order on the same
The petitioners appealed the decision to the Court date, to wit:
of Appeals. However, as stated earlier, the ORDER
appellate court affirmed the lower court's decision. When these cases were called for continuation of
The Petitioners' motion for reconsideration was trial, Atty. Benito P. Fabie appeared before this
likewise denied. Court, however, the defendants and their lawyer
Hence, this petition. despite receipt of the Order setting the case for
In a resolution dated February 5, 1992, we gave hearing today failed to appear. On Motion of Atty.
due course to the petition. Fabie, further cross examination of witness Victoria
The petitioners now contend: Vallarta is hereby considered as having been
I Respondent Court erred when it refused to pierce waived.
the veil of corporate fiction over private respondent The plaintiff is hereby given twenty (20) days from
and maintain the petitioners in their possession today within which to submit formal offer of
and/or occupancy of the subject premises evidence and defendants are also given ten (10)
considering that petitioners are owners of aliquot days from receipt of such formal offer of evidence to
part of the properties of private respondent. file their objection thereto.
Besides, private respondent itself discarded the In the meantime, hearing in these cases is set to
mantle of corporate fiction by acts and/or omissions September 29, 1986 at 10:00 o'clock in the
of its board of directors and/or stockholders. morning. (Original Records, p. 286)
II The respondent Court erred in not holding that Copies of the Order were sent and received by the
petitioners were in fact denied due process or their petitioners and their counsel on the following dates
day in court brought about by the gross negligence — Rebecca Boyer-Roxas on August 20, 1986,
of their former counsel. Guillermo Roxas on August 26, 1986, and Atty.
III The respondent Court misapplied the law when it Conrado Manicad on September 19, 1986. (Original
ordered petitioner Rebecca Boyer-Roxas to remove Records, pp. 288-290)
the unfinished building in RTC Case No. 802-84-C, On September 1, 1986, the respondent corporation
when the trial court opined that she spent her own filed its "Formal Offer of Evidence." In an Order
funds for the construction thereof. (CA Rollo, pp. 17- dated September 29, 1986, the lower court issued
18) an Order admitting exhibits "A" to "M-3" submitted
Were the petitioners denied due process of law in by the respondent corporation in its "Formal Offer of
the lower court? Evidence . . . there being no objection . . ." (Original
After the cases were re-raffled to the sala of Records, p. 418) Copies of this Order were sent and
Presiding Judge Odilon Bautista of Branch 37 the received by the petitioners and their counsel on the
following events transpired: following dates: Rebecca Boyer-Roxas on October
On July 3, 1986, the lower court issued an Order 9, 1986; Guillermo Roxas on October 9, 1986 and
setting the hearing of the cases on July 21, 1986. Atty. Conrado Manicad on October 4, 1986 (Original
Petitioner Rebecca V. Roxas received a copy of the Records, pp. 420, 421, 428).
Order on July 15, 1986, while petitioner Guillermo The scheduled hearing on September 29, 1986 did
Roxas received his copy on July 18, 1986. Atty. not push through as the petitioners and their
Conrado Manicad, the petitioners' counsel received counsel were not present prompting Atty. Benito
another copy of the Order on July 11, 1986. Fabie, the respondent corporation's counsel to
(Original Records, p. 260) move that the cases be submitted for decision. The
On motion of the respondent corporation's counsel, lower court denied the motion and set the cases for
hearing on October 22, 1986. However, in its Order same in court but the messenger thought that it was
dated September 29, 1986, the court warned that in the secretary who would file it; it was only later on
the event the petitioners and their counsel failed to when it was discovered that the copy for the Court
appear on the next scheduled hearing, the court has not yet been filed and that such failure to file the
shall consider the cases submitted for decision motion for reconsideration was due to excusable
based on the evidence on record. (Original neglect and/or accident. The motion for
Records, p. 429, 430 and 431) reconsideration contained the following allegations:
Separate copies of this Order were sent and that on the date set for hearing (October 22, 1986),
received by the petitioners and their counsel on the he was on his way to Calamba to attend the hearing
following dates: Rebecca Boyer-Roxas on October but his car suffered transmission breakdown; and
9, 1986, Guillermo Roxas on October 9, 1986; and that despite efforts to repair said transmission, the
Atty. Conrado Manicad on October 1, 1986. car remained inoperative resulting in his absence at
(Original Records, pp. 429-430) the said hearing. (Original Records, pp. 460-469)
Despite notice, the petitioners and their counsel On February 3, 1987, Atty. Manicad filed a motion
again failed to attend the scheduled October 22, for reconsideration of the January 15, 1987
1986 hearing. Atty. Fabie representing the decision. He explained that he had to file the motion
respondent corporation was present. Hence, in its because the receiving clerk refused to admit the
Order dated October 22, 1986, on motion of Atty. motion for reconsideration attached to the ex-parte
Fabie and pursuant to the order dated September manifestation because there was no proof of service
29, 1986, the Court considered the cases submitted to the other party. Included in the motion for
for decision. (Original Records, p. 436) reconsideration was a notice of hearing of the
On November 14, 1986, the respondent motion on February 3, 1987. (Original Records, p.
corporation, filed a "Manifestation", stating that ". . . 476-A)
it is submitting without further argument its On February 4, 1987, the respondent corporation
"Opposition to the Motion for Reconsideration" for through its counsel filed a Manifestation and Motion
the consideration of the Honorable Court in manifesting that they received the copy of the
resolving subject incident." (Original Records, p. motion for reconsideration only today (February 4,
442) 1987), hence they prayed for the postponement of
On December 16, 1986, the lower court issued an the hearing. (Original Records, pp. 478-479)
Order, to wit: On the same day, February 4, 1987, the lower court
ORDER issued an Order setting the hearing on February 13,
Considering that the Court up to this date has not 1987 on the ground that it received the motion for
received any Motion for Reconsideration filed by the reconsideration late. Copies of this Order were sent
defendants in the above-entitled cases, the Court separately to the petitioners and their counsel. The
cannot act on the Opposition to Motion for records show that Atty. Manicad received his copy
Reconsideration filed by the plaintiff and received by on February 11, 1987. As regards the petitioners,
the Court on November 14, 1986. (Original the records reveal that Rebecca Boyer-Roxas did
Records, p. 446) not receive her copy while as regards Guillermo
On January 15, 1987, the lower court rendered the Roxas, somebody signed for him but did not
questioned decision in the two (2) cases. (Original indicate when the copy was received. (Original
Records, pp. 453-459) Records, pp. 481-483)
On January 20, 1987, Atty. Conrado Manicad, the At the scheduled February 13, 1987 hearing, the
petitioners' counsel filed an Ex-Parte Manifestation counsels for the parties were present. However, the
and attached thereto, a motion for reconsideration hearing was reset for March 6, 1987 in order to
of the October 22, 1986 Order submitting the cases allow the respondent corporation to file its
for decision. He prayed that the Order be set aside opposition to the motion for reconsideration. (Order
and the cases be re-opened for reception of dated February 13, 1987, Original Records, p. 486)
evidence for the petitioners. He averred that: 1) Copies of the Order were sent and received by the
within the reglementary period he prepared the petitioners and their counsel on the following dates:
motion for reconsideration and among other Rebecca Boyer-Roxas on February 23, 1987;
documents, the draft was sent to his law office thru Guillermo Roxas on February 23, 1987 and Atty.
his messenger; after signing the final copies, he Manicad on February 19, 1987. (Original Records,
caused the service of a copy to the respondent pp. 487, 489-490)
corporation's counsel with the instruction that the The records are not clear as to whether or not the
copy of the Court be filed; however, there was a scheduled hearing on March 6, 1987 was held.
miscommunication between his secretary and Nevertheless, the records reveal that on March 13,
messenger in that the secretary mailed the copy for 1987, the lower court issued an Order denying the
the respondent corporation's counsel and placed motion for reconsideration.
the rest in an envelope for the messenger to file the The well-settled doctrine is that the client is bound
by the mistakes of his lawyer. (Aguila v. Court of accordance with the facts and circumstances of
First Instance of Batangas, Branch I, 160 SCRA 352 each case. Adherence to the general rule would, in
[1988]; See also Vivero v. Santos, et al., 98 Phil. the instant case, result in the outright deprivation of
500 [1956]; Isaac v. Mendoza, 89 Phil. 279 [1951]; their property through a technicality.
Montes v. Court of First Instance of Tayabas, 48 In its questioned decision dated November 19, 1989
Phil. 640 [1926]; People v. Manzanilla, 43 Phil. 167 the Court of Appeals found, in no uncertain terms,
[1922]; United States v. Dungca, 27 Phil. 274 the negligence of the then counsel for petitioners
[1914]; and United States v. Umali, 15 Phil. 33 when he failed to file the proper motion to dismiss or
[1910]) This rule, however, has its exceptions. Thus, to draw a compromise agreement if it was true that
in several cases, we ruled that the party is not they agreed on a settlement of the case; or in
bound by the actions of his counsel in case the simply filing an answer; and that after having been
gross negligence of the counsel resulted in the furnished a copy of the decision by the court he
client's deprivation of his property without due failed to appeal therefrom or to file a petition for
process of law. In the case of Legarda v. Court of relief from the order declaring petitioners in default.
Appeals (195 SCRA 418 [1991]), we said: In all these instances the appellate court found said
In People's Homesite & Housing Corp. v. Tiongco counsel negligent but his acts were held to bind his
and Escasa (12 SCRA 471 [1964]), this Court ruled client, petitioners herein, nevertheless.
as follows: The Court disagrees and finds that the negligence
Procedural technicality should not be made a bar to of counsel in this case appears to be so gross and
the vindication of a legitimate grievance. When such inexcusable. This was compounded by the fact, that
technicality deserts from being an aid to Justice, the after petitioner gave said counsel another chance to
courts are justified in excepting from its operation a make up for his omissions by asking him to file a
particular case. Where there was something fishy petition for annulment of the judgment in the
and suspicious about the actuations of the former appellate court, again counsel abandoned the case
counsel of petitioners in the case at bar, in that he of petitioner in that after he received a copy of the
did not give any significance at all to the processes adverse judgment of the appellate court, he did not
of the court, which has proven prejudicial to the do anything to save the situation or inform his client
rights of said clients, under a lame and flimsy of the judgment. He allowed the judgment to lapse
explanation that the court's processes just escaped and become final. Such reckless and gross
his attention, it is held that said lawyer deprived his negligence should not be allowed to bind the
clients of their day in court, thus entitling said clients petitioner. Petitioner was thereby effectively
to petition for relief from judgment despite the lapse deprived of her day in court. (at pp. 426-427)
of the reglementary period for filing said period for The herein petitioners, however, are not similarly
filing said petition. situated as the parties mentioned in the abovecited
In Escudero v. Judge Dulay (158 SCRA 69 [1988]), cases. We cannot rule that they, too, were victims of
this Court, in holding that the counsel's blunder in the gross negligence of their counsel.
procedure is an exception to the rule that the client The petitioners are to be blamed for the October 22,
is bound by the mistakes of counsel, made the 1986 order issued by the lower court submitting the
following disquisition: cases for decision. They received notices of the
Petitioners contend, through their new counsel, that scheduled hearings and yet they did not do
the judgment rendered against them by the anything. More specifically, the parties received
respondent court was null and void, because they notice of the Order dated September 29, 1986 with
were therein deprived of their day in court and the warning that if they fail to attend the October 22,
divested of their property without due process of 1986 hearing, the cases would be submitted for
law, through the gross ignorance, mistake and decision based on the evidence on record. Earlier,
negligence of their previous counsel. They at the scheduled hearing on September 29, 1986,
acknowledge that, while as a rule, clients are bound the counsel for the respondent corporation moved
by the mistake of their counsel, the rule should not that the cases be submitted for decision for failure
be applied automatically to their case, as their trial of the petitioners and their counsel to attend despite
counsel's blunder in procedure and gross ignorance notice. The lower court denied the motion and gave
of existing jurisprudence changed their cause of the petitioners and their counsel another chance by
action and violated their substantial rights. rescheduling the October 22, 1986 hearing.
We are impressed with petitioner's contentions. Indeed, the petitioners knew all along that their
xxx xxx xxx counsel was not attending the scheduled hearings.
While this Court is cognizant of the rule that, They did not take steps to change their counsel or
generally, a client will suffer consequences of the make him attend to their cases until it was too late.
negligence, mistake or lack of competence of his On the contrary, they continued to retain the
counsel, in the interest of Justice and equity, services of Atty. Manicad knowing fully well his
exceptions may be made to such rule, in lapses vis-a-vis their cases. They, therefore, cannot
raise the alleged gross negligence of their counsel as Hidden Valley Springs Resort where the
resulting in their denial of due process to warrant questioned properties are located.
the reversal of the lower court's decision. In a These facts, however, do not justify the position
similar case, Aguila v. Court of First Instance of taken by the petitioners.
Batangas, Branch 1 (supra), we ruled: The respondent is a bona fide corporation. As such,
In the instant case, the petitioner should have it has a juridical personality of its own separate from
noticed the succession of errors committed by his the members composing it. (Western Agro Industrial
counsel and taken appropriate steps for his Corporation v. Court of Appeals, 188 SCRA 709
replacement before it was altogether too late. He [1990]; Tan Boon Bee & Co., Inc. v. Jarencio, 163
did not. On the contrary, he continued to retain his SCRA 205 [1988]; Yutivo Sons Hardware Company
counsel through the series of proceedings that all v. Court of Tax Appeals, 1 SCRA 160 [1961]; Emilio
resulted in the rejection of his cause, obviously Cano Enterprises, Inc. v. Court of Industrial
through such counsel's "ineptitude" and, let it be Relations, 13 SCRA 290 [1965]) There is no dispute
added, the clients' forbearance. The petitioner's that title over the questioned land where the Hidden
reverses should have cautioned him that his lawyer Valley Springs Resort is located is registered in the
was mishandling his case and moved him to seek name of the corporation. The records also show that
the help of other counsel, which he did in the end the staff house being occupied by petitioner
but rather tardily. Rebecca Boyer-Roxas and the recreation hall which
Now petitioner wants us to nullify all of the was later on converted into a residential house
antecedent proceedings and recognize his earlier occupied by petitioner Guillermo Roxas are owned
claims to the disputed property on the justification by the respondent corporation. Regarding
that his counsel was grossly inept. Such a reason is properties owned by a corporation, we stated in the
hardly plausible as the petitioner's new counsel case of Stockholders of F. Guanzon and Sons, Inc.
should know. Otherwise, all a defeated party would v. Register of Deeds of Manila, (6 SCRA 373
have to do to salvage his case is claim neglect or [1962]):
mistake on the part of his counsel as a ground for xxx xxx xxx
reversing the adverse judgment. There would be no . . . Properties registered in the name of the
end to litigation if these were allowed as every corporation are owned by it as an entity separate
shortcoming of counsel could be the subject of and distinct from its members. While shares of stock
challenge by his client through another counsel constitute personal property, they do not represent
who, if he is also found wanting, would likewise be property of the corporation. The corporation has
disowned by the same client through another property of its own which consists chiefly of real
counsel, and so on ad infinitum. This would render estate (Nelson v. Owen, 113 Ala., 372, 21 So. 75;
court proceedings indefinite, tentative and subject to Morrow v. Gould, 145 Iowa 1, 123 N.W. 743). A
reopening at any time by the mere subterfuge of share of stock only typifies an aliquot part of the
replacing counsel. (at pp. 357-358) corporation's property, or the right to share in its
We now discuss the merits of the cases. proceeds to that extent when distributed according
In the first assignment of error, the petitioners to law and equity (Hall & Faley v. Alabama
maintain that their possession of the questioned Terminal, 173 Ala., 398, 56 So. 235), but its holder
properties must be respected in view of their is not the owner of any part of the capital of the
ownership of an aliquot portion of all the properties corporation (Bradley v. Bauder, 36 Ohio St., 28).
of the respondent corporation being stockholders Nor is he entitled to the possession of any definite
thereof. They propose that the veil of corporate portion of its property or assets (Gottfried V. Miller,
fiction be pierced, considering the circumstances 104 U.S., 521; Jones v. Davis, 35 Ohio St., 474).
under which the respondent corporation was The stockholder is not a co-owner or tenant in
formed. common of the corporate property (Harton v.
Originally, the questioned properties belonged to Johnston, 166 Ala., 317, 51 So. 992). (at pp. 375-
Eugenia V. Roxas. After her death, the heirs of 376)
Eugenia V. Roxas, among them the petitioners The petitioners point out that their occupancy of the
herein, decided to form a corporation — Heirs of staff house which was later used as the residence
Eugenia V. Roxas, Incorporated (private respondent of Eriberto Roxas, husband of petitioner Rebecca
herein) with the inherited properties as capital of the Boyer-Roxas and the recreation hall which was
corporation. The corporation was incorporated on converted into a residential house were with the
December 4, 1962 with the primary purpose of blessings of Eufrocino Roxas, the deceased
engaging in agriculture to develop the inherited husband of Eugenia V. Roxas, who was the majority
properties. The Articles of Incorporation of the and controlling stockholder of the corporation. In his
respondent corporation were amended in 1971 to lifetime, Eufrocino Roxas together with Eriberto
allow it to engage in the resort business. Roxas, the husband of petitioner Rebecca Boyer-
Accordingly, the corporation put up a resort known Roxas, and the father of petitioner Guillermo Roxas
managed the corporation. The Board of Directors corporation in deference to the wishes of Eufrocino
did not object to such an arrangement. The Roxas, who during his lifetime, controlled and
petitioners argue that . . . the authority thus given by managed the corporation. Eufrocino Roxas' actions
Eufrocino Roxas for the conversion of the recreation could not have bound the corporation forever. The
hall into a residential house can no longer be petitioners have not cited any provision of the
questioned by the stockholders of the private corporation by-laws or any resolution or act of the
respondent and/or its board of directors for they Board of Directors which authorized Eufrocino
impliedly but no leas explicitly delegated such Roxas to allow them to stay within the company
authority to said Eufrocino Roxas. (Rollo, p. 12) premises forever. We rule that in the absence of
Again, we must emphasize that the respondent any existing contract between the petitioners and
corporation has a distinct personality separate from the respondent corporation, the corporation may
its members. The corporation transacts its business elect to eject the petitioners at any time it wishes for
only through its officers or agents. (Western Agro the benefit and interest of the respondent
Industrial Corporation v. Court of Appeals, supra). corporation.
Whatever authority these officers or agents may The petitioners' suggestion that the veil of the
have is derived from the board of directors or other corporate fiction should be pierced is untenable.
governing body unless conferred by the charter of The separate personality of the corporation may be
the corporation. An officer's power as an agent of disregarded only when the corporation is used "as a
the corporation must be sought from the statute, cloak or cover for fraud or illegality, or to work
charter, the by-laws or in a delegation of authority to injustice, or where necessary to achieve equity or
such officer, from the acts of the board of directors, when necessary for the protection of the creditors."
formally expressed or implied from a habit or (Sulong Bayan, Inc. v. Araneta, Inc., 72 SCRA 347
custom of doing business. (Vicente v. Geraldez, 52 [1976] cited in Tan Boon Bee & Co., Inc., v.
SCRA 210 [1973]) Jarencio, supra and Western Agro Industrial
In the present case, the record shows that Eufrocino Corporation v. Court of Appeals, supra) The
V. Roxas who then controlled the management of circumstances in the present cases do not fall under
the corporation, being the majority stockholder, any of the enumerated categories.
consented to the petitioners' stay within the In the third assignment of error, the petitioners insist
questioned properties. Specifically, Eufrocino Roxas that as regards the unfinished building, Rebecca
gave his consent to the conversion of the recreation Boyer-Roxas is a builder in good faith.
hall to a residential house, now occupied by The construction of the unfinished building started
petitioner Guillermo Roxas. The Board of Directors when Eriberto Roxas, husband of Rebecca Boyer-
did not object to the actions of Eufrocino Roxas. Roxas, was still alive and was the general manager
The petitioners were allowed to stay within the of the respondent corporation. The couple used
questioned properties until August 27, 1983, when their own funds to finance the construction of the
the Board of Directors approved a Resolution building. The Board of Directors of the corporation,
ejecting the petitioners, to wit: however, did not object to the construction. They
R E S O L U T I O N No. 83-12 allowed the construction to continue despite the fact
RESOLVED, That Rebecca B. Roxas and Guillermo that it was within the property of the corporation.
Roxas, and all persons claiming under them, be Under these circumstances, we agree with the
ejected from their occupancy of the Hidden Valley petitioners that the provision of Article 453 of the
Springs compound on which their houses have Civil Code should have been applied by the lower
been constructed and/or are being constructed only courts.
on tolerance of the Corporation and without any Article 453 of the Civil Code provides:
contract therefor, in order to give way to the If there was bad faith, not only on the part of the
Corporation's expansion and improvement program person who built, planted or sown on the land of
and obviate prejudice to the operation of the Hidden another but also on the part of the owner of such
Valley Springs Resort by their continued land, the rights of one and the other shall be the
interference. same as though both had acted in good faith.
RESOLVED, Further that the services of Atty. In such a case, the provisions of Article 448 of the
Benito P. Fabie be engaged and that he be Civil Code govern the relationship between
authorized as he is hereby authorized to effect the petitioner Rebecca-Boyer-Roxas and the
ejectment, including the filing of the corresponding respondent corporation, to wit:
suits, if necessary to do so. (Original Records, p. Art. 448 — The owner of the land on which anything
327) has been built, sown or planted in good faith, shall
We find nothing irregular in the adoption of the have the right to appropriate as his own the works,
Resolution by the Board of Directors. The sowing or planting after payment of the indemnity
petitioners' stay within the questioned properties provided for in articles 546 and 548, or to oblige the
was merely by tolerance of the respondent one who built or planted to pay the price of the land,
and the one who sowed, the proper rent. However, Punsalan's request, Saño sent PAWCI another
the builder or planter cannot be obliged to buy the letter-proposal ("Second Contract") formalizing its
land if its value is considerably more than that of the proposal for consultancy services in the amount of
building or trees. In such case, he shall pay P400,000. On 10 January 1987, Andy Villaceren,
reasonable rent, if the owner of the land does not vice president of PAWCI, received the operations
choose to appropriate the buildings or trees after manual prepared by Saño. PAWCI submitted said
proper indemnity. The parties shall agree upon the operations manual to the Bureau of Customs in
terms of the lease and in case of disagreement, the connection with the former's application to operate a
court shall fix the terms thereof. bonded warehouse; thereafter, in May 1987, the
WHEREFORE, the present petition is partly Bureau issued to it a license to operate, enabling it
GRANTED. The questioned decision of the Court of to become one of the three public customs bonded
Appeals affirming the decision of the Regional Trial warehouses at the international airport. Saño also
Court of Laguna, Branch 37, in RTC Civil Case No. conducted, in the third week of January 1987 in the
802-84-C is MODIFIED in that subparagraphs (c) warehouse of PAWCI, a three-day training seminar
and (d) of Paragraph 1 of the dispositive portion of for the latter's employees. On 25 March 1987, Saño
the decision are deleted. In their stead, the joined the Bureau of Customs as special assistant
petitioner Rebecca Boyer-Roxas and the to then Commissioner Alex Padilla, a position he
respondent corporation are ordered to follow the held until he became technical assistant to then
provisions of Article 448 of the Civil Code as Commissioner Miriam Defensor-Santiago on 7
regards the questioned unfinished building in RTC March 1988. Meanwhile, Punsalan sold his shares
Civil Case No. 802-84-C. The questioned decision in PAWCI and resigned as its president in 1987. On
is affirmed in all other respects. 9 February 1988, Saño filed a collection suit against
SO ORDERED. PAWCI. He alleged that he had prepared an
operations manual for PAWCI, conducted a
seminar-workshop for its employees and delivered
People's Aircargo and Warehousing Co. Inc. vs. to it a computer program; but that, despite demand,
Court of Appeals PAWCI refused to pay him for his services. PAWCI,
[GR 117847, 7 October 1998] in its answer, denied that Saño had prepared an
First Division, Panganiban (J): 4 concur operations manual and a computer program or
conducted a seminar-workshop for its employees. It
further alleged that the letter-agreement was signed
Facts: People's Aircargo and Warehousing Co. Inc.
by Punsalan without authority, in collusion with
(PAWCI) is a domestic corporation, which was
Saño in order to unlawfully get some money from
organized in the middle of 1986 to operate a
customs bonded warehouse at the old Manila PAWCI, and despite his knowledge that a group of
International Airport in Pasay City. To obtain a employees of the company had been commissioned
by the board of directors to prepare an operations
license for the corporation from the Bureau of
manual. The Regional Trial Court (RTC) of Pasay
Customs, Antonio Punsalan Jr., the corporation
City, Branch 110, rendered a Decision dated 26
president, solicited a proposal from Stefani Saño for
October 1990 declared the Second Contract
the preparation of a feasibility study. Saño
submitted a letter-proposal dated 17 October 1986 unenforceable or simulated. However, since Saño
("First Contract") to Punsalan, for the project had actually prepared the operations manual and
conducted a training seminar for PAWCI and its
feasibility study (market, technical, and financial
employees, the trial court awarded P60,000 to the
feasibility) and preparation of pertinent
former, on the ground that no one should be
documentation requirements for the application,
worth P350,000. Initially, Cheng Yong, the majority unjustly enriched at the expense of another (Article
stockholder of PAWCI, objected to Saño's offer, as 2142, Civil Code). The trial Court determined the
amount "in light of the evidence presented by
another company priced a similar proposal at only
defendant on the usual charges made by a leading
P15,000. However, Punsalan preferred Saño's
consultancy firm on similar services." Upon appeal,
services because of the latter's membership in the
and on 28 February 1994, the appellate court
task force, which was supervising the transition of
the Bureau of Customs from the Marcos modified the decision of the trial court, and declared
government to the Aquino Administration. On 17 the Second Contract valid and binding on PAWCI,
which was held liable to Saño in the full amount of
October 1986, PAWCI, through Punsalan, sent
P400,000, representing payment of Saño services
Saño a letter confirming their agreement.
in preparing the manual of operations and in the
Accordingly, Saño prepared a feasibility study for
conduct of a seminar for PAWCI. As no new ground
PAWCI which eventually paid him the balance of
the contract price, although not according to the was raised by PAWCI, reconsideration of the
schedule agreed upon. On 4 December 1986, upon decision was denied in the Resolution promulgated
on 28 October 1994. PAWCI filed the Petition for Article 1403(2) is ratified "by the acceptance of
Review. benefits under them" under Article 1405.

Issue: Whether a single instance where the G.R. No. L-20451 December 28, 1964
corporation had previously allowed its president to
enter into a contract with another without a board R. F. SUGAY and CO., INC., petitioner,
resolution expressly authorizing him, has clothed its vs.
president with apparent authority to execute the PABLO C. REYES, CESAR CURATA, PACIFIC
subject contract. PRODUCTS, INC., and WORKMEN'S
COMPENSATION COMMISSION, respondents.
Held: Apparent authority is derived not merely from
practice. Its existence may be ascertained through Facts:
(1) the general manner in which the corporation
holds out an officer or agent as having the power to
Respondents Pablo Reyes and Cesar Curata
act or, in other words, the apparent authority to act suffered burns of various degrees, while painting
in general, with which it clothes him; or (2) the the building of the Pacific Products, Inc., caused by
acquiescence in his acts of a particular nature, with
a fire of accidental origin, resulting in their
actual or constructive knowledge thereof, whether
temporary disability from work.
within or beyond the scope of his ordinary powers. It
requires presentation of evidence of similar act(s)
executed either in its favor or in favor of other For said injuries they filed claims for disability and
parties. It is not the quantity of similar acts which medical expenses against the R. F. Sugay & Co.,
establishes apparent authority, but the vesting of a Inc., Romulo F. Sugay and the Pacific Products, Inc.
corporate officer with the power to bind the
corporation. Herein, PAWCI, through its president The R. F. Sugay & Co., Inc., answered the claim,
Antonio Punsalan Jr., entered into the First Contract alleging that the corporation was not the employer
without first securing board approval. Despite such of the claimants but it was the Pacific Products, Inc.,
lack of board approval, PAWCI did not object to or which had an administration and supervision job
repudiate said contract, thus "clothing" its president contract with Romulo F. Sugay, who, aside from
with the power to bind the corporation. The grant of being the President of the corporation, bearing his
apparent authority to Punsalan is evident in the name, had also a business of his own, distinct and
testimony of Yong — senior vice president, separate from said corporation; and that the
treasurer and major stockholder of PAWCI. The Regional Office of the Department of Labor had no
First Contract was consummated, implemented and jurisdiction over the subject matter.
paid without a hitch. Hence, Sano should not be
faulted for believing that Punsalan's conformity to Romulo F. Sugay did not file an Answer, but
the contract in dispute was also binding on voluntarily appeared during the hearing and
petitioner. It is familiar doctrine that if a corporation disclaimed liability.
knowingly permits one of its officers, or any other
agent, to act within the scope of an apparent The Answer of Pacific Products, Inc., contained the
authority, it holds him out to the public as customary admissions and denials, and averred that
possessing the power to do those acts; and thus, its business was mainly in the manufacture and sale
the corporation will, as against anyone who has in of lacquer and other painting materials. As
good faith dealt with it through such agent, be defenses, it stated that the claimants were the
estopped from denying the agent's authority. employees of respondents R. F. Sugay
Furthermore, Saño prepared an operations manual Construction Co., Inc., and/or Romulo F. Sugay that
and conducted a seminar for the employees of as a result of the, fire, it incurred a loss of
PAWCI in accordance with their contract. PAWCI P2,000,000.00, occasioned by the employment of
accepted the operations manual, submitted it to the incompetent men in the painting of its factory by the
Bureau of Customs and allowed the seminar for its Sugays.
employees. As a result of its aforementioned
actions, PAWCI was given by the Bureau of
The Hearing Officer dismissed the case with
Customs a license to operate a bonded warehouse.
respect, to R. F. Sugay & Co., Inc., and Romulo F.
Granting arguendo then that the Second Contract
Sugay "for want of employer-employee relationship
was outside the usual powers of the president,
with the claimants, either directly or through an
PAWCI's ratification of said contract and
independent contractor and adjudged Pacific
acceptance of benefits have made it binding,
Products, Inc., is to pay claimant for the benefits.
nonetheless. The enforceability of contracts under
Pacific Products, Inc., appealed the above decision
to the Commission. Commissioner Jose Sanchez
rendered judgment affirming the compensability of MANUEL A. TORRES, JR., (Deceased),
the injuries and the amounts due them, but modified GRACIANO J. TOBIAS, RODOLFO L. JOCSON,
the decision of the Hearing Officer, by finding that JR., MELVIN S. JURISPRUDENCIA, AUGUSTUS
R. F. Sugay & Co., Inc., was the statutory employer CESAR AZURA and EDGARDO D. PABALAN
of the claimants and should be liable to them. vs.
Pacific Products, Inc., was absolved from all COURT OF APPEALS, SECURITIES AND
responsibility. EXCHANGE COMMISSION, TORMIL REALTY &
DEVELOPMENT CORPORATION, ANTONIO P.
TORRES, JR., MA. CRISTINA T. CARLOS, MA.
LUISA T. MORALES and DANTE D. MORALES.
The Commission en banc denied the motion for G.R. No. 120138 September 5,
reconsideration stating that there was "nothing to 1997
warrant a modification much less a reversal, of the
decision sought to be reviewed." FACTS:

The late Manuel A. Torres, Jr. was the


In the appeal of R. F. Sugay & Co., to this Court, it major stockholder of Tormil Realty & Development
is insisted that Pacific Products, Inc. was the Corporation while private respondents who are the
employer of the claimants. children of Judge Torres' deceased brother Antonio
A. Torres, constituted the minority stockholders. In
Issue: particular, their respective shareholdings and
positions in the corporation.
W/N Pacific Products is the employer of the In 1984, Judge Torres, in order to make
claimants. substantial savings in taxes, adopted an "estate
planning" scheme under which he assigned to
Ruling: No. Tormil Realty & Development Corporation (Tormil
for brevity) various real properties he owned and his
shares of stock in other corporations in exchange
R. F. Sugay & Co., is the statutory employer of the
for 225,972 Tormil Realty shares. Hence, on various
claimants. The decisive elements showing that it is
dates in July and August of 1984, ten (10) deeds of
the employer, are present, such as selection and
assignment were executed by the late Judge
engagement; payment of wages; power of
Torres.Consequently, the aforelisted properties
dismissal, and control. These powers were lodged
were duly recorded in the inventory of assets of
in R. F. Sugay & Co. On this very score alone, the
Tormil Realty and the revenues generated by the
petition for review should be dismissed.
said properties were correspondingly entered in the
corporation's books of account and financial
There was a faint attempt by the petitioning records.
corporation, to evade liability, by advancing the Due to the insufficient number of shares of
theory that Romulo P. Sugay, its President, was the stock issued to Judge Torres and the alleged
one who entered into a contract of administration refusal of private respondents to approve the
and supervision for the painting of the factory of the needed increase in the corporation's authorized
Pacific Products, Inc., and making it appear that capital stock (to cover the shortage of 972 shares
said Romulo F. Sugay acted as an agent of the due to Judge Torres under the "estate planning"
Pacific Products, Inc., and as such, the latter should scheme), on 11 September 1986, Judge Torres
be made answerable to the compensation due to revoked the two (2) deeds of assignment covering
the claimants. the properties in Makati and Pasay City.

We, however, agree with the Commission that "the ISSUE:


dual roles of Romulo F. Sugay should not be
allowed to confuse the facts relating to employer- Whether or not the deed of assignment
employee relationship." It is a legal truism that when executed can be revoked.
the veil of corporate fiction is made as a shield to
perpetrate a fraud and/or confuse legitimate issues RULING:
(here, the relation of employer-employee), the same
should be pierced. Verily the R. F. Sugay & Co., Inc. NO.
is a business conduit of R. F. Sugay.
The shortage of 972 shares would not be file a motion seeking the nullification of the sale on
valid ground for respondent Torres to unilaterally the ground that VECCI is not the lawful and
revoke the deeds of assignment he had executed absolute owner thereof and that she has not been
on July 13, 1984 and July 24, 1984 wherein he notified nor consulted as to the terms and conditions
voluntarily assigned to TORMIL real properties of the sale. The trial court ruled that the sale to
covered by TCT No. 374079 (Makati) and TCT No. Sureste was valid.
41527, 41528 and 41529 (Pasay) respectively. A
comparison of the number of shares that ISSUES:
respondent Torres received from TORMIL by virtue
of the "deeds of assignment" and the stock Whether or not the sale of Esguerra
certificates issued by the latter to the former readily Building II is a valid exercise of corporate power.
shows that TORMIL had substantially performed
what was expected of it. In fact, the first two RULING:
issuances were in satisfaction to the properties
being revoked by respondent Torres. Hence, the YES.
shortage of 972 shares would never be a valid
ground for the revocation of the deeds covering VECCI's sale of all the properties
Pasay and Quezon City properties. mentioned in the judicially-approved compromise
Moreover, we agree with the contention of agreement was done on the basis of its Corporate
the Solicitor General that the shortage of shares Secretary's Certification of these two resolutions.
should not have affected the assignment of the The partial decision did not require any further
Makati and Pasay City properties which were board or stockholder resolutions to make VECCI's
executed in 13 and 24 July 1984 and the sale of these properties valid. Being regular on its
consideration for which have been duly paid or face, the Secretary's Certification was sufficient for
fulfilled but should have been applied logically to the private respondent Sureste Properties, Inc. to rely
last assignment of property — Judge Torres' Ayala on. It did not have to investigate the truth of the
Fund shares — which was executed on 29 August facts contained in such certification. Otherwise,
1984. business transactions of corporations would
become tortuously slow and unnecessarily
hampered. Ineluctably, VECCI's sale of Esguerra
Building II to private respondent was not ultra vires
JULIETA V. ESGUERRA but a valid execution of the trial court's partial
vs. decision.
COURT OF APPEALS and SURESTE Based on the foregoing, the sale is also
PROPERTIES, INC. deemed to have satisfied the requirements of
G.R. No. 119310. February 3, 1997 Section 40 of the Corporation Code.

FACTS:

Julieta Esguerra filed a complaint for JULIETA V. ESGUERRA, petitioner,


administration of conjugal partnership or separation vs.
of property against her husband Vicente Esguerra, COURT OF APPEALS and SURESTE
Jr. and V. Esguerra Construction Co., Inc. (VECCI) PROPERTIES, INC., respondents
and other family corporations as defendants before
the trial court. Julieta Esguerra filed a complaint for administration
The parties entered into a compromise of conjugal partnership or separation of property
agreement. By virtue of said agreement, Esguerra
against her husband Vicente Esguerra, Jr. The said
Bldg. I was sold and the net proceeds distributed
complaint was later amended on 31 October 1985
according to the agreement. The controversy arose
impleading V. Esguerra Construction Co., Inc.
with respect to Esguerra Building II. Herein
(VECCI for brevity) and other family corporations as
petitioner started claiming one-half of the rentals of defendants.
the said building which VECCI refused. Thus,
petitioner filed a motion with respondent court
praying that VECCI be ordered to remit one-half of The parties entered into a compromise agreement
the rentals to her. The trial court ruled in favor of which was submitted to the court.
petitioner.
Meanwhile, Esguerra Bldg. II was sold to On the basis of the said agreement, the court
private respondent Sureste Properties. Inc. for rendered two partial judgments: one between
P150,000,000.00 prompting Julieta V. Esguerra to Vicente and Julieta and the other as between the
latter and VECCI. The compromise agreement investigate further the truth and veracity
between Julieta and VECCI provides that VECCI thereof?
shall sell/alienate/transfer or dispose of some of its
properties under the terms and conditions recited in RULING:
the enabling resolutions of its Board of Directors.
The Civil Code provides that a contract is
By virtue of said agreement, Esguerra Bldg. I was unenforceable when it is ". . . entered into in the
sold and the net proceeds distributed according to name of another person by one who has been given
the agreement. no authority or legal representation, or who has
acted beyond his powers." And that "(a) contract
The controversy arose with respect to Esguerra entered into in the name of another by one who has
Building II. Julieta started claiming one-half of the no authority or legal representation, or who has
rentals of the said building which VECCI refused. acted beyond his powers, shall be unenforceable, . .
Thus, Julieta filed a motion with respondent court ." After a thorough review of the case at bench, the
praying that VECCI be ordered to remit one-half of Court finds the sale of Esguerra Building II by
the rentals to until the same be sold. VECCI VECCI to private respondent Sureste Properties,
opposed said motion. Inc. valid. The sale was expressly and clearly
authorized under the judicially-approved
Trial court ruled in favor of herein petitioner compromise agreement freely consented to and
voluntarily signed by petitioner Julieta Esguerra.
Meanwhile, Esguerra Bldg. II was sold to herein Thus, petitioner's contention that the sale is
private respondent Sureste Properties. Inc. for unenforceable as to her share for being
unauthorized is plainly incongruous with the express
P150,000,000.00. Julieta filed a motion seeking the
authority granted by the compromise agreement to
nullification of the sale on the ground that VECCI is
VECCI, which specified no condition that the latter
not the lawful and absolute owner thereof and that
she has not been notified nor consulted as to the shall first consult with the former prior to selling any
terms and conditions of the sale. of the properties listed there. As astutely and
correctly found by the appellate Court:
Sureste filed a Manifestation pointing out that in the
The compromise agreement entered between
compromise agreement, Julieta gave her express
consent to the sale of the said building. private respondent (Julieta Esguerra) and
VECCI, which was approved by the court,
expressly provides, among others, that the latter
RTC rendered its order ruling that the sale of shall sell or otherwise dispose of certain
Esguerra Bldg. II to Sureste Properties, Inc. is properties, among them, Esguerra Bldgs. I and
declared valid with respect to one-half of the value II, and fifty (50%) percent of the net proceeds
thereof but ineffectual and unenforceable with thereof to be given to the former. Pursuant to
respect to the other half as the acknowledged said agreement, VECCI sold the buildings. . . .
owner of said portion was not consulted as to the
terms and conditions of the sale.
. . . The compromise agreement expressly
authorizes VECCI to sell the subject properties,
Petitioner filed this petition alleging that the trial with the only condition that the sale be in a lawful
court acted without jurisdiction and/or abused its and convenient manner and under the terms and
discretion when it held that the questioned sale of conditions recited in the enabling resolutions of
the property ineffectual and unenforceable as to its Board of Directors and stockholders. There is
herein petitioner's one-half (1/2) ownership/interest nothing in the said agreement requiring VECCI
in the property since the sale was made without her to consult the private respondent (Julieta
knowledge and consent. BECAUSE No proper Esguerra) before any sale (can be concluded).
corporate action of VECCI was made to effect such Thus, when VECCI sold the property to (Sureste
sale as required under the compromise agreement; Properties, Inc.) as agreed upon, it need not
consult the private respondent
ISSUE:
VECCI'S Sale of Esguerra
Is the corporate secretary's certification of the Building II A Valid Exercise of Corporate
shareholders' and directors resolution Power
authorizing such sale sufficient, or does the
buyer need to go behind such certification and
Petitioner contends that VECCI violated the Furthermore, petitioner Julieta Esguerra is estopped
condition in the compromise agreement requiring from contesting the validity of VECCI's corporate
that the sale be made "under the terms and action in selling Esguerra Building II on the basis of
conditions recited in the enabling resolutions of its said resolutions and certification because she never
Board of Directors and stockholders. She rues that raised this issue in VECCI's prior sales of the other
no shareholders' or directors' meeting, wherein properties sold including the Esguerra Building I.
these resolutions were passed, was actually held. The same identical resolutions and certification
She thus bewails this sale as improper for not were used in such prior sales.
having complied with the requirements mandated by
Section 40 of the Corporation Code.

Petitioner's contention is plainly unmeritorious. The SALOME PABON and VICENTE CAMONAYAN
trial court's partial decision dated January 11, 1990 vs.
approving the compromise agreement clearly NATIONAL LABOR RELATIONS COMMISSION
showed that the "enabling resolutions of its and SENIOR MARKETING CORPORATION
(VECCI's) board of directors and stockholders" G.R. No. 120457 September 24, 1998
referred to were those then already existing; to wit:
(1) "the resolution of the stockholders of VECCI FACTS:
dated November 9, 1989, (where) the stockholders
authorized VECCI to sell and/or disposed all or On May 24, 1994 and June 22, 1994,
substantially all its property and assets upon such complaints for illegal dismissal and non-payment of
terms and conditions and for such consideration as benefits were filed by petitioners Salome Pabon and
the board of directors may deem expedient." 24 (2) Vicente Camonayan against private respondent
the "resolution dated 9 November 1989, (where) the Senior Marketing Corporation (SMC) and its Field
board of directors of VECCI authorized VECCI to Manager, R-Jay Roxas Summons and notices of
sell and/or dispose all or substantially all the hearings were sent to Roxas at private respondent's
property and assets of the corporation, at the provincial office in 13 Valley Homes, Patul Road,
highest available price/s they could be sold or Santiago, Isabela which were received by its
disposed of in cash, and in such manner as may be bookkeeper, Mina Villanueva.
held convenient under the circumstances, and On September 15, 1994, the Labor Arbiter
authorized the President Vicente B. Esguerra. Jr. to rendered a judgment by default after finding that
negotiate. contract, execute and sign such sale for private respondent tried to evade all the summons
and in behalf of the corporation." and orders of hearing by refusing to claim all the
registered mail addressed to it.
VECCI's sale of all the properties mentioned in
the judicially-approved compromise agreement ISSUE:
was done on the basis of its Corporate
Secretary's Certification of these two Whether or not Petitioners herein are
resolutions. The partial decision did not require authorized to receive summons in behalf of the
any further board or stockholder resolutions to corporation.
make VECCI's sale of these properties valid.
Being regular on its face, the Secretary's RULING:
Certification was sufficient for private
respondent Sureste Properties, Inc. to rely on. It YES.
did not have to investigate the truth of the facts
contained in such certification. Otherwise, Bookkeeper can be considered as an agent
business transactions of corporations would of private respondent corporation within the purview
become tortuously slow and unnecessarily of Section 13, Rule 14 of the old Rules of Court.
hampered. The rationale of all rules with respect to service of
process on a corporation is that such service must
Ineluctably, VECCI's sale of Esguerra Building II to be made to an agent or a representative so
private respondent was not ultra vires but a valid integrated with the corporation sued as to make it a
execution of the trial court's partial decision. Based priori supposable that he will realize his
on the foregoing, the sale is also deemed to have responsibilities and know what he should do with
satisfied the requirements of Section 40 of the any legal papers served on him. The bookkeeper's
Corporation Code. task is one under consideration. The job of a
bookkeeper is so integrated with the corporation
that his regular recording of the corporation's
"business accounts" and "essential facts about the approval of the sale; and the balance of P50,000.00
transactions of a business or enterprise" safeguards "shall be paid by the BUYER to the different
the corporation from possible fraud being committed suppliers of the SELLER." The very same day that
adverse to its own corporate interest. the contract of sale was executed, the parties
Although it may be true that the service of thereto immediately applied with the PSC for its
summons was made on a person not authorized to approval, with a prayer for the issuance of a
receive the same in behalf of the petitioner, provisional authority in favor of the vendee
nevertheless since it appears that the summons and Corporation to operate the service therein involved.
complaint were in fact received by the corporation On 19 May 1959, the PSC granted the provisional
through its said clerk, the Court finds that there was permit prayed for, upon the condition that "it may be
substantial compliance with the rule on service of modified or revoked by the Commission at any time,
summons. Indeed the purpose of said rule as above shall be subject to whatever action that may be
stated to assure service of summons on the taken on the basic application and shall be valid
corporation had thereby been attained. The need for only during the pendency of said application."
speedy justice must prevail over technicality. Before the PSC could take final action on said
application for approval of sale, however, the Sheriff
of Manila, on 7 July 1959, levied on 2 of the five
Villa Rey Transit vs. Ferrer certificates of public convenience involved therein,
[GR L-23893, 29 October 1968] namely, those issued under PSC cases 59494 and
En Banc, Angeles (J): 6 concur, 2 took no part, 1 on 63780, pursuant to a writ of execution issued by the
leave Court of First Instance of Pangasinan in Civil Case
13798, in favor of Eusebio E. Ferrer against
Facts: [preceding case] Prior to 1959, Jose M. Valentin Fernando. The Sheriff made and entered
the levy in the records of the PSC. On 16 July 1959,
Villarama was an operator of a bus transportation,
a public sale was conducted by the Sheriff of the
under the business name of Villa Rey Transit,
said two certificates of public convenience. Ferrer
pursuant to certificates of public convenience
was the highest bidder, and a certificate of sale was
granted him by the Public Service Commission
(PSC) in Cases 44213 and 104651, which issued in his name. Thereafter, Ferrer sold the two
authorized him to operate a total of 32 units on certificates of public convenience to Pantranco, and
jointly submitted for approval their corresponding
various routes or lines from Pangasinan to Manila,
contract of sale to the PSC. Pantranco therein
and vice-versa. On 8 January 1959, he sold the two
prayed that it be authorized provisionally to operate
certificates of public convenience to the Pangasinan
the service involved in the said two certificates. The
Transportation Company, Inc. (Pantranco), for
P350,000.00 with the condition, among others, that applications for approval of sale, filed before the
the seller (Villarama) "shall not for a period of 10 PSC, by Fernando and the Corporation, Case
124057, and that of Ferrer and Pantranco, Case
years from the date of this sale, apply for any TPU
126278, were scheduled for a joint hearing. In the
service identical or competing with the buyer."
meantime, to wit, on 22 July 1959, the PSC issued
Barely 3 months thereafter, or on 6 March 1959: a
an order disposing that during the pendency of the
corporation called Villa Rey Transit, Inc. (the
Corporation) was organized with a capital stock of cases and before a final resolution on the aforesaid
P500,000.00 divided into 5,000 shares of the par applications, the Pantranco shall be the one to
operate provisionally the service under the two
value of P100.00 each; P200,000.00 was the
certificates embraced in the contract between Ferrer
subscribed stock; Natividad R. Villarama (wife of
and Pantranco. The Corporation took issue with this
Jose M. Villarama) was one of the incorporators,
and she subscribed for P1,000.00; the balance of particular ruling of the PSC and elevated the matter
P199,000.00 was subscribed by the brother and to the Supreme Court, which decreed, after
deliberation, that until the issue on the ownership of
sister-in-law of Jose M. Villarama; of the subscribed
the disputed certificates shall have been finally
capital stock, P105,000.00 was paid to the treasurer
settled by the proper court, the Corporation should
of the corporation, who was Natividad R. Villarama.
be the one to operate the lines provisionally.
In less than a month after its registration with the
Securities and Exchange Commission (10 March
1959), the Corporation, on 7 April 1959, bought 5 [present case] On 4 November 1959, the
certificates of public convenience, 49 buses, tools Corporation filed in the Court of First Instance of
and equipment from one Valentin Fernando, for the Manila, a complaint for the annulment of the
sum of P249,000.00, of which P100,000.00 was sheriff's sale of the aforesaid two certificates of
paid upon the signing of the contract; P50,000.00 public convenience (PSC Cases 59494 and 63780)
was payable upon the final approval of the sale by in favor of Ferrer, and the subsequent sale thereof
the PSC; P49,500.00 one year after the final by the latter to Pantranco, against Ferrer, Pantranco
and the PSC. The Corporation prayed therein that merely as an aggregation of individuals. Hence, the
all the orders of the PSC relative to the parties' Villa Rey Transit, Inc. is an alter ego of Jose M.
dispute over the said certificates be annulled. The Villarama, and that the restrictive clause in the
CFI of Manila declared the sheriff's sale of two contract entered into by the latter and Pantranco is
certificates of public convenience in favor of Ferrer also enforceable and binding against the said
and the subsequent sale thereof by the latter to Corporation. For the rule is that a seller or promisor
Pantranco null and void; declared the Corporation to may not make use of a corporate entity as a means
be the lawful owner of the said certificates of public of evading the obligation of his covenant. Where the
convenience; and ordered Ferrer and Pantranco, Corporation is substantially the alter ego of the
jointly and severally, to pay the Corporation, the covenantor to the restrictive agreement, it can be
sum of P5,000.00 as and for attorney's fees. The enjoined from competing with the covenantee.
case against the PSC was dismissed. All parties
appealed.

Issue: Whether the stipulation, "SHALL NOT FOR Delta Motor Sales vs. Mangosing
A PERIOD OF 10 YEARS FROM THE DATE OF
THIS SALE, APPLY FOR ANY TPU SERVICE
IDENTICAL OR COMPETING WITH THE BUYER"
in the contract between Villarama and Pantranco, Facts:
binds the Corporation (the Villa Rey Transit, Inc.).
Delta Motors is sued by their customer Pamintuan
Held: Villarama supplied the organization expenses because of the defective car sold by delta. Summon
and the assets of the Corporation, such as trucks was serve not to the proper person who is
and equipment; there was no actual payment by the
authorized to receive summon. And because of that
original subscribers of the amounts of P95,000.00
and P100,000.00 as appearing in the books; delta motors was in default because of failure to
Villarama made use of the money of the answer the complaint. They filed a motion but the
Corporation and deposited them to his private same failed.
accounts; and the Corporation paid his personal
accounts. Villarama himself admitted that he
mingled the corporate funds with his own money.
These circumstances are strong persuasive Issue:
evidence showing that Villarama has been too much
involved in the affairs of the Corporation to Whether or not summon is properly served.
altogether negative the claim that he was only a
part-time general manager. They show beyond
doubt that the Corporation is his alter ego. The
interference of Villarama in the complex affairs of Ruling:
the corporation, and particularly its finances, are
much too inconsistent with the ends and purposes Rule 14 of the Revised Rules of Court provides:
of the Corporation law, which, precisely, seeks to
separate personal responsibilities from corporate
undertakings. It is the very essence of incorporation SEC. 13. Service upoin private
that the acts and conduct of the corporation be domestic corporation or
carried out in its own corporate name because it partnership. — If defendant is a
has its own personality. The doctrine that a corporation organized under the
corporation is a legal entity distinct and separate laws of the Philippines or a
from the members and stockholders who compose partnership duly registered, service
it is recognized and respected in all cases which are may be made on the president,
within reason and the law. When the fiction is urged manager, secretary, cashier, agent,
as a means of perpetrating a fraud or an illegal act or any of its directors.
or as a vehicle for the evasion of an existing
obligation, the circumvention of statutes, the For the purpose of receiving service of summons
achievement or perfection of a monopoly or and being bound by it, a corporation is Identified
generally the perpetration of knavery or crime, the with it agent or officer who under the rule is
veil with which the law covers and isolates the designated to accept service of process. "The
corporation from the members or stockholders who corporate power to receive and act on such service,
compose it will be lifted to allow for its consideration
so far as to make it known to the corporation, is thus petitioner in default and appointed a commissioner
vested in such officer or agent. to receive evidence ex parte. Petitioner filed a
Motion to Dismiss and to Stop Ex Parte Reception
A strict compliance with the mode of service is of Evidence. It asserted that it was not properly
necessary to confer jurisdiction of the court over a served with summons and consequently, the trial
court did not acquire jurisdiction over its person. It
corporation. The officer upon whon service is made
argued that none of the officers enumerated in
be one who is named in the statute; otherwise the Section 13, Rule 14 of the Revised Rules of Court
service is insufficient. So, where the statute required (namely, the corporation's president, manager,
that in the case of a domestic corporation summons secretary, cashier, agent or any of its directors)
should be served on "the president or head of the received any summons. The trial court denied
corporation secretary treasurer, cashier or petitioner's motion and allowed private respondent
to adduce its evidence ex parte.
managing agent thereof", service of summons on
The Court of Appeals dismissed the
the secretary's wife did not confer jurisdiction over petitioner’s petition for certiorari ruling that the trial
the corporation in the foreclosure proceeding court did not commit any grave abuse of discretion
against it. Hence, the the decree of forclosure and in declaring the petitioner in default and in denying
the deficiency judgment were void and should be petitioner's motion for reconsideration.
vacated.
ISSUE:
The purpose is to render it reasonably certain that
the corporation will receive prompt and proper Whether or not there was valid service of
summons.
notice in an action against it or to insure that the
summons be served on a representative so RULING:
integrated with the corporation that such person will
know what to do with the legal papers served on YES.
him. In other words, "to bring home to the
corporation notice of the filing of the action. As a general rule, service of summons must
be made on the persons named in Section 13, Rule
14 of the Revised Rules of Court which provides: If
the defendant is a corporation organized under the
In the instant case the Manila court did not acquire laws of the Philippines or a partnership duly
registered, service may be made on the president,
jurisdiction over Delta Motor because it was not
manager, secretary, cashier, agent or any of its
properly served with summons. The service of directors.
summons on Dionisia G. Miranda, who is not Thus service on persons other than those
among the persons mentioned in section 13 of Rule mentioned in said Rule has been held as improper.
14, was insufficient. It did not bind the Delta Motor. Through the years, the rule on service of summons
has been liberalized. Such liberalization is to give
life to the rationale behind Section 13 of Rule 14.
Service of summons on persons other than those
R. TRANSPORT CORPORATION enumerated in Section 13 of Rule 14 have been
vs. held proper on the theory that those persons served
HON. COURT OF APPEALS, Former 15th were holding positions of responsibility and could
Division, Manila, HON. SALVADOR S. ABAD appreciate the importance of the papers handed
SANTOS, as Presiding Judge, Regional Trial them, and could be expected to deliver the papers
Court of, Metro Manila, Branch 65 and to the proper officer. These individuals were
FLOSERIDA L. CASTAÑEDA considered "agents" within the contemplation of
G.R. No. 111187. February 1, 1995 Section 13 of Rule 14. Thus, the Court holds that
service of summons on petitioner's Operations
FACTS: Manager was valid. He is an officer who may be
relied upon to appreciate the importance of the
On November 22, 1991, private respondent papers served on him.
filed a complaint for damages arising from breach of The fact that service was made at
contract of carriage against petitioner. In an Order petitioner's bus terminal at the address stated in the
dated January 28, 1991, the trial court upon ex summons and not at its office in Makati does not
parte motion of private respondent, declared render the service of summons invalid. Petitioner is
engaged in the transportation business, operating
over 100 buses. Its central bus terminal is located at
Sucat, Parañaque, from where it conducts the bulk
of its business. It was at that terminal where HELD:
petitioner's Operations Manager was found and
upon whom service was made.

The service of summons upon the branch manager


of petitioner at its branch office at Cagayan de Oro,
E.B. Villarosa & Partners Co., Ltd. i. Benito, 312 instead of upon the general manager at its principal
SCRA 65 ,1999| office at Davao City is improper. Consequently, the
trial court did not acquire jurisdiction over the
person of the petitioner.

FACTS:

E.B. Villarosa & Partners is a limited


partnership with principal office address at 102 Juan “Notably, under the new Rules, service of summons
Luna St., Davao City and with branch offices at
Parañaque and Cagayan de Oro City (CDO). upon an AGENT of the corporation is NO LONGER
Villarosa and Imperial Development (ID) executed authorized.”
an Agreement wherein Villarosa agreed to develop
certain parcels of land in CDO belonging to ID into a
housing subdivision. ID, filed a Complaint for
Breach of Contract and Damages against Villarosa
before the RTC allegedly for failure of the latter to “The designation of persons or officers who are
comply with its contractual obligation. authorized to accept summons for a domestic
corporation or partnership is now limited and more
clearly specified in Section11, Rule 14. The rule
now states "general manager" instead of only
"manager";"corporate secretary" instead of
Summons, together with the complaint, were served "secretary"; and "treasurer" instead of "cashier."
upon Villarosa, through its Branch Manager Wendell The phrase “agent, or any of its directors" is
Sabulbero at the address at CDO but the Sheriff’s conspicuously deleted in the new rule.”
Return of Service stated that the summons was duly
served "E.B. Villarosa & Partner thru its Branch
Manager at their new office Villa Gonzalo, CDO,
and evidenced by the signature on the face of the
original copy of the summons." Villarosa prayed for “A strict compliance with the mode of service is
the dismissal of the complaint on the ground of necessary to confer jurisdiction of the court over a
improper service of summons and for lack of corporation. The officer upon whom service is made
jurisdiction over the person of the defendant. must be one who is named in the statute; otherwise
Villarosa contends that the RTC did not acquire the service is insufficient. . . The liberal construction
jurisdiction over its person since the summons was rule cannot be invoked and utilized as a substitute
improperly served upon its employee in its branch for the plain legal requirements as to the manner in
office at CDO who is not one of those persons which summons should be served on a domestic
named in Sec. 11, Rule 14 upon whom service of corporation. .”
summons may be made. ID filed a Motion to
Declare Villarosa in Default alleging that Villarosa
has failed to file an Answer despite its receipt
allegedly on May 5, 1998 of the summons and the The purpose is to render it reasonably certain that
complaint, as shown in the Sheriff's Return. the corporation will receive prompt and proper
notice in an action against it or to insure that the
summons be served on a representative so
integrated with the corporation that such person will
Issue: Won an agent of a corporation can receive know what to do with the legal papers served on
summons in behalf of their corporation? him. In other words, to bring home to the
corporation notice of the filing of the action.
defendant Fernando Busuego, and alleging that the
agreement for the purchase of 4,000 cavans of
G.R. No. L-48930 February 23, 1944 palay and the payment of the price of P8,400 were
made by the plaintiff with and to the Natividad-
Vasquez Sabani Development Co., Inc., a
ANTONIO VAZQUEZ, petitioner, corporation organized and existing under the laws
vs. of the Philippines, of which the defendant Antonio
FRANCISCO DE BORJA, respondent. Vazquez was the acting manager at the time the
transaction took place.
x---------------------------------------------------------x
The trial court rendered judgment ordering the
G.R. No. L-48931 February 23, 1944 defendant Antonio Vazquez to pay the plaintiff the
sum of P3,175.20 plus the sum of P377.50, with
FRANCISCO DE BORJA, petitioner, legal interest on both sums, and absolving the
vs. defendant Fernando Busuego (treasurer of the
ANTONIO VAZQUEZ, respondent. corporation) from the complaint and the plaintiff
from the defendant Antonio Vazquez' counterclaim.
Facts:
Upon appeal to the Court of Appeals, the latter
This action was commenced in the Court of First modified that judgment. But by a subsequent
Instance of Manila by Francisco de Borja against resolution upon the defendant's motion for
Antonio Vazquez and Fernando Busuego to recover reconsideration, the Court of Appeals set aside its
from them jointly and severally the total sum of judgment and ordered that the case be remanded to
P4,702.70 upon three alleged causes of action, to the court of origin for further proceedings.
wit:
The defendant Vazquez, not being agreeable to that
First, that the defendants jointly and severally result, filed the present petition for certiorari to
obligated themselves to sell to the plaintiff 4,000 review and reverse the judgment of the Court of
cavans of palay at P2.10 per cavan, to be delivered Appeals.
during the month of February, 1932, the said
defendants having subsequently received from the Issue:
plaintiff in virtue of said agreement the sum of
P8,400; that the defendants delivered to the plaintiff W/N de Borja entered into the contract with the
during the months of February, March, and April, defendant Antonio Vazquez in his personal capacity
1932, only 2,488 cavans of palay of the value of or as manager of the Natividad-Vazquez Sabani
P5,224.80 and refused to deliver the balance of Development Co., Inc.
1,512 cavans of the value of P3,175.20
notwithstanding repeated demands. Ruling: No.

Second, that because of defendants' refusal to It is well known that a corporation is an artificial
deliver to the plaintiff the said 1,512 cavans of palay being invested by law with a personality of its own,
within the period above mentioned, the plaintiff separate and distinct from that of its stockholders
suffered damages in the sum of P1,000. and from that of its officers who manage and run its
affairs. The mere fact that its personality is owing to
And, third, that on account of the agreement above a legal fiction and that it necessarily has to act thru
mentioned the plaintiff delivered to the defendants its agents, does not make the latter personally liable
4,000 empty sacks, of which they returned to the on a contract duly entered into, or for an act lawfully
plaintiff only 2,490 and refused to deliver to the performed, by them for an in its behalf. The legal
plaintiff the balance of 1,510 sacks or to pay their fiction by which the personality of a corporation is
value amounting to P377.50; and that on account of created is a practical reality and necessity. Without
such refusal the plaintiff suffered damages in the it no corporate entities may exists and no corporate
sum of P150. business may be transacted. Such legal fiction may
be disregarded only when an attempt is made to
The defendant Antonio Vazquez denied having use it as a cloak to hide an unlawful or fraudulent
entered into the contract mentioned in the first purpose. No such thing has been alleged or proven
cause of action in his own individual and personal in this case. It has not been alleged nor even
capacity, either solely or together with his co intimated that Vazquez personally benefited by the
contract of sale in question and that he is merely those of the persons composing it as well as from
invoking the legal fiction to avoid personal liability. that of any other legal entity to which it may be
Neither is it contended that he entered into said related. As a general rule, a corporation may not be
contract for the corporation in bad faith and with made to answer for acts or liabilities of its
intent to defraud the plaintiff. We find no legal and stockholders or those of the legal entities to which it
factual basis upon which to hold him liable on the may be connected and vice versa. However, the veil
contract either principally or subsidiarily. of corporate fiction may be pierced when it is used
as a shield to further an end subversive of justice; or
for purposes that could not have been intended by
the law that created it; or to defeat public
Palay, Inc. v. Clave [No. L – 56076. Sept. 21, convenience, justify wrong, protect fraud, or defend
1983. 124 SCRA 638] crime; or to perpetuate fraud or confuse legitimate
Post under case digests, Commercial Law at issues; or to circumvent the law or perpetuate
Wednesday, March 07, 2012 Posted by deception; or as an alter ego, adjunct or business
Schizophrenic Mind conduit for the sole benefit of the stockholders.
Facts: That Palay, Inc., through its President, Albert
Onstott executed in favor of private respondent, In this case, there was no finding of fraud on
Nazario Dumpit, a Contract to Sell a parcel of Land petitioners' part. They had literally relied, although
payable with a downpayment and monthly mistakenly, on paragraph 6 of its contract with
installments until fully paid. Paragraph 6 of the private respondent when it rescinded the contract to
contract provided for automatic extrajudicial sell extrajudicially and had sold it to a third person.
rescission upon default in payment of any monthly
installment after the lapse of 90 days from the (2) No. The SC held that no sufficient proof exists
expiration of the grace period of one month, without on record that said petitioner used the corporation
need of notice and with forfeiture of all installments to defraud private respondent. He cannot, therefore,
paid. Private respondent Dumpit paid the be made personally liable just because he "appears
downpayment and several installments. However, to be the controlling stockholder". Mere ownership
Dumpit failed to continue paying the installments for
by a single stockholder or by another corporation is
almost 6 years. Thereafter, Dumpit wrote petitioner
offering to update all his overdue accounts with not of itself sufficient ground for disregarding the
interest, and seeking its written consent to the separate corporate personality.
assignment of his rights to a certain Lourdes Dizon.
Petitioners replied that the Contract to Sell had long
been rescinded pursuant to paragraph 6 of the
contract, and that the lot had already been resold. G.R. No. 89879 April 20, 1990
Consequently, Dumpit filed a complaint questioning
the validity of the rescission with the National JAIME PABALAN AND EDUARDO LAGDAMEO,
Housing Authority (NHA) for reconveyance with an petitioners,
alternative prayer for refund. The NHA found the vs.
rescission void in the absence of either judicial or NATIONAL LABOR RELATIONS COMMISSION,
notarial demand. Thus, it ordered Palay, Inc. and LABOR ARBITER AMBROSIO B. SISON,
Alberto Onstott in his capacity as President of the ELIZABETH RODEROS, ET AL., and THE
corporation, jointly and severally, to refund SHERIFF OF THE NATIONAL LABOR
immediately to Dumpit the amount paid with 12% RELATIONS COMMISSION, respondents.
interest from the filing of the complaint. On appeal,
respondent Clave, the Presidential Executive
Assistant affirmed. Hence, this petition. Facts:

Issues: 84 workers of the Philippine Inter-Fashion (PIF) filed


(1) Whether or not the doctrine of piercing the veil of a complaint against the latter for illegal transfer
corporate fiction applies. simultaneous with illegal dismissal in violation of the
Labor Code. PIF was notified about the complaint
(2) Whether or not petitioner Onstott is solidarily and summons but hearings were continually re-set
liable with Palay, Inc. for the refund. for failure of its officers (petitioners herein) to
appear. Complainant workers thus moved to
Held: implead petitioners as officers of PIF in the
(1) No. The SC held that a corporation is invested complaint for their illegal transfer to a new firm. The
by law with a personality separate and distinct from Labor Arbiter ruled in favor of workers holding
petitioners-officers jointly and severally liable with Sulo ng Bayan, Inc. filed an accion de
PIF to pay them their benefits. Petitioners’ appeal revindicacion with the CFI against defendants-
was dismissed. appellees to recover the ownership and possession
of a large tract of land. The complaint specifically
Issue: alleged that plaintiff is a corporation organized and
existing under the laws of the Philippines, with its
principal office and place of business at San Jose
Whether or not petitioners as officers may be held
del Monte, Bulacan; that its membership is
jointly and severally liable with the corporation for its
composed of natural persons residing at San Jose
liability.
del Monte, Bulacan; that the members of the plaintiff
corporation, through themselves and their
Ruling: NO. predecessors-in-interest, had pioneered in the
clearing of the fore-mentioned tract of land,
The settled rule is that the corporation is vested by cultivated the same since the Spanish regime and
law with a personality separate and distinct from the continuously possessed the said property openly
persons composing it, including its officers as well and public under concept of ownership adverse
as from that of any other legal entity to which it may against the whole world.
be related. Thus, a company manager acting in
good faith within the scope of his authority in ISSUE:
terminating the services of certain employees
cannot be held personally liable for damages. Whether or not plaintiff corporation may
However, the legal fiction that a corporation has a institute an action in behalf of its individual members
personality separate and distinct from stockholders for the recovery of certain parcels of land allegedly
and members may be disregarded when the notion owned by said members; for the nullification of the
of legal entity is used as a means to perpetrate transfer certificates of title issued in favor of
fraud or an illegal act or as a vehicle for the evasion defendants appellees covering the aforesaid parcels
of an existing obligation, the circumvention of of land; for a declaration of "plaintiff's members as
statutes, and or (to) confuse legitimate issues the absolute owners of the property" and the issuance
veil which protects the corporation will be lifted. of the corresponding certificate of title; and for
damages.
In this particular case complainants did not allege or
show that petitioners, as officers of the corporation RULING:
deliberately and maliciously designed to evade the
financial obligation of the corporation to its NO.
employees, or used the transfer of the employees
as a means to perpetrate an illegal act or as a It is a doctrine well-established and obtains
vehicle for the evasion of existing obligations, the both at law and in equity that a corporation is a
circumvention of statutes, or to confuse the distinct legal entity to be considered as separate
legitimate issues. and apart from the individual stockholders or
members who compose it, and is not affected by the
Not one of the above circumstances has been personal rights, obligations and transactions of its
shown to be present. Hence petitioners cannot be stockholders or members. The property of the
held jointly and severally liable with the PIF corporation is its property and not that of the
corporation under the questioned decision and stockholders, as owners, although they have
resolution of the public respondent. equities in it. Properties registered in the name of
the corporation are owned by it as an entity
separate and distinct from its members.
Conversely, a corporation ordinarily has no interest
in the individual property of its stockholders unless
SULO NG BAYAN INC.
transferred to the corporation, "even in the case of a
vs.
one-man corporation. The mere fact that one is
GREGORIO ARANETA, INC., PARADISE FARMS,
president of a corporation does not render the
INC., NATIONAL WATERWORKS & SEWERAGE
property which he owns or possesses the property
AUTHORITY, HACIENDA CARETAS, INC, and
of the corporation, since the president, as individual,
REGISTER OF DEEDS OF BULACAN
and the corporation are separate similarities.
G.R. No.L-31061. August 17, 1976
Similarly, stockholders in a corporation engaged in
buying and dealing in real estate whose certificates
FACTS:
of stock entitled the holder thereof to an allotment in
the distribution of the land of the corporation upon
surrender of their stock certificates were considered under the terms and conditions of the stock
not to have such legal or equitable title or interest in certificates.
the land, as would support a suit for title, especially
against parties other than the corporation. ISSUES:
It must be noted, however, that the juridical
personality of the corporation, as separate and Whether or not there is a difference
distinct from the persons composing it, is but a legal between a preferred share from a redeemable
fiction introduced for the purpose of convenience share.
and to subserve the ends of justice. This separate Whether or not petitioner can be compelled
personality of the corporation may be disregarded, by defendant to redeem the preferred shares issued
or the veil of corporate fiction pierced, in cases to the private respondent.
where it is used as a cloak or cover for fraud or
illegality, or to work -an injustice, or where RULING:
necessary to achieve equity.
Clearly, no right of action exists in favor of YES.
plaintiff corporation, for as shown heretofore it does
not have any interest in the subject matter of the A preferred share of stock is one which
case which is material and, direct so as to entitle it entitles the holder thereof to certain preferences
to file the suit as a real party in interest. over the holders of common stock. The preferences
are designed to induce persons to subscribe for
shares of a corporation. Preferred shares take a
multiplicity of forms. The most common forms may
REPUBLIC PLANTERS BANK be classified into two: (1) preferred shares as to
vs. assets; and (2) preferred shares as to dividends.
COURT OF APPEALS and FERMIN CANLAS The former is a share which gives the holder thereof
G.R. No. 93073. December 21, 1992 preference in the distribution of the assets of the
corporation in case of liquidation; the latter is a
FACTS: share the holder of which is entitled to receive
dividends on said share to the extent agreed upon
On September 18, 1961, private respondent before any dividends at all are paid to the holders of
Corporation secured a loan from petitioner in the common stock. There is no guaranty, however, that
amount of P120,000.00. As part of the proceeds of the share will receive any dividends.
the loan, preferred shares of stocks were issued to Under the old Corporation Law in force at
private respondent Corporation, through its officers the time the contract between the petitioner and the
then, private respondent Adalia F. Robes and one private respondents was entered into, it was
Carlos F. Robes. In other words, instead of giving provided that "no corporation shall make or declare
the legal tender totaling to the full amount of the any dividend except from the surplus profits arising
loan, which is P120,000.00, petitioner lent such from its business, or distribute its capital stock or
amount partially in the form of money and partially property other than actual profits among its
in the form of stock certificates. Said stock members or stockholders until after the payment of
certificates were in the name of private respondent its debts and the termination of its existence by
Adalia F. Robes and Carlos F. Robes, who limitation or lawful dissolution." Similarly, the
subsequently, however, endorsed his shares in present Corporation Code provides that the board of
favor of Adalia F. Robes. Said certificates of stock directors of a stock corporation may declare
bear the following terms and conditions: dividends only out of unrestricted retained earnings.
The Preferred Stock shall have the The Code, in Section 43, adopting the change made
following rights, preferences, qualifications and in accounting terminology, substituted the phrase
limitations, to wit: Of the right to receive a quarterly "unrestricted retained earnings," which may be a
dividend of One Per Centum (1%), cumulative and more precise term, in place of "surplus profits
participating. That such preferred shares may be arising from its business" in the former law.
redeemed, by the system of drawing lots, at any Thus, the declaration of dividends is
time after two (2) years from the date of issue at the dependent upon the availability of surplus profit or
option of the Corporation unrestricted retained earnings, as the case may be.
On January 31, 1979, private respondents Preferences granted to preferred
proceeded against petitioner and filed a Complaint stockholders, moreover, do not give them a lien
anchored on private respondents' alleged rights to upon the property of the corporation nor make them
collect dividends under the preferred shares in creditors of the corporation, the right of the former
question and to have petitioner redeem the same being always subordinate to the latter. Dividends
are thus payable only when there are profits earned
by the corporation and as a general rule, even if
there are existing profits, the board of directors has
the discretion to determine whether or not dividends FACTS:
are to be declared. Shareholders, both common and
preferred, are considered risk takers who invest
capital in the business and who can look only to
what is left after corporate debts and liabilities are 1. Since 1982 when, as admitted, [Growth
fully paid. Link] was pre-qualified as NPC supplier, up
Redeemable shares, on the other hand, are to the time in 1987 when x x x NPC refused
shares usually preferred, which by their terms are to do business with petitioner, the latter had
redeemable at a fixed date, or at the option of either numerous sales through public biddings
issuing corporation, or the stockholder, or both at a with a total value of over P60 million x x x.
certain redemption price. Redemption by the
corporation of its stock is, in a sense, a repurchase
of it for cancellation. The present Code allows 2. [Growth Link] was the lowest bidder and the
redemption of shares even if there are no most advantageous bidder in several other
unrestricted retained earnings on the books of the biddings x x x but NPC did not issue the
corporation. This is a new provision which in effect awards.
qualifies the general rule that the corporation cannot
purchase its own shares except out of current
retained earnings. However, while redeemable 3. As a matter [of] procedure, NPC dealt only
shares may be redeemed regardless of the with accredited suppliers and NPC
existence of unrestricted retained earnings, this is recognized [Growth Link] as duly
subject to the condition that the corporation has, accredited. x x x
after such redemption, assets in its books to cover
debts and liabilities inclusive of capital stock.
Redemption, therefore, may not be made where the 4. At the start in 1982 [Growth Link] complied
corporation is insolvent or if such redemption will with the accreditation requirements of NPC
cause insolvency or inability of the corporation to by submitting voluminous documents like
meet its debts as they mature. the articles of incorporation of GLI,
corporate profile, appointment of [Growth
YES. Link] as exclusive supplier and distributor of
spare parts by foreign manufacturers x x x,
While it is true that the very wordings of the suppliers warranties x x x catalogues,
terms and conditions in said stock certificates company profile and other information
clearly allows redemption, the option to do was about foreign suppliers x x x. And, more
clearly vested in the petitioner bank. The importantly, it did not anymore undergo the
redemption therefore is clearly the type known as same process ad (sic) subsequent biddings
"optional". Thus, except as otherwise provided in [that Growth Link] participated in. So that
the stock certificate, the redemption rests entirely the accreditation was a continuing one and
with the corporation and the stockholder is without not on a per transaction basis.
right to either compel or refuse the redemption of its
stock. Furthermore, the terms and conditions set
forth therein use the word "may". It is a settled
doctrine in statutory construction that the word
"may" denotes discretion, and cannot be construed 5. On February 13, 1987 NPC announced its
as having a mandatory effect. decision to stop transacting business with
CA decision is reversed and set aside. [Growth Link] x x x and was blacklisted due
Judgement is hereby rendered declaring private to violation of the conditions of the contract.
respondent Fermin Canlas jointly and severally xxx
liable on all the nine promissory notes with the
following sums and at 16% interest per annum from
the dates indicated. 6. The grounds for the cancellation of [Growth
Links] accreditation x x x are three, namely:

NAPOCOR VS CA a. that [Growth Link] supplied second hand


piston skirts; F2c84-3/5-1027 and 1028Tr for Pielstick
Engine Spares, plus 12% interest thereto
b. that piston rings supplied by it did not reach [sic] per annum from September, 1996 until
the required running hours fully paid;
h) P6,216,583.00 representing unrealized
commissions on various item bidded where
[Growth Link] was the lowest bidder but
c. that [Growth Link] supplied exhaust valve which was not awarded by NPC to it, plus
bodies manufactured by Fuji Diesel Ltd. 12% interest thereto [sic] per annum from
which was not licensed by SEMT. July, 1986 until fully paid;
i) P 1,419,853.00 representing unpaid
7. [Growth Link] refuted the charges in several commission for the disregarded lowest bid
letters x x x and was asking for opportunity of [Growth Links] principal on NPC Foreign
to be heard at a formal hearing on [the] Inquiry Nos. FPS85-11/6-005AA, plus 12 %
request for reconsideration but same was interest thereto [sic] per annum from
not acted upon by NPC. October, 1987 until fully paid;
j) P2,000,000.00 for compensatory damage[s]
suffered by petitioner due to loss of
8. [Growth Link] filed [a] petition for business relationship and standing here
mandamus with preliminary injunction and and abroad;
damages with the trial court on February 8, k) P1,500,00.00 for moral and exemplary
1988 which was granted by the court and damages suffered by [Growth Link];
the decision as to the award of damages l) P30,000.00 plus 30% of the principal
was in favor of Growth Link ordering the amount recoverable, as and for attorneys
NPC and its officers and members of the fees;
Board of Directors, to jointly and severally m) P40,000.00 as litigation expenses
pay Growth Link the following amounts: (premiums paid on the injunction bond,
etc.); and
n) Costs of suit.

a) P230,000.00 representing the cost of the


replaced piston skirts under P.O No. ISSUE: WON THE LOWER COURT ERRED IN
086653 plus 12% interest thereto [sic] per HOLDING NAPOCOR JOINTLY AND SEVERALLY
annum from April 9, 1986 until fully paid; LIABLE WITH ITS OFFICERS.
b) P16,870.00 [which was] the amount
deducted by [NPC] from [Growth Link]s
outstanding collectibles, plus 12% interest
thereto [sic] per annum from November 18, HELD:The finding solidarity liability among the NPC
1985 until fully paid;
and its officers and Members of the Board of
c) P144,000.00 for payment of items delivered
under P.O. No. 095435 plus 12 % interest Directors, is patently baseless. The decision of the
thereto [sic] per annum from November 13, trial court contains no such allegation, finding or
1986 until fully paid; conclusion regarding particular acts committed by
d) P27,650.00 for payment of items delivered said officers and members of the Board of Directors
under P.O. No. 096345 plus 12% interest that show them to have individually guilty of
thereto [sic] per annum from April 4, 1987 unmistakable malice, bad faith, or ill-motive in their
until fully paid;
personal dealings with Growth Link. In fact, it was
e) P182,070.00 for payment of items delivered
under P.O. No. 096626 plus 12% interest only in the dispositive portion of the decision of the
thereto [sic] per annum from April 4, 1987 court a quo that solidary liability as such was first
until fully paid; mentioned.
f) P176,356.00 representing unrealized
commission on the cancelled Indent Order
No 08114 dated May 24, 1985 plus 12%
interest thereto [sic] per annum from NPCs officers and members of the Board of
November, 1985 until fully paid; Directors were sued merely as nominal parties in
g) P1,249,745.00 representing unrealized their official capacities as such. They were
commission on the Foreign Inquiry Nos.
impleaded by Growth Link not in their personal
capacities as individuals but in their official
capacities as officers and members of the Board of
Directors through whom the NPC conducts EMILIO CANO ENTERPRISES, INC., petitioner,
business and undertakes its operations pursuant to vs.
its avowed corporate purposes. Therefore, as a COURT OF INDUSTRIAL RELATIONS, ET
bonafide government corporation, NPC should AL., respondents.
alone be liable for its corporate acts as duly
Facts:
authorized by its officers and directors.

A complaint for unfair labor practice was filed


before the Court of Industrial Relations against
This is so, because a corporation is invested by law Emilio, Ariston and Rodolfo, all surnamed Cano, in
their capacity as president and proprietor, field
with a separate personality, separate and distinct
supervisor and manager, respectively, of Emilio
from that of the person composing it as well as from Cano Enterprises, Inc.
any other legal entity to which it may be related.
(Tan Boon Be & Co. v. Jarencio, 163 SCRA 205 After trial, Presiding Judge Jose S. Bautista
[1988] citing Yutivo and Sons Hardware Company rendered decision finding Emilio Cano and Rodolfo
v. Court of Tax Appeals, 1 SCRA 160 [1961]; Emilio Cano guilty of the unfair labor practice charge, but
Cano Enterprises, Inc. v. Court of Industrial absolved Ariston for insufficiency of evidence. As a
Relations, 13 SCRA 290 [1965]). A corporation is an consequence, the two were ordered, jointly and
artificial person and can transact its business only severally, to reinstate Honorata Cruz, to her former
position with payment of backwages from the time
through its officers and agents. Necessarily, of her dismissal up to her reinstatement, together
somebody has to act for it. Separate personality of with all other rights and privileges thereunto
the corporation may be disregarded, or the veil of appertaining.
corporate fiction pierced and the individual
stockholders may be personally liable to obligations Emilio Cano died and the attempt to have the case
of the corporation only when the corporation is used dismissed against him having failed, the case was
as a cloak or cover from fraud or illegality, or to appealed to the court en banc, which in due course
work an injustice, or where necessary to achieve affirmed the decision of Judge Bautista.
equity or when necessary for the protection of
The order of execution have been directed against
creditors. (Sulo ng Bayan, Inc. v. Araneta, Inc., 72
the properties of Emilio Cano Enterprises, Inc.
SCRA 347 [1976] xxx). instead of those of the respondents named in the
decision, said corporation filed an ex parte motion to
quash the writ on the ground that the judgment
sought to be enforced was not rendered against it
We repeat, there was nothing in Growth Links which is a juridical entity separate and distinct from
petition not in the mass of evidence proffered, its officials. This motion was denied. Motion for
before the court a quo that established the factual or reconsideration was also denied. Hence, the
legal basis to hold the officers and members of the corporation interposed the present petition
Board of Directors of the NPC jointly and severally for certiorari.
liable with the NPC for the damages suffered by
Growth Link because of acts of gross evident bad Issue:
faith on the part of the NPC as a corporate entity
Can the judgment rendered against Emilio and
acting through its officers and directors. The records
Rodolfo Cano in their capacity as officials of the
even bear out that every single offense taken by the corporation Emilio Cano Enterprises, Inc. be made
NPC against Growth Link arose from a corporate effective against the property of the latter which was
decision and was executed as a corporate act. not a party to the case?
Thus, the trial court gravely erred in holding said
officers and directors to be jointly and severally Ruling:
liable with the NPC for the damages suffered by
Growth Link but caused by the NPC alone as a The answer must be in the affirmative. While it is an
corporate entity. undisputed rule that a corporation has a personality
separate and distinct from its members or  You are to remit the amount of P16,000
stockholders because of a fiction of the law, here immediately after accepting the
we should not lose sight of the fact that the Emilio appointment as a guarantee bond for the
Cano Enterprises, Inc. is a closed family corporation
faithful performance of your duties and
where the incorporators and directors belong to one
single family. Thus, the following are its responsibilities;
incorporators: Emilio Cano, his wife Juliana, his  All government licenses, permits, utilities
sons Rodolfo and Carlos, and his daughter-in-law and services in the premises shall be your
Ana D. Cano. Here is an instance where the account;
corporation and its members can be considered as  The sole control and management of the
one. And to hold such entity liable for the acts of its
premises shall belong to you and your are
members is not to ignore the legal fiction but merely
to give meaning to the principle that such fiction not responsible to Any Board of Directors
cannot be invoked if its purpose is to use it as a except to me alone;
shield to further an end subversive of justice. 1 And  You are empowered to make any
so it has been held that while a corporation is a renovation, repairs and improvements but
legal entity existing separate and apart from the expenses shall be for your account as well
persons composing it, that concept cannot be
as to change or add personnel therein;
extended to a point beyond its reason and policy,
and when invoked in support of an end subversive  Please take all good care of all the
of this policy it should be disregarded by the courts. equipment and facilities presently existing
therein and see to it that they are always in
A factor that should not be overlooked is that Emilio good working condition.
and Rodolfo Cano are here indicted, not in their
private capacity, but as president and manager, This case arose from the petitioner’s act of allegedly
respectively, of Emilio Cano Enterprises, Inc. terminating the respondent’s appointment as
Having been sued officially their connection with the manager-administrator as a result of his alleged
case must be deemed to be impressed with the failure to comply with the terms and conditions of
representation of the corporation. In fact, the court's
his appointment. Private Respondent filed with the
order is for them to reinstate Honorata Cruz to her
former position in the corporation and incidentally CFI of Manila a case for specific performance and
pay her the wages she had been deprived of during damages with prayer for writ of preliminary
her separation. Verily, the order against them is in mandatory injunction and attorney’s fees against
effect against the corporation. petitioner. Private Respondent amended his
complaint for breach of contract with damages with
the same prayer.

Paradise Sauna vs Ng After the trial, the lower court rendered a judgment
G.R. No. L-66394 in favor of private respondent, declaring the letter-
February 5, 1990 contract as a contract of lease covering the
paradise sauna bath and massage clinic.
FACTS:
On appeal, the then Intermediate Appellate Court
On December 30, 1975, petitioners and private
affirmed the decision of the trial court. In its petition
respondent entered into a letter-contract (Exhibit A)
for review, petitioner contends that IAC sanctioned
signed by petitioner Juanito Uy, President of
a legal error made by the trial court which is a
Paradise Sauna which provides that:
reformation of Exhibit A from a management
 Alejandro Ng is appointed to manage and contract to a lease contract contrary to Art. 1367 of
administer the Paradise Sauna and the New Civil Code. In support of their contention,
Massage Corporation effective January 1, they averred that when respondent filed an action
1976, under a commission basis over and for specific performance then for breach of contract,
above the amount of P8,000 which should he should have been presumed to have admitted
be remitted to the petitioner not later than the due execution and contents of the letter-contract
the first 5 days of each month starting marked as Exhibit A whereby he was appointed as
January 1, 1976; manager-administrator of the petitioner corporation
and he should never have been allowed to deny the corporation on different occasions in 1980, 1981
contents thereof for purposes of reforming the said and 1982, represented by Antonio Rodriguez but
instrument. denied that its total obligation was P100,753.80.
WESGRO alleged that this amount is bloated
Issue: because it had already made various payments on
different dates.
Whether petitioner Uy can be held liable for his
actions.

Held: Antonio Rodriguez filed a motion to dismiss on the


ground that the complaint states no cause of action
Yes.
against him. He alleged that he is a director and
Anent the argument that the respondent Court, in officer of WESGRO and that he entered into the
holding petitioner Uy severally liable with the purchase contract with the respondent corporation
petitioner corporation, departed from the rule that a in his capacity as officer or agent of WESGRO and
stockholder or officer of a corporation has a therefore such contract was with WESGRO as a
personality distinct from the corporation, we hold distinct legal entity and did not confer rights much
that the corporate entity theory cannot apply in the less liabilities on him.
instant case where it is being invoked as a cloak or
shield for illegality. (see Tan Boon Bee & Co., Inc. v.
Judge Jarencio, 163 SCRA 205 [1988]), There is RTC denied the motion to dismiss filed by
proof obtaining in the case at bar as to the real Rodriguez.
nature of Exhibit A. Thus, being a party to a
simulated contract of management, petitioner Uy
cannot be permitted to escape liability under the
said contract by using the corporate entity theory. The appellate courts ruled that Antonio Rodriguez is
This is one instance when the veil of corporate solidarily liable with WESGRO for the latter's
entity has to be pierced to avoid injustice and P84,626.70 obligation to SIA is based principally on
inequity. the ground that Rodriguez represented WESGRO in
its dealings with SIA.
WESTERN AGRO VS CA

ISSUE: whether or not petitioner Antonio Rodriguez


FACTS: can be made solidarily liable with the petitioner
corporation for debts incurred by the latter.

WESGRO is doing business through Antonio


Rodriguez and on different occasions in 1980, 1981 HELD:
and 1982, Rodriguez, representing WESGRO
bought on credit different automotive spare parts
from the private respondent amounting to
It is significant to note that SIA never questioned the
P100,753.80; that the said amount has long
legal personality of WESGRO. Hence, we can
become over due and yet the petitioners refused to
assume that WESGRO is a bona fide corporation.
pay despite repeated demands. The complaint
Therefore, as a bona fide corporation, WESGRO
prayed among others that the defendants jointly and
should alone be liable for its corporate acts as duly
severally pay the plaintiff.
authorized by its officers and directors. (Caram Jr.
v. Court of Appeals, 151 SCRA 372 [1987]). This is
so, because a corporation "is invested by law with a
WESGRO admitted that it bought on credit various separate personality, separate and distinct from that
automotive spare parts from the respondent of the persons composing it as well as from any
other legal entity to which it may be related." (Tan G.R. No. 70789 October 19, 1992
Boon Bee & Co Inc. v. Jarencio, 163 SCRA 205
[1988] citing Yutivo and Sons Hardware Company RUSTAN PULP & PAPER MILLS, INC.,
v. Court of Tax Appeals, 1 SCRA 160 [1961]; Emilio BIENVENIDO R. TANTOCO, SR., and ROMEO S.
Cano Enterprises, Inc. v. Court of Industrial VERGARA, petitioners,
vs.
Relations, 13 SCRA 290 [1965]). A corporation is an
THE INTERMEDIATE APPELLATE COURT and
artificial person and can transact its business only ILIGAN DIVERSIFIED PROJECTS, INC., ROMEO
through its officers or agents. Necessarily, A. LLUCH and ROBERTO G.
somebody has to act for it. The separate personality BORROMEO, respondents.
of the corporation may be disregarded, or the veil of
corporate fiction pierced and the individual Facts:
stockholders may be personally liable to obligations
of the corporation only when the corporation is used Petitioner Rustan established a pulp and paper mill
"as a cloak or cover for fraud or illegality, or to work in Baloi, Lano del Norte. Respondent Lluch, who is
a holder of a forest products license, transmitted a
an injustice, or where necessary to achieve equity
letter to petitioner Rustan for the supply of raw
or when necessary for the protection of creditors." materials by the former to the latter.
(Sulo ng Bayan, Inc. v. Araneta, Inc., 72 SCRA 347
[1976] cited in Tan Boon Bee & Co., Inc. v. Petitioner Rustan proposed “that the contract to
Jarencio, supra). supply is not exclusive because Rustan shall have
the option to buy from other suppliers who are
qualified and holder of appropriate government
authority or license to sell and dispose pulp wood”.
In the case at bar, there is no showing that Antonio
Rodriguez, a director and officer of WESGRO was These business proposals resulted to the execution
not authorized by the corporation to enter into of a contract of sale whereby Romeo A. Lluch
purchase contracts with SIA. Moreover, the agreed to sell, and Rustan Pulp and Paper Mill, Inc.
respondent corporation has not shown any undertook to pay the price of P30.00 per cubic
circumstances which would necessitate the piercing meter of pulp wood raw materials to be delivered at
the buyer's plant in Baloi, Lanao del Norte.
of the corporate veil so as to make Rodriguez
personally liable for the obligations incurred by the
Of pertinent significance to the issue at hand are the
petitioner. Hence, the inevitable conclusion is that following stipulations in the bilateral undertaking:
he was acting in behalf of the corporation when he
executed the purchase contracts with the 3. That BUYER shall have the option to buy from
respondent corporation. In other words, Rodriguez' other SELLERS who are equally qualified and
acts in representing the petitioner corporation in its holders of appropriate government authority or
dealings with the respondent corporation are license to sell or dispose, that BUYER shall not buy
corporate acts for which only the corporation should from any other seller whose pulp woods being sold
shall have been established to have emanated from
be made liable for any obligations arising from
the SELLER'S lumber and/or firewood concession. .
them. ..

And that SELLER has the priority to supply the pulp


wood materials requirement of the BUYER;
WHEREFORE, the instant petition is hereby
PARTLY GRANTED. The questioned decision of
7. That the BUYER shall have the right to stop
the Court of Appeals is modified in that petitioner
delivery of the said raw materials by the seller
Antonio Rodriguez is declared not liable jointly and covered by this contract when supply of the same
severally or otherwise with petitioner WESTERN shall become sufficient until such time when need
AGRO INDUSTRIAL CORPORATION for the for said raw materials shall have become
money awards in favor of respondent Sia's necessarily provided, however, that the SELLER is
Automotive and Diesel Parts, Inc. The decision is given sufficient notice.
affirmed in other respects.
During the test run of the pulp mill, the machinery
line thereat had major defects while deliveries of the
raw materials piled up, which prompted the EPG Construction Co, Inc. entered into a
Japanese supplier of the machinery to recommend contract with the University of the Philippines to
the stoppage of the deliveries. Private respondent construct the UP Law Library Building for the
Romeo Lluch sought to clarify the tenor of the letter
stipulated price of ₱7,545,000.00. The contract also
as to whether stoppage of delivery or termination of
the contract of sale was intended, but the query was included an agreement which stated that upon the
not answered by petitioners. This alleged ambiguity guaranty of the contractor, the work was completed
notwithstanding, Lluch and the other suppliers in accordance with the architect's prepared plans
resumed deliveries after the series of talks between and specifications. And, that the contractor would
Romeo S. Vergara and Romeo Lluch. repair at his own cost and expenses for a period of
1 year from the completion date all work covered
The complaint for contractual breach was filed under their contract and change any order that
which was dismissed.
would prove to be deffective except maintenance
works. The guarantee clause of the contract also
Respondent Court found it ironic that petitioners had
expressly provided for the contractor's liability under
to exercise the prerogative regarding the stoppage
of deliveries via the letter addressed to Iligan Art. 1723 should the building collapse because of
Diversified Project, Inc. because petitioners never any defects in construction or use of materials of
really stopped accepting deliveries from private inferior quality furnished by him or due to any
respondents until December 23, 1968. violation of the terms of their contract.

Issue:

W/N Tantoco and Vergara should be liable to pay EPG turned the building over to UP upon its
for moral damages and atty’s fees completion and the latter institution issued a
certification of acceptance on January 3, 1983.
Ruling: No. Sometime in July of the same year, the private
respondent complained to company that 6 air-
We have to agree with petitioners' citation of conditioning units on the third floor of the building
authority to the effect that the President and were not cooling properly. After inspecting the
Manager of a corporation who entered into and equipment, the petitioner agreed to shoulder the
signed a contract in his official capacity, cannot be
expenses to get the air-conditioning units repaired,
made liable thereunder in his individual capacity in
the absence of stipulation to that effect due to the including labor and materials amounting to ₱38,000.
personality of the corporation being separate and
distinct from the person composing it.
However, the units were not repaired. Upon
filing another complaint with the company, the latter
sent its representatives to check the broken units.
After inspection, EPG made a written offer to repair
the cooling system for ₱194,000. When EPG
G.R. No. 103372 June 22, 1992
demurred to UP's insistence that it was it's
obligation to repair the defects under the guarantee
EPG CONSTRUCTION COMPANY, INC., and
clause of their contract, the school had to contract
EMMANUEL P. DE GUZMAN, petitioner,
another company for repairs and the costs totalled
vs.
to ₱190,000.
HONARABLE COURT OF APPEALS (17th
Division), ( Republic of the Philippines),
UNIVERSITY OF THE PHILIPPINES, respondents.
UP demanded reimbursement from EPG
plus damages for the cost of repairs, but it was
rejected. So, the private respondent sued the
FACTS: company and its president Emmanuel P. De
Guzman in the RTC. The trial court ruled in favor of Notably, when Emmanuel de Guzman moved to
UP and ordered the company and its president to dismiss the complaint as to him, UP said in its
pay the plaintiff jointly and severally for the repairs, opposition to the motion that it was suing him "in his
damages and other costs. The company elevated official capacity and not in his personal capacity."
the case to the CA, but the appellate court His inclusion as President of the company was
sustained the ruling of the trial court. therefore superfluous, as De Guzman correctly
contended, because his acts as such were
corporate acts imputable to EPG itself as his
principal. It is settled that;

ISSUE: A corporation is invested by law


with a personality separate and
distinct from those of the persons
composing it as well as from that of
Whether EPG was relieved from liability any other entity to which it may be
when UP issued a certificate of acceptance upon related. Mere ownership by a single
the completion of the building. stockholder or by another
corporation of all or nearly all of the
capital stock of a corporation is not
HELD: of itself sufficient ground for
disregarding the separate corporate
personality. The general manager
of a corporation therefore should
The final point of the petition is that not be made personally answerable
Emmanuel P. de Guzman has a separate legal for the payment of the employee's
personality from EPG Construction Co., Inc. and backwages unless he had acted
should not be held solidarity liable with it. He maliciously or in bad faith in
stresses that the acts of the company are its own
terminating the services of the
responsibility and there is no reason why any
liability arising from such acts should be ascribed to employee. 4
him. Thus:
The exception noted is where the official "had acted
It is a doctrine well-established and maliciously or in bad faith," in which event he may
obtains both at law and in equity be made personally liable for his own act. That
that a corporation is a distinct legal exception is not applicable in the case at bar,
entity to be considered as separate because it has not been proved that De Guzman
and apart from the individual acted maliciously or in bad faith when, as President
stockholders or members who of EPG, he sought to protect its interests and
compose it, and is not affected by resisted UP's claims. Whatever damage was
the personal rights, obligations and caused to UP as a result of his acts is the sole
transactions of its stockholders or responsibility of EPG even though De Guzman was
members. its principal officer and controlling stockholder.

The trial court did not explain why Emmanuel de In sum, we hold that the lower court did not err in
Guzman was held solidarity liable with EPG holding EPG liable for the repair of the air-
Construction Co., Inc., and neither did the conditioning system at its expense pursuant to the
respondent court when it affirmed the appealed guarantee provision in the construction contract with
decision, In its Comment on the present petition, UP UP. However, Emmanuel de Guzman is not
also did not refute the petitioners' argument and solidarily liable with it, having acted on its
simply passed upon it sub silentio although the behalf within the scope of his authority and
matter was squarely raised and discussed in the without any demonstrated malice or bad faith.
petition.
use of the innocuous phrase "in his capacity as
President" found in the dispositive portion of the
G.R. No. 89804 October 23, 1992 challenged Amended Decision — making the same
a sanctuary for a defense which he, as hereinafter
CALVIN S. ARCILLA, petitioner, vs. discussed, had long since abandoned or waived
either deliberately or through his obliviscence. His
THE HONORABLE COURT OF APPEALS and
sole purpose, of course, is to avoid complying with
EMILIO RODULFO, respondents.
the liability adjudged against him by the public
Facts: Petitioner Calvin Arcilla, taking advantage of respondent; such avoidance is premiered on the so-
his close relationship with respondent Emilio called newly discovered evidence offered after the
Rodulfo, succeeded in securing on credit from the public respondent had bent over backwards to grant
latter, items, cash and checks. him a new trial despite the availability of such
evidence during pendency of the proceedings
RODULFO willingly extended the credit because of before the trial court. It is to be noted that he failed
the representations of ARCILLA that he was a to assign as error in his Brief the denial by the said
successful financial consultant. court of his motion for new trial on the basis thereof.

The indebtedness is shown and described in 30 if We are to assume arguendo that the obligation
“vales” signed by ARCILLA. was incurred in the name of the corporation, the
petitioner would still be personally liable therefor
When RODULFO demanded payment, ARCILLA, because for all legal intents and purposes, he and
acting in gross and evident bad faith, refused to the corporation are one and the same. Csar Marine
satisfy the claim. Hence, he filed a complaint for Resources, Inc. is nothing more than his business
RECOVERY OF SUM OF MONEY. conduit and alter ego. The fiction of a separate
juridical personality conferred upon such
In his Answer, ARCILLA did not deny having had
corporation by law should be disregarded.
business transactions with RODULFO. He explicitly
Significantly, petitioner does not seriously challenge
admits that the loan was in the name of his family
the public respondent's application of the doctrine
corporation, CSAR MARINE RESOURCES, INC.
which permits the piercing of the corporate veil and
After hearing, the Court rendered judgment against the disregarding of the fiction of a separate juridical
ARCILLA ordering him to pay private respondent personality; this is because he knows only too well
Emilio Rodulfo in his capacity as President of CSAR that from the very beginning, he merely used the
MARINE. corporation for his personal purposes.

ARCILLA argued that he cannot be ordered to pay Petitioner's volunteered admission that he procured
in his capacity as President of CSAR MARINE since the pro-forma invoice from the private respondent in
the said corporation was not impleaded in the case. connection with his loan from the KKK, using his
The personality and liability of ARCILLA and CSAR family corporation in the process, and his deliberate
MARINE as a corporation are separate and distinct. waiver of the aforementioned defense provide an
insurmountable obstacle to the viability of this
Issue: Whether or not the petitioner can raise the petition.
issue of separate corporate identity in the pleadings
for the first time in a Motion for Clarificatory
Judgment?
TRAMAT MERCANTILE, INC. and DAVID HONG
vs.
Ruling: The records bear nothing to prop up the COURT OF APPEALS, MELCHOR DE LA
instant petition. The arguments adduced by the CUESTA
petitioner breathe no life to it. GR 111008, 07 November 1994

On the contrary, the pleadings lead Us to the FACTS:


inescapable conclusion that the petitioner, who is
himself a lawyer, is merely taking advantage of the
On 09 April 1984, Melchor de la Cuesta, MAM REALTY CORPORATION
doing business under the name and style of vs.
"Farmers Machineries," sold to Tramat Mercantile, NLRC, CELSO BALBASTRO
Inc. one unit Hinomoto Tractor Model MB 1100D GR 114787, 02 June 1995
powered by a 13 H.P. diesel engine. In payment,
David Ong, Tramat's president and manager, FACTS:
issued a check for P33,500.00 (apparently replacing
an earlier postdated check for P33,080.00). Tramat, Celso B. Balbastro filed a case against
in turn, sold the tractor, together with an attached MAM Realty Development Corporation ("MAM") and
lawn mower fabricated by it, to the Metropolitan its Vice President Manuel P. Centeno, for unfair
Waterworks and Sewerage System/NAWASA for labor practice in violation of the Labor Code.
P67,000.00. David Ong caused a stop payment of Balbastro alleged that he was employed by MAM as
the check when NAWASA refused to pay the tractor a pump operator in 1982 and had since performed
and lawn mower after discovering that, aside from such work at its Rancho Estate, Marikina, Metro
some stated defects of the attached lawn mower, manila.
the engine (sold by de la Cuesta) was a MAM countered that Balbastro had
reconditioned unit. previously been employed by Francisco Cacho and
On 28 May 1985, de la Cuesta filed an co., Inc., the developer of Rancho Estates.
action for the recovery of P33,500.00, as well as Sometime in May 1982, his services were
attorney's fees of P10,000.00, and the costs of suit. contracted by MAM for the operation of the Rancho
Ong, in his answer, averred, among other things, Estates' water pump. He was engaged, however,
that de la Cuesta had no cause of action; that the not as an employee, but as a service contractor, at
questioned transaction was between plaintiff and an agreed fee of P1,590.00 a month. Similar
Tramat Mercantile, Inc., and not with Ong in his arrangements were likewise entered into by MAM
personal capacity; and that the payment of the with one Rodolfo Mercado and with a security guard
check was stopped because the subject tractor had of Rancho Estates III Homeowners' Association.
been priced as a brand new, not as a reconditioned Under the agreement, Balbastro was merely made
unit. to open and close on a daily basis the water supply
system of the different phases of the subdivision in
ISSUE: accordance with its water rationing scheme. He
worked for only a maximum period of three hours a
Whether or not Ong should be held liable day, and he made use of his free time by offering
for the unpaid tractor. plumbing services to the residents of the
subdivision. He was not at all subject to the control
RULING: or supervision of MAM for, in fact, his work could so
also be done either by Mercado or by the security
NO. guard. On 23 May 1990, prior to the filing of the
complaint, MAM executed a Deed of Transfer, 1
It is an error to hold David Ong jointly and effective 01 July 1990, in favor of the Rancho
severally liable with TRAMAT to de la Cuesta under Estates Phase III Homeowners Association, Inc.,
the questioned transaction. Ong had acted, not in conveying to the latter all its rights and interests
his personal capacity, but as an officer of a over the water system in the subdivision.
corporation, TRAMAT, with a distinct and separate NLRC found the corporation guilty as
personality. As such, it should only be the charged, and likewise held Centeno liable together
corporation, not the person acting for and on its with said corporation.
behalf, that properly could be made liable thereon.
Tramat, however, should be held liable for ISSUE:
the unpaid tractor because at the time of the
purchase, the appellants did not reveal to the Whether or not Centeno should be held
appellee the true purpose for which the tractor liable together with MAM Realty.
would be used. Granting that the appellants
informed the appellee that they would be reselling RULING:
the unit to the MWSS, an entity admittedly not
engaged in farming, and that they ordered the NO.
tractor without the power tiller, an indispensable
accessory if the tractor would be used in farming, A corporation, being a juridical entity, may
these in themselves would not constitute the act only through its directors, officers and
required implied notice to the appellee as seller. employees. Obligations incurred by them, acting as
such corporate agents, are not theirs but the direct
accountabilities of the corporation they represent. b. act in bad faith or with
True, solidarily liabilities may at times be incurred gross negligence in
but only when exceptional circumstances. In labor directing the corporate
cases, for instance, the Court has held corporate affairs;
directors and officers solidarily liable with the c. are guilty of conflict of
corporation for the termination of employment of interest to the prejudice of
employees done with malice or in bad faith. the corporation, its
In the case at bench, there is nothing stockholders or members,
substantial on record that can justify, prescinding and other persons.
from the foregoing, petitioner Centeno's solidary 2. When a director or officer has
liability with the corporation. Nothing states that he consented to the issuance of
acted in bad faith. watered stocks or who, having
Although the Court found that there is an knowledge thereof, did not forthwith
employer-employee relationship between Balbastro file with the corporate secretary his
and MAM Realty, the case was remanded to NLRC written objection thereto.
for the recomputation of Balbastro’s monetary 3. When a director, trustee or officer
awards, such as backwages and wage differentials. has contractually agreed or
stipulated to hold himself personally
and solidary liable with the
Corporation.
4. When a director, trustee or officer is
MAM Realty VS. NLRC (1995) made, by specific provision of law,
personally liable for his corporate
Facts: action.
A complaint filed with the Labor Arbiter by
private respondent Celso Balbastro against In labor cases, for instance, the Court has
petitioners, MAM Realty Development Corporation held corporate directors and officers solidary liable
and its Vice President Manuel Centeno, for wage with the corporation for the termination of
differentials, ECOLA, overtime pay, incentive leave employment of employees done with malice or in
pay, 13th month pay, holiday pay and rest day pay. bad faith.
Balbastro alleged that he was employed by MAM as
a pump operator and had since performed such In the case at Bench, there is nothing
work at its Rancho Estate, Marikina. LA dismissed substantial on record that can justify, prescinding
said complaint for lack of merit. The CA however from the foregoing, petitioner Centeno’s solidary
reversed the same and held that Centeno is jointly liability with the corporation.
and severally liable with MAM.
Issue:
W/N the CA erred in holding Centeno jointly
and severally liable with MAM.

Ruling:
Yes, the court agrees with petitioners that
the NLRC erred in holding Centeno jointly and
severally liable with MAM. A corporation, being a
juridical personality, may act only through its
directors, officers and employees. Obligations
incurred by them, acting as such corporate agents,
are not theirs but the direct accountabilities of the
corporation they represent. True, solidary liabilities
may at times be incurred but only when exceptional
circumstances warrant such as, generally, in the
following cases:

1. When directors and trustees or, in


appropriate cases, the officers of a
corporation –
a. vote for or assent to
patently unlawful acts of
the corporation;
NATIONAL FOOD AUTHORITY vs. CA G.R. No. 96453, remuneration to the owner of the ships for the detention
August 4, 1999 of his vessel beyond the number of days allowed by the
charter party for loading or unloading or for sailing.
Thursday, January 29, 2009 Posted by Coffeeholic Liability for demurrage, using the word in its technical
Writes sense, exists only when expressly stipulated in the
Labels: Case Digests, Commercial Law contract.

Facts: National Food Authority (NFA), thru its officers, Shipper or charterer is liable for the payment of
entered into a “Letter of Agreement for Vessel/Barge demurrage claims when he exceeds the period for
Hire with Hongfil for the shipment of 200,000 bags of loading and unloading as agreed upon or the agreed
corn grains from Cagayan de Oro City to Manila. “laydays”. The period for such may or may not be
stipulated in the contract. A charter party may either
The loading of bags of corn grains in the vessel provide for a fixed laydays or contain general or
commenced but it took a longer period of 21 days, 15 indefinite words such as “customary quick dispatch” or
hours, and 18 minutes to finish than as was certified by “as fast as the streamer can load”. In the case at bar, the
the arrastre firm as there was a strike staged by the charter party provides merely for a general or indefinite
arrastre workers in view of the refusal of the striking words of “customary quick dispatch”. Such stipulation
stevedores to attend to their work. The vessel was implies that loading and unloading of the cargo should
allowed to depart for the port of Manila and arrived there, be within a reasonable time.
but unfortunately, it took a longer period of 20 days, 14
hours and 33 minutes to finish the unloading than the The charterer NFA could not be held liable for
discharging rate certified by the Port of Manila, due to demurrage for it appears that cause of delay was not
the unavailability of a berthing space for the vessel M/V imputable to either of the parties. The cause of delay
CHARLIE/DIANE. Only 166,798 bags were unloaded at during the loading was the strike staged by the crew of
the Port of Manila. the arrastre operator, and the unavailability of a berthing
space for the vessel during the unloading. Here, the
After the discharging was completed, NFA paid Hongfil Court holds that the delay sued upon was still within the
the amount of P1,006,972.11 covering the shipment of “reasonable time” embraced in the stipulation of
corn grains. Thereafter, Hongfil sent its billing to NFA “Customary Quick Dispatch”.
claiming payment for freight covering the shut-out load
or deadfreight as well as demurrage, allegedly sustained Furthermore, considering the subject contract of
during the loading and unloading of subject shipment of affreightment contains an express provision
corn grains. When NFA refused to pay the amount “Demurrage/Dispatch: NONE”, the same left the parties
reflected in the billing, Hongfil brought the present action
with no recourse but to apply the literal meaning of such
against NFA.
stipulation.

Issues:
1) Can petitioners be held liable for deadfreight?
2) Can petitioners be held liable for demurrage? AC Ransom Labor Union v. NLRC
G.R. No. L-69494

Held: 1) Yes. It bears stressing that subject Letter of Facts:


Agreement is considered a Charter Party. A charter
party is classified into (1) “bareboat” or “demise” charter AC Ransom Labor Union is claiming unfair labor practice
and (2) contract of affreightment. Subject contract is one against AC Ransom. The CIR and the Labor Arbiter
of affreightment, whereby the owner of the vessel leases ruled in favor of the labor union stating that the strike
part or all of its space to haul goods for others. It is a was legal and justified thereby requiring the company to
contract for special service to be rendered by the owner
pay for backwages and to immediately reinstate the
of the vessel. Under such contract, the ship retains
possession, command, and navigation of the ship, the members of the union which the NLRC reversed the
charterer or freighter merely having use of the space in decision. Hence the special civil action of certiorari filed
the vessel in return for his payment of the charter hire. by the Union moving for the Officers of AC Ransom and
ROSARIO Company to be held liable because although
Under the law, the cargo not loaded is considered a RANSOM had assumed a posture of suffering from
deadfreight. It is the amount paid by or recoverable from business reverse, its officers and principal stockholders
a charterer of a ship for the portion of the ship’s capacity
had organized a new corporation, the Rosario Industrial
the latter contracted for but failed to occupy. Explicit and
succinct is the law that the liability for deadfreight is on Corporation (thereinafter called ROSARIO), using the
the charterer. (Article 680 of the Code of Commerce). same equipment, personnel, business stocks and the
same place of business. AC Ransom used as a defense
2) No. Demurrage is the sum fixed in a charter party as a the clearance given by SEC to cease to operate due to
financial difficulties in order to lessen the award given by wrong, protect fraud, or defend crime, the law will regard
the court. It also declared that ROSARIO is a distinct the corporation as an association or persons, or, in the
and separate corporation, which was organized long case of two corporations, will merge them into one.
before these instant cases were decided adversely
against RANSOM. As to the officers and agents

Issue/s: The inclusion of the officers and agents was but proper
since a corporation, as an artificial being, can act only
 Whether or not ROSARIO Company should be through them.
held liable for the claims of AC Ransom Labor
Union?
 Whether or not the officers and directors of AC
VILLANUEVA VS ADRE
Ransom should be held liable for backwages?

Held:
FACTS:
The questioned Decision of the National Labor
Relations Commission is SET ASIDE, and the Order of
Labor Arbiter Tito F. Genilo of March 11, 1980 is
reinstated with the modification that Rosario Industrial The case began from a complaint, dated January 6,
Corporation and its officers and agents are hereby held 1977, for recovery of unpaid thirteenth-month pay filed
jointly and severally liable with the surviving private by the Sarangani Marine and General Workers Union-
respondents for the payment of the backwages due the ALU with the Department of Labor (Regional Office No.
22 union members. XI, General Santos City) against the South Cotabato
Integrated Port Services, Inc. (SCIPSI), a Philippine
Rosario Industrial Corporation is hereby ordered to corporation. Later, thirty-seven SCIPSI employees, non-
reinstate the 22 union members or, if this is not possible, union members apparently, filed their own complaint.
to award them separation pay equivalent at least to one The labor arbiter consolidated the twin complaints and
(1) month pay or to one (1) month salary for every year after hearing, ordered a dismissal on December 29,
of service actually rendered by them with A.C. Ransom 1977. On appeal, however, the National Labor Relations
(Phils). Corporation, whichever is higher. Commission, on June 9, 1981, reversed and
accordingly, ordered the private respondents, SCIPSI
Rosario Company is held liable because the
and its president and general, Lucio Velayo, to pay the
organization of a "run-away corporation," ROSARIO, in
thirteenth-month pays demanded. The private
1969 at the time the unfair labor practice case was
respondents' motion for reconsideration was denied, and
pending before the CIR by the same persons who were
the decision has since attained finality.
the officers and stockholders of RANSOM, engaged in
the same line of business as RANSOM, producing the
same line of products, occupying the same compound,
using the same machineries, buildings, laboratory, Thereafter, the parties, on orders of the labor arbiter,
bodega and sales and accounts departments used by were made to appear before a corporate auditing
RANSOM, and which is still in existence. Both examiner to determine the private respondents' exact
corporations were closed corporations owned and liability. On October 24, 1986, the corporate auditing
managed by members of the same family. Its examiner submitted an accounting and found the private
organization proved to be a convenient instrument to respondents liable in the total sum of Pl,134,000.00.
avoid payment of backwages and the reinstatement of Thereupon, the private respondents interposed an
the 22 workers. This is another instance where the objection and prayed for a revision. It appears, however,
fiction of separate and distinct corporate entities should that the private respondents never pursued their
be disregarded. exceptions.

It is very obvious that the second corporation


seeks the protective shield of a corporate fiction whose
veil in the present case could, and should, be pierced as On January 16,1987, the union moved for execution and
it was deliberately and maliciously designed to evade its pursuant thereto, the labor arbiter issued a writ of
financial obligation to its employees.... When a notion of execution. As a result, the sheriff levied on two parcels
legal entity is used to. defeat public convenience, justify
of land, both registered in Lucio Velayo's name, with an defense of his case.
area of 400 and 979 square meters.

In A.C. Ransom Labor Union-CCLU v. NLRC 12 we


On February 16,1987, Velayo alone filed a petition with held that in case of corporations. It is the president
the respondent court (Special Case No. 227) on a cause who responds personally for violation of the labor
of action based on an alleged irregular execution, on the pay laws. We quote:
ground that he "was never a party to the labor case" 3
and that "a corporation (that is, SCIPSI has a separate Article 273 of the Code provides that:
and distinct personality from this incorporators,
Any person violating any of the provisions of Article
stockholders and officers."
265 of this Code shall be punished by a fine of not
exceeding five hundred pesos and/or imprisonment
for not less than one (1) day nor more than six (6)
On February 17, 1987, the respondent court issued a months.
temporary restraining order enjoining execution of the
judgment in the aforementioned labor cases. On March (b) How can the foregoing provisions be
5, 1987, the petitioner moved for dismissal for lack of implemented when the employer is a corporation?
jurisdiction and litis pendentia whichwas denied by the The answer is found in Article 212 (c) of the Labor
Court. Code which provides:

(c) 'Employer' of the Labor Code which provides:


which 'Employer' includes any person acting in the
Meanwhile, on April 27,1988, the parties (in G.R. Nos. interest of an employer, directly or indirectly. The
7730001) submitted a Compromise Agreement whereby term shall not include any labor organization or any
the private respondents agreed to pay, in installments, of its officers or agents except when acting as
the reduced sum of P637,400.00 to the workers. On May employer.
11, 1988, we issued a Resolution approving the
Compromise Agreement, and considering the cases
(G.R. Nos. 77300-01) closed and terminated.
The responsible officer of an employer corporation
can be held personally, not to say even criminally,
liable for non-payment of back wages. That is the
policy of the law. In the Minimum Wage Law, Section
15(b) provided:
ISSUE: WON the courts can enjoin execution of
judgment rendered by the National Labor Relations (b) If any violation of this Act is committed by a
Commission on the basis that Velayo is a stranger to the corporation, trust, partnership or association, the
proceedings? manager or in his default, the person acting as such
when the violation took place, shall be responsible.
In the case of a government corporation, the
managing head shall be made responsible, except
HELD:
when shown that the violation was due to an act or
commission of some other person, over whom he
has no control, in which case the latter shall be held
The respondent court must be held to be in error responsible.
assuming jurisdiction over Special Case No. 227. It is
well-established that the courts cannot enjoin execution
of judgment rendered by the National Labor Relations
Accordingly, Velayo cannot be excused from payment of
Commission.
SCIPSI's liability by mere reason of SCIPSI's separate
corporate existence. The theory of corporate entity, in
the first place, was not meant to promote unfair
First, and as pointed out by the Solicitor General, and as objectives or otherwise, to shield them. This Court has
the records will amply show, he, Velayo, was a party to not hesitated in penetrating the veil of corporate fiction
the proceedings below where he took part actively in
14
when it would defeat the ends envisaged by law, not On September 12, 1986, a local union election was held
to mention the clear decree of the Labor Code. under the auspices of the ULGWP wherein the herein
petitioner, Beda Magdalena Villanueva, and the other
union officers were proclaimed as winners

The instant case has been rendered moot and academic On March 21, 1987, a Petition for Impeachment was
by reason of the out-of-court settlement between the filed with the national federation ULGWP by the defeated
parties candidates in the aforementioned election.

On April 17, 1988, the local union held a general


membership meeting at the Caruncho Complex in Pasig.
G.R. No. 113907 February 28, 2000 Several union members failed to attend the meeting,
prompting the Executive Board to create a committee
tasked to investigate the non-attendance of several
MALAYANG SAMAHAN NG MGA MANGGAGAWA SA union members in the said assembly, pursuant to
M. GREENFIELD (MSMG-UWP Sections 4 and 5, Article V of the Constitution and By-
vs. Laws of the union, which read:
HON. CRESENCIO J. RAMOS, NATIONAL LABOR
RELATIONS COMMISSION, M. GREENFIELD (B),
Seksyon 4. Ang mga kinukusang hindi pagdalo o
INC., SAUL TAWIL, CARLOS T. JAVELOSA, RENATO
hindi paglahok sa lahat ng hakbangin ng unyon
C. PUANGCO, WINCEL LIGOT, MARCIANO HALOG,
GODOFREDO PACENO, SR., GERVACIO ng sinumang kasapi o pinuno ay maaaring
CASILLANO, LORENZO ITAOC, ATTY. GODOFREDO maging sanhi ng pagtitiwalag o pagpapataw ng
PACENO, JR., MARGARITO CABRERA, GAUDENCIO multa ng hindi hihigit sa P50.00 sa bawat araw
RACHO, SANTIAGO IBANEZ, AND RODRIGO na nagkulang.
AGUILING, respondents.
Seksyon 5. Ang sinumang dadalo na aalis ng
Facts: hindi pa natatapos ang pulong ay ituturing na
pagliban at maparusahan itong alinsunod sa
Article V, Seksyong 4 ng Saligang Batas na ito.
The petitioner, Malayang Samahan ng mga Sino mang kasapi o pisyales na mahuli and
Manggagawa sa M. Greenfield, Inc., (B) (MSMG), dating sa takdang oras ng di lalampas sa isang
hereinafter referred to as the "local union", is an affiliate oras ay magmumulta ng P25.00 at babawasin
of the private respondent, United Lumber and General sa sahod sa pamamagitan ng salary deduction
Workers of the Philippines (ULGWP), referred to as the at higit sa isang oras ng pagdating ng huli ay
"federation". ituturing na pagliban.3

An agreement made and entered into by and between: On June 27, 1988, the local union wrote respondent
company a letter requesting it to deduct the union fines
M. GREENFIELD, INC. (B) a corporation duly from the wages/salaries of those union members who
organized in accordance with the laws of the failed to attend the general membership meeting.
Republic of the Philippines with office address at
Km. 14, Merville Road, Parañaque, Metro The Secretary General of the national federation,
Manila, represented in this act by its General Godofredo Paceño, Jr. disapproved the resolution of the
manager, Mr. Carlos T. Javelosa, hereinafter local union imposing the P50.00 fine. The union officers
referred to as the Company; protested such action by the Federation in a Reply dated
July 4, 1988.
-and-
The imposition of P50.00 fine became the subject of
MALAYANG SAMAHAN NG MGA bitter disagreement between the Federation and the
MANGGAGAWA SA M. GREENFIELD (B) local union culminating in the latter's declaration of
(MSMG)/UNITED LUMBER AND GENERAL general autonomy from the former through Resolution
WORKERS OF THE PHILIPPINES (ULGWP), a No. 10 passed by the local executive board and ratified
legitimate labor organization with address at by the general membership on July 16, 1988.
Suite 404, Trinity Building, T. M. Kalaw Street,
Manila, represented in this act by a Negotiating In retaliation, the national federation asked respondent
Committee headed by its National President, Mr. company to stop the remittance of the local union's
Godofredo Paceno, Sr., referred to in this share in the education funds effective August 1988. This
Agreement as the UNION.1 was objected to by the local union which demanded that
the education fund be remitted to it in full.
The company was thus constrained to file a Complaint (d) Giving ULGWP a period of five (5) days to
for Interpleader with a Petition for Declaratory Relief with cease and desist from further committing acts of
the Med-Arbitration Branch of the Department of Labor coercion, intimidation and harassment.8
and Employment, docketed as Case No. OD-M-8-435-
88. This was resolved on October 28, 1988, by Med- However, as early as November 21, 1988, the officers
Arbiter Anastacio Bactin in an Order. were expelled from the ULGWP.

On appeal, Director Pura-Ferrer Calleja issued a On the same day, the federation advised respondent
Resolution dated February 7, 1989, which modified in company of the expulsion of the 30 union officers and
part the earlier disposition. demanded their separation from employment pursuant to
the Union Security Clause in their collective bargaining
Meanwhile, on September 2, 1988, several local unions agreement. This demand was reiterated twice, through
(Top Form, M. Greenfield, Grosby, Triumph letters dated February 21 and March 4, 1989,
International, General Milling, and Vander Hons respectively, to respondent company.
chapters) filed a Petition for Audit and Examination of
the federation and education funds of ULGWP which Thereafter, the Federation filed a Notice of Strike with
was granted by Med-Arbiter Rasidali Abdullah on the National Conciliation and Mediation Board to compel
December 25, 1988 in an Order which directed the audit the company to effect the immediate termination of the
and examination of the books of account of ULGWP. expelled union officers.

On September 30, 1988, the officials of ULGWP called a Under the pressure of a threatened strike, respondent
Special National Executive Board Meeting at Nasipit, company terminated the 30 union officers from
Agusan del Norte where a Resolution was passed employment, serving them identical copies of the
placing the MSMG under trusteeship and appointing termination letter.
respondent Cesar Clarete as administrator.
On that same day, the expelled union officers assigned
On October 27, 1988, the said administrator wrote the in the first shift were physically or bodily brought out of
respondent company informing the latter of its the company premises by the company's security
designation of a certain Alfredo Kalingking as local union guards. Likewise, those assigned to the second shift
president and "disauthorizing" the incumbent union were not allowed to report for work. This provoked some
officers from representing the employees. This action by of the members of the local union to demonstrate their
the national federation was protested by the petitioners protest for the dismissal of the said union officers. Some
in a letter to respondent company dated November 11, union members left their work posts and walked out of
1988. the company premises.

On November 13, 1988, the petitioner union officers On the other hand, the Federation, having achieved its
received identical letters from the administrator requiring objective, withdrew the Notice of Strike filed with the
them to explain within 72 hours why they should not be NCMB.
removed from their office and expelled from union
membership.
The petitioners filed a Notice of Strike with the NCMB,
DOLE, Manila, docketed as Case No. NCMB-NCR-NS-
Petitioners replied: 03-216-89, alleging the following grounds for the strike:

(a) Questioning the validity of the alleged (a) Discrimination


National Executive Board Resolution placing
their union under trusteeship;
(b) Interference in union activities

(b) Justifying the action of their union in (c) Mass dismissal of union officers and shop
declaring a general autonomy from ULGWP due
stewards
to the latter's inability to give proper educational,
organizational and legal services to its affiliates
and the pendency of the audit of the federation (d) Threats, coercion and intimidation
funds;
(e) Union busting
(c) Advising that their union did not commit any
act of disloyalty as it has remained an affiliate of A strike vote referendum was conducted and out of 2,
ULGWP; 103 union members who cast their votes, 2,086
members voted to declare a strike.
On March 10, 1989, the thirty (30) dismissed union The First Division affirmed the Labor Arbiter's
officers filed an urgent petition, docketed as Case No. disposition. With the denial of their motion for
NCMB-NCR-NS-03-216-89, with the Office of the reconsideration on January 28, 1994, petitioners
Secretary of the Department of Labor and Employment elevated the case to this Court, attributing grave abuse
praying for the suspension of the effects of their of discretion to public respondent NLRC in:
termination from employment. However, the petition was
dismissed by then Secretary Franklin Drilon which held Issue:
that the dispute is purely an intra-union matter.
W/N respondent company was justified in dismissing
A total of 78 union shop stewards were placed under petitioner employees merely upon the labor federation's
preventive suspension by respondent company. This demand for the enforcement of the union security clause
prompted the union members to again stage a walk-out embodied in their collective bargaining agreement.
and resulted in the official declaration of strike at around
3:30 in the afternoon of March 14, 1989. The strike was Ruling:
attended with violence, force and intimidation on both
sides resulting to physical injuries to several employees,
both striking and non-striking, and damage to company The Court is of the opinion, and so holds, that
properties. respondent company officials cannot be held personally
liable for damages on account of the employees'
dismissal because the employer corporation has a
The employees who participated in the strike and
personality separate and distinct from its officers who
allegedly figured in the violent incident were placed
merely acted as its agents.
under preventive suspension by respondent company.
The company also sent return-to-work notices to the
home addresses of the striking employees thrice It has come to the attention of this Court that the 30-day
successively, on March 27, April 8 and April 31, 1989, prior notice requirement for the dismissal of employees
respectively. However, respondent company admitted has been repeatedly violated and the sanction imposed
that only 261 employees were eventually accepted back for such violation enunciated in Wenphil Corporation
to work. Those who did not respond to the return-to-work vs. NLRC32has become an ineffective deterrent. Thus,
notice were sent termination letters. the Court recently promulgated a decision to reinforce
and make more effective the requirement of notice and
hearing, a procedure that must be observed before
The petitioners filed a verified complaint with the
termination of employment can be legally effected.
Arbitration Branch, National Capital Region, DOLE,
Manila, docketed as Case No. NCR-00-09-04199-89,
charging private respondents of unfair labor practice In Ruben Serrano vs. NLRC and Isetann Department
which consists of union busting, illegal dismissal, illegal Store (G.R. No. 117040, January 27, 2000), the Court
suspension, interference in union activities, ruled that an employee who is dismissed, whether or not
discrimination, threats, intimidation, coercion, violence, for just or authorized cause but without prior notice of his
and oppression. termination, is entitled to full backwages from the time he
was terminated until the decision in his case becomes
final, when the dismissal was for cause; and in case the
The complaint for unfair labor practice was assigned to dismissal was without just or valid cause, the backwages
Labor Arbiter Manuel Asuncion but was thereafter
shall be computed from the time of his dismissal until his
reassigned to Labor Arbiter Cresencio Ramos when
actual reinstatement. In the case at bar, where the
respondents moved to inhibit him from acting on the
requirement of notice and hearing was not complied
case.
with, the aforecited doctrine laid down in the Serrano
case applies.
On December 15, 1992, finding the termination to be
valid in compliance with the union security clause of the
collective bargaining agreement, Labor Arbiter
Cresencio Ramos dismissed the complaint.
FRANCISCO V. DEL
ROSARIO
Petitioners then appealed to the NLRC. During its vs.
pendency, Commissioner Romeo Putong retired from NATIONAL LABOR RELATIONS COMMISSION
the service, leaving only two commissioners, G.R. No. 85416. July 24, 1990
Commissioner Vicente Veloso III and Hon. Chairman
Bartolome Carale in the First Division. When FACTS:
Commissioner Veloso inhibited himself from the case,
Commissioner Joaquin Tanodra of the Third Division In POEA Case No. 85-06-0394, the POEA
was temporarily designated to sit in the First Division for promulgated a decision dismissing the complaint for
the proper disposition of the case. money claims for lack of merit. The decision was
appealed to the NLRC, which reversed the POEA
decision and ordered Philsa Construction and Trading
Co., Inc., the recruiter and Arieb Enterprises, the foreign undertaking required by the rules of the POEA, together
employer to jointly and severally pay private respondent with the filing of cash and surety bonds, in order to
their salary differentials and vacation leave benefits. ensure that overseas workers shall find satisfaction for
A writ of execution was issued by the POEA but awards in their favor.
it was returned unsatisfied as Philsa was no longer
operating and was financially incapable of satisfying the
judgment. Private respondent moved for the issuance of G.R. No. 79907 March 16, 1989
an alias writ against the officers of Philsa. This motion
was opposed by the officers, led by petitioner, the
president and general manager of the corporation. SAMUEL CASAS LIM, petitioner,
Petitioner appealed to the NLRC. On September vs.
23, 1988, the NLRC dismissed the appeal on the theory THE NATIONAL LABOR RELATIONS COMMISSION
that the corporate personality of Philsa should be and VICTORIA R. CALSADO, respondents.
disregarded. According to the NLRC, Philsa
Construction & Trading Co., Inc. and Philsa International FACTS:
Placement & Services Corp are one and the same
because both corporations has the same set of directors These two cases have been consolidated because they
and officers. Petitioner's motion for reconsideration was relate to the same factual antecedents and the same
denied. private respondent. The issues are:
Thus, this petition was filed, alleging that the NLRC
gravely abused its discretion. 1. In G.R. No. 79975, whether or not the private
respondent was an employee of the petitioner and, if
ISSUE: so, had been illegally dismissed; and corollarily,
whether or not the NLRC had jurisdiction over their
Whether or not the NLRC acted with grave dispute.
abuse of discretion.
2. In G.R. No. 79907, whether or not the petitioner
RULING:
could be held solidarity liable with Sweet Lines, Inc.
to the private respondent.
YES.

Under the law a corporation is bestowed juridical Private respondent Victoria Calsado was hired by Sweet
personality, separate and distinct from its stockholders. Lines, Inc. as Senior Branch Officer of its International
But when the juridical personality of the corporation is Accounts Department for a fixed salary and a stipulated
used to defeat public convenience, justify wrong, protect 5 % commission on sales production.
fraud or defend crime, the corporation shall be
considered as a mere association of persons and its After tendering her resignation to accept another offer of
responsible officers and/or stockholders shall be held employment, she was persuaded to remain with an offer
individually liable. For the same reasons, a corporation of her promotion to Manager of the Department with
shall be liable for the obligations of a stockholder, or a corresponding increase in compensation, which she
corporation and its successor-in-interest shall be accepted. She was also allowed to buy a second-hand
considered as one and the liability of the former shall Colt Lancer pursuant to a liberal car plan under which
attach to the latter. one-half of the cost was to be paid by the company and
But for the separate juridical personality of a the other half was to be deducted from her salary.
corporation to be disregarded, the wrongdoing must be
clearly and convincingly established. It cannot be Relations began to sour later, however, when she
presumed. Thus, at the time Philsa allowed its license to repeatedly asked for payment of her commissions, which
lapse in 1985 and even at the time it was delisted in had accumulated and were long overdue. She also
1986, there was yet no judgment in favor of private complained of the inordinate demands on her time even
respondent. An intent to evade payment of his claims when she was sick and in the hospital.
cannot therefore be implied from the expiration of
Philsa's license and its delisting. Likewise, substantial She was served with a letter from Samuel Casas Lim,
identity of the incorporators of the two corporations does the other petitioner, informing her that her "employment
not necessarily imply fraud. with Sweet Lines" would terminate on August 5, 1985.
In this case, not only has there been a failure to Efforts were also taken by Sweet Lines to forcibly take
establish fraud, but it has also not been shown that the car from her, culminating in an action for replevin
petitioner is the corporate officer responsible for private against her in the regional trial court of Manila.
respondent's predicament. It must be emphasized that
the claim for differentials and benefits was actually
On August 14, 1985, Calsado filed a complaint against
directed against the foreign employer. Philsa became
both petitioners for illegal dismissal, illegal deduction,
liable only because of its undertaking to be jointly and
and unpaid wages and commissions plus moral and
severally bound with the foreign employer, an
exemplary damages, among other claims.
The respondents' defenses were based mainly on the The case of Ransom v. NLRC is not in point because
claim that Calsado was not an employee of Sweet Lines there the debtor corporation actually ceased operations
but an independent contractor and that therefore their after the decision of the Court of Industrial Relations was
dispute with her came under the jurisdiction of the civil promulgated against it, making it necessary to enforce it
courts and not of the Labor Arbiter. against its former president. Sweet lines is still existing
and able to satisfy the judgment in favor of the private
ISSUE: respondent.

Whether or not Victoria Calsado is an employee of The Solicitor General, invoking equity rather than law,
Sweet Lines. observes that making Lim solidarity liable with Sweet
Lines will ensure payment of Calsado's claim. But this
Whether or not Samuel Lim can be held personally precaution, even assuming it to be valid, is really
liable with Sweet Lines for signing the separation unnecessary. In fact, as a condition for the issuance of
letter. our temporary restraining order of October 14, 1987,
Sweet Lines posted as required a bond in the amount of
P850,000.00, which should cover the amounts awarded
RULING: to the private respondent.

The employee-employer relations between Calsado and


Sweet Lines have been sufficiently established. The
petitioner cannot now argue that the grant to her of the G.R. No. 78345 September 21, 1990
13th month pay and even the differential pay was a mere
accomodation like the car plan (which, for that matter, is JOSE M. MAGLUTAC, petitioner, vs.
a benefit usually extended only to employees).
NATIONAL LABOR RELATIONS COMMISSION,
Such relationship having been established, the third COMMART (PHIL.), INC. AND JESUS T. MAGLUTAC,
issue is automatically resolved and requires not much
respondents.
elaboration. Suffice it only to stress that the damages
claimed by private respondent as a result of her
G.R. No. 78637 September 21, 1990
illegal dismissal and the violation of the terms and
conditions of her employment also come within the
COMMART (PHIL.), INC., petitioner, vs.
jurisdiction of the Labor Arbiter as a contrary rule
would result in the splitting of actions and the
NATIONAL LABOR RELATIONS COMMISSION AND
consequent multiplication of suits.
JOSE M. MAGLUTAC, respondents.
On the fourth issue, we agree with petitioner Lim that he
Facts: Jose M. Maglutac, petitioner in G.R. No. 78345
cannot be held personally liable with Sweet Lines for
merely having signed the letter informing Calsado of her (hereinafter referred to as complainant) was employed
separation. There is no evidence that he acted with by Commart (Phils.), Inc. (hereinafter referred to as
malice or bad faith. The letter, in fact, informed her Commart) rose to become the Manager of its Energy
not only of her separation but also of the benefits Equipment Sales. He received a notice of termination
due her as a result of the termination of her services. signed by Joaquin S. Cenzon, Vice-President-General
Manager and Corporate Secretary of CMS International,
It is true that Lim has raised this matter rather tardily and a corporation controlled by Commart.
also that he belongs to a closed corporation controlled
by the members of one family only. But these Thereafter, Jose Maglutac filed a complaint for illegal
circumstances should not be allowed to operate against
dismissal against Commart and Jesus T. Maglutac,
him if he is to be accorded substantial justice in the
resolution of the private respondent's claim. As we said President and Chairman of the Board of Directors of
in Ortigas vs. Lufthansa German Airlines, the Court is Commart. The complainant alleged that his dismissal
"clothed with ample authority to review matters, even if was part of a vendetta drive against his parents who
they are not assigned as errors in the appeal, if it finds dared to expose the massive and fraudulent diversion of
that its consideration is necessary in arriving at a just company funds to the company president's private
decision of the case." As for the second charge, the accounts, stressing that complainant's efficiency and
mere fact that Lim is part of the family corporation
does not mean that all its acts are imputable to him effectiveness were never put to question when very
directly and personally. His acts were official acts, suddenly he received his notice of termination.
done in his capacity as Vice President of Sweet
Lines and on its behalf. There is no showing that he Commart and Jesus T. Maglutac, on the other hand,
acted without or in excess of his authority or was justified the dismissal for lack of trust and confidence
motivated by personal ill-will toward Calsado. brought about by complainant and his family's
establishment of a company, MM International, in direct recovery of unpaid wages for the period November 15,
competition with Commart. the Labor Arbiter rendered a 1984 up to March 31, 1985. The Labor Arbiter ordered to
decision finding that complainant was illegally dismissed. pay the complainant which affirmed by the NLRC. The
petitioner now, in this petition, argues that he cannot be
Commart and Jesus T. Maglutac filed a motion for held personally liable for the unpaid wages of the private
respondent. He contends that the corporation alone,
reconsideration. The NLRC affirmed the finding of the
which has a personality separate and distinct from its
Labor Arbiter that complainant was illegally dismissed by members or stockholders, should be the one held liable
Commart but it deleted the award for moral and for corporate obligations.
exemplary damages in favor of complainant and
absolved Jesus T. Maglutac from any personal liability to
the complainant. Issue:
W/N the vice-president be held jointly and
Both parties filed their respective motions for severally liable with the corporation for the unpaid wages
reconsideration of the decision of the NLRC but both of the company's former president?
were denied. Hence, the instant petitions both alleging
Ruling:
grave abuse of discretion on the part of respondent Yes.
NLRC. In the instant case, it was correct for the private
respondent to have impleaded the petitioner in the
Issue: Whether or not respondent Maglutac should be complaint considering that the latter was the highest and
held jointly and severally liable with Commart? most ranking official of the corporation after the private
respondent had resigned. Certainly, there should be an
Ruling: Yes, The responsible officer of an employer officer directly responsible for the failure to pay the
corporation can be held personally, not to say even wages of the corporation's president. In this case, such
criminally, liable for non-payment of backwages. In the officer happened to be the vice-president.
We resolve the issue in the light of the precedent
case of Chua v. NLRC, G.R. 81450, Feb. 15, 1990,
set in the case of A.C. Ransom Labor Union-CCLU v.
citing the case of A.C. Ransom Labor Union-CCLU v. National Labor Relations Commission (142 SCRA 269
NLRC, 142 SCRA 269, We affirmed the finding of the [1986]). In this case, the Court set aside the decision of
Labor Arbiter and the NLRC that the vice-president of a the NLRC upholding the personal non-liability of the
corporation who was the most ranking officer of the individual officers and agents of the corporation unless
corporation can be held jointly and severally liable with they have acted beyond the scope of their authority. In
the corporation for the payment of the unpaid wages of thus reversing in the NLRC decision, the Court ruled that
the president or presidents of the corporation may be
its president. It was held:
held liable for the corporation's obligations to its workers.
The Court explained:
(c) Employer includes any person acting in the interest of
an employer directly or indirectly. The term shall not
We resolve the issue in the light of the precedent set in include any labor organization or any of its officers or
the case of A.C. Ransom Labor Union-CCLU v. National agents except when acting as employer.
Labor Relations Commission (142 SCRA 269 [1986]). In . . . Since RANSOM is an artificial person, it must have
this case, the Court set aside the decision of the NLRC an officer who can be presumed to be the employer,
upholding the personal non-liability of the individual being the 'person acting in the interest of employer.'
officers and agents of the corporation unless they have RANSOM. The Corporation, only in the technical sense,
is the employer.
acted beyond the scope of their authority. In thus
The responsible officer of an employer corporation can
reversing the NLRC decision, the Court ruled that the be held personally, not to say even criminally, liable for
president or presidents of the corporation may be held non-payment of backwages. ... (At p. 273-274; Emphasis
liable for the corporations's obligations to its workers. supplied)
xxx xxx xxx
This Court continued:
xxx xxx xxx
Chua VS. NLRC (d) The record does not clearly Identify the officer or
Facts: officers of RANSOM directly responsible for failure to
City Air International Brokerage Corporation is a family pay the backwages of the 22 strikers. In the absence of
corporation with the private respondent, Jesus Chua, definite proof in that regard, we believe it should be
serving as its president from October, 1984 up to the presumed that the responsible officer is the President of
time of his resignation in March, 1985 while the the corporation who can be deemed the chief operation
petitioner, Johnson Chua is the vice-president. Private officer thereof. Thus, in RA 602, criminal responsibility is
respondent filed a complaint with the National Labor with the manager or in his default, the person acting as
Relations Commission (NLRC) for illegal dismissal and
such.' In RANSOM, the President appears to be the FACTS:
Manager. (At p. 274)
Moreover, there are peculiar circumstances attendant to American Hospital Supplies was engaged in the
this case which point to the petitioner as the person also sale and manufacture of medicines and pharmaceuticals
directly responsible to the private respondent for his in the country and did substantial business with
salaries. These lead us further to sustain the decision of government hospitals. On 1 June 1970 it hired Alfonso
the NLRC. Bayani as an Area Manager for Visayas and Mindanao,
It is the general rule that findings of fact of quasi-judicial and later appointed him Manager of its Cebu branch. On
agencies which have acquired expertise because their 30 January 1978 private respondent was dismissed from
jurisdiction is confined to specific matters are accorded the service. At that time he was receiving a monthly
not only respect by this Court but at times even finality if compensation of P3,180.00.
such findings are supported by substantial evidence. On 5 May 1978 private respondent filed a
(Arica v. National Labor Relations Commission, G.R. No. complaint for damages before the trial court alleging that
78210, February 28, 1989; Johnson and Johnson Labor in the course of their business petitioners were directly
Union - FFW v. Director of Labor Relations, G.R. No. encouraging, abetting and promoting bribery in the guise
76427, February 21, 1989; Reyes v. Ministry of Labor, of "commissions," "entertainment expenses" and
G.R. No. L-48705, February 9, 1989). The instant case "representation expenses" which were given to various
is not an exception to the rule. There is substantial government hospital officials in exchange for favorable
evidence to support the decision. recommendations, approvals and actual purchases of
As found by the labor arbiter, the private respondent and medicines and pharmaceuticals. For his refusal to take
petitioner are brothers, serving as the president and direct and personal hand in giving "bribe money" he was
vice-president respectively of the family corporation. At dismissed. He then implicated AHS President Gervacio
the time of the filing of the complaint for illegal dismissal Amistoso and Vice President Constancio Halili as
and unpaid wages or commission by the private responsible for his illegal dismissal.
respondent the petitioner, as Vice-President, was the
highest and most ranking official of said corporation. He ISSUE:
caused the preparation and verified the position paper
dated August 6, 1985, the appeal memorandum dated Whether or not Amistoso and Halili be held
December 20, 1985, and the motion for the solidarily liable with the corporation.
reconsideration of the NLRC decision on August 10,
1987. He showed personal interest in the case of Jesus RULING:
Chua despite the fact that a new corporate president in
the person of Jose Beltran had been elected on May 20, NO.
1985. Beltran could have handled this case in .behalf of
the corporation but he did not do so. Since the records Corporate officers are not personally liable for
fail to show that the petitioner was authorized by the money claims of discharged corporate employees unless
corporation to pursue or defend the case filed against it they acted with evident malice and bad faith in
by the private respondent and yet the petitioner terminating their employment. In the case at bar, while
personally acted in the case and showed keen interest in petitioners Amistoso and Halili may have had a hand in
its progress, he must be held responsible for its the relief of respondent. Bayani, there are no indications
outcome. The records show that personal animosity of malice and bad faith on their part. We take exception
existed between these two (2) brothers. With the to the conclusion of respondent Court of Appeals that
petitioner's manifest interest in the case and his being "the manner by which Halili and Amistoso acted is
the top officer after his brother was eased out, there is characterized by bad faith and malice, thus binding them
enough reason to believe that the petitioner had a hand personally liable to plaintiff-appellee,'' On the contrary it
in the dismissal of the private respondent. We, therefore is apparent that the relief order was a business judgment
see no reversible error committed by the NLRC in on the part of the officers, with the best interest of the
applying Article 289 of the Labor Code which provides: corporation in mind, based on their opinion that
Art. 289. Who are liable when committed by other than respondent Bayani had failed to perform the duties
natural person. — If the offense is committed by a expected of him. Hence both the trial court and
corporation, trust, firm, partnership, association or any respondent Court of Appeals committed a reversible
other entity, the penalty shall be imposed upon the guilty error in holding petitioners Amistoso and Halili jointly and
officer or officers of such corporation, trust, firm, solidarily liable with Petitioner Corporation.
partnership, association or entity.
REAHS CORP., SEVERO CASTULO, et al.
vs.
NLRC, BONIFACIO RED, et al.
AMERICAN HOSPITAL SUPPLIES/PHILIPPINES et al. GR 117473, 15 April 1997
vs.
COURT OF APPEALS, ALFONSO BAYANI FACTS:
GR 111807, 14 June 1996
Private respondents sued Reahs Corp. for unfair to perform functions at the push of a button. The issue,
labor practice and illegal dismissal. They claim that they however, is not limited to payment of separation pay
were unlawfully dismissed and were not awarded nor under Article 283 but also payment of labor standard
given any separation pay. benefits such as underpayment of wages, holiday pay
On the other hand, respondents allege that and 13th month pay to two of the private respondents.
sometime in 1986, a certain Ms Soledad Domingo, the While there is no sufficient evidence to conclude that
sole proprietress and operator of Rainbow Sauna petitioners have indiscriminately stopped the entity's
located at 316 Araneta Avenue, Quezon City, offered to business, at the same time, petitioners have opted to
sell her business to respondent Reah's Corporation After abstain from presenting sufficient evidence to establish
the sale, all the assets of Ms Domingo were turned over the serious and adverse financial condition of the
to respondent Reah's, which put a sing-along coffee company.
shop and massage clinic; that complainant Red started
his employment on the first week of December 1988 as
a room boy at P50.00/day and was given living quarters
inside the premises as he requested; that sometime in G.R. No. 121434 June 2, 1997
March 1989, complainant Red asked permission to go to
Bicol for a period of ten (10) days, which was granted,
and was given an advance money of P1,200.00 to bring ELENA F. UICHICO, SAMUEL FLORO, VICTORIA F.
some girls from the province to work as attendants at the BASILIO, petitioners,
respondent's massage clinic, that it was only on January vs.
1, 1990 that complainant Red returned and was re-hired NATIONAL LABOR RELATIONS COMMISSION,
under the same terms and conditions of his previous LUZVIMINDA SANTOS, SHIRLEY PORRAS, CARMEN
employment with the understanding that he will have to ELIZARDE, ET. AL., respondents.
refund the P1,200.00 cash advance given to him; that
due to poor business, increase in the rental cost and the
failure of Meralco to reconnect the electrical services in
the establishment, it suffered losses leading to its Facts:
closure.
The NLRC ruled in favor of respondents. Private respondents were employed by Crispa, Inc. for
Together with the corporation, the NLRC also held many years in the latter's garments factory located in
Castulo, Romeo Pascua, and Daniel Valenzuela Pasig Boulevard, Pasig City.
solidarily liable due to their capacity as Chairman, Board
Member and Accountant, and Acting Manager,
Sometime in September, 1991, private respondents'
respectively.
services were terminated on the ground of retrenchment
due to alleged serious business losses suffered by
ISSUE:
Crispa, Inc. in the years immediately preceding 1990.
Whether or not Pascua, Castulo, and
Valenzuela, may be held liable. Thereafter, respondent employees filed before the
NLRC, National Capital Region, Manila, three (3)
RULING: separate complaints for illegal dismissal and diminution
of compensation against Crispa, Inc., Valeriano Floro,
YES. and the petitioners.

They acted in bad faith in dismissing the Valeriano Floro was a major stockholder, incorporator
respondents. As a general rule established by legal and Director of Crispa, Inc., while the petitioners were
fiction, the corporation has a personality separate and high ranking officers and directors of the company. Said
distinct from its officers, stockholders and members. complaints were consolidated in order to expedite the
Hence, officers of a corporation are not personally liable proceedings. The case was assigned to Labor Arbiter
for their official acts unless it is shown that they have Raul Aquino.
exceeded their authority. This fictional veil, however, can
be pierced by the very same law which created it when Labor Arbiter Aquino rendered a decision dismissing the
"the notion of the legal entity is used as a means to complaints for illegal dismissal but at the same time
perpetrate fraud, an illegal act, as a vehicle for the ordering Crispa, Inc., Floro and the petitioners to pay
evasion of an existing obligation, and to confuse respondent employees separation pay equivalent to
legitimate issues". Under the Labor Code, for instance, seventeen (17) days for every year of service.
when a corporation violates a provision declared to be
penal in nature, the penalty shall be imposed upon the Dissatisfied, private respondents appealed before the
guilty officer or officers of the corporation. public respondent NLRC. In a Resolution, the Second
In the case at bar, the thrust of petitioners' Division of the NLRC found Crispa, Inc., Valeriano Floro,
arguments was aimed at confining liability solely to the together with the petitioners liable for illegal dismissal,
corporation, as if the entity were an automaton designed
and modified the award of separation pay in the amount MANCAO SUPERMARKET, INC. AND/OR MANAGER,
of one (1) month for every year of service. ANTONIO MANCAO, respondents.

Petitioners filed a MR but the same was denied by the FACTS:


NLRC.
Petitioner, Emelita Nicario, was employed with
Private respondents sought a clarification of public respondent company, Mancao Supermarket, on June 6,
respondent NLRC's Resolution insofar as the 1986 as a salesgirl and was later on promoted as sales
computation of separation pay by the Examination and supervisor. However, private respondent terminated her
Computation Division was concerned as well as the services on February 7, 1989.
failure of the Resolution to award them full backwages
despite the finding of illegal dismissal. A complaint for illegal dismissal with prayer for
backwages, wage differential, service incentive leave
The NLRC, treating the Motion to Clarity Judgment as an pay, overtime pay, 13th month pay and unpaid wages
Appeal, granted the same. was filed by petitioner before the National Labor
Relations Commission,
Petitioners filed a Motion for Reconsideration of the April
21, 1995 Resolution, which was denied. In a decision dated May 23, 1994, Labor Arbiter
Macaraig-Guillen awarded petitioner's claims for unpaid
Hence, this petition. service incentive leave pay, 13th month pay, overtime
pay and rest day pay for the entire period of her-
Issue: W/N a director, trustee or officer be made employment, but dismissed her claims for holiday
personally liable for his corporate action. premium pay and unpaid salaries from February 3 to 5,
1989.
Ruling: Yes.
Not satisfied with the decision, private respondent
appealed to the NLRC, and in a resolution dated August
In labor cases, particularly, corporate directors and 16, 1995, the Commission affirmed in toto Labor Arbiter
officers are solidarily liable with the corporation for the Macaraig-Guillen's decision. Private respondent then
termination of employment of corporate employees done filed a motion for reconsideration. In a resolution dated
with malice or in bad faith. 23 In this case, it is undisputed December 21, 1995, public respondent NLRC modified
that petitioners have a direct hand in the illegal dismissal its earlier resolution by deleting the award for overtime
of respondent employees. They were the ones, who as pay and ruling that private respondent Antonio Mancao
high-ranking officers and directors of Crispa, Inc., signed is not jointly and severally liable with Mancao
the Board Resolution retrenching private respondents on Supermarket to pay petitioner the monetary award
the feigned ground of serious business losses that had adjudged.
no basis apart from an unsigned and unaudited Profit
and Loss Statement which, to repeat, had no evidentiary
value whatsoever. This is indicative of bad faith on the ISSUE:
part of petitioners for which they can be held jointly and
severally liable with Crispa, Inc. for all the money claims Whether or not Antonio Mancao is jointly and severally
of the illegally terminated respondent employees in this liable with respondent supermarket as to the monetary
case. award.

RULING:

As to the liability of private respondent Antonio Mancao,


petitioner contends that as manager of Mancao
establishment, he should be jointly and severally liable
with respondent corporation as to the monetary award
adjudged.

The general rule is that officers of a corporation are not


personally liable for their official acts unless it is shown
that they have exceeded their authority. However, the
G.R. No. 125340 September 17, 1998 legal fiction that a corporation has a personality separate
and distinct from stockholders and members may be
EMELITA NICARIO, petitioner, disregarded if it is used as a means to perpetuate fraud
vs. or an illegal act or as a vehicle for the evasion of an
NATIONAL LABOR RELATIONS COMMISSION, existing obligation, the circumvention of statutes, or to
confuse legitimate issues.
In this case, there is no showing that Antonio Mancao, Issue: Whether or not petitioners are personally liable to
as manager of respondent company, deliberately and private respondents?
maliciously evaded the respondent's company financial
obligation to the petitioner. Hence, there appearing to be Ruling: Yes. Assuming that indeed, the Restaurant
no evidence on record that Antonio Mancao acted Services Corporation was the owner of the Restaurante
maliciously or deliberately in the non-payment of benefits
Las Conchas and the employer of private respondents,
to petitioner he cannot he held jointly and severally liable
with Mancao supermarket. this will not absolve petitioners David Gonzales and
Elizabeth Anne Gonzales from their liability as corporate
officers. Although as a rule, the officers and members of
a corporation are not personally liable for acts done in
the performance of their duties, this rule admits of
exceptions, one of which is when the employer
G.R. No. 119085 September 9, 1999 corporation is no longer existing and is unable to satisfy
the judgment in favor of the employee, the officers
RESTAURANTE LAS CONCHAS and/or DAVID
should be held liable for acting on behalf of the
GONZALES, petitioners, vs.
corporation. Here, the corporation does not appear to
LYDIA LLEGO, SERGIO DANO, EDWARD ARDIANTE, exist anymore.
FEDERICO DE LA CRUZ, SHERILITA ANIEL, LORNA
In the present case, the employees can no longer claim
AZUELA, ZENAIDA HERMOCILLA, FELICIDAD
their separation benefits and 13th month pay from the
ROLDAN, HELEN MANALAYSAY, LUZ
corporation because it has already ceased operation. To
BALDELAMAR, FELICIDAD MENDOZA, DOLORES
require them to do so would render illusory the
BAQUIZO, RODOLFO BAS, CIRIACO BATITES, and
separation and 13th month pay awarded to them by the
THE HONORABLE NATIONAL LABOR RELATIONS
NLRC. Their only recourse is to satisfy their claim from
COMMISSION, respondents.
the officers of the corporation who were, in effect, acting
Facts: Private respondents were employees of petitioner in behalf of the corporation. It would appear that,
Restaurante Las Conchas which was allegedly operated originally, Restaurante Las Conchas was a single
by the Restaurant Services Corporation and by proprietorship put up by the parents of Elizabeth Anne
petitioners David Gonzales and Elizabeth Anne Gonzales, who together with her husband, petitioner
Gonzales who are members of the board of directors David Gonzales, later took over its management. Private
and officers of the corporation. respondents claim, and rightly so, that the former were
the real owners of the restaurant. The conclusion is
The Restaurant Services Corporation got involved in a bolstered by the fact that petitioners never revealed who
legal battle with the Ayala Land, Inc. over the land were the other officers of the Restaurant Services
allegedly being occupied by petitioners for their Corporation, if only to pinpoint responsibility in the
restaurant. closure of the restaurant that resulted in the dismissal of
the private respondents from employment. Petitioners
Ayala Land, Inc. obtained a favorable judgment in the David Gonzales and Elizabeth Anne Gonzales are,
case filed against Restaurant Services Corporation for therefore, personally liable for the payment of the
unlawful detainer and the latter were ordered to vacate separation and 13th month pay due to their former
the premises. The case was appealed to the Court of employees.
Appeals and ultimately to this Court which affirmed the
decision of the trial court.

Petitioners attempted to look for a suitable place for their Valderrama VS. NLRC
restaurant business at the Ortigas Center but to no avail, Facts:
thus, they shut down their business. This resulted in the Maria Andrea Saavedra, herein private
respondent, filed a complaint against the COMMODEX
termination of employment of private respondents.
(Phils.), Inc., petitioner Consuelo Valderrama as owner,
Tranquilino Valderrama as executive vice president and
Private respondents filed a complaint with the Labor
Jose Ma. Togle as vice president and general manager,
Arbiter for payment of separation pay and 13th month for reinstatement and backwages. On December 2,
pay. This was, however, dismissed by the Labor Arbiter 1986, the Labor Arbiter rendered a decision, finding
prompting the private respondents to appeal the case to private respondent to have been illegally dismissed and
the respondent NLRC. The NLRC rendered a Decision holding the respondent COMMODEX liable. It was
favorable to private respondents. shown that private respondent had been dismissed from
her employment due to her pregnancy, contrary to
allegations of petitioner and her co-respondents therein IN VIEW OF ALL THE FOREGOING, . . . the A. C.
that the termination of her employment was due to Ransom Philippine Corporation is guilty of unfair labor
redundancy and retrenchment. practice of interference and discrimination hereinabove
A writ of execution was granted, but it was held and specified, ordering its officers and agents to
returned unsatisfied. The sheriff reported that cease and desist from committing the same, finding the
COMMODEX had ceased operation, while the individual strike legal and justified; and to reinstate immediately . . .
officers, who were co-respondents in the case, took the to their respective positions with backwages from July
position that the writ could not be enforced against them 25, 1969 until actually reinstated, without loss of
on the ground that the dispositive portion of the decision seniority rights and other privileges appurtenant to their
mentioned only COMMODEX. employment.
Private respondent filed a Motion for Clarification A corporation can only act through its officers
in which she prayed that the body of the decision clearly and agents. That is why the cease and desist order was
held the petitioner and her co-respondents therein to be directed to the officers and agents of A. C. Ransom,
liable and that therefore, this Office is not precluded from which was actually found guilty of unfair labor practice.
correcting the inadvertence by clarifying the words But that case clearly also holds that any decision against
respondent company which ought to have been the company can be enforced against the officers in their
respondents jointly and severally in order to make the personal capacities should the corporation fail to satisfy
fallo or dispositive part correspond or correlate with the the judgment against it. The quoted portion of that
body of the final decision, considering that the unjust decision explaining the basis for such ruling makes that
dismissal of the complainant constitutes tort or quasi- clear. Agreeably with the ruling in A. C. Ransom Labor
delict. (Article 2176, New Civil Code). Petitioner and her Union-CCLU it was held in another case that where the
co-respondents therein filed an opposition to the motion Employer corporation is no longer existing and [is]
for clarification. They contended that the decision of the unable to satisfy the judgment in favor of the employee,
Labor Arbiter had become final and executory and could the officer should be held liable for acting on behalf of
no longer be amended. the corporation.
The Labor Arbiter, citing our ruling in A. C. Similarly it was held in Carmelcraft Corp. v.
Ransom Labor Union-CCLU v. NLRC, which held the NLRC:
president of a corporation responsible and personally We also find untenable the contention of Carmen Yulo
liable for payment of backwages, granted the private that she is not liable for the acts of the petitioner
respondents motion and set it for hearing for reception of company, assuming it had acted illegally, because the
evidence of the relationship of the petitioner and her Carmelcraft Corporation is a distinct and separate entity
corespondents therein to COMMODEX. Private with a legal personality of its own. Yulo claims she is
respondent then presented the Articles of Incorporation, only an agent of the company carrying out the decisions
List of Stockholders and the General Information Sheet of its board of directors. We do not agree. Our finding is
of COMMODEX, which showed that of the 2,000 shares that she is in fact and legal effect the corporation, being
of stocks of the corporation, Consuelo Valderrama not only its president and general manager but also its
owned 1,993 and that she was chairman of the board owner.
and president of respondent company. The NLRC In this case, the documents presented by the
affirmed the LA’s decision. Hence, this petition. private respondent show that petitioner controlled the
company owning 1,993 of its 2,000 shares, with the rest
Issue: of the stockholders owning only nominal amounts.
W/N Petitioner cannot and should not be held
personally liable jointly and severally with Commodex
(Phils.), Inc. for the awards adjudged in favor of herein GARCIA, petitioner
Private Respondent Saavedra. vs.
LIM CHU SING, respondent
Ruling: G.R. No. L-39427 February 24, 1934
No, petitioner can be and should be held liable.
First. The rule that once a judgment becomes FACTS:
final it can no longer be disturbed, altered, or modified is
not an inflexible one. It admits of exceptions, as where On June 20, 1930, the defendant-appellant Lim
facts and circumstances transpire after a judgment has Chu Sing executed and delivered to the Mercantile Bank
become final and executory which render its execution of China promissory note for the sum of P19,605.17 with
impossible or unjust. In such a case the modification of interest thereon at 6 per cent per annum, payable
the decision may be sought by the interested party and monthly as follows: P1,000 on July 1, 1930; P500 on
the court will modify and alter the judgment to harmonize August 1, 1930; and P500 on the first of every month
it with justice and the facts. thereafter until the amount of the promissory note
Petitioner seeks to distinguish that case from the together with the interest thereon is fully paid (Exhibit A).
one at bar on the ground that the dispositive portion of One of the conditions stipulated in said promissory note
the decision in that case actually ordered the officers is that in case of defendant's default in the payment of
and agents of A. C. Ransom to cease and desist from any of the monthly installments, as they become due,
committing further acts of certain labor practice thus: the entire amount or the unpaid balance thereof together
with interest thereon at 6 per cent per annum, shall Exchange Commission. From this order the Quezon
become due and payable on demand. The defendant College, Inc. has appealed.
had been, making several partial payments thereon,
leaving an unpaid balance of P9,105.17. However, he ISSUE:
defaulted in the payment of several installments by
reason of which the unpaid balance of P9,105.17 on the Whether or not the subscription applied for by
promissory note has ipso facto become due and Damasa Crisostomo is an enforceable contract.
demandable.
RULING:
ISSUE:
NO.
Whether or not it is proper to compensate the
defendant-appellant's indebtedness of P9,105.17, which It appears that the application sent by Damasa
is claimed in the complaint, with the sum of P10,000 Crisostomo to the Quezon College, Inc. was written on a
representing the value of his shares of stock with the general form indicating that an applicant will enclose an
plaintiff entity, the Mercantile Bank of China. amount as initial payment and will pay the balance in
accordance with law and the regulations of the College.
RULING: On the other hand, in the letter actually sent by Damasa
Crisostomo, the latter (who requested that her
NO. subscription for 200 shares be entered) not only did not
enclose any initial payment but stated that "babayaran
According to the weight of authority, a share of kong lahat pagkatapos na ako ay makapagpahuli ng
stock or the certificate thereof is not indebtedness to the isda." There is nothing in the record to show that the
owner or evidence of indebtedness and, therefore, it is Quezon College, Inc. accepted the term of payment
not a credit. Stockholders, as such, are not creditors of suggested by Damasa Crisostomo, or that if there was
the corporation. It is the prevailing doctrine of the any acceptance the same came to her knowledge during
American courts, repeatedly asserted in the broadest her lifetime. As the application of Damasa Crisostomo is
terms, that the capital stock of a corporation is a trust obviously at variance with the terms evidenced in the
fund to be used more particularly for the security of form letter issued by the Quezon College, Inc., there was
creditors of the corporation, who presumably deal with it absolute necessity on the part of the College to express
on the credit of its capital stock. Therefore, the its agreement to Damasa's offer in order to bind the
defendant-appellant Lim Chu Sing not being a creditor of latter. Conversely, said acceptance was essential,
the Mercantile Bank of China, although the latter is a because it would be unfair to immediately obligate the
creditor of the former, there is no sufficient ground to Quezon College, Inc. under Damasa's promise to pay
justify compensation. the price of the subscription after she had caused fish to
be caught. In other words, the relation between Damasa
Crisostomo and the Quezon College, Inc. had only thus
reached the preliminary stage whereby the latter offered
TRILLANA, petitioner its stock for subscription on the terms stated in the form
vs. letter, and Damasa applied for subscription fixing her
QUEZON COLLEGE, INC., respondent own plan of payment, — a relation, in the absence as in
G.R. No. L-5003 June 27, 1953 the present case of acceptance by the Quezon College,
Inc. of the counter offer of Damasa Crisostomo, that had
FACTS: not ripened into an enforceable contract.
Indeed, the need for express acceptance on the
Damasa Crisostomo sent a letter to the Board of part of the Quezon College, Inc. becomes the more
Trustees of the Quezon College subscribing to 200 imperative, in view of the proposal of Damasa
shares of its capital stock at par value of Php100 each.
Crisostomo to pay the value of the subscription after she
Damasa Crisostomo died on October 26, 1948. As no
payment appears to have been made on the has harvested fish, a condition obviously dependent
subscription mentioned in her letter, the Quezon upon her sole will and, therefore, facultative in nature,
College, Inc. presented a claim before the Court of First rendering the obligation void, under article 1115 of the
Instance of Bulacan in her testate proceeding, for the old Civil Code which provides as follows: "If the
collection of the sum of P20,000, representing the value fulfillment of the condition should depend upon the
of the subscription to the capital stock of the Quezon exclusive will of the debtor, the conditional obligation
College, Inc. This claim was opposed by the
shall be void. If it should depend upon chance, or upon
administrator of the estate, and the Court of First
Instance of Bulacan, after hearing issued an order the will of a third person, the obligation shall produce all
dismissing the claim of the Quezon College, Inc. on the its effects in accordance with the provisions of this
ground that the subscription in question was neither code." It cannot be argued that the condition solely is
registered in nor authorized by the Securities and void, because it would have served to create the
obligation to pay, wherein only the potestative condition
was held void because it referred merely to the subscription, properly speaking, is the mutual agreement
fulfillment of an already existing indebtedness. of the subscribers to take and pay for the stock of a
corporation, while a purchase is an independent
agreement between the individual and the corporation to
buy shares of stock from it at stipulated price." In some
BAYLA, et al., petitioner particulars the rules governing subscriptions and sales of
vs. shares are different. For instance, the provisions of our
SILANG TRAFFIC CO., INC., respondent Corporation Law regarding calls for unpaid subscription
G.R. Nos. L-48195 and 48196 May 1, 1942 and assessment of stock do not apply to a purchase of
stock. Likewise the rule that corporation has no legal
FACTS: capacity to release an original subscriber to its capital
stock from the obligation to pay for his shares, is
Petitioners in G.R. No. 48195 instituted this inapplicable to a contract of purchase of shares.
action in the Court of First Instance of Cavite against the
respondent Silang Traffic Co., Inc. (cross-petitioner in
G.R. No. 48196), to recover certain sums of money VELASCO VS. POIZAT
which they had paid severally to the corporation on
account of shares of stock they individually agreed to G.R. No. L-11528
take and pay for under certain specified terms and
conditions. The agreements signed by the other
petitioners were of the same date (March 30, 1935) and
in identical terms as the foregoing except as to the
number of shares and the corresponding purchase price. FACTS: The plaintiff, as assignee in insolvency of "The
The petitioners agreed to purchase a total of 46 shares Philippine Chemical Product Company" (Ltd.) is seeking
and, up to April 30, 1937, had paid the corresponding to recover of the defendant, Jean M. Poizat, the sum of
amount on account thereof.Petitioners' action for the P1,500, upon a subscription made by him to the
recovery of the sums above mentioned is based on a corporate stock of said company. It appears that the
resolution by the board of directors of the respondent corporation in question was originally organized by
corporation on August 1, 1937.
several residents of the city of Manila, where the
The respondent corporation set up the following
defenses: (1) That the above-quoted resolution is not company had its principal place of business, with a
applicable to the petitioners Sofronio T. Bayla, Josefa capital of P50,000, divided into 500 shares. The
Naval, and Paz Toledo because on the date thereof defendant subscribed for 20 shares of the stock of the
"their subscribed shares of stock had already company, an paid in upon his subscription the sum of
automatically reverted to the defendant, and the P500, the par value of 5 shares . The action was brought
installments paid by them had already been forfeited";
to recover the amount subscribed upon the remaining
and (2) that said resolution of August 1, 1937, was
revoked and cancelled by a subsequent resolution of the shares. It appears that the defendant was a stock holder
board of directors of the defendant corporation dated in the company from the inception of the enterprise, and
August 22, 1937. for sometime acted as its treasurer and manager. While
serving in this capacity he called in and collected all
ISSUE: subscriptions to the capital stock of the company, except
the aforesaid 15 shares subscribed by himself and
Whether or not the agreement was a contract of
another 15 shares owned by Jose R. Infante.
subscription to the capital stock of the respondent
corporation.
A meeting of the board of directors of the company was
RULING: held at which a majority of the stock was presented.
Upon this occasion two resolutions were adopted. The
NO. first was a proposal that the directors, or shareholders,
of the company should make good by new subscriptions,
Whether a particular contract is a subscription or in proportion to their respective holdings, 15 shares
a sale of stock is a matter of construction and depends which had been surrendered by Infante. It seems that
upon its terms and the intention of the parties. In the this shareholder had already paid 25 per cent of his
Unson case just cited, this Court held that a subscription subscription upon 20 shares, leaving 15 shares unpaid
to stock in an existing corporation is, as between the for, and an understanding had been reached by him and
subscriber and the corporation, simply a contract of the management by which he was to be released from
purchase and sale. the obligation of his subscription, it being understood
It seems clear from the terms of the contracts in that what he had already paid should not be refunded.
question that they are contracts of sale and not of Accordingly the directors present at this meeting
subscription. The lower courts erred in overlooking the subscribed P1,200 toward taking up his shares, leaving
distinction between subscription and purchase "A a deficiency of P300 to be recovered by voluntary
subscriptions from stockholders not present at the the court. It is now quite well settled that when the
meeting. The other proposition was o the effect that corporation becomes insolvent, with proceedings
Juan [Jean] M. Poizat, who was absent, should be instituted by creditors to wind up and distribute its
required to pay the amount of his subscription upon the assets, no call or assessment is necessary before
15 shares for which he was still indebted to the the institution of suits to collect unpaid balances on
company. The resolution further provided that, in case subscription. It evidently cannot be permitted that a
he should refuse to make such payment, the subscriber should escape from his lawful obligation
management of the corporation should be authorized to by reason of the failure of the officers of the
undertake judicial proceedings against him. When corporation to perform their duty in making a call;
notification of this resolution reached Poizat through the and when the original model of making the call
mail it evoked from him a manifestation of surprise and becomes impracticable, the obligation must be
pain, which found expression in a letter written by him in treated as due upon demand. The better doctrine is
reply, dated July 27, 1914, and addressed to Velasco, as that when insolvency supervenes all unpaid
treasurer and administrator. In this letter Poizat states subscriptions become at once due and enforceable.
that he had been given to understand by some member
of the board of directors that he was to be relieved from
his subscription upon the terms conceded to Infante. The
company soon went into voluntary insolvency, Velasco
The circumstance that the board of directors in their
being named as the assignee. At the hearing of the meeting of July 13, 1914, resolved to release Infante
Court of First Instance, judgment was rendered in favor from his obligation upon a subscription for 15
of the defendant, and the complaint was dismissed. shares is no wise prejudicial to the right of the
From this action the plaintiff has appealed.
corporation or its assignee to recover from Poizat
upon a subscription made by him. In releasing
ISSUE: WON Poizat is liable upon this subscription? Infante the board transcended its powers, and he no
doubt still remained liable on such of his shares as
HELD: Poizat is liable upon his subscription. Section were not taken up and paid for by other persons.The
36 of the Corporation Law clearly recognizes that a general doctrine is that the corporation has no legal
stock subscription is subsisting liability from the capacity to release an original subscriber to its
time the subscription is made, since it requires the capital stock from the obligation of paying for his
subscriber to pay interest quarterly from that date shares, in whole or in part.The suggestion contained
unless he is relieved from such liability by the by- in Poizat's letter of July 27, 1914, to the effect that he
laws of the corporation. The subscriber is as much understood that he was to be relieved upon the
bound to pay the amount of the share subscribed by same terms as Infante is, for the same reason, of no
him as he would be to pay any other debt, and the merit as matter of defense, even if an agreement to
right of the company to demand payment is no less that effect had been duly proved.
incontestable. The provisions of the Corporation
Law (Act No. 1459) given recognition of two
remedies for the enforcement of stock
subscriptions. The first and most special remedy PHILIPPINE NATIONAL BANK, plaintiff
given by the statute consists in permitting the vs.
corporation to put up the unpaid stock for sale and BITULOK SAWMILL INC., defendant
dispose of it for the account of the delinquent G.R. Nos. L-24177-85 June 29, 1968
subscriber. In this case the provisions of section 38
to 48, inclusive, of the Corporation Law are FACTS:
applicable and must be followed. Nothing in this Act
shall prevent the directors from collecting, by action The Philippine Lumber Distributing Agency, Inc.,
in any court of proper jurisdiction, the amount due according to the lower court, "was organized sometime
on any unpaid subscription, together with accrued
in the early part of 1947 upon the initiative and
interest and costs and expenses incurred. The
insistence of the late President Manuel Roxas of the
assignee of the insolvent corporation succeeds to all
Republic of the Philippines who for the purpose, had
the corporate rights of action vested in the
called several conferences between him and the
corporation prior to its insolvency; and the assignee subscribers and organizers of the Philippine Lumber
therefore has the same freedom with respect to Distributing Agency, Inc." The purpose was praiseworthy,
suing upon the stock subscription as the directors
to insure a steady supply of lumber, which could be sold
themselves would have had under section 49 above
at reasonable prices to enable the war sufferers to
cited. There is another reason why the present
rehabilitate their devastated homes. At the beginning,
plaintiff must prevail in this case. That reason is this:
the lumber producers were reluctant to organize the
When insolvency supervenes upon a corporation cooperative agency as they believed that it would not be
and the court assumes jurisdiction to wind up, all easy to eliminate from the retail trade the alien
unpaid stock subscriptions become payable on
middlemen who had been in this business from time
demand, and are at once recoverable in an action
immemorial, but because the late President Roxas made
instituted by the assignee or receiver appointed by
it clear that such a cooperative agency would not be
successful without a substantial working capital which FACTS:
the lumber producers could not entirely shoulder, and as
an inducement he promised and agreed to finance the This action was instituted in the Court of First
agency by making the Government invest P9.00 by way Instance of Manila by the National Exchange Co., Inc.,
of counterpart for every peso that the members would as assignee (through the Philippine National Bank) of C.
invest therein." S. Salmon & Co., for the purpose of recovering from I. B.
Accordingly, "the late President Roxas instructed Dexter a balance of P15,000, the par value of one
the Hon. Emilio Abello, then Executive Secretary and hundred fifty shares of the capital stock of C. S. Salmon
Chairman of the Board of Directors of the Philippine & co., with interest and costs. Upon hearing the cause
National Bank, for the latter to grant said agency an the trial judge gave judgment for the plaintiff to recover
overdraft in the original sum of P250,000.00 which was the amount claimed, with lawful interest from January 1,
later increased to P350,000.00, which was approved by 1920, and with costs. From this judgment the defendant
said Board of Directors of the Philippine National Bank appealed.
on July 28, 1947, payable on or before April 30, 1958, It appears that on August 10, 1919, the
with interest at the rate of 6% per annum, and secured defendant, I. B. Dexter, signed a written subscription to
by the chattel mortgages on the stock of lumber of said the corporate stock of C. S. Salmon & Co. in the
agency." The Philippine Government did not invest the following form: I hereby subscribe for three hundred
P9.00 for every peso coming from defendant lumber (300) shares of the capital stock of C. S. Salmon and
producers. The loan extended to the Philippine Lumber Company, payable from the first dividends declared on
Distributing Agency by the Philippine National Bank was any and all shares of said company owned by me at the
not paid. time dividends are declared, until the full amount of this
subscription has been paid.
ISSUE: Upon this subscription the sum of P15,000 was
paid in January, 1920, from a dividend declared at about
Whether or not the non-compliance with a plain that time by the company, supplemented by money
statutory command, considering the persuasiveness of supplied personally by the subscriber. Beyond this
the plea that defendants-appellees would "not have nothing has been paid on the shares and no further
subscribed to the capital stock" of the Philippine Lumber dividend has been declared by the corporation. There is
Distributing Agency "were it not for the assurance of the therefore a balance of P15,000 still paid upon the
then President of the Republic that the Government subscription.
would back it up by investing P9.00 for every
peso" subscribed, a condition which was not fulfilled, ISSUE:
such commitment not having been complied with, be
justified. Whether or not the stipulation contained in the
subscription to the effect that the subscription is payable
RULING: from the first dividends declared on the shares has the
effect of relieving the subscriber from personal liability in
NO. an action to recover the value of the shares.

It would be unwarranted to ascribe to the late RULING:


President Roxas the view that the payment of the stock
subscriptions, as thus required by law, could be NO.
condoned in the event that the counterpart fund to be
invested by the Government would not be available. In discussing this problem we accept as sound
Even if such were the case, however, and such a law the proposition propounded by the appellant's
promise were in fact made, to further the laudable attorneys and taken from Fletcher's Cyclopedia as
purpose to which the proposed corporation would be follows: In the absence of restrictions in its character, a
devoted and the possibility that the lumber producers corporation, under its general power to contract, has the
would lose money in the process, still the plain and power to accept subscriptions upon any special terms
specific wording of the applicable legal provision as not prohibited by positive law or contrary to public policy,
interpreted by this Court must be controlling. It is a well- provided they are not such as to require the performance
settled principle that with all the vast powers lodged in of acts which are beyond the powers conferred upon the
the Executive, he is still devoid of the prerogative of corporation by its character, and provided they do not
suspending the operation of any statute or any of its constitute a fraud upon other subscribers or
terms. stockholders, or upon persons who are or may become
creditors of the corporation.
NATIONAL EXCHANGE CO., Pursuant to such, we find that the Philippine
INC., petitioner Commission inserted in the Corporation Law, enacted
vs. March 1, 1906, the following provision: "no corporation
I.B. DEXTER, respondent shall issue stock or bonds except in exchange for actual
G.R. No. L-27872 February 25, 1928 cash paid to the corporation or for property actually
received by it at a fair valuation equal to the par value of another P70 million 3 to FLADC and P20 million to the
the stock or bonds so issued." Tius over and above their P100 million investment, the
The prohibition against the issuance of shares total sum of which (P190 million) was used to settle the
by corporations except for actual cash to the par value of P190 million mortgage indebtedness of FLADC to PNB.
the stock to its full equivalent in property is thus The business harmony between the Ongs and the Tius
enshrined in both the organic and statutory law of the in FLADC, however, was shortlived because the Tius, on
Philippine; Islands; and it would seem that our 23 February 1996, rescinded the Pre-Subscription
lawmakers could scarely have chosen language more Agreement. The Tius accused the Ongs of (1) refusing
directly suited to secure absolute equality stockholders to credit to them the FLADC shares covering their real
with respect to their liability upon stock subscriptions. property contributions; (2) preventing David S. Tiu and
Now, if it is unlawful to issue stock otherwise than as Cely Y. Tiu from assuming the positions of and
stated it is self-evident that a stipulation such as that performing their duties as Vice-President and Treasurer,
now under consideration, in a stock subcription, is illegal, respectively, and (3) refusing to give them the office
for this stipulation obligates the subcriber to pay nothing spaces agreed upon. The controversy finally came to a
for the shares except as dividends may accrue upon the head when the case was commenced by the Tius on 27
stock. In the contingency that dividends are not paid, February 1996 at the Securities and Exchange
there is no liability at all. This is discrimination in favor of Commission (SEC), seeking confirmation of their
the particular subcriber, and hence the stipulation is rescission of the Pre-Subscription Agreement.
unlawful.
ISSUE:

Whether or not the rescission of Pre-


ONG YONG, et al., petitioner Subscription Agreement would result in unauthorized
vs. liquidation.
TIU, et al., respondent
G.R. No. 144476 8 April 2003
RULING:
FACTS:
YES.
In 1994, the construction of the Masagana
Citimall in Pasay City was threatened with stoppage and The rescission of the Pre-Subscription
incompletion when its owner, the First Landlink Asia Agreement will effectively result in the unauthorized
distribution of the capital assets and property of the
Development Corporation (FLADC), which was owned
corporation, thereby violating the Trust Fund Doctrine
by David S. Tiu, Cely Y. Tiu, Moly Yu Gow, Belen See
and the Corporation Code, since rescission of a
Yu, D. Terence Y. Tiu, John Yu and Lourdes C. Tiu (the
subscription agreement is not one of the instances when
Tius), encountered dire financial difficulties. It was
heavily indebted to the Philippine National Bank (PNB) distribution of capital assets and property of the
for P190 million. To stave off foreclosure of the mortgage corporation is allowed. Rescission will, in the final
analysis, result in the premature liquidation of the
on the two lots where the mall was being built, the Tius
corporation without the benefit of prior dissolution in
invited Ong Yong, Juanita Tan Ong, Wilson T. Ong,
accordance with Sections 117, 118, 119 and 120 of the
Anna L. Ong, William T. Ong and Julia Ong Alonzo (the
Corporation Code.
Ongs), to invest in FLADC. Under the Pre-Subscription
Agreement they entered into, the Ongs and the Tius
agreed to maintain equal shareholdings in FLADC: the
What is a “Subscription”
Ongs were to subscribe to 1,000,000 shares at a par
value of P100.00 each while the Tius were to subscribe
ONG YONG, et al., petitioner
to an additional 549,800 shares at P100.00 each in
vs.
addition to their already existing subscription of 450,200
TIU, et al., respondent
shares. Furthermore, they agreed that the Tius were
G.R. No. 144476 8 April 2003
entitled to nominate the Vice-President and the
Treasurer plus 5 directors while the Ongs were entitled
FACTS:
to nominate the President, the Secretary and 6 directors
(including the chairman) to the board of directors of
FLADC. Moreover, the Ongs were given the right to In 1994, the construction of the Masagana
manage and operate the mall. Accordingly, the Ongs Citimall in Pasay City was threatened with stoppage and
incompletion when its owner, the First Landlink Asia
paid P100 million in cash for their subscription to
Development Corporation (FLADC), which was owned
1,000,000 shares of stock while the Tius committed to
by David S. Tiu, Cely Y. Tiu, Moly Yu Gow, Belen See
contribute to FLADC a four-storey building and two
Yu, D. Terence Y. Tiu, John Yu and Lourdes C. Tiu (the
parcels of land respectively valued at P20 million (for
200,000 shares), P30 million (for 300,000 shares) and Tius), encountered dire financial difficulties. It was
P49.8 million (for 49,800 shares) to cover their additional heavily indebted to the Philippine National Bank (PNB)
for P190 million. To stave off foreclosure of the mortgage
549,800 stock subscription therein. The Ongs paid in
on the two lots where the mall was being built, the Tius 1,000,000 shares of stock was, from the viewpoint of the
invited Ong Yong, Juanita Tan Ong, Wilson T. Ong, law, one between the Ongs and FLADC, not between
Anna L. Ong, William T. Ong and Julia Ong Alonzo (the the Ongs and the Tius.
Ongs), to invest in FLADC. Under the Pre-Subscription
Agreement they entered into, the Ongs and the Tius
agreed to maintain equal shareholdings in FLADC: the
Ongs were to subscribe to 1,000,000 shares at a par G.R. No. 144476 April 8, 2003
value of P100.00 each while the Tius were to subscribe
to an additional 549,800 shares at P100.00 each in
addition to their already existing subscription of 450,200 ONG YONG, JUANITA TAN ONG, WILSON T. ONG,
shares. Furthermore, they agreed that the Tius were ANNA L. ONG, WILLIAM T. ONG, WILLIE T. ONG, and
entitled to nominate the Vice-President and the JULIE ONG ALONZO, petitioners,
Treasurer plus 5 directors while the Ongs were entitled vs.
to nominate the President, the Secretary and 6 directors DAVID S. TIU, CELY Y. TIU, MOLY YU GAW, BELEN
(including the chairman) to the board of directors of SEE YU, D. TERENCE Y. TIU, JOHN YU, LOURDES C.
FLADC. Moreover, the Ongs were given the right to TIU, INTRALAND RESOURCES DEVELOPMENT
manage and operate the mall. Accordingly, the Ongs CORP., MASAGANA TELAMART, INC., REGISTER
paid P100 million in cash for their subscription to OF DEEDS OF PASAY CITY, and the SECURITIES
1,000,000 shares of stock while the Tius committed to AND EXCHANGE COMMISSION, respondents.
contribute to FLADC a four-storey building and two
parcels of land respectively valued at P20 million (for x-----------------------------x
200,000 shares), P30 million (for 300,000 shares) and
P49.8 million (for 49,800 shares) to cover their additional G.R. No. 144629 April 8, 2003
549,800 stock subscription therein. The Ongs paid in
another P70 million 3 to FLADC and P20 million to the DAVID S. TIU, CELY Y. TIU, MOLY YU GAW, BELEN
Tius over and above their P100 million investment, the SEE YU, D. TERENCE Y. TIU, JOHN YU, LOURDES C.
total sum of which (P190 million) was used to settle the TIU, and INTRALAND RESOURCES DEVELOPMENT
P190 million mortgage indebtedness of FLADC to PNB. CORP., petitioners,
vs.
ISSUE: ONG YONG, JUANITA TAN ONG, WILSON T. ONG,
ANNA L. ONG, WILLIAM T. ONG, WILLIE T. ONG, and
Whether or not the pre-Subscription Agreement JULIA ONG ALONZO, respondents.
executed by the Ongs is actually a subscription contract.
FACTS:
RULING:

YES. In 1994, the construction of the Masagana Citimall in


Pasay City was threatened with stoppage and
FLADC was originally incorporated with an incompletion when its owner, the First Landlink Asia
authorized capital stock of 500,000 shares with the Tius Development Corporation (FLADC), which was owned
owning 450,200 shares representing the paid-up capital. by the Tius, encountered dire financial difficulties. It was
When the Tius invited the Ongs to invest in FLADC as heavily indebted to the Philippine National Bank (PNB)
stockholders, an increase of the authorized capital stock for P190 million.
became necessary to give each group equal (50-50)
shareholdings as agreed upon in the Pre-Subscription To stave off foreclosure of the mortgage on the two lots
Agreement. The authorized capital stock was thus where the mall was being built, the Tius invited the
increased from 500,000 shares to 2,000,000 shares with Ongs to invest in FLADC.
a par value of P100 each, with the Ongs subscribing to
1,000,000 shares and the Tius to 549,800 more shares Under the Pre-Subscription Agreement they entered
in addition to their 450,200 shares to complete into, the Ongs and the Tius agreed to maintain equal
1,000,000 shares. Thus, the subject matter of the shareholdings in FLADC.
contract was the 1,000,000 unissued shares of FLADC
stock allocated to the Ongs. Since these were unissued Furthermore, they agreed that the Tius were entitled to
shares, the parties' Pre-Subscription Agreement was in nominate the Vice-President and the Treasurer plus five
fact a subscription contract as defined under Section 60, directors while the Ongs were entitled to nominate the
Title VII of the Corporation Code. A subscription contract President, the Secretary and six directors (including the
necessarily involves the corporation as one of the chairman) to the board of directors of FLADC.
contracting parties since the subject matter of the
transaction is property owned by the corporation — its
shares of stock. Thus, the subscription contract Moreover, the Ongs were given the right to manage and
(denominated by the parties as a Pre-Subscription operate the mall.
Agreement) whereby the Ongs invested P100 million for
The business harmony between the Ongs and the Tius ISSUE:
in FLADC, however, was shortlived because the Tius, on
February 23, 1996, rescinded the Pre-Subscription Whether or not the Tius could legally rescind the Pre-
Agreement. The Tius accused the Ongs of: Subscription Agreement.

(1) refusing to credit to them the FLADC shares RULING:


covering their real property contributions;
In the instant case, the rescission of the Pre-
(2) preventing David S. Tiu and Cely Y. Tiu from Subscription Agreement will effectively result in the
unauthorized distribution of the capital assets and
assuming the positions of and performing their
property of the corporation, thereby violating the Trust
duties as Vice-President and Treasurer,
Fund Doctrine and the Corporation Code, since
respectively, and
rescission of a subscription agreement is not one of the
instances when distribution of capital assets and
(3) refusing to give them the office spaces property of the corporation is allowed.
agreed upon.
Contrary to the Tius' allegation, rescission will, in the
In their defense, the Ongs said that David S. Tiu and final analysis, result in the premature liquidation of the
Cely Y. Tiu had in fact assumed the positions of Vice- corporation without the benefit of prior dissolution in
President and Treasurer of FLADC but that it was they accordance with Sections 117, 118, 119 and 120 of the
who refused to comply with the corporate duties Corporation Code.
assigned to them.
The Tius maintain that rescinding the subscription
This case was commenced by the Tius on February 27,
1996 at the SEC, seeking confirmation of their rescission contract is not synonymous to corporate liquidation
because all rescission will entail would be the simple
of the Pre-Subscription Agreement. After hearing, the
restoration of the status quo ante and a return to the two
SEC issued a decision confirming the rescission sought
groups of their cash and property contributions. We wish
by the Tius.
it were that simple. Very noticeable is the fact that the
SEC en banc affirmed and confirmed the rescission of Tius do not explain why rescission in the instant case will
the Pre-Subscription Agreement. not effectively result in liquidation. The Tius merely refer
in cavalier fashion to the end-result of rescission (which
incidentally is 100% favorable to them) but turn a blind
On appeal, the Court of Appeals likewise affirmed the
eye to its unfair, inequitable and disastrous effect on the
SEC decision. However, the CA concluded that both the
Ongs and the Tius were in pari delicto (which would not corporation, its creditors and the Ongs.
have legally entitled them to rescission) but, "for practical
considerations," that is, their inability to work together, it The Tius claim that rescission of the agreement will not
was best to separate the two groups by rescinding the result in an unauthorized liquidation of the corporation
Pre-Subscription Agreement, returning the original because their case is actually a petition to decrease
investment of the Ongs and awarding practically capital stock pursuant to Section 38 of the Corporation
everything else to the Tius. Code. Section 122 of the law provides that "(e)xcept by
decrease of capital stock…, no corporation shall
Both parties filed separate petitions for review before the distribute any of its assets or property except upon lawful
Supreme Court. dissolution and after payment of all its debts and
liabilities." The Tius claim that their case for rescission,
This Court affirmed the fact that both the Ongs and the being a petition to decrease capital stock, does not
Tius violated their respective obligations under the Pre- violate the liquidation procedures under our laws. All that
Subscription Agreement. The Ongs prevented the Tius needs to be done, according to them, is for this Court to
from assuming the positions of Vice-President and order (1) FLADC to file with the SEC a petition to issue a
Treasurer of the corporation. On the other hand, the certificate of decrease of capital stock and (2) the SEC
Decision established that the Tius failed to turn over to approve said decrease. This new argument has no
FLADC funds to the Ongs and that the Tius diverted merit.
rentals due to FLADC to their MATTERCO account.
Consequently, it held that rescission was not possible The Tius' case for rescission cannot validly be deemed a
since both parties were in pari delicto. petition to decrease capital stock because such action
never complied with the formal requirements for
However, this Court agreed with the Court of Appeals decrease of capital stock under Section 33 of the
that the remedy of specific performance, as espoused by Corporation Code. No majority vote of the board of
the Ongs, was not practical and sound either and would directors was ever taken. Neither was there any
only lead to further "squabbles and numerous litigations" stockholders meeting at which the approval of
between the parties. stockholders owning at least two-thirds of the
outstanding capital stock was secured. There was no
revised treasurer's affidavit and no proof that said the law but is also prejudicial to corporate creditors who
decrease will not prejudice the creditors' rights. On the enjoy absolute priority of payment over and above any
contrary, all their pleadings contained were alleged acts individual stockholder thereof.
of violations by the Ongs to justify an order of rescission.
Stripped to its barest essentials, the issue of rescission
Furthermore, it is an improper judicial intrusion into in this case is not difficult to understand. If rescission is
the internal affairs of the corporation to compel denied, will injustice be inflicted on any of the parties?
FLADC to file at the SEC a petition for the issuance The answer is no because the financial interests of both
of a certificate of decrease of stock. Decreasing a the Tius and the Ongs will remain intact and safe within
corporation's authorized capital stock is an FLADC. On the other hand, if rescission is granted, will
amendment of the Articles of Incorporation. It is a any of the parties suffer an injustice? Definitely yes
decision that only the stockholders and the directors because the Ongs will find themselves out in the streets
can make, considering that they are the contracting with nothing but the money they had in 1994 while the
parties thereto. In this case, the Tius are actually not Tius will not only enjoy a windfall estimated to be
just asking for a review of the legality and fairness of a anywhere from P450 million to P900 million but will also
corporate decision. They want this Court to make a take over an extremely profitable business without much
corporate decision for FLADC. We decline to intervene effort at all.
and order corporate structural changes not voluntarily
agreed upon by its stockholders and directors.
G.R. No. L-19893 March 31, 1923
Truth to tell, a judicial order to decrease capital stock
without the assent of FLADC's directors and ARNALDO F. DE SILVA, plaintiff-appellant, vs.
stockholders is a violation of the "BUSINESS
JUDGMENT RULE" which states that: ABOITIZ & COMPANY, INC., defendant-appellee.

xxx xxx xxx (C)ontracts intra vires entered into Facts: The plaintiff subscribed for 650 shares of stock of
by the board of directors are binding upon the the defendant corporation of the value of P500 each, of
corporation and courts will not interfere unless which he has paid only the total value of 200 shares,
such contracts are so unconscionable and there remaining 450 shares unpaid, for which he was
oppressive as to amount to wanton destruction
indebted to the corporation in the sum of P225,000, the
to the rights of the minority, as when plaintiffs
aver that the defendants (members of the value thereof. He was notified by the secretary of the
board), have concluded a transaction among corporation of a resolution adopted by the board of
themselves as will result in serious injury to the directors of the corporation on the preceding day,
plaintiffs stockholders.29 declaring the unpaid subscriptions to the capital stock of
the corporation to have become due and payable on the
The reason behind the rule is aptly explained by Dean following May 31st at the office thereof, the payment to
Cesar L. Villanueva, an esteemed author in corporate be made to the treasurer, and stating that all such
law, thus: shares as may have not been paid then, with the
accrued interest up to that date, will be declared
Courts and other tribunals are wont to override
delinquent, advertised for sale at public auction, and sold
the business judgment of the board mainly
because, courts are not in the business of on the following June 16th, for the purpose of paying up
business, and the laissez faire rule or the free the amount of the subscription and accrued interest, with
enterprise system prevailing in our social and the expenses of the advertisement and sale, unless said
economic set-up dictates that it is better for the payment was made before.
State and its organs to leave business to the
businessmen; especially so, when courts are ill- The proper advertisement having been published, as
equipped to make business decisions. More announced in the aforesaid notice, the plaintiff filed a
importantly, the social contract in the corporate complaint against the said corporation, wherein, after
family to decide the course of the corporate
business has been vested in the board and not relating the above-mentioned facts, he prayed for a
with courts.30 judgment in his favor, decreeing that, in prescribing
another method of paying the subscription to the capital
Apparently, the Tius do not realize the illegal stock different from that provided in article 46 of its by-
consequences of seeking rescission and control of the laws, in declaring the aforesaid 450 shares delinquent,
corporation to the exclusion of the Ongs. Such an act and in directing the sale thereof, as advertised, the
infringes on the law on reduction of capital stock. corporation had exceeded its executive authority, and as
Ordering the return and distribution of the Ongs' capital a consequence thereof he asked that a writ of injunction
contribution without dissolving the corporation or
be issued against the said defendant, enjoining it from
decreasing its authorized capital stock is not only against
taking any further action of whatever nature in "Nothing in this Act prevent the directors from collecting,
connection with the acts complained of and that it pay by action in any court of proper jurisdiction, the amount
the costs of this suit. due on any unpaid subscription, together with accrued
interest and costs and expenses incurred."
The plaintiff alleged as the grounds of his petition: (1)
That, according to aforesaid article 46 of the by-law of Admitting that the provision of article 46 of the said by-
the corporation, which was inserted in the complaint, all laws maybe regarded as a contract between the
the shares subscribed to by the incorporation that were defendant corporation and its stockholders , yet as it is
not paid for at the time of the incorporation, shall be paid only to the board of directors of the corporation that said
out of the 70 per cent of the profit obtained, the same to articles gives the authority or right to apply on the
be distributed among the subscribers, who shall not payment of unpaid subscriptions such amount of the 70
receive any dividend until said shares were paid in full; per cent of the profit distributable among the
(2) that in declaring the plaintiff's unpaid subscription to shareholders in equal parts as may be deemed fit, it
the capital stock to have become due and payable on cannot be maintained that the said article has prescribe
May 31st, and in publishing the aforesaid notice an operative method for the payment of said subscription
declaring his unpaid shares delinquent, the defendant continuously until their full amortization, or, what would
corporation has violated the aforesaid article, which be the same thing, that said article has prescribe that
prescribes an operative method of paying for the shares sole and exclusive method for that purpose, for, in the
continuously until their full amortization, thus violating first place, the adoption of that method for the purpose of
and disregarding a right of the plaintiff vested under the collecting the value of subscriptions due and unpaid lies,
said by-laws; (3) that the aforesaid acts of the defendant according to said article, within the discretion of the
corporation were in excess of its powers and executive board of directions, that is, it is subject to this condition,
authority and the plaintiff had no other plain, speedy and and this can in no way be reconciled with the idea of
adequate remedy in the ordinary course of law than that method, which implies something fixed as a rule or
prayed for in the said complaint, to prevent the permanent standard, and not variable at the will of
defendant from taking any further action in connection somebody and according to the circumstances; and, in
with the sale and alienation of the said shares. the second place, in connection with the provision of the
said article relative to the aforesaid discretionary power
A preliminary injunction having been issued against the of the board of directors to adopt that method, there is
defendant, period elapsed without the plaintiff having also the discretionary power granted the same board of
amended his complaint, upon motion of the defendant, directors to avail itself, for the same purpose, to either of
that court, dismissed the complaint and ordered the the two remedies prescribed in sections 38 to 49,
dissolution of the preliminary injunction previously inclusive, of the aforecited Corporation Law.
issued, with costs, to which orders the plaintiff excepted,
asking at the same time for the annulment thereof and a In the instant case, the defendant corporation, through
new hearing, which motion was denied by the lower its board of directors, made use of its discretionary
court. To that ruling the plaintiff also excepted, and power, taking advantage of the first of the two remedies
brought the case to this court by the proper bill of provided by the aforesaid law. On the other hand, the
exceptions. plaintiff has no right whatsoever under the provision of
the above cited article 46 of the said by-laws to prevent
Issue: Whether or not, under the provision of article 46 of the board of directors from following, for that purpose,
the by-laws of the defendant corporation, the latter may any other method than that mentioned in the said article,
declare the unpaid shares delinquent, or collect their for the very reason that the same does not give the
value by another method different from that prescribed in stockholders any right in connection with the
the aforecited article? determination of the question whether or not there
should be deducted from the 70 per cent of the profit
Ruling: Yes. The first and most special remedy given by
distributable among the stockholders such amount as
the statute consists in permitting the corporation to put
may be deemed fit for the payment of subscriptions due
the unpaid stock for sale and dispose of it for the
and unpaid. Therefore, it is evident that the defendant
account of the delinquent subscriber. In this case the
corporation has not violated, nor disregarded any right of
provisions of sections 38 to 48, inclusive, of the
the plaintiff recognized by the said by-laws, nor
Corporation Law are applicable and must be followed.
exceeded its authority in the discharge of its executive
The other remedy is by action in court concerning which
functions, nor abused its discretion when it performed
we find in section 49 the following provision:
the acts mentioned in the complaint as grounds thereof,
and, consequently, the facts therein alleged do not Together with another documentary stamp tax
constitute a cause of action. assessment which it also questioned, petitioner
appealed the Commissioner's ruling to the Court of Tax
For the foregoing, the orders appealed from are Appeals. The CTA rendered its decision holding that the
affirmed, with the costs of both instances against the amount of the documentary stamp tax should be based
appellant. So ordered. on the par value stated on each certificate of stock.

Respondent Commissioner of Internal Revenue


appealed to the Court of Appeals which, reversed the
G.R. No. 118043 July 23, 1998 CTA's decision and held that, in assessing the tax in
question, the basis should be the actual value
LINCOLN PHILIPPINE LIFE INSURANCE COMPANY, represented by the subject shares on the assumption
INC. (now JARDINE-CMG LIFE INSURANCE CO. that stock dividends, being a distinct class of shares, are
INC.), petitioner,vs. not subject to the qualification in the law as to the type of
certificate of stock used (with or without par value).
COURT OF APPEALS and COMMISSIONER OF
INTERNAL REVENUE, respondents. Issue: Whether or not respondent court of appeals erred
in holding that stock dividends involving shares with par
Facts: Petitioner, now the Jardine-CMG Life Insurance
value are subject to documentary stamp tax based on
Company, Inc., is a domestic corporation engaged in the
the book value of said shares?
life insurance business. It issued 50,000 shares of stock
as stock dividends, with a par value of P100 or a total of Ruling: Yes. Conformably, in the case of stock
P5 million. Petitioner paid documentary stamp taxes on certificates with par value, the documentary stamp tax is
each certificate on the basis of its par value. The based on the par value of the stock; for stock certificates
question in this case is whether in determining the without par value, the same tax is computed from the
amount to be paid as documentary stamp tax, it is the actual consideration received by the corporation,
par value of the certificates of stock or the book value of association or company; but for stock dividends,
the shares which should be considered. The pertinent documentary stamp tax is to be paid "on the actual value
provision of law, as it stood at the time of the questioned represented by each share."
transaction, reads as follows:

Sec. 224.Stamp tax on original issues of certificates of


stock. — On every original issue, whether on Since in dividends, no consideration is technically
organization, reorganization or for any lawful purpose, of received by the corporation, petitioner is correct in
certificates of stock by any association, company or basing the assessment on the book value thereof
corporation, there shall be collected a documentary rejecting the principles enunciated in Commissioner of
stamp tax of one peso and ten centavos on each two Internal Revenue vs. Heald Lumber Co. (10 SCRA 372)
hundred pesos, or fractional part thereof, of the par as the said case refers to purchases of no-par
value of such certificates: Provided, That in the case of certificates of stocks and not to stock dividends.
the original issue of stock without par value the amount
of the documentary stamp tax herein prescribed shall be Apparently, the Court of Appeals treats stock dividends
based upon the actual consideration received by the as distinct from ordinary shares of stock for purposes of
association, company, or corporation for the issuance of the then §224 of the National Internal Revenue Code.
such stock, and in the case of stock dividends on the There is, however, no basis for considering stock
actual value represented by each share. dividends as a distinct class from ordinary shares of
stock since under this provision only certificates of stock
The Commissioner of Internal Revenue took the view are required to be distinguished (into either one with par
that the book value of the shares, amounting to value or one without) rather than the classes of shares
P19,307,500.00, should be used as basis for themselves.
determining the amount of the documentary stamp tax.
Accordingly, respondent Internal Revenue Stock dividends are in the nature of shares of stock, the
Commissioner issued a deficiency documentary stamp consideration for which is the amount of unrestricted
tax assessment in the amount of P78,991.25 in excess retained earnings converted into equity in the
of the par value of the stock dividends. corporation's books. Thus,
A "stock dividend'' is any dividend payable in shares of Lingayen Gulf Electric Power Company, Inc. VS.
stock of the corporation declaring or authorizing such Baltazar
dividend. It is, what the term itself implies, a distribution Facts:
The plaintiff, Lingayen Gulf Electric Power
of the shares of stock of the corporation among the
Company is a domestic corporation with an authorized
stockholders as dividends. A stock dividend of a capital stock of P300,000 divided into 3,000 shares with
corporation is a dividend paid in shares of stock instead a par value of P100 per share. The defendant, Irineo
of cash, and is properly payable only out of surplus Baltazar appears to have subscribed for 600 shares on
profits. So, a stock dividend is actually two things: (1) a account of which he had paid upon the organization of
dividend and (2) the enforced use of the dividend money the corporation the sum of P15,000. After incorporation,
to purchase additional shares of stock at par. the defendant made further payments on account of his
subscription, leaving a balance of P18,500 unpaid for,
which amount, the plaintiff now claims in this action.
On July 23, 1946, a majority of the stockholders
LINGAYEN GULF ELECTRIC POWER COMPANY, of the corporation, among them the herein defendant,
INC. held a meeting and adopted stockholders' resolution No.
vs. 17. By said resolution, it was agreed upon by the
IRINEO BALTAZAR stockholders present to call the balance of all unpaid
G.R. No. L-4824, June 30, 1953 subscribed capital stock as of July 23, 1946, the first 50
per cent payable within 60 days beginning August 1,
FACTS: 1946, and the remaining 50 per cent payable within 60
days beginning October 1, 1946. The resolution also
The respondent subscribed stocks of the provided, that all unpaid subscription after the due dates
petitioner. After paying several amount, the respondent of both calls would be subject to 12 per cent interest per
failed to pay its outstanding balance, even after a annum. Lastly, the resolution provided, that after the
demand made by the corporation. The latter hence expiration of 60 days' grace which would be on
opted to collect the unpaid balance of the subscription December 1, 1946, for the first call, and on February 1,
made. However, the respondent refused to pay on the 1947, for the second call, all subscribed stocks
contention that he has been released from his liability remaining unpaid would revert to the corporation.
under Resolution No. 17. Furthermore, he countered On September 22, 1946, the plaintiff corporation
that, as the President of the corporation, he was entitled wrote a letter to the defendant reminding him that the
to compensation. first 50 per cent of his unpaid subscription would be due
The trial court rendered judgment in favor of respondent. on October 1, 1946. The plaintiff requested the
defendant to "kindly advise the company thru the
ISSUE: undersigned your decision regarding this matter. The
defendant answered on September 25, 1946, asking the
Whether or not the respondent is entitled to corporation that he be allowed to pay his unpaid
compensation. subscription by February 1, 1947. In his answer, the
defendant also agreed that if he could not pay the
RULING: balance of his subscription by February 1, 1947, his
unpaid subscription would be reverted to the corporation.
NO. A case was file in which the lower court found
that the call for payment embodied in resolution No. 17
It is clear that he is not entitled to the same. The of July 23, 1946 was null and void for lack of publication;
by-laws of the company are silent as to the salary of the consequently, he dismissed the complaint as premature.
President. And, while resolutions of the incorporators He further held said resolution null and void in so far as it
and stockholders provide salaries for the general tried to relieve the defendant from liability on his unpaid
manager, secretary-treasurer and other employees, subscription, on the ground that the resolution was not
there was no provision for the salary of the President. approved by all the stockholders of the corporation. He
On the other hand, other resolutions provide for per also dismissed the defendant's counterclaim for
diems to be paid to the President and the directors of compensation as president of the corporation.
each meeting attended, P10 for the President and P8 for
each director, which were later increased to P25 and Issue:
P15, respectively. This leads to the conclusions that the W/N Resolution No. 17 of 1946 released him
President and the board of directors were expected to from the obligation to pay for his unpaid subscription.
serve without salary, and that the per diems paid to them
were sufficient compensation for their services. Ruling:
Furthermore, for defendant's several years of service as No.
President and up to the filing of the action against him, The authorities are generally agreed that in
he never filed a claim for salary. order to effect the release, there must be unanimous
consent of the stockholders of the corporation. We quote
some authorities:
Subject to certain exceptions, considered in subdivision or release from the obligation without the consent of the
(3) of this section, the general rule is that a valid and corporation and all the stockholders; . . . . (2 Thompson
binding subscription for stock of a corporation cannot be on Corporation, p. 186).
cancelled so as to release the subscriber from liability He states the reason for the rule as follows:
thereon without the consent of all the stockholders or SEC. 855. Right to withdraw as against subscribers. —
subscribers. Furthermore, a subscription cannot be A contract of subscription is, at least in the sense which
cancelled by the company, even under a secret or creates as estoppel, a contract among the several
collateral agreement for cancellation made with the subscribers. For this reason no one of the subscribers
subscriber at the time of the subscription, as against can withdraw from the contract without the consent of all
persons who subsequently subscribed or purchased the others, and thereby diminish, without the universal
without notice of such agreement. (18 C.J.S. 874). consent, the common fund in which all have acquired an
(3) Exceptions. interest. . . . (2 Thompson on Corporations, p. 194.).
In particular circumstances, as where it is given As already found by the trial court, the release
pursuant to a bona fide compromise, or to set off a debt attempted in Resolution No. 17 of 1946 was not valid for
due from the corporation, a release, supported by lack of a unanimous vote. If found that at least seven
consideration, will be effectual as against dissenting stockholders were absent from the meeting when said
stockholders and subsequent and existing creditors. A resolution was approved.
release which might originally have been held invalid Defendant and appellant, however, contends
may be sustained after a considerable lapse of time. (18 that after dismissing the complaint for being premature,
C.J.S. 874). there was no necessity or reason for the trial court to go
In the present case, the release claimed by further and say that defendant was not validly released
defendant and appellant does not fall under the from the payment for his unpaid subscription. It must be
exception above referred to, because it was not given borne in mind, however, that this was one of the
pursuant to a bona fide compromise, or to set off a debt principal issues involved in the case and the trial court
due from the corporation, and there was no was called upon to pass upon it, because unless so
consideration for it. passed upon and deter- mined, it might decisively affect
Section 66. Interest on unpaid subscriptions. the case on appeal. Supposing that on appeal the
- Subscribers for stock shall pay to the corporation appellate court decides that the call was valid, then it
interest on all unpaid subscriptions from the date of would be important to know whether or not in spite of the
subscription, if so required by, and at the rate of validity of the call, defendant was nevertheless not liable
interest fixed in the by-laws. If no rate of interest is because he had been validly released by a resolution of
fixed in the by-laws, such rate shall be deemed to be the corporation. If that question was not decided by the
the legal rate. trial court, and naturally was not touched upon in the
Section 67. Payment of balance of appeal, then the appellate court would have no occasion
subscription. - Subject to the provisions of the to pass upon it, and it might be necessary to bring
contract of subscription, the board of directors of another action to determine the point, which means
any stock corporation may at any time declare due multiplicity of suits. Moreover, the authority given to the
and payable to the corporation unpaid subscriptions courts to render judgments for declaratory relief in order
to the capital stock and may collect the same or to determine the rights or duties of parties over a certain
such percentage thereof, in either case with accrued transaction or under a certain written instrument, or to
interest, if any, as it may deem necessary. Payment remove the uncertainty or controversy over the same
of any unpaid subscription or any percentage (Rule 66 of the Rules of Court), justified the trial court in
thereof, together with the interest accrued, if any, passing upon this question of release.
shall be made on the date specified in the contract of
subscription or on the date stated in the call made
by the board. Failure to pay on such date shall
render the entire balance due and payable and shall
make the stockholder liable for interest at the legal PHILIPPINE TRUST COMPANY, as assignee in
rate on such balance, unless a different rate of insolvency of "La Cooperativa Naval Filipina"
interest is provided in the by-laws, computed from vs.
such date until full payment. If within thirty (30) days MARCIANO RIVERA
from the said date no payment is made, all stocks G.R. No. L-19761. January 29, 1923
covered by said subscription shall thereupon
become delinquent and shall be subject to sale as FACTS:
hereinafter provided, unless the board of directors
orders otherwise. In 1918 the Cooperativa Naval Filipina was duly
Another authority: incorporated under the laws of the Philippine Islands,
SEC. 850. Unanimous consent of stockholders with a capital of P100,000, divided into one thousand
necessary to release subscriber. — It may be asserted shares of a par value of P100 each. Among the
as the first rule under this proposition that, after a valid incorporators of this company was numbered the
subscription to the capital stock of a corporation has defendant Mariano Rivera, who subscribed for 450
been made and accepted, there can be no cancellation
shares representing a value of P45,000, the remainder On October 4, 1924, the appellant subscribed to
of the stock being taken by other persons. sixteen shares of the capital stock of the Compañia
In the course of time the company became Hispano-Filipina, Inc., a corporation which is duly formed
insolvent and went into the hands of the Philippine Trust and organized. Of the sixteen subscribed shares, at the
Company, as assignee in bankruptcy; and by it this par value of P100 each, the appellant only paid P400,
action was instituted to recover one-half of the stock the value of four shares. On June 5, 1931, the plaintiff-
subscription of the defendant, which admittedly has appellee was appointed by the court receiver of the
never been paid. Compañia Hispano-Filipina, Inc., to collect all the credits
The reason given for the failure of the defendant of said corporation, pay its debts and dispose of the
to pay the entire subscription is a meeting of its remainder of its assets and of its properties. On June 18,
stockholders occurred, at which a resolution was 1931, the plaintiff-appellee in vain made demand upon
adopted to the effect that the capital should be reduced the defendant-appellant to pay the balance of his
by 50 per centum and the subscribers released from the subscription. On July 10, 1933, the plaintiff, as receiver,
obligation to pay any unpaid balance of their subscription brought an action in the Court of First Instance of Manila
in excess of 50 per centum of the same. to recover from the defendant-appellant and other
shareholders the balance of their subscriptions, but the
ISSUE: complaint was dismissed for lack of prosecution. On
October 10, 1935, a similar complaint was filed against
Whether or not the reduction of the company’s the appellant, and after trial, judgment was rendered
capital by 50 per centum and the subscribers released therein ordering the said defendant to pay to the plaintiff,
from the obligation to pay any unpaid balance of their as receiver of Compañia Hispano-Filipina, Inc., the sum
subscription in excess of 50 per centum is valid. of P1,200, with legal interest thereon from October 4,
1924, and the costs. The defendant appealed and in this
RULING: instance contends that the trial court erred in holding that
the action of the plaintiff-appellee has not prescribed,
NO. and that the appellant has not been released from his
obligation to pay the balance of his subscription.
The Court ruled that defendant was still liable for
the unpaid balance of his subscription. It is established ISSUE:
doctrine that subscription to the capital of a corporation
constitute a find to which creditors have a right to look Whether or not the obligation contracted by the
for satisfaction of their claims and that the assignee in appellant to pay the value of his subscription was
insolvency can maintain an action upon any unpaid demandable from the date of subscription in the absence
stock subscription in order to realize assets for the of any stipulation to the contrary.
payment of its debts.
A corporation has no power to release an RULING:
original subscriber to its capital stock from the obligation
of paying for his shares, without a valuable consideration NO.
for such release; and as against creditors a reduction of
the capital stock can take place only in the manner an Section 37 of the Corporation Law provides
under the conditions prescribed by the statute or the when the obligation to pay interest arises and when
charter or the articles of incorporation. Moreover, strict payment should be made, but it is absolutely silent as to
compliance with the statutory regulations is necessary. when the subscription to a stock should be paid. Of
In the case before us the resolution releasing the course, the obligation to pay arises from the date of the
shareholders from their obligation to pay 50 per centum subscription, but the coming into being of an obligation
of their respective subscriptions was an attempted should not be confused with the time when it becomes
withdrawal of so much capital from the fund upon which demandable. In a loan for example, the obligation to pay
the company's creditors were entitled ultimately to rely arises from the time the loan is taken; but the maturity of
and, having been effected without compliance with the that obligation, the date when the debtor can be
statutory requirements, was wholly ineffectual. compelled to pay, is not the date itself of the loan,
because this would be absurd. The date when payment
can be demanded is necessarily distinct from and
subsequent to that the obligation is contracted.
By the same token, the subscription to the
GARCIA, plaintiff capital stock of the corporation, unless otherwise
vs. stipulation, is not payable at the moment of the
SUAREZ, defendant subscription but on a subsequent date which may be
G.R. No. L-45493 April 21, 1939 fixed by the corporation. Hence, section 38 of the
Corporation Law, amended by Act No. 3518, provides
FACTS: that: The board of directors or trustees of any stock
corporation formed, organized, or existing under this Act
may at any time declare due and payable to the
corporation unpaid subscriptions to the capital stock.
The board of directors of the Compañia
Hispano-Filipino, Inc., not having declared due and
payable the stock subscribed by the appellant, the
prescriptive period of the action for the collection thereof
only commenced to run from June 18, 1931 when the
plaintiff, in his capacity as receiver and in the exercise of
the power conferred upon him by the said section 38 of
the Corporation Law, demanded of the appellant to pay
the balance of his subscription. The present action
having been filed on October 10, 1935, the defense of
prescription is entirely without basis.
TAN, petitioner Besides, in Philippine jurisprudence, a certificate
vs. of stock is not a negotiable instrument. "Although it is
SECURITIES AND EXCHANGE COMMISSION, sometime regarded as quasi-negotiable, in the sense
respondent that it may be transferred by endorsement, coupled with
G.R. No. 95696 March 3, 1992 delivery, it is well-settled that it is non-negotiable,
because the holder thereof takes it without prejudice to
FACTS: such rights or defenses as the registered owner/s or
transferor's creditor may have under the law, except
Respondent corporation was registered on insofar as such rights or defenses are subject to the
October 1, 1979. As incorporator, petitioner had four limitations imposed by the principles governing
hundred (400) shares of the capital stock standing in his estoppel."
name at the par value of P100.00 per share, evidenced To follow the argument put up by petitioner
by Certificate of Stock No. 2. He was elected as which was upheld by the Cebu SEC Extension Office
President and subsequently reelected, holding the Hearing Officer, Felix Chan, that the cancellation of
position as such until 1982 but remained in the Board of Stock Certificate Nos. 2 and 8 was null and void for lack
Directors until April 19, 1983 as director. of delivery of the cancelled "mother" Certificate No. 2
On January 31, 1981, while petitioner was still whose endorsement was deliberately withheld by
the president of the respondent corporation, two other petitioner, is to prescribe certain restrictions on the
incorporators, namely, Antonia Y. Young and Teresita Y. transfer of stock in violation of the corporation law itself
Ong, assigned to the corporation their shares, as the only law governing transfer of stocks. While
represented by certificate of stock No. 4 and 5 after Section 47(s) grants stock corporations the authority to
which, they were paid the corresponding 40% corporate determine in the by-laws "the manner of issuing
stock-in-trade. certificates" of shares of stock, however, the power to
Petitioner's certificate of stock No. 2 was regulate is not the power to prohibit, or to impose
cancelled by the corporate secretary and respondent unreasonable restrictions of the right of stockholders to
Patricia Aguilar by virtue of Resolution No. 1981 (b), transfer their shares.
which was passed and approved while petitioner was
still a member of the Board of Directors of the
respondent corporation. APOLINARIO G. DE LOS SANTOS and ISABELO
Due to the withdrawal of the aforesaid ASTRAQUILLO
incorporators and in order to complete the membership vs.
of the five (5) directors of the board, petitioner sold fifty J. HOWARD MCGRATH ATTORNEY GENERAL OF
(50) shares out of his 400 shares of capital stock to his THE UNITED STATES, SUCCESSOR TO THE
brother Angel S. Tan. Another incorporator, Alfredo B. PHILIPPINE ALIEN PROPERTY ADMINISTRATION OF
Uy, also sold fifty (50) of his 400 shares of capital stock THE UNITED STATES, REPUBLIC OF THE
to Teodora S. Tan and both new stockholders attended PHILIPPINES
the special meeting, Angel Tan was elected director and G.R. No. L-4818 February 28, 1955
on March 27, 1981, the minutes of said meeting was
filed with the SEC. These facts stand unchallenged. FACTS:

ISSUE: This action involves the title to 1,600,000 shares


of stock of the Lepanto Consolidated Mining Co., Inc., a
Whether or not the cancellation and transfer of corporation duly organized and existing under the laws
petitioner's shares and Certificate of Stock No. 2 as well of the Philippines, hereinafter referred to, for the sake of
as the issuance and cancellation of Certificate of Stock brevity, as the Lepanto. Originally, one-half of said
No. 8 was patently and palpably unlawful, null and void, shares of stock were claimed by plaintiff, Apolinario de
invalid and fraudulent. los Santos, and the other half, by his co-plaintiff Isabelo
Astraquillo. During the pendency of this case, the latter
RULING: has allegedly conveyed and assigned his interest in and
to said half claimed by him to the former. The shares of
YES. stock in question are covered by several stock
certificates issued in favor of Vicente Madrigal, who is
Under the instant case, the fact of the matter is, registered in the books of the Lepanto as owner of said
the new holder, Angel S. Tan has already exercised his stocks and whose indorsement in blank appears on the
rights and prerogatives as stockholder and was even back of said certificates, all of which, except certificates
elected as member of the board of directors in the No. 2279 — marked Exhibit 2 — covering 55,000
respondent corporation with the full knowledge and shares, are in plaintiffs' possession. So was said Exhibit
acquiescence of petitioner. Due to the transfer of fifty 2, up to sometime in 1945 or 1946 when said
(50) shares, Angel S. Tan was clothed with rights and possession was lost under the conditions set forth in
responsibilities in the board of the respondent subsequent pages.
corporation when he was elected as officer thereof.
ISSUE:
1968, VCC was renamed Floro Cement Corporation
Whether or not the plaintiffs had the owners of (FCC); that on 22 October 1990, FCC was renamed
the shares of stock in question. Alsons Cement Corporation (ACC); that from the time of
incorporation of VCC up to the present, no certificates of
RULING: stock corresponding to the 239,500 subscribed and fully
paid shares of Gaid were issued in the name of Fausto
NO. G. Gaid and/or Ponce; and that despite repeated
demands, ACC and Giron refused and continue to
In the case at bar, neither madrigal nor the refuse without any justifiable reason to issue to Ponce
Mitsuis had alienated shares of stock in question. It is the certificates of stocks corresponding to the 239,500
not even claimed that either had, through negligence, shares of Gaid, in violation of Ponce's right to secure the
given — occasion for an improper or irregular disposition corresponding certificate of stock in his name. ACC and
of the corresponding stock certificates. Plaintiffs Giron moved to dismiss. SEC Hearing Officer Enrique L.
merely argue without any evidence whatsoever thereon Flores, Jr. granted the motion to dismiss in an Order
— that Kitajimamight have, or must have, assigned the dated 29 February 1996. Ponce appealed the Order of
certificates on or before December 1942, although, as dismissal. On 6 January 1997, the Commission En Banc
above stated, this is, not only, improbable, under the reversed the appealed Order and directed the Hearing
conditions, then obtaining, but, also., impossible, Officer to proceed with the case. In ruling that a transfer
considering that, in April 1943, Kitajima delivered the or assignment of stocks need not be registered first
instruments to Miwa, who kept them in its before it can take cognizance of the case to enforce
possession until 1945. At any rate, such assignment by Ponce's rights as a stockholder, the Commission En
Miwa — granting for the sake of argument the accuracy Banc cited the Supreme Court's ruling in Abejo vs. De la
of the surmise of plaintiffs herein — was unauthorized by Cruz, 149 SCRA 654 (1987). Their motion for
the mitsuis, who, in the light of the precedents cited reconsideration having been denied, ACC and Giron
above, are not chargeable with negligence. In other appealed the decision of the SEC En Banc and the
words, assuming that Kitajima had been guilty of resolution denying their motion for reconsideration to the
embezzlement, by negotiating the stock certificates in Court of Appeals. In its decision, the Court of Appeals
question for his personal benefit, as claimed by the held that in the absence of any allegation that the
plaintiffs, the title of his assignees and successors in transfer of the shares between Gaid and Ponce was
interest would still be subject to the rights of the registered in the stock and transfer book of ACC, Ponce
registered owner, namely, Madrigal, and consequently, failed to state a cause of action. Thus, said the appellate
of the party for whose benefit and account the latter held court, "the complaint for mandamus should be dismissed
the corresponding shares of stock, that is to say, the for failure to state a cause of action." Ponce's motion for
Mitsuis. reconsideration was denied in a resolution dated 10
In conclusion, when the Property Custodian August 1999. Ponce filed the petition for review on
issued the Vesting Order complained of, the shares of certiorari.
stock in question belonged to the Mitsuis, admittedly an
enemy corporation, so that Vesting Order is in Issue: Whether Ponce can require the corporate
conformity with law and should be upheld. Wherefore, secretary, Giron, to register Gaid’s shares in his name.
the decision appealed from is hereby reversed, and the
complaint, accordingly, dismissed, with costs against the
Held: Fausto Gaid was an original subscriber of ACC's
plaintiffs-appellees. It is so ordered.
239,500 shares. From the Amended Articles of
Incorporation approved on 9 April 1995, each share had
a par value of P1.00 per share. Ponce had not made a
previous request upon the corporate secretary of ACC,
Ponce vs. Alsons Cement Corporation Francisco M. Giron Jr., to record the alleged transfer of
[GR 139802, 10 December 2002] stocks. Pursuant to Section 63 of the Corporation Code,
Second Division, Quisumbing (J): 4 concur a transfer of shares of stock not recorded in the stock
and transfer book of the corporation is non-existent as
Facts: On 25 January 1996, Vicente C. Ponce, filed a far as the corporation is concerned. As between the
complaint with the SEC for mandamus and damages corporation on the one hand, and its shareholders and
against Alsons Cement Corporation and its corporate third persons on the other, the corporation looks only to
secretary Francisco M. Giron, Jr. In his complaint, Ponce its books for the purpose of determining who its
alleged, among others, that "the late Fausto G. Gaid was shareholders are. It is only when the transfer has been
an incorporator of Victory Cement Corporation (VCC), recorded in the stock and transfer book that a
having subscribed to and fully paid 239,500 shares of corporation may rightfully regard the transferee as one of
said corporation; that on 8 February 1968, Ponce and its stockholders. From this time, the consequent
Fausto Gaid executed a "Deed of Undertaking" and obligation on the part of the corporation to recognize
"Indorsement" whereby the latter acknowledges that the such rights as it is mandated by law to recognize arises.
former is the owner of said shares and he was therefore Hence, without such recording, the transferee may not
assigning/endorsing the same to Ponce; that on 10 April be regarded by the corporation as one among its
stockholders and the corporation may legally refuse the
issuance of stock certificates in the name of the corporate funds thereby causing losses to the
transferee even when there has been compliance with corporation.
the requirements of Section 64 of the Corporation Code.
The stock and transfer book is the basis for ascertaining Plaintiff Ricardo Gurrea sold and transferred a number of
the persons entitled to the rights and subject to the his shares of stock in the corporation to several
liabilities of a stockholder. Where a transferee is not yet individuals, but the defendants, as president-manager
recognized as a stockholder, the corporation is under no and secretary respectively, refused to register in the
specific legal duty to issue stock certificates in the corporate books to transfer and to issue the
transferee's name. A petition for mandamus fails to state corresponding certificates of stock.
a cause of action where it appears that the petitioner is
not the registered stockholder and there is no allegation
To avoid irreparable damages to the corporation which is
that he holds any power of attorney from the registered
on the verge of insolvency, the plaintiffs pray that an
stockholder, from whom he obtained the stocks, to make order ex parte be issued appointing Ricardo Gurrea
the transfer. The deed of undertaking with indorsement receiver of all funds, assets and business of the
presented by Ponce does not establish, on its face, his corporation.
right to demand for the registration of the transfer and
the issuance of certificates of stocks. Under the
provisions of our statute touching the transfer of stock, After the issues had been joined by the parties and while
the mere indorsement of stock certificates does not in the case was pending trial, a motion was filed by Ricardo
itself give to the indorsee such a right to have a transfer Jeruta, Jr., Felipe Espinosa and Isidro Perlado alleging
of the shares of stock on the books of the company as that on 4 August 1958 plaintiff Ricardo Gurrea, who is
will entitle him to the writ of mandamus to compel the the registered owner of 134 shares of stock of the La
company and its officers to make such transfer at his Paz Ice Plant & Cold Storage Co., Inc., evidenced by
demand, because, under such circumstances the duty, certificate of stock No. 14 of the corporation, had sold
the legal obligation, is not so clear and indisputable as to seven shares thereof to the movants and that before and
justify the issuance of the writ. As a general rule, as after the court had appointed Jose Dineros receiver of
between the corporation on the one hand, and its the corporation on 26 September 1958 the movants
shareholders and third persons on the other, the exerted efforts to have said shares of stock registered in
corporation looks only to its books for the purpose of their names in the books of the corporation, "but the
determining who its shareholders are, so that a mere persons concerned refused to do so;" and praying that
indorsee of a stock certificate, claiming to be the owner, the trial court order the receiver to record such transfer
will not necessarily be recognized as such by the of shares of stock in the books of the corporation and to
corporation and its officers, in the absence of express issue in their names the certificates of the shares of
instructions of the registered owner to make such stock sold to them in lieu of those of plaintiff Gurrea.
transfer to the indorsee, or a power of attorney
authorizing such transfer. Thus, absent an allegation that Acting on the motion, the Court rendered an order
the transfer of shares is recorded in the stock and ordering the Secretary of the corporation to transfer in
transfer book of ACC, there appears no basis for a clear the books of the defendant corporation the sale of the
and indisputable duty or clear legal obligation that can shares of stock of Ricardo Gurrea in favor of Ricardo
be imposed upon the corporate secretary, so as to justify Jeruta, Jr., Felipe Espinosa and Isidro Perlado after
the issuance of the writ of mandamus to compel him to surrendering the certificates to the secretary of the
perform the transfer of the shares to Ponce. corporation.

On 18 November 1958, the defendant secretary moved


for a reconsideration and setting aside of the above
C. N. HODGES and RICARDO GURREA, in their own order, on the ground that the duly-appointed receiver of
behalf as majority stockholders and for and in behalf the corporation should decide what to do with the
of the La Paz Ice Plant & Cold Storage Co., Inc., aforesaid transfer of shares.
plaintiffs-appellees,
vs. The movants filed an objection to the defendant's
JOSE MANUEL LEZAMA, defendant,
(Paquita B. Lezama's) motion for reconsideration of the
PAQUITA B. LEZAMA, defendant-appellant.
order of 29 November 1958 contending that as a result
of the appointment of a receiver, the court may order the
FACTS:
secretary of the corporation to transfer in the corporate
A complaint was filed by plaintiffs C. N. Hodges and books the sales of the shares of stock of plaintiff Gurrea
Ricardo Gurrea (stockholders of La Paz Ice Plant & Cold to the three movants, the conveyance being a business
Storage Co., Inc.) against the defendants Jose Manuel transaction and the properties of the corporation being in
Lezama and Paquita B. Lezama (President-Manager the hands of the receiver, or in custodia legis.
and Secretary) alleging that Jose Manuel Lezama
mismanaged the corporation and misappropriated the ISSUE:
Whether or not the order of the Court is proper. Sec. 224.Stamp tax on original issues of certificates of
stock. — On every original issue, whether on
RULING: organization, reorganization or for any lawful purpose, of
certificates of stock by any association, company or
The plaintiff Ricardo Gurrea is the registered owner of corporation, there shall be collected a documentary
215 shares of the capital stock of the corporation. On 4 stamp tax of one peso and ten centavos on each two
August 1958, in a notarized instrument, he sold seven of
hundred pesos, or fractional part thereof, of the par
the shares to herein movants-appellees and on 26
September 1958 the corporation was placed under value of such certificates: Provided, That in the case of
receivership. Despite surrender of the owner's (Gurrea's) the original issue of stock without par value the amount
certificate No. 14 and efforts exerted by the purchasers of the documentary stamp tax herein prescribed shall be
(movants) to have the transfer registered in the "stock based upon the actual consideration received by the
and transfer books" of the corporation, the appellant as association, company, or corporation for the issuance of
secretary of the corporation refused the registration and such stock, and in the case of stock dividends on the
transfer. The appellant never questioned the legality of
the transfer sought to be registered, she having admitted actual value represented by each share.
in her answer that Gurrea owned 215 shares of stock of
the corporation. The Commissioner of Internal Revenue took the view
that the book value of the shares, amounting to
This Court has ruled and held that a trial court has P19,307,500.00, should be used as basis for
jurisdiction to order a receiver of a corporation under determining the amount of the documentary stamp tax.
receivership to do any act so as to protect and preserve Accordingly, respondent Internal Revenue
his properties, and to that end it may order the secretary Commissioner issued a deficiency documentary stamp
to do an act within the internal affairs of the corporation tax assessment in the amount of P78,991.25 in excess
aimed at protecting the interests of the stockholders.1 of the par value of the stock dividends.
Sections 35 and 52 of the Corporation Law, as amended
by Act 3471, which require that all transfers of shares to
Together with another documentary stamp tax
be valid as far as the corporation is concerned must be
entered and noted upon the books of the corporation, assessment which it also questioned, petitioner
contemplate no restriction as to whom the shares may appealed the Commissioner's ruling to the Court of Tax
be transferred or sold.2 The assets and business of the Appeals. The CTA rendered its decision holding that the
corporation having been placed under receivership, the amount of the documentary stamp tax should be based
court is in duty bound and has the authority to require on the par value stated on each certificate of stock.
the appellant as secretary of the corporation to perform
her duties under the law. Respondent Commissioner of Internal Revenue
appealed to the Court of Appeals which, reversed the
CTA's decision and held that, in assessing the tax in
question, the basis should be the actual value
G.R. No. 118043 July 23, 1998
represented by the subject shares on the assumption
LINCOLN PHILIPPINE LIFE INSURANCE COMPANY, that stock dividends, being a distinct class of shares, are
INC. (now JARDINE-CMG LIFE INSURANCE CO. not subject to the qualification in the law as to the type of
INC.), petitioner,vs. certificate of stock used (with or without par value).

COURT OF APPEALS and COMMISSIONER OF Issue: Whether or not respondent court of appeals erred
INTERNAL REVENUE, respondents. in holding that stock dividends involving shares with par
value are subject to documentary stamp tax based on
Facts: Petitioner, now the Jardine-CMG Life Insurance the book value of said shares?
Company, Inc., is a domestic corporation engaged in the
life insurance business. It issued 50,000 shares of stock Ruling: Yes. Conformably, in the case of stock
as stock dividends, with a par value of P100 or a total of certificates with par value, the documentary stamp tax is
P5 million. Petitioner paid documentary stamp taxes on based on the par value of the stock; for stock certificates
each certificate on the basis of its par value. The without par value, the same tax is computed from the
question in this case is whether in determining the actual consideration received by the corporation,
amount to be paid as documentary stamp tax, it is the association or company; but for stock dividends,
par value of the certificates of stock or the book value of documentary stamp tax is to be paid "on the actual value
the shares which should be considered. The pertinent represented by each share."
provision of law, as it stood at the time of the questioned
transaction, reads as follows:
Since in dividends, no consideration is technically mismanagement in directing the affairs of the
received by the corporation, petitioner is correct in corporation to the prejudice of the stockholders. She
basing the assessment on the book value thereof alleges that certain transactions entered into by the
corporation were not supported by any stockholder’s
rejecting the principles enunciated in Commissioner of
resolution. The complaint sought to enjoin Apostol from
Internal Revenue vs. Heald Lumber Co. (10 SCRA 372) further acting as president-director of the corporation
as the said case refers to purchases of no-par and from disbursing any money or funds.
certificates of stocks and not to stock dividends. Apostol contends that Bitong was merely a
holder-in-trust of the JAKA shares of the corporation,
Apparently, the Court of Appeals treats stock dividends hence, not entitled to the relief she prays for. SEC
as distinct from ordinary shares of stock for purposes of Hearing Panel issued a writ enjoining Apostol. After
the then §224 of the National Internal Revenue Code. hearing the evidence, SEC Hearing Panel dissolved the
There is, however, no basis for considering stock writ and dismissed the complaint filed by Bitong. Bitong
appealed to the SEC en banc which reversed SEC
dividends as a distinct class from ordinary shares of
Hearing Panel decision. Apostol filed petition for review
stock since under this provision only certificates of stock with the CA. CA reversed SEC en banc ruling holding
are required to be distinguished (into either one with par that Bitong was not the owner of any share of stock in
value or one without) rather than the classes of shares the corporation and therefore, not a real party in interest
themselves. to prosecute the complaint.

Stock dividends are in the nature of shares of stock, the ISSUE:


consideration for which is the amount of unrestricted
Whether or not petitioner validly declared
retained earnings converted into equity in the
dividends.
corporation's books. Thus,
RULING:
A "stock dividend'' is any dividend payable in shares of
stock of the corporation declaring or authorizing such YES.
dividend. It is, what the term itself implies, a distribution
of the shares of stock of the corporation among the The records show that the original stock and
stockholders as dividends. A stock dividend of a transfer book and the stock certificate book of Mr. & Ms.
corporation is a dividend paid in shares of stock instead were in the possession of petitioner before their custody
was transferred to the Corporate Secretary, Atty.
of cash, and is properly payable only out of surplus
Augusto San Pedro. On 25 May 1988, Assistant
profits. So, a stock dividend is actually two things: (1) a Corporate Secretary Renato Jose Unson wrote Mr.& Ms.
dividend and (2) the enforced use of the dividend money about the lost stock and transfer book which was also
to purchase additional shares of stock at par. noted by the corporation's external auditors,
Punongbayan and Araullo, in their audit. Atty. Unson
even informed respondent Eugenia D. Apostol as
President of Mr. & Ms. that steps would be undertaken to
prepare and register a new Stock and Transfer Book
with the SEC. Incidentally, perhaps strangely, upon
NORA A. BITONG verification with the SEC, it was discovered that the
vs. general file of the corporation with the SEC was missing.
COURT OF APPEALS (FIFTH DIVISION), EUGENIA D. Hence, it was even possible that the original Stock and
APOSTOL, JOSE A. APOSTOL, MR. & MS. Transfer Book might not have been registered at all.
PUBLISHING CO., LETTY J. MAGSANOC, AND This simply shows that as of 1988 there still
ADORACION G. NUYDA existed certain issues affecting the ownership of the
G.R. No. 123553. July 13, 1998 JAKA shares, thus raising doubts whether the alleged
NORA A. BITONG transactions recorded in the Stock and Transfer Book
vs. were proper, regular and authorized. JAKA retained its
COURT OF APPEALS (FIFTH DIVISION) and ownership of Mr.& Ms. shares as clearly shown by its
EDGARDO B. ESPIRITU receipt of the dividends issued in December 1986. This
(CA-G.R. No. 33873) July 13, 1998 only means, very obviously, that Mr.& Ms. shares in
question still belonged to JAKA and not to petitioner.
FACTS: For, dividends are distributed to stockholders pursuant to
their right to share in corporate profits. When a dividend
Bitong alleged that she was the treasurer and is declared, it belongs to the person who is the
member of the BoD of Mr. & Mrs. Corporation. She filed substantial and beneficial owner of the stock at the time
a complaint with the SEC to hold respondent spouses regardless of when the distribution profit was earned.
Apostol liable for fraud, misrepresentation, disloyalty,
evident bad faith, conflict of interest and
Bachrach Motor Co. VS. Lacson Ledesma Plaintiff said it had a preferred right over the
Facts: 6,300 shares because the stocks were in custodia legis
Bachrach obtained judgment (in 1927) against by virtue of the attachment/garnishment when Cert 772
Ledesma in two civil cases. The sheriff, in compliance was delivered to PNB, and when Talisay issued Cert
with the writ of execution issued in favor of Bachrach, 1155 in favor of PNB. This contention was unfounded as
attached and sold the right of redemption of Ledesma it appeared that the stocks were pledged to the bank
over several properties, and attached as well all right, prior to the garnishment.
title to and interest that Ledesma had in “Any bonus, Cert 772 was delivered to PNB on Feb 27, 1930.
dividend, shares of stock, money, or other property The garnishment was notified to the parties and became
which Ledesma was entitled to receive from Talisay- effective on August 11, 1930, more than five months
Silay Milling Co. Inc. on account of being a stockholder after delivery. On Feb, 1931, Talisay issued Cert 1155 in
in that corporation or which he is entitled to receive from favor of PNB.
that corporation for any other cause or pretext According to Article 1865 of the Civil Code then,
whatsoever.” The properties and the shares Ledesma in order that a pledge may be effective as against third
owned in Talisay were mortgaged to PNB as securities persons, evidence of its date must appear in a public
to ensure his payment of P624,000. There was another instrument in addition to the delivery of the thing pledged
mortgage over the real properties in favor of PNB to to the creditor. However, Sec. 4 of the Chattel Mortgage
answer for the debts of Central Talisay-Silay Milling. Law implicitly modified 1865 – a contract of pledge and
Central resolved to grant a bonus or compensation to that of chattel mortgage need not appear in public
the owners of the properties mortgaged for the risk instruments to be effective against third persons,
incurred from being subjected to said mortgage lien. provided that delivery was made. Therefore, the pledge
Under the resolution, Ledesma was allotted P19,911.11. of the 6,300 shares was valid against Bachrach.
This was payable only in January, 1930. PNB brought an The contention that a certificate of stock or of
action against Ledesma and his wife for recovery of stock dividends cannot be the subject matter of contract
mortgage credit (1928). In 1929, they amended the of pledge or chattel mortgage was untenable.
complaint to include Bachrach, “because they claim to Certificates of stock or of stock dividends are quasi
have some right to certain properties which are the negotiable instruments. They may be given in pledge or
subject matter of the complaint.” The court ruled in favor mortgage to secure an obligation. They are transferable,
of PNB, and ordered the sale of properties mortgaged. when properly indorsed, by mere delivery, and by
PNB was also granted the authority to sell the stock estoppel against the corporation or against prior holders,
certificates. as good a title to the transferee as if they were
During the pendency of the case of PNB v. Ledesma, negotiable. It is to the public interest that such use
Bachrach filed an action against Talisay to recover should be simplified and facilitated by placing them as
P13,850 which by virtue of the resolution was bestowed nearly as possible on the plane of commercial paper.
upon Ledesma by Central. PNB intervened, alleging a
preferred right, as said bonus being a civil fruit of the
mortgaged lands, the bank became entitled to it as the
mortgage had become due. Judgment was rendered in ENRIQUE RAZON
favor of Bachrach. The SC held that the bonus had no vs.
immediate relation to the lands in question but merely a INTERMEDIATE APPELLATE COURT and VICENTE
remote and accidental one. It was not a civil fruit, being a B. CHUIDIAN, in his capacity as Administrator of the
mere personal right of Ledesma. Estate of the Deceased JUAN T. CHUIDIAN
In January, 1930, Stock Cert. 772 was issued in G .R. No. 74306 March 16, 1992
favor of Ledesma by Talisay. Ledesma ordered this to
be delivered to PNB. The 6,300 shares constituted the FACTS:
2,100 original shares that was given as pledge to PNB
under the deed of mortgage. On Feb. 1931, the sheriff
In his complaint filed on June 29, 1971, and
sold the whole 6,300 shares covered by 772, and not amended on November 16, 1971, Vicente B. Chuidian
only the 2,100 original shares. PNB informed Talisay of prayed that defendants Enrique B. Razon, E. Razon,
the sale, and Talisay issued Stock Cert. 1155
Inc., Geronimo Velasco, Francisco de Borja, Jose
representing 8,968 shares (6,300 + 2,100).
Francisco, Alfredo B. de Leon, Jr., Gabriel Llamas and
Luis M. de Razon be ordered to deliver certificates of
Issues:
stocks representing the shareholdings of the deceased
-W/N Bachrach had a preferred right by virtue of Juan T. Chuidian in the E. Razon, Inc. with a prayer for
the judgment and attachment made (1927) - NO W/N the an order to restrain the defendants from disposing of the
pledge was ineffective as against Bachrach because
said shares of stock, for a writ of preliminary attachment
evidence of its date was not made to appear in a public
v. properties of defendants having possession of shares
instrument – NO
of stock and for receivership of the properties of
-W/N the pledge could not legally exist as the
defendant corporation.
Cert. was not the shares themselves - NO In their answer filed on June 18, 1973,
HELD: defendants alleged that all the shares of stock in the
name of stockholders of record of the corporation were
fully paid for by defendant, Razon; that said shares are vs.
subject to the agreement between defendants and HONORABLE COURT OF APPEALS, HONORABLE
incorporators; that the shares of stock were actually COMMISSION EN BANC, SECURITIES AND
owned and remained in the possession of Razon. EXCHANGE COMMISSION, HONORABLE ENRIQUE
Appellees also alleged . . . that neither the late Juan T. L. FLORES, JR., in his capacity as Hearing Officer,
Chuidian nor the appellant had paid any amount REYNALDO VILLANUEVA, SR, AVELINA M.
whatsoever for the 1,500 shares of stock in question VILLANUEVA, CATALINO VILLANUEVA, ANDRES
GONZALES, AURORA LACERNA, CELSO LAYGO,
ISSUE: EDGARDO REYES, ALEJANDRA TONOGAN
G.R. No. 124535 September 28, 2001
Whether or not petitioner have right over the
ownership of the 1,500 shares of stock in E. Razon, Inc. FACTS:

RULING: Private respondent Reynaldo Villanueva, Sr., a


stockholder of the Rural Bank of Lipa City, executed a
NO. Deed of Assignment, wherein he assigned his shares, as
well as those of eight (8) other shareholders under his
In the instant case, there is no dispute that the control with a total of 10,467 shares, in favor of the
questioned 1,500 shares of stock of E. Razon, Inc. are in stockholders of the Bank represented by its directors
the name of the late Juan Chuidian in the books of the Bernardo Bautista, Jaime Custodio and Octavio
corporation. Moreover, the records show that during his Katigbak. Sometime thereafter, Reynaldo Villanueva, Sr.
lifetime Chuidian was ellected member of the Board of and his wife, Avelina, executed an Agreement wherein
Directors of the corporation which clearly shows that he they acknowledged their indebtedness to the Bank in the
was a stockholder of the corporation. From the point of amount of Four Million Pesos (P4,000,000.00), and
view of the corporation, therefore, Chuidian was the stipulated that said debt will be paid out of the proceeds
owner of the 1,500 shares of stock. In such a case, the of the sale of their real property described in the
petitioner who claims ownership over the questioned Agreement. At a meeting of the Board of Directors of the
shares of stock must show that the same were Bank on November 15, 1993, the Villanueva spouses
transferred to him by proving that all the requirements for assured the Board that their debt would be paid on or
the effective transfer of shares of stock in accordance before December 31 of that same year; otherwise, the
with the corporation's by laws, if any, were followed or in Bank would be entitled to liquidate their shareholdings,
accordance with the provisions of law. including those under their control.
The petitioner failed in both instances. The When the Villanueva spouses failed to settle
petitioner did not present any by-laws which could show their obligation to the Bank on the due date, the Board
that the 1,500 shares of stock were effectively sent them a letter3 demanding: (1) the surrender of all
transferred to him. In the absence of the corporation's the stock certificates issued to them; and (2) the delivery
by-laws or rules governing effective transfer of shares of of sufficient collateral to secure the balance of their debt
stock, the provisions of the Corporation Law are made amounting to P3,346,898.54.
applicable to the instant case.
The law is clear that in order for a transfer of ISSUE:
stock certificate to be effective, the certificate must be
properly indorsed and that title to such certificate of Whether or not the transfer of title to such
stock is vested in the transferee by the delivery of shares is ineffective until and unless the duly indorsed
the duly indorsed certificate of stock. Since the certificate certificate of stock is delivered to them notwithstanding
of stock covering the questioned 1,500 shares of stock the execution of the deed of assignment in favor of the
registered in the name of the late Juan Chuidian was petitioners.
never indorsed to the petitioner, the inevitable
conclusion is that the questioned shares of stock belong RULING:
to Chuidian. The petitioner's asseveration that he did not
require an indorsement of the certificate of stock in view NO.
of his intimate friendship with the late Juan Chuidian can
not overcome the failure to follow the procedure required The rule is that the delivery of the stock
by law or the proper conduct of business even among certificate duly endorsed by the owner is the operative
friends. To reiterate, indorsement of the certificate of act of transfer of shares from the lawful owner to the
stock is a mandatory. transferee. Thus, title may be vested in the transferee
only by delivery of the duly indorsed certificate of stock.
We have uniformly held that for a valid transfer
of stocks, there must be strict compliance with the mode
BERNARDO BAUTISTA, JAIME CUSTODIO, of transfer prescribed by law. The requirements are: (a)
OCTAVIO KATIGBAK, FRANCISCO CUSTODIO, and There must be delivery of the stock certificate: (b) The
JUANITA BAUTISTA OF THE RURAL BANK OF LIPA certificate must be endorsed by the owner or his
CITY, INC., attorney-in-fact or other persons legally authorized to
make the transfer; and (c) To be valid against third members of the Board in the annual stockholders'
parties, the transfer must be recorded in the books of the meeting held in May 1954, largely on the vote of their co-
corporation. As it is, compliance with any of these defendant Acena, while the other two (2) were elected
requisites has not been clearly and sufficiently shown. mainly on the vote of the plaintiffs and their group of
It may be argued that despite non-compliance stockholders. Let the first group be called theUngson
with the requisite endorsement and delivery, the group and the second, the Baltazar group.
assignment was valid between the parties, meaning the
private respondents as assignors and the petitioners as ISSUE:
assignees. While the assignment may be valid and
binding on the petitioners and private respondents, it Whether or not a stockholder, in a stock
does not necessarily make the transfer effective. corporation, subscribes to a certain number of shares of
Consequently, the petitioners, as mere assignees, stock, and he pays only partially, for which he is issued
cannot enjoy the status of a stockholder, cannot vote nor certificates of stock, is he entitled to vote the latter,
be voted for, and will not be entitled to dividends, insofar notwithstanding the fact that he has not paid the balance
as the assigned shares are concerned. Parenthetically, of his subscription, which has been called for payment or
the private respondents cannot, as yet, be deprived of declared delinquent.
their rights as stockholders, until and unless the issue of
ownership and transfer of the shares in question is RULING:
resolved with finality.
YES.

The cases at bar do not come under the aegis of


the principle enunciated in the Fua Cun v.
BALTAZAR, plaintiff
Summers case, because it was the practice and
vs.
LINGAYEN GULF ELECTRIC POWER CO., INC., procedure, since the inception of the corporation, to
respondent issue certificates of stock to its individual subscribers for
G.R. No. L-16236 June 30, 1965 unpaid shares of stock and gave voting power to shares
of stock fully paid. And even though no agreement
FACTS: existed, the ruling in said case does not now reflect the
correct view on the matter, for better than an agreement
The Lingayen Gulf Electric Power Co., Inc., or practice, there is the law, which renders the said case
hereinafter referred to as Corporation, was doing of Fua Cun-Summers, obsolescent.
In the cases at bar, the defendant-corporation
business in the Philippines, with principal offices at
had chosen to apply payments by its stockholders to
Lingayen, Pangasinan, and with an authorized capital
definite shares of the capital stock of the corporation and
stock of P300.000.00 divided into 3,000 shares of voting
had fully paid capital stock shares certificates for said
stock at P100.00 par value, per share. Plaintiffs Baltazar
and Rose were among the incorporators, having payments; its call for payment of unpaid subscription and
subscribed to 600 and 400 shares of the capital stock, or its declaration of delinquency for non-payment of said
call affecting only the remaining number of shares of its
a total par value of P60,000.00 and P40.000.00,
capital stock for which no fully paid capital stock shares
respectively. It is alleged that it has always been the
certificates have been issued, "and only these have
practice and procedure of the Corporation to issue
been legally shorn of their voting rights by said
certificates of stock to its individual subscribers for
unpaid shares of stock. Of the 600 shares of capital declaration of delinquency" (amended decision).
stock subscribed by Baltazar, he had fully paid 535
shares of stock, and the Corporation issued to him
several fully paid up and non-assessable certificates of
stock, corresponding to the 535 shares. After having SANTAMARIA, assisted by her husband,
made transfers to third persons and acquired new ones, FRANCISCO SANTAMARIA, Jr. vs.
Baltazar had to his credit, on the filing of the complaint THE HONGKONG AND SHANGHAI BANKING
341 shares fully paid and non-assessable. CORPORATION and R. W. TAPLIN.
The respondents Ungson, Estrada, Fernandez G.R. No. L-2808 August 31, 1951
and Yuson were small stockholders of the Corporation,
all holding a total number of fully paid-up shares of FACTS:
stock, of not more than 100 shares, with a par value of
P10,000.00 and the defendant Acena, was likewise an Mrs. Josefa T. Santamaria bought 10,000
incorporator and stockholder, holding 600 shares of shares of the Batangas Minerals, Inc., through the
stock, for which certificate of stock were issued to him offices of Woo, Uy-Tioco & Naftaly, a stock brokerage
and as such, was the largest individual stockholder firm and pay therefore the sum of P8,041.20 as shown
thereof. Defendants Ungson, Estrada, Fernandez and by receipt Exh. B. The buyer received Stock Certificate
Yuzon, constituted the majority of the holdover seven- No. 517 issued in the name of Woo, Uy-Tioco & Naftaly
member Board of Directors of the Corporation, in 1955, and indorsed in bank by this firm.
two (2) of said defendants having been elected as
On March 9, 1937, Mrs. Santamaria placed an of hypothecation. Even if said certificate had been in the
order for the purchase of 10,000 shares of the Crown name of the plaintiff but indorsed in blank, the Bank
Mines, Inc. with R.J. Campos & Co., a brokerage firm, would still have been justified in believing that R.J.
and delivered Certificate No. 517 to the latter as security Campos & Co., Inc. had title thereto for the reason that it
therefor with the understanding that said certificate is a well-known practice that a certificate of stock,
would be returned to her upon payment of the 10,000 indorsed in blank, is deemed quasi negotiable, and as
Crown Mines, Inc. shares. Exh. D. is the receipt of the such the transferee thereof is justified in believing that it
certificate in question signed by one Mr. Cosculluela, belongs to the holder and transferor.
Manager of the R.J. Campos & Co., Inc. According to
certificate Exh. E, R. J. Campos & Co., Inc. bought for
Mrs. Josefa Santamaria 10,000 shares of the Crown
Mines, Inc. at .225 a share, or the total amount of
NEUGENE M ARKETING INC., ET AL, PETITIONERS
P2,250. Two days later, on March 11, Mrs. Santamaria
V.
went to R.J. Campos & Co., Inc. to pay for her order of
10,000 Crown Mines shares and to get back Certificate COURT OF APPEALS ET AL, RESPONDENTS 1
No. 517. Cosculluela then informed her that R.J. GR no. 112941 February 18, 1999
Campos & Co., Inc. was no longer allowed to transact Purisima, J.
business due to a prohibition order from Securities and SV: 3 out of 5 shareholders of Neugene voted for the
Exchange Commission. She was also inform that her dissolution of the corporation. The dissolution was
Stock certificate was in the possession of the Hongkong
acknowledged by the SEC. The 2 other stockholders
and Shanghai Banking Corporation.
questioned the dissolution alleging that at the time the
ISSUE: meeting for voting for the dissolution was held, the 3 did
not own the requisite 2/3 of the outstanding capital stock
Whether or not the obligation of the defendant to validly vote for the dissolution since majority of their
Bank to have inquired into the ownership of the shares of stock has already been transferred to new
certificate when it received it from R.J. Campos & Co., stockholders. SEC held that the dissolution is not valid
Inc. and not conclude that the Bank was negligent for not
but CA reversed the decision
having done so, contrary to the claim of the plaintiff that
defendant Bank acted negligently, if not in bad faith, in SC: the dissolution is valid. The transfers were
accepting delivery of said certificate from RJ. Campos & fraudulently made. The stocks after issuance by the
Co., Inc. corporation, were endorsed in blank to the UY family
(the beneficial owner of Neugene) for safekeeping. This
RULING: was agreed upon by the stockholders and the Uy Family.
For the transfer to be valid it was agreed upon that the
YES.
owners of stocks intended to be transferred must give its
Certificate No. 517 came into the possession of approval first. It was also found by the court that there
the defendant Bank because R.J. Campos & Co., Inc. was no consideration for the transfer of stocks.
had opened an overdraft account with said Bank and to
this effect it had executed on April 16, 1946, a letter of
hypothecation by the terms of which R.J. Campos & Co., 1. Neugene was duly registered with the SEC to
Inc. pledged to the said Bank "all Stocks, Shares and engage in the trading business for a term of 50
Securities which I/we may hereafter come into their years with the ff incorporators: Johnson Lee, Lok
possession on my/our account and whether originally Chun Suen, Charles O. Sy, Eugenio Flores, Jr.,
deposited for safe custody only or for any other purpose Arsenio Yang Jr.
whatever or which may hereafter be deposited by me/us 2. The authorized capital stock of Neugene is
in lieu of or in addition to the Stocks, Shares, and PhP3Million divided into 30,000 shares with a
Securities now deposited or for any other purpose par value of 100 pesos each. 600,000 has been
whatsoever." subscribed by the ff
It should be noted that the certificate of stock in a. Johnson Lee: 600 shares
question was issued in the name of the brokerage firm- P60,000
Woo, Uy-Tioco & Naftaly and that it was duly indorsed in b. Lok Chun Suen: 1,200 shares
blank by said firm, and that said indorsement was P120,000
guaranteed by R.J. Campos & Co., Inc., which in turn
indorsed it in blank. This certificate is what it is known as 1NEUGENE MARKETING INC., LEONCIO TAN, NICANOR
street certificate. Upon its face, the holder was entitled to
MARTIN, SONNY MORENO, JOHNSON LEE and SECURITIES
demand its transfer into his name from the issuing
AND EXCHANGE COMMISSION, petitioners,
corporation. The Bank was not obligated to look beyond
vs.
the certificate to ascertain the ownership of the stock at
the time it received the same from R.J. Campos & Co., COURT OF APPEALS, ARSENIO YANG, JR., CHARLES O. SY,
Inc., for it was given to the Bank pursuant to their letter LOK CHUN SUEN, BAN HUA U. FLORES, BAN HA U. CHUA
and ROGER REYES, respondents.
c. Charles O. Sy 1,800 Shares d. Lee sold the said shares of stock to
P180,000 Leoncio Tan (2,100 out of 2,800 shares
d. Eugenio Flores, Jr. 2,100 shares of Charles O. Sy were assigned to him
P210,000 in addition to the 350 out of 1,050
e. Arsenio Yang Jr. 300 shares shares of Yang) and Nicanor Martin
P30,000 (1,400 shares of Suen were assigned to
3. Out of the aforesaid subscription 150,000 had Martin)
been paid by the ff: e. That Yang and Sy who each had 700
a. Johnson Lee shares, and Suen, who was no longer a
P15,000 stockholder could no longer validly vote
b. Lok Chun Suen for the dissolution of Neugene, hence
P30,000 the proceedings and the meetings were
c. Charles O. Sy improperly called and are null and void
P450,000 11. The respondents , SY, YANG and SUEN
d. Eugenio Flores, Jr. theorized that the alleged assignments of shares
P52,500 of stock in favor of Tan and Martin were
e. Arsenio Yang Jr. simulated and fraudulently effected as there
P7,500 were never an agreement entered into by the Uy
4. The shareholdings were increased by 10% by Family to award the NEugene shares of stock.
virtue of stock dividend declaration in the amt of They delivered the shares of stock to the Uy
60,000 pesos. Again it declared a stock dividend Family for safe keeping and the said certificates
later amounting to P40,000 in proportion to the were kept inside the confidential vault of the Uy
shareholdings of the stockholders as of April 30, Family but were stolen by Johnny KH Uy and
1981. Magdalena Go-Uy without the knowledge and
5. Eugenio Flores Jr. transferred and conveyed hi authority of the Uy Family
entire shareholdings of 2450 shares to a. That Sonny Moreno conspired with the
a. Sonny Moreno 1,050 shares fraudulent transfer of stocks when he
b. Arsenio Yang Jr. 700 shares recorded the fraudulent assignments in
c. Charles O. Sy 700 shares the stock and transfer book
6. So now, the stockholders appearing in the stock 12. SEC nullified the certification on the dissolution
and transfer book are as follows of Neugene, ruling that the Sy and Yang no
a. Johnson Lee 700 longer hold the 2/3 of the outstanding capital
b. Lok Chun Suen 1,400 stocks of NEugene at the time they presented
c. Sonny Moreno 1,050 the petition for dissolution.
d. Charles O. Sy 2,800 13. CA reversed and upheld the validity of the
e. Arsenio Yang Jr. 1,050 dissolution ruling that the stock certificates were
7. Sy and Yang (total shares 5,250)sent notice to not validly transferred to Martin and Tan. That to
the directors for a board meeting. They also sent constitute a valid transfer a stock certificate must
notice for a special stockholder’s meeting on the be delivered and its delivery must be coupled
same day, November 30, 1987 to consider the with an intention of constituting the person to
dissolution of Neugene whom the stock is delivered the transferee. And
8. Sy, Yang and Suen, the directors and in order that there is a valid transfer, the person
stockholders then present voted for and to whom the certificates are endorsed to must be
approved a resolution dissolving NEugene a bona fide transferee for value
9. SEC issued a certificate of dissolution a. No receipt or transaction showing
10. The petitioners, Johnson Lee, Sonny Moreno, payment for the stocks.
Leoncio Tan and Nicanor Martin brought an b. The supposed partition and division of
action to annul or set aside the SEC certificate the properties of the Uy Family was a
on the dissolution of Neugene they alleged that: mere Xerox copy whose original was
a. They are the majority stockholders of never produced in court. It also
Neugene owning 80% of its outstanding contained erasures and/insertion and it
capital stock is written in Chinese with no translation
b. That on July 1, 1987 Yang , Sy and submitted.
Suen divested themselves of their 14. CA denied MR hence this petition
stockholdings when they endorsed their
stock certificates in blank and delivered ISSUE: Whether or not the dissolution was valid
the same to the Uy Family, the (meaning that the stockholders who voted for it
beneficial owners of NEugene. owned at least 2/3 of the outstanding capital stock)
c. The Uy Family agreed to award
YES
Neugene’s stock certificates to Johnny
KH Uy who authorized Lee to dispose of
the same
 Entries in the Stock and Transfer book support Corporation at a par value of P100 per share [P 50,000].
the conclusion arrived at by the CA that Suen, However, he only paid the sum of P 25,000 which is 1/2
SY and Yang owned at least 2/3 of the of the subscription price, in cash for which a receipt
outstanding capital stock on the day of the was issued.
meetings
 It was shown that they owned 5,250 shares out On May 18, 1921, Chua Soco executed a promissory
of the 7,000 outstanding capital stock, note in favor of the plaintiff Fua Cun for the sum of
constituting at least 2/3 P25,000 payable in ninety days and drawing interest at
 SC agreed with the conclusion that the the rate of 1 per cent per month, securing the note with a
certificates of stock were stolen and therefore chattel mortgage on the shares of stock subscribed for
not validly transferred and the transfers of stock by Chua Soco, who also endorsed the receipt above
relied upon by the petitioners were fraudulently mentioned and delivered it to the mortgagee.
recorded in the STB of NEugene under the
column “Certificates Cancelled” (this was relied The Fua Cun, the plaintiff, thereupon took the receipt to
upon by the petitioners as evidence that the 3 the manager of the defendant Bank and informed him of
did not own at least 2/3 of the outstanding the transaction with Chua Soco, but was told to await
capital stock) action upon the matter by the Board of Directors.
 SEC overlooked certain facts of substance and
value when it declared the dissolution as not Meanwhile, Chua Soco appears to have become
valid indebted to the China Banking Corporation in the sum of
o It misappreciated the true nature of the P37,731.68 for dishonored acceptances of commercial
relationship between the stockholders paper and in an action brought against him to recover
and the Uy Family. They had an this amount, Chua Soco's interest in the five hundred
understanding that subject shares of shares subscribed for was attached and the receipt
stock were, immediately upon issuance, seized by the sheriff.
endorsed in blank by the shareholders
and entruseed to the Uy Family for Fua Cun thereupon brought the present action
safekeeping. maintaining that by virtue of the payment of the one-half
o Johnson Lee and Sonny Moreno knew of the subscription price of the 500 shares Chua Soco in
this and were in bad faith in assigning effect became the owner 250 shares and praying that
the stock certificates to Martin and Tan his, the plaintiff's, lien on said shares, by virtue of the
and in recording it in the STB chattel mortgage, be declared to hold priority over the
o As nominees of the UY family, Sy, Suen claim of the defendant Banking Corporation; that the
and Yang’s approval was necessary for defendants be ordered to deliver the receipt in question
the effectivity of the transfer. In this case to him; and that he be awarded the sum of P5,000 in
the transfer lacked the requisite damages for wrongful attachment.
approval
o No valuable consideration for the The trial court rendered judgment in favor of the Fua
supposed transfer of stocks. The Cun declaring that Chua Soco, through the payment of
complete absence of a cause or the P25,000, acquired the right to two hundred and fifty
consideration renders the contract shares fully paid up, upon which shares the plaintiff
absolutely void and inexistent. holds a lien superior to that of the China Banking
Corporation and ordering that the receipt be returned to
Petition Dismissed. CA decision Affirmed said plaintiff. From this judgment the defendants appeal.

Justin Benedict A. Moreto ISSUE/S


1) Whether Chua Soco, through the payment of
the P25,000, acquired the right to 250 shares
which were fully paid up. (main issue for
corp)
G.R. No. L-19441 March 27, 1923 2) Whether the plaintiff holds a lien superior to
that of the defendant Banking Corporation
FUA CUN (alias Tua Cun), plaintiff-appellee, and ordering that the receipt be returned to
vs. said plaintiff. (main issue in the case)
RICARDO SUMMERS, in his capacity as Sheriff ex-
oficio of the City of Manila, and the CHINA BANKING HELD:
CORPORATION, defendants-appellants
1) No.
FACTS:
The Court stated that the payment of half the
On August 26, 1920, one Chua Soco subscribed for subscription price does not make the holder of stock the
500 shares of stock of the defendant Banking owner of half the subscribed shares. Plaintiff's rights
consist in equity in 500 shares and upon payment of the On May 18, 1921, Chua Soco executed a
unpaid portion of the subscription price he becomes promissory note in favor of the plaintiff Fua Cun for the
entitled to the issuance of certificate for the said 500 sum of P25,000 payable in ninety days and drawing
shares in his favor. interest at the rate of 1 per cent per month, securing the
note with a chattel mortgage on the shares of stock
However, shares for which no certificate of stock has subscribed for by Chua Soco, who also endorsed the
been issued may validly be mortgaged in whole (and not receipt above mentioned and delivered it to the
just with respect to the portion paid-up) and the mortgagee. The plaintiff thereupon took the receipt to the
corporation receiving notice thereof is bound to respect manager of the defendant Bank and informed him of the
the security arrangement. transaction with Chua Soco, but was told to await action
upon the matter by the Board of Directors.
Equity in shares of stock may be assigned. The In the meantime Chua Soco appears to have
assignment becomes effective as between the parties become indebted to the China Banking Corporation in
and as to third parties with notice. Equity in shares of the sum of P37,731.68 for dishonored acceptances of
stock may be a subject of a chattel mortgage but such commercial paper and in an action brought against him
will operate as a conditional equitable assignment. to recover this amount, Chua Soco's interest in the five
hundred shares subscribed for was attached and the
2) Yes, the plaintiff has a superior lien over the receipt seized by the sheriff. The attachment was levied
bank. after the defendant bank had received notice of the facts
that the receipt had been endorsed over to the plaintiff.
The claim of the defendant Banking Corporation upon Fua Cun thereupon brought the present action
which it brought the action in which the writ of maintaining that by virtue of the payment of the one-half
attachment was issued, was for the non-payment of of the subscription price of five hundred shares Chua
drafts accepted by Chua Soco and had no direct Soco in effect became the owner of two hundred and
connection with the shares of stock in question. fifty shares and praying that his, the plaintiff's, lien on
said shares, by virtue of the chattel mortgage, be
At common law a corporation has no lien upon the declared to hold priority over the claim of the defendant
shares of stockholders for any indebtedness to the Banking Corporation; that the defendants be ordered to
corporation and our attention has not been called to any deliver the receipt in question to him; and that he be
statute creating such lien here. awarded the sum of P5,000 in damages for wrongful
attachment.
On the contrary, section 120 of the Corporation Act
provides that "no bank organized under this Act shall ISSUE:
make any loan or discount on the security of the shares
of its own capital stock, nor be the purchaser or holder of Whether or not the interest held by Chua Soco
any such shares, unless such security or purchase shall was merely an equity which could not be made the
be necessary to prevent loss upon a debt previously subject of a chattel mortgage.
contracted in good faith, and stock so purchased or
acquired shall, within six months from the time of its RULING:
purchase, be sold or disposed of at public or private
sale, or, in default thereof, a receiver may be appointed NO.
to close up the business of the bank in accordance with
law." Though the courts have uniformly held that
chattel mortgages on shares of stock and other choses
The reasons for this doctrine are obvious; if banking in action are valid as between the parties, there is still
corporations were given a lien on their own stock for the much to be said in favor of the defendants' contention
indebtedness of the stockholders, the prohibition against that the chattel mortgage here in question would not
granting loans or discounts upon the security of the prevail over liens of third parties without notice; an equity
stock would become largely ineffective. in shares of stock is of such an intangible character that
it is somewhat difficult to see how it can be treated as a
chattel and mortgaged in such a manner that the
recording of the mortgage will furnish constructive notice
FUA CUN, petitioner to third parties.
vs. In regard to a chattel mortgage of shares of
SUMMERS, respondent stock: These certificates of stock are in the pockets of
G.R. No. L-19441 March 27, 1923 the owner, and go with him where he may happen to
locate, as choses in action, or evidence of his right,
FACTS: without any means on the part of those with whom he
proposes to deal on the faith of such a security of
It appears from the evidence that on August 26, ascertaining whether or not this stock is in pledge or
1920, one Chua Soco subscribed for five hundred mortgaged to others. The chief office of the company
shares of stock of the defendant Banking Corporation. may be at one place to-day and at another tomorrow.
The owner may have no fixed or permanent abode, and which he has in it to the latter, and a chattel mortgage is
with his notes in one pocket and his certificates of stock not within the meaning of such term.
in the other. Therefore, the chattel mortgage is not the
But a determination of this question is not transfer referred to in section 35 of Act No. 1459
essential in the present case. There can be no doubt commonly known as the Corporation law, which transfer
that an equity in shares of stock may be assigned and should be entered and noted upon the books of a
that the assignment is valid as between the parties and corporation in order to be valid, and which, as has
as to persons to whom notice is brought home. Such an already been said, means the absolute and
assignment exists here, though it was made for the unconditional conveyance of the title and ownership of a
purpose of securing a debt. share of stock.
If, in accordance with said section 35 of the
Corporation Law, only the transfer or absolute
conveyance of the ownership of the title to a share need
ENRIQUE MONSERRAT be entered and noted upon the books of the corporation
vs. in order that such transfer may ba valid, therefore,
CARLOS G. CERON, ET AL. ERMA, INC., and, THE inasmuch as a chattel mortgage of the aforesaid title is
SHERIFF OF MANILA not a complete and absolute alienation of the dominion
G.R. No. 37078 September 27, 1933 and ownership thereof, its entry and notation upon the
books of the corporation is not necessary requisite to its
FACTS: validity.

GONZALO CHUA
Enrique Monserrat, was the president and
GUAN
manager of the Manila Yellow Taxicab Co., Inc., and the
vs.
owner of P1,200 common shares of stock thereof. On
SAMAHANG MAGSASAKA, INC., and SIMPLICIO
March 25, 1930, in consideration of the interest shown
OCAMPO, ADRIANO G. SOTTO, and EMILIO
and the financial aid extended him in the organization of
VERGARA, as president, secretary and treasurer
the corporation by Carlos G. Ceron, one of the
respectively of the same
defendants herein, Enrique Monserrat assigned to the
G.R. No. L-42091 November 2, 1935
former the usufruct of half of the aforesaid common
shares of stock, the corresponding certificate of stock
FACTS:
No. 7, having been issued in the name of said Carlos G,
Ceron to that effect on March 24, 1930. (Exhibit 1.) Said
Samahang Magsasaka, Inc., is a corporation
assignment or transfer only gave the transferee the right
duly organized under the laws of the Philippine Islands
to enjoy, during his lifetime, the profits which might be
with principal office in Cabanatuan, Nueva Ecija, and
derived from the shares assigned him, prohibiting him
that the individual defendants are the president,
from selling, mortgaging, encumbering, alienating or
otherwise exercising any act implying absolute secretary and treasurer respectively of the same; that on
ownership of all or any of the shares in question, the June 18, 1931, Gonzalo H. Co Toco was the owner of
5,894 shares of the capital stock of the said corporation
transferor having reserved for himself and his heirs the
represented by nine certificates having a par value of P5
right to vote derived from said shares of stock and to
per share; that on said date Gonzalo H. Co Toco, a
recover the ownership thereof at the termination of the
resident of Manila, mortgaged said 5,894 shares to Chua
usufruct (Exhibit A). Stock certificate No. 7 was recorded
in the name of Carlos G. Ceron and the aforesaid deed Chiu to guarantee the payment of a debt of P20,000 due
on or before June 19, 1932. The said certificates of stock
of transfer Exhibit A, was noted by himself as secretary,
were delivered with the mortgage to the mortgagee,
on page 22 of the Stock and Transfer Book of the Manila
Chua Chiu. The said mortgage was duly registered in
Yellow Taxicab Co., Inc.
the office of the register of deeds of Manila on June 23,
ISSUE: 1931, and in the office of the said corporation on
September 30, 1931. On November 28, 1931, Chua
Chiu assigned all his right and interest in the said
Whether or not it is necessary to enter upon the
mortgage to the plaintiff and the assignment was
books of the corporation a mortgage constituted on
registered in the office of the register of deeds in the City
common shares of stock in order that such mortgage
of Manila on December 28, 1931, and in the office of the
may be valid and may have force and effect as against
third persons. said corporation on January 4, 1932.
The debtor, Gonzalo H. Co Toco, having
RULING: defaulted in the payment of said debt at maturity, the
plaintiff foreclosed said mortgage and delivered the
YES. certificates of stock and copies of the mortgage and
assignment to the sheriff of the City of Manila in order to
A "transfer" is the act by which owner of a thing sell the said shares at public auction. The sheriff
delivers it to another with the intent of passing the rights auctioned said 5,894 shares of stock on December 22,
1932, and the plaintiff having been the highest bidder for
the sum of P14,390, the sheriff executed in his favor a registration. However, in this case the court held
certificate of sale of said shares. The plaintiff tendered that registration accords validity or invalidity to the
the certificates of stock standing in the name of Gonzalo transfer. If the same is not registered, the
H. Co Toco to the proper officers of the corporation for transaction is void as to the fact of those who have
cancellation and demanded that they issue new no notice and it is also void as to the transaction
certificates in the name of the plaintiff. The said officers itself.
(the individual defendants) refused and still refuse to When the corporation is party to the
issue said new shares in the name of the plaintiff. transaction and it accepts the notice duly given to
it, then it binds the corporation, even in the transfer
ISSUE: of ownership; most especially if the corporation
already recognized the transferee as the owner.
Whether or not the mortgage registered With the latter situation, the corporation can no
considered the certificate of registration in the longer assert non-registration. However, when the
corporation. corporation is not a party to the transaction, then
the corporation cannot be bound by the notice.
RULING: (The notice pertained herein is the sale of the
shares of stock to Jollye and not the attachment of
YES. Uson.)

By analogy with the foregoing and considering


the ownership of shares in a corporation as property RICARDO NAVA
distinct from the certificates which are merely the vs.
evidence of such ownership, it seems to us a reasonable PEERS MARKETING CORP., RENATO CUSI and
construction of section 4 of Act No. 1508 to hold that the AMPARO CUSI
property in the shares may be deemed to be situated in GR L-28120, 25 November 1976
the province in which the corporation has its principal
office or place of business. If this province is also the FACTS:
province of the owner's domicile, a single registration
sufficient. If not, the chattel mortgage should be Teofilo Po as an incorporator subscribed to
registered both at the owner's domicile and in the eighty shares of Peers Marketing Corporation at one
province where the corporation has its principal office or hundred pesos a share or a total par value of eight
place of business. In this sense the property mortgaged thousand pesos. Po paid two thousand pesos or twenty-
is not the certificate but the participation and share of the five percent of the amount of his subscription. No
owner in the assets of the corporation. certificate of stock was issued to him or, for that matter,
to any incorporator, subscriber or stockholder.
On April 2, 1966 Po sold to Ricardo A. Nava for
two thousand pesos twenty of his eighty shares. In the
deed of sale Po represented that he was "the absolute
Uson VS. Diosomito
and registered owner of twenty shares" of Peers
Facts:
Marketing Corporation.
Toribia Uson attached the shares of stocks
Nava requested the officers of the corporation to
owned by Vicente Diosomito, which was given by register the sale in the books of the corporation. The
the latter as security for a loan obtained from Uson. request was denied because Po has not paid fully the
However, it was later on found out that the same
amount of his subscription. Nava was informed that Po
shares were sold to Jollye even before the
was delinquent in the payment of the balance due on his
attachment. But the problem is, the sale was not
subscription and that the corporation had a claim on his
registered in the stock and transfer book of the entire subscription of eighty shares which included the
corporation. The same was only brought to the twenty shares that had been sold to Nava.
attention of the corporation after nine months when
the attachment was levied. ISSUE:
Issue:
Whether or not Peers may be compelled by
W/N the attachment will take precedence mandamus to register the stocks in Nava’s name.
over the sale.
RULING:
Ruling:
Yes, the Court ruled that the NO.
attachment is to take precedence over the sale,
and the same shall be respected. There’s no certificate of stock issued in favor of
Prior to the ruling in Uson, registration Po. Shares of stock may be transferred by delivery to the
merely meant notice to the corporation. The same
transferee of the certificate properly indorsed. "Title may
has nothing to do with the validity or invalidity of
be vested in the transferee by delivery of the certificate
the transaction. As such actual knowledge >
with a written assignment or indorsement thereof" There BLTB, except Henry John A. Potenciano, Michael A.
should be compliance with the mode of transfer Potericiano and Candido A. Potenciano; a revocable
prescribed by law. proxy to vote the subject shares made by the sellers in
The usual practice is for the stockholder to sign favor of the buyer; a Declaration of Trust made by the
the form on the back of the stock certificate. The sellers in favor of the buyer acknowledging that the
certificate may thereafter be transferred from one person subject shares shall be held in trust by the sellers
to another. If the holder of the certificate desires to for the buyer pending their transfer to the latter's
assume the legal rights of a shareholder to enable him to name; and the duly executed capital gains tax return
vote at corporate elections and to receive dividends, he forms covering the sale, indicating no taxable gain on
fills up the blanks in the form by inserting his own name the same. Furthermore, the buyer guaranteed that it
as transferee. Then he delivers the certificate to the shall take over the management and operations of BLTB
secretary of the corporation so that the transfer may be but shall immediately surrender the same to the sellers
entered in the corporation's books. The certificate is then in case it fails to pay the balance of the purchase price
surrendered and a new one issued to the transferee. on November 26, 1997. On November 21, 1997, at a
That procedure cannot be followed in the instant meeting of the stockholders of BLTB, Benjamin Bitanga
case because, as already noted, the twenty shares in and Monina Grace Lim were elected as directors of the
question are not covered by any certificate of stock in corporation Subsequently, on November 28, 1997,
Po's name. Moreover, the corporation has a claim on the another stockholders' meeting was held, wherein
said shares for the unpaid balance of Po's subscription. Laureano A. Siy and Renato L. Leveriza were elected as
A stock subscription is a subsisting liability from the time directors. At the same meeting, the Board of Directors of
the subscription is made. The subscriber is as much BLTB elected James Olayvar, Eduardo Azucena, Evelio
bound to pay his subscription as he would be to pay any Custodia, and Gemma Santos as officers. During a
other debt. The right of the corporation to demand meeting of the Board of Directors on April 14, 1998, the
payment is no less incontestable. newly elected directors of BLTB scheduled the annual
In this case no stock certificate was issued to stockholders' meeting on May 19, 1998, to be held at the
Po. Without the stock certificate, which is the evidence of principal office of BLTB in San Pablo, Laguna. Before
ownership of corporate stock, the assignment of the scheduled meeting, Michael Potenciano wrote
corporate shares is effective only between the parties to Benjamin Bitanga, requesting for a postponement of the
the transaction. stockholders' meeting due to the absence of a thirty-day
advance notice. However, no response from Bitanga on
whether or not the request for postponement was
favorably acted upon. On the scheduled date of the
meeting, inasmuch as there was no notice of
BATANGAS LAGUNA TAYABAS BUS COMPANY,
postponement prior to that, a total of 286 stockholders,
INC vs. BITANGA representing 87% of the shares of stock of BLTB, arrived
and attended the meeting. The majority of the
G.R. No. 137934/ G.R. No. 137936 August 10, stockholders present rejected the postponement and
2001 voted to proceed with the meeting. The Potenciano
group was re-elected to the Board of Directors, and a
new set of officers was thereafter elected. On May 21,
1998, the Bitanga group filed with the SEC a Complaint
for Damages and Injunction. Their prayer for the
FACTS: On October 28, 1997, Dolores Potenciano, Max
issuance of a temporary restraining order was, however,
Joseph Potenciano, Mercedelin Potenciano, Delfin
denied at the ex-parte summary hearing conducted by
Yorro, and Maya Industries, Inc., entered into a Sale and
SEC Chairman Perfecto Yasay, Jr. Likewise, the
Purchase Agreement, whereby they sold to BMB
Potenciano group filed on May 25, 1998, a Complaint for
Property Holdings, Inc., represented by its President,
Injunction and Damages with Preliminary Injunction and
Benjamin Bitanga, their 21,071,114 shares of stock in
Temporary Restraining Order with the SEC. The SEC
BLTB. The said shares represented 47.98% of the total
Chairman Perfecto Yasay, Jr. issued a temporary
outstanding capital stock of BLTB. The purchase price
restraining order enjoining the Bitanga group from acting
for the shares of stock was P72,076,425.00. A
as officers and directors of BLTB. On June 8, 1998, the
downpayment was made while the balance was payable
Bitanga group filed another complaint with application for
on November 26, 1997. The contracting parties
a writ of preliminary injunction and prayer for temporary
stipulated that the downpayment was conditioned upon
restraining order, seeking to annul the May 19, 1998
receipt by the buyer of certain documents upon signing
stockholders' meeting. A joint hearing was conducted.
of the Agreement, namely, the Secretary's Certificate
On June 17, 1998, the SEC Hearing Panel granted the
stating that the Board of Directors of Maya Industries,
Bitanga group's application for a writ of preliminary
Inc. authorized the sale of its shares in BLTB and the
injunction upon the posting of a bond in the amount of
execution of the Agreement, and designating Dolores A.
P20,000,000.00. It declared that the May 19, 1998
Potenciano as its Attorney-in-Fact; the Special Power of
stockholders' meeting was void on the grounds that, first,
Attorney executed by each of the sellers in favor of
Michael Potenciano had himself asked for its
Dolores A. Potenciano for purposes of the Agreement;
postponement due to improper notice; and, second,
the undated written resignation letters of the Directors of
there was no quorum, since BMB Holdings, Inc., July 21, 1998 and July 27, 1998 in SEC Case No. EB
represented by the Bitanga group, which then owned 611 are ordered REINSTATED.
50.26% of BLTB's shares having purchased the same
from the Potenciano group, was not present at the said BATANGAS LAGUNA TAYABAS BUS COMPANY,
meeting. The Hearing Panel further held that the Bitanga INC., DOLORES A. POTENCIANO, MAX JOSEPH A.
Board remains the legitimate Board in a hold-over POTENCIANO, MERCEDELIN A. POTENCIANO, and
capacity. The Potenciano group filed a petition for DELFIN C. YORRO
certiorari with the SEC En Banc on June 29, 1998, vs.
seeking a writ of preliminary injunction to restrain the BENJAMIN M. BITANGA, RENATO L. LEVERIZA,
implementation of the Hearing Panel's assailed Order. LAUREANO A. SIY, JAMES A. OLAYVAR, EDUARDO
On July 21, 1998, the SEC En Banc set aside the June A. AZUCENA, MONINA GRACE S. LIM, and GEMMA
17, 1998 Order of the Hearing Panel and issued the writ M. SANTOS
of preliminary injunction prayed for. The Bitanga group G.R. No. 137934 August 10, 2001
immediately filed a petition for certiorari with the Court of
Appeals on July 22, 1998, followed by a Supplemental FACTS:
Petition on August 10, 1998. Meanwhile, on July 29,
1998, the SEC En Banc issued a writ of preliminary Dolores A. Potenciano, Max Joseph A.
injunction against the Bitanga group, after the Potenciano, Mercedelin A. Potenciano, Delfin C. Yorro,
Potencianos posted the required bond of and Maya Industries, Inc., entered into a Sale and
P20,000,000.00. On November 23, 1998, the CA Purchase Agreement, whereby they sold to BMB
rendered the now assailed Decision, reversing the Property Holdings, Inc., represented by its President,
assailed Orders of the SEC En Banc and reinstating the Benjamin Bitanga, their 21,071,114 shares of stock in
Order of the Hearing Panel ordered dated June 17, BLTB. The said shares represented 47.98% of the total
1998. The CA denied the Motions for Reconsideration in outstanding capital stock of BLTB. The contracting
a Resolution dated March 25, 1999. Hence, this petition parties stipulated that the downpayment was conditioned
for review. upon receipt by the buyer of certain documents upon
signing of the Agreement, namely, the Secretary's
ISSUE: Whether or not the stockholders' meeting on Certificate stating that the Board of Directors of Maya
May 19, 1998 was void since BMB Holdings, Inc., Industries, Inc. authorized the sale of its shares in BLTB
represented by the Bitanga group was not present at the and the execution of the Agreement, and designating
said meeting. Dolores A. Potenciano as its Attorney-in-Fact; the
Special Power of Attorney executed by each of the
RULING: Until registration is accomplished, the sellers in favor of Dolores A. Potenciano for purposes of
transfer, though valid between the parties, cannot be the Agreement; the undated written resignation letters of
effective as against the corporation. Thus, the the Directors of BLTB, except Henry John A.
unrecorded transferee, the Bitanga group in this Potenciano, Michael A. Potericiano and Candido A.
case, cannot vote nor be voted for. The purpose of Potenciano); a revocable proxy to vote the subject
registration, therefore, is two-fold: to enable the shares made by the sellers in favor of the buyer; a
transferee to exercise all the rights of a stockholder, Declaration of Trust made by the sellers in favor of the
including the right to vote and to be voted for, and to buyer acknowledging that the subject shares shall be
inform the corporation of any change in share held in trust by the sellers for the buyer pending their
ownership so that it can ascertain the persons transfer to the latter's name; and the duly executed
entitled to the rights and subject to the liabilities of a capital gains tax return forms covering the sale,
stockholder. Until challenged in a proper indicating no taxable gain on the same.
proceeding, a stockholder of record has a right to
participate in any meeting; his vote can be properly ISSUE:
counted to determine whether a stockholders'
resolution was approved, despite the claim of the Whether or not the Bitanga group vote or be
alleged transferee. On the other hand, a person who voted upon.
has purchased stock, and who desires to be
recognized as a stockholder for the purpose of RULING:
voting, must secure such a standing by having the
transfer recorded on the corporate books. Until the NO.
transfer is registered, the transferee is not a
stockholder but an outsider. We are in full accord with the SEC En Banc on
this matter. Indeed, until registration is accomplished,
the transfer, though valid between the parties, cannot be
WHEREFORE, in view of all the foregoing, the instant
effective as against the corporation. Thus, the
petitions for review are GRANTED. The Decision of the
Court of Appeals dated November 23, 1998 in CA-G.R. unrecorded transferee, the Bitanga group in this case,
SP No. 48374 and its resolution dated March 25, 1999 cannot vote nor be voted for. The purpose of
registration, therefore, is two-fold: to enable the
are SET ASIDE. The Orders of the SEC En Banc dated
transferee to exercise all the rights of a stockholder,
including the right to vote and to be voted for, and to
inform the corporation of any change in share ownership The transfer of the subject certificate made by
so that it can ascertain the persons entitled to the rights Dico to petitioner was not valid as to the spouses Atinon,
and subject to the liabilities of a stockholder. Until the judgment creditors, as the same still stood in the
challenged in a proper proceeding, a stockholder of name of Dico, the judgment debtor, at the time of the
record has a right to participate in any meeting; his vote levy on execution. In addition, as correctly ruled by the
can be properly counted to determine whether a CA, the entry in the minutes of the meeting of the Club's
stockholders' resolution was approved, despite the claim board of directors noting the resignation of Dico as
of the alleged transferee. On the other hand, a person proprietary member thereof does not constitute
who has purchased stock, and who desires to be compliance with Section 63 of the Corporation Code.
recognized as a stockholder for the purpose of voting, Said provision of law strictly requires the recording of the
must secure such a standing by having the transfer transfer in the books of the corporation, and not
recorded on the corporate books. Until the transfer is elsewhere, to be valid as against third parties.
registered, the transferee is not a stockholder but an Accordingly, the CA committed no reversible error in
outsider. rendering the assailed decision.
The Court finds no error either in jurisdiction or
judgment on the part of the SEC En Banc, since its
conclusions of law were anchored on established
principles and jurisprudence. The petition is denied. CHEMPHIL EXPORT AND IMPORT CORPORATION
vs.
THE HON. COURT OF APPEALS (Former Twelfth
Division), PHILIPPINE INVESTMENTS SYSTEMS
NEMESIO GARCIA ORGANIZATION (PISO), BANK OF THE PHILIPPINE
vs. ISLANDS (BPI), PHILIPPINE COMMERCIAL
NICOLAS JOMOUAD, Ex-officio Provincial Sheriff of INDUSTRIAL BANK (PCIB), RIZALCOMMERCIAL
Cebu and SPOUSES JOSE ATINON & SALLY BANKING CORPORATION (RCBC) and LAND BANK
ATINON OF THE PHILIPPINES (LBP)
G.R. No. 133969 January 26, 2000 G.R. No. 97217 April 10, 1992

FACTS: FACTS

Petitioner filed an action for injunction with Dynetics, Inc. and Antonio M. Garcia filed a
prayer for preliminary injunction against respondents complaint for declaratory relief and/or injunction against
spouses Jose and Sally Atinon and Nicolas the PISO, BPI, LBP, PCIB and RCBC or the consortium
Jomouad, ex-officio sheriff of Cebu. with the Regional Trial Court of Makati, Branch 45 (Civil
Said action stemmed from an earlier case for Case No. 8527), seeking judicial declaration,
collection of sum of money, docketed as Civil Case No. construction and interpretation of the validity of the
CEB-10433, before the RTC, Branch 10 of Cebu, filed by surety agreement that Dynetics and Garcia had entered
the spouses Atinon against Jaime Dico. In that case into with the consortium and to perpetually enjoin the
(collection of sum of money), the trial court rendered latter from claiming, collecting and enforcing any
judgment ordering Dico to pay the spouses Atinon the purported obligations which Dynetics and Garcia might
sum of P900,000.00 plus interests. have undertaken in said agreement.
After said judgment became final and executory, The consortium filed their respective answers
respondent sheriff proceeded with its execution. In the with counterclaims alleging that the surety agreement in
course thereof, the Proprietary Ownership Certificate question was valid and binding and that Dynetics and
(POC) No. 0668 in the Cebu Country Club, which was in Garcia were liable under the terms of the said
the name of Dico, was levied on and scheduled for agreement. It likewise applied for the issuance of a writ
public auction. Claiming ownership over the subject of preliminary attachment against Dynetics and Garcia.
certificate, petitioner filed the aforesaid action for
injunction with prayer for preliminary injunction to enjoin ISSUE:
respondents from proceeding with the auction.
Whether or not the attachment of shares of
ISSUE: stock, in order to bind third persons, must be recorded in
the stock and transfer book of the corporation.
Whether or not the appellate court erroneously
rely on Section 63 of the Corporation Code in upholding RULING:
the levy on the subject certificate to satisfy the judgment
debt of Dico in Civil Case No. CEB-14033. NO.

RULING: Section 7(d), Rule 57 of the Rules of Court was


complied with by the consortium (through the Sheriff of
NO. the trial court) when the notice of garnishment over the
Chemphil shares of Garcia was served on the president Deeds be ordered to cancel TCT No. 22431 and to issue
of Chemphil on July 19, 1985. Indeed, to bind third a new title in her favor.
persons, no law requires that an attachment of shares of
stock be recorded in the stock and transfer book of a ISSUE:
corporation. The statement attributed by the Regional
Trial Court to the Supreme Court in Samahang Whether or not petitioner’s ownership in the
Magsasaka, Inc. vs. Gonzalo Chua Guan, G.R. No. L- outstanding capital stock of SUBIC entitles them to a
7252, February 25, 1955 (unreported), to the effect that significant vote in the corporate affairs.
"as between two attaching creditors, the one whose
claim was registered first on the books of the corporation RULING:
enjoys priority," is an obiter dictum that does not modify
the procedure laid down in Section 7(d), Rule 57 of the NO.
Rules of Court.
Therefore, ruled the Court of Appeals, the The words "an interest in the subject" mean a
attachment made over the Chemphil shares in the name direct interest in the cause of action as pleaded, and
of Garcia on July 19, 1985 was made in accordance with which would put the intervenor in a legal position to
law and the lien created thereby remained valid and litigate a fact alleged in the complaint, without the
subsisting at the time Garcia sold those shares to FCI establishment of which plaintiff could not recover.
(predecessor-in-interest of appellee CEIC) in 1988. Here, the interest, if it exists at all, of petitioners-
movants is indirect, contingent, remote, conjectural,
consequential and collateral. At the very least, their
MAGSAYSAY-LABRADOR, petitioner interest is purely inchoate, or in sheer expectancy of a
vs. right in the management of the corporation and to share
COURT OF APPEALS, respondent in the profits thereof and in the properties and assets
G.R. No. 58168 December 19, 1989 thereof on dissolution, after payment of the corporate
debts and obligations.
FACTS: While a share of stock represents a
proportionate or aliquot interest in the property of the
On February 9, 1979, Adelaida Rodriguez- corporation, it does not vest the owner thereof with any
Magsaysay, widow and special administratix of the legal right or title to any of the property, his interest in the
estate of the late Senator Genaro Magsaysay, brought corporate property being equitable or beneficial in
before the then Court of First Instance of Olongapo an nature. Shareholders are in no legal sense the owners of
action against Artemio Panganiban, Subic Land corporate property, which is owned by the corporation as
Corporation (SUBIC), Filipinas Manufacturer's Bank a distinct legal person.
(FILMANBANK) and the Register of Deeds of Zambales.
In her complaint, she alleged that in 1958, she and her
husband acquired, thru conjugal funds, a parcel of land
with improvements, known as "Pequena Island", covered Asset Privatization Trust VS. Sandiganbayan
by TCT No. 3258; that after the death of her husband,
Facts:
she discovered [a] an annotation at the back of TCT No.
Roberto Garcia, Rosario Olivares, Tuynita Salud
3258 that "the land was acquired by her husband from
Soriano, Rosita Sosing, Jose Luna Castro, Araceli
his separate capital;" [b] the registration of a Deed of
Linsangan, Luisa Linsangan, Reynaldo Naval, Manuel
Assignment dated June 25, 1976 purportedly executed Salak and Augusto Villianueva, and the Development
by the late Senator in favor of SUBIC, as a result of
Bank of the Philippines represented by Alicia Ll. Reyes,
which TCT No. 3258 was cancelled and TCT No. 22431
signed an instrument denominated as a Deed of
issued in the name of SUBIC; and [c] the registration of
Assignment of Voting Shares.
Deed of Mortgage dated April 28, 1977 in the amount of
In another instrument dated January 16, 1979,
P 2,700,000.00 executed by SUBIC in favor of denominated as Supplement to the Deed of Assignment,
FILMANBANK; that the foregoing acts were void and the parties made the following covenants and
done in an attempt to defraud the conjugal partnership
stipulations, to wit: This assignment shall secure not only
considering that the land is conjugal, her marital consent
the foreign currency loan of US $124,140.00 referred to
to the annotation on TCT No. 3258 was not obtained, the
above but also all previous and future accommodations
change made by the Register of Deeds of the
granted/to be granted in favor of PJI by the ASSIGNEE.
titleholders was effected without the approval of the The assignment is subject to the same terms
Commissioner of Land Registration and that the late and conditions of the deed of assignment of June 17,
Senator did not execute the purported Deed of
1977, and said terms and conditions are hereby
Assignment or his consent thereto, if obtained, was
reiterated, confirmed and continued to be in full force
secured by mistake, violence and intimidation. She
and effect, making said terms and conditions integral
further alleged that the assignment in favor of SUBIC
parts hereof by reference. The ASSIGNORS further bind
was without consideration and consequently null and themselves to assign such additional shares as may
void. She prayed that the Deed of Assignment and the later be registered in their names, so as to complete and
Deed of Mortgage be annulled and that the Register of
maintain at all times the assignment in favor of the
ASSIGNEE at not less than 67% of the total subscribed subject to a resolutory condition, which was the
and paid-up shares of the company. On February 27, settlement of PJI’s loan obligation with DBP.”
1987, the DBP pursuant to the provisions of
Proclamation No. 50, transferred its rights in PJI to the
Asset Privatization Trust (APT). In the exercise of its
assigned rights, APT opted to enforce the provisions of
the above-mentioned deed of assignment and its LIM TAY
supplement. vs.
On January 31, 1992, respondent Rosario B. COURT OF APPEALS, GO FAY AND CO. INC., SY
Olivares filed with the Sandiganbayan an urgent ex-parte GUIOK, and THE ESTATE OF ALFONSO LIM
motion to order the appearance of DBP or APT G.R. No. 126891, August 5, 1998
representatives to vote the 67% shares in the
stockholders meeting of PJI set for February 4, 1992 at FACTS:
2:00 in the afternoon.
On January 8, 1980, Respondent-Appellee Sy
Issue: Guiok secured a loan from the petitioner in the amount
W/N the assignment to DBP and later to APT of of P40,000 payable within six (6) months. To secure the
voting shares of the PJI was an assignment of voting payment of the aforesaid loan and interest thereon,
rights or voting shares. Respondent Guiok executed a Contract of Pledge in
favor of the [p]etitioner whereby he pledged his three
Ruling: hundred (300) shares of stock in the Go Fay & Company
Voting shares. Inc., Respondent Corporation, for brevity's sake.
The Deed of Assignment is very clear that what Respondent Guiok obliged himself to pay interest on
was assigned to DBP (APT) were voting shares as said loan at the rate of 10% per annum from the date of
distinguished from non-voting shares. Obviously, it said contract of pledge. On the same date, Alfonso Sy
meant that the assignees of the shares had the right as Lim secured a loan from the [p]etitioner in the amount of
though they were owners of the shares. It is true that the P40,000 payable in six (6) months. To secure the
assignment was predicated on the intention that it would payment of his loan, Sy Lim executed a "Contract of
serve as security vis--vis DBPs financial accommodation Pledge" covering his three hundred (300) shares of stock
extended to PJI, but it was a valid and duly executed in Respondent Corporation. Under said contract, Sy Lim
assignment, subject to a resolutory condition, which was obliged himself to pay interest on his loan at the rate of
the settlement of PJIs loan obligation with DBP. 10% per annum from the date of the execution of said
Private respondent Olivares submitted the view contract.
that what was assigned to DBP/APT was merely voting However, Respondent Guiok and Sy Lim failed
rights and adduced the endorsements attached to each to pay their respective loans and the accrued interests
stock certificate representing the assigned shares to thereon to the [p]etitioner. In October, 1990, the
DBP/APT. petitioner filed a "Petition for Mandamus" against
It is clear in the endorsements that what was Respondent Corporation, with the SEC entitled "Lim Tay
given to DBP was not just voting rights but a transfer and versus Go Fay & Company. Inc., SEC Case No. 03894".
assignment of the shares. The phrase counter-signed by
Delia S. Tantuico simply clarifies that the transfer and ISSUE:
assignment include also the right to vote the shares. The
deed executed between DBP and the assignors was Whether or not there is there dacion en pago.
denominated as a Deed of Assignment of voting shares,
not of voting rights. RULING:
The private respondent contends that since the
shares involved were sequestered shares, jurisdiction NO.
over the subject of the dispute lies with the
Sandiganbayan. The issue between APT and At the outset, it must be underscored that
respondent Olivares involved a dispute between petitioner did not acquire ownership of the shares by
stockholders, clearly intra-corporate in nature, hence, virtue of the contracts of pledge. Article 2112 of the Civil
outside the jurisdiction of the Sandiganbayan and Code states: The creditor to whom the credit has not
squarely was within the jurisdiction of the Securities and been satisfied in due time, may proceed before a Notary
Exchange Commission. Public to the sale of the thing pledged. This sale shall be
It seems that the assignment of voting shares as made at a public auction and with notification to the
security for a loan operates to give the assignee not only debtor and the owner of the thing pledged in a proper
the right to vote on the shares, but would also treat the case, stating the amount for which the public sale is to
assignee as the owner of the shares (not just an be held. If at the first auction the thing is not sold, a
equitable mortgage): “It is true that the assignment was second one with the same formalities shall be held; and
predicated on the intention that it would serve as security if at the second auction there is no sale either, the
vis-à-vis DBP’s financial accommodation extended to creditor may appropriate the thing pledged. In this case
PJI, but it was a valid and duly executed assignment,
he shall be obliged to give an acquaintance for his entire In October 1990, Lim Tay filed a "Petition for Mandamus"
claim. against Go Fay & Co., with SEC praying that an order be
There is no showing that petitioner made any issued directing the corporate secretary to register the
attempt to foreclose or sell the shares through public or stock transfers and issue new certificates in favor of Lim
private auction, as stipulated in the contracts of pledge Tay; and ordering the corporation to pay all dividends
and as required by Article 2112 of the Civil Code. due and unclaimed on the said certificates to Lim Tay. In
Therefore, ownership of the shares could not have the interim, Sy Lim died. Guiok and the Intestate Estate
passed to him. The pledgor remains the owner during of Alfonso Sy Lim, represented by Conchita Lim, filed
the pendency of the pledge and prior to foreclosure and their Answer-In-Intervention with the SEC.
sale, as explicitly provided by Article 2103 of the same
Code: Unless the thing pledged is expropriated, the The SEC hearing officer dismissed Lim Tay's Complaint
debtor continues to be the owner thereof. on the ground that although the SEC had jurisdiction
Neither did petitioner acquire the shares by over the action, he failed to prove the legal basis for the
virtue of a novation of the contract of pledge. Novation is secretary of the Corporation to be compelled to register
defined as "the extinguishment of an obligation by a stock transfers in favor of Lim Tay and to issue new
subsequent one which terminates it, either by changing certificates of stock under his name. His appeal was
its object or principal conditions, by substituting a new denied by SEC. He appealed with CA.
debtor in place of the old one, or by subrogating a third
person to the rights of the creditor." Novation of a The CA debunked Lim Tay's claim that he had acquired
contract must not be presumed. "In the absence of an ownership over the shares by virtue of novation, holding
express agreement, novation takes place only when the that Guiok's and Sy Lim's endorsement and delivery of
old and the new obligations are incompatible on every the shares were pursuant to Articles 2093 and 2095 of
point. the Civil Code and that Lim Tay's receipt of dividends
was in compliance with Article 2102 of the same Code.
G.R. No. 126891 August 5, 1998
Arguments of Lim Tay: He contends that it has acquired
LIM TAY, petitioner, ownership of the shares "through extraordinary
prescription," pursuant to Article 1132 of the Civil Code,
vs.
COURT OF APPEALS, GO FAY AND CO. INC., SY and through respondents' subsequent acts, which
GUIOK, and THE ESTATE OF ALFONSO LIM, amounted to a novation of the contracts of pledge.
respondents. Petitioner also claims that there was dacion en pago, in
which the shares of stock were deemed sold to
petitioner, the consideration for which was the
FACTS: extinguishment of the loans and the interests thereon.
Petitioner likewise claims that laches bars respondents
On 8 January 1980, Sy Guiok secured a loan from Lim from recovering the subject shares.
Tay in the amount of P40,000 payable within 6 months.
To secure the payment of the aforesaid loan and interest ISSUE:
thereon, Guiok executed a Contract of Pledge in favor of
Lim Tay. He pledged his 300 shares of stock in the Go WON Lim Tay is the owner of the shares previously
Fay & Company Inc. Guiok obliged himself to pay subjected to pledge, for him to cause the registration of
interest on said loan at the rate of 10% per annum from said shares in his own name.
the date of said contract of pledge.
RULING: NO.
On the same date, Alfonso Sy Lim secured a loan, from
Lim Tay in the amount of P40,000 payable in 6 months. Lim Tay's ownership over the shares was not yet
To secure the payment of his loan, Sy Lim executed a perfected when the Complaint was filed. The contract of
"Contract of Pledge" covering his 300 shares of stock in pledge certainly does not make him the owner of the
Go Fay & Co. Under said contract, Sy Lim obliged shares pledged.
himself to pay interest on his loan at the rate of 10% per
annum from the date of the execution of said contract. Further, whether prescription effectively transferred
ownership of the shares, whether there was a novation
The contract provided that Lim Tay was merely of the contracts of pledge, and whether laches had set in
authorized to foreclose the pledge upon maturity of the were difficult legal issues, which were unpleaded and
loans, not to own them. The foreclosure is not automatic, unresolved when Lim Tay asked the corporate secretary
for it must be done in a public or private sale. of Go Fay to effect the transfer, in his favor, of the
shares pledged to him. Lim Tay has failed to establish a
Guiok and Sy Lim endorsed their respective shares of clear legal right.
stock in blank and delivered the same to Lim Tay.
However, Guiok and Sy Lim failed to pay their respective Lim Tay's contention that he is the owner of the said
loans and the accrued interests thereon to Lim Tay. shares is completely without merit. Lim Tay does not
have any ownership rights at all. At the time Lim Tay
instituted his suit at the SEC, his ownership claim had no of stocks (sic) of the corporation from stockholder Peter
prima facie leg to stand on. At best, his contention was Chiu.
disputable and uncertain.
Ting Ping Lay purchased another one-thousand four-
Lim Tay cannot claim to have acquired ownership over hundred (1,400) shares from his brother Teng Ching
the certificates of stock through extraordinary
Lay.
prescription, as provided for in Article 1132 of the Civil
Code. What is required by Article 1132 is possession in
Ting Ping Lay acquired 1,440 more shares from
the concept of an owner. Herein, Lim Tay's possession
of the stock certificates came about because they were Ismaelita Maluto.
delivered to him pursuant to the contracts of pledge. His
possession as a pledgee cannot ripen into ownership by Teng Ching Lay served as president and operations
prescription. Lim Tay expressly repudiated the pledge, manager until his death. Respondent Anna Teng served
only when he filed his Complaint and claimed that he as the Corporate Secretary.
was not a mere pledgee, but that he was already the
owner of the shares. Henry Teng took over the management of the company
after his father's death.
Based on the foregoing, Lim Tay has not acquired the
certificates of stock through extraordinary prescription. Ting Ping Lay in order to protect his shareholdings with
Neither did Lim Tay acquire the shares by virtue of a the company requested Anna Teng to enter the transfer
novation of the contract of pledge. Novation cannot be
of shares of stocks for the proper recording of his
presumed by Guiok's and Sy Lim's indorsement and
delivery of the certificates of stock covering the 600 acquisitions in the Stock and Transfer Book of the
shares, nor Lim Tay's receipt of dividends from 1980 to corporation. Likewise, he demanded the issuance of the
1983, nor the fact that Guiok and Sy Lim have not new certificates of stock in his favor. However,
instituted any action to recover the shares since 1980. respondents refused despite repeated demands.
Novation is never presumed inferred.
Ting Ping Lay filed a petition for mandamus with the
Notes: There is a contract of pledge between Guiok Securities and Exchange Commission against TCL
(respondent) and Lim Tay (petitioner) & Sy Lim Corporation and Anna Teng.
(respondent) and Lim Tay (petitioner). What was
mortgaged? 300 shares of stock in the Go Fay & After the trial, the hearing officer found for the petitioner.
Company Inc. by Guiok and 300 shares of stock in the
Go Fay & Company Inc. by Sy Lim. Date - January 8, The Commission en banc modified the aforequoted
1980.
ruling by deleting the liability of TCL Corporation relative
to the award of moral damages and attorney's fees.
Subsequently, herein petitioners filed with respondent
G.R. No. 129777 January 5, 2001 Court of Appeals a petition for review of the Order of the
SEC, the Court of Appeals dismissed the petition.
TCL SALES CORPORATION and ANNA TENG,
petitioners, vs. Issue: Whether or not the alleged transfer of shares in
favor of private respondent are valid and can be ordered
HON. COURT OF APPEALS and TING PING LAY, recorded?
respondents.
Ruling: Yes, Moreover, the SEC en banc found that the
Facts: Respondent TCL Corporation was organized and petitioners did not refute the validity of the transfers of
registered sometime in 1973. The incorporators were shares of stock to Ting Ping Lay, insofar as those shares
Teng Ching Lay, Henry Teng (son of Teng Ching Lay), covered duly indorsed stock certificates were concerned.
Anna Teng (daughter of Teng Ching Lay), Ismaelita Petitioners themselves conceded that they could not
Maluto and Peter Chiu. The corporation started with an assail the documents evincing the transfer of the shares
authorized capital stock of 5,000 shares valued at to Ting Ping Lay.
P1,000.00 per share with an aggregate value of
P500,000.00. In 1974 the Articles of Incorporation was In Lim Tay vs. Court of Appeals, we held that the
amended increasing its authorized capital stock to registration of shares in a stockholder's name, the
20,000 shares valued at P2,000,000.00 of which 8,000 issuance of stock certificates, and the right to receive
shares were subscribed and fully paid. dividends which pertain to the shares are all rights that
flow from ownership. Respondent Ting Ping Lay was
Petitioner Ting Ping Lay (the brother of Teng Ching Lay) able to establish prima facie ownership over the shares
acquired by purchase four-hundred eighty (480) shares of stocks in question, through deeds of transfer of shares
of stock of TCL Corporation. Petitioners could not its power of control and supervision over all corporations
repudiate these documents. Hence, the transfer of to encourage active public participation in the affairs of
shares to him must be recorded on the corporation's private corporations by way of investments.
stock and transfer book.

Noteworthy, of the petition before us contains a listing of


the corporation's stockholders and their respective Tayag vs Benguet Consolidated Inc
shares before and after the execution of a certain deed
26 SCRA 242 [GR No. L-23145 November 27,1968]
of assignment. Respondent Ting Ping Lay is listed as a
stockholder of the corporation in this document. By this
inclusion, petitioners have in effect rebutted their own
claim in their petition that Ting Ping Lay "is not and has Facts: County Trust Company of New York, United
neither been an incorporator nor a stockholder of the States of America is the domiciliary administration of the
corporation". Undoubtedly then, the dispute is an intra- decedent, Idonah Slade Perkins who owned 33,002
corporate controversy, involving as it does stockholders shares of stocks in the appellant, domestic corporation,
of TCL Corporation. Benguet Consolidated Inc. located in the Philippines. A
dispute arose between the appellee, Tayag who is the
The determination of whether or not a shareholder is appointed ancillary of Perkins in the Philippines and the
entitled to exercise the rights of a stockholder is within domiciliary administration as to who is entitled to the
the jurisdiction of the SEC.21 As held by the Court, thru possession of the certificate of shares, however, County
Justice A. Panganiban in Lim Tay: Trust Company refuses to transfer the said certificate to
Tayag despite the order of the court. Hence, the
"The duty of a corporate secretary to record transfers of
appellee was compelled to petition the court for the
stocks is ministerial. However, he cannot be compelled
appellant to declare the subject certificates as lost to
to do so when the transferee's title to said shares has no
which appellant allegeed that no new certificate can be
prima facie validity or is uncertain. More specifically, a
issued and the same cannot be rendered as lost in
pledgor, prior to foreclosure and sale, does not acquire
accordance with their by-laws.
ownership rights over the pledged shares and thus
cannot compel the corporate secretary to record his
alleged ownership of such shares on the basis merely of
the contract of pledge. Similarly, the SEC does not Issue: Whether or not the certificate of shares of stock
acquire jurisdiction over a dispute when a party's claim can be declared lost.
to being a shareholder is, on the face of the complaint,
invalid or inadequate or is otherwise negated by the very
allegations of such complaint. Mandamus will not issue
Held: Yes. Administration whether principal or ancillary
to establish a right, but only to enforce one that is
certainly extends to the assets of a decedent found
already established."
within the state or country where it was granted.
The fact that Ting Ping Lay is allegedly not yet a
stockholder of record does not remove the case from the
jurisdiction of the SEC, for it is precisely the right of It is often necessary to have more than one
recording and the right to be issued stock certificates administration of an estate. When a person dies
that said respondent sought to enforce by mandamus. intestate owning property located in the country of
his domicile as well as in a foreign country,
In addition, even if Ting Ping Lay were not a stockholder,
administration is had in both countries. That which
he is nonetheless a member of the public whose
is granted in the jurisdiction of decedent’s last
investment in the corporation the law seeks to protect
domicile is termed the principal administration, while
and encourage, as his purchase of the shares of stock
any other administration is termed the ancillary
has been established. After all, the principal function of
administration. The reason for the latter is because a
the SEC is the supervision and control of corporations,
grant of administration does not ex proprio vigore
partnerships and associations with the end in view that
have any effect beyond the limits of the country in
investments in these entities may be encouraged and
which it is granted.Hence, an administration
protected, and their activities pursued for the protection
appointed in a foreign state has no authority in the
of economic development. In other words, the
Philippines. The ancillary administration is proper,
jurisdiction of the SEC should be construed in relation to
whenever a person dies, leaving in a country other
than that of his last domicile, property to be
administered in the nature of the deceased’s liable
for his individual debts or to be distributed among
his heirs.

Since there is refusal, persistently adhered to by the


domiciliary administration in New York, to deliver the
shares of stocks of appellant corporation owned by the
decedent to the ancillary administration in the
Philippines, there was nothing unreasonable or arbitrary
in considering them lost and requiring the appellant to
issue new certificates in lieu thereof. Thereby the task
incumbent under the law on the ancillary administration
could be discharged and his responsibility fulfilled.

Assuming that a contrariety exist between the provision


of the laws and the command of a court decree, the
latter is to be followed.

A corporation as known to Philippine jurisprudence is a


creature without any existence until it has received the
imprimatur of state according to law. It is logically
inconceivable therefore it will have rights and privileges
of a higher priority than that of its creator, more than
that, it cannot legitimately refuse to yield obedience to
acts of its state organs, certainly not excluding the
judiciary, whenever called upon to do so.

S-ar putea să vă placă și